You are on page 1of 153

PDF created with pdfFactory Pro trial version www.pdffactory.

com
‫ﺣﺪﻭﺩ ﺍﻟﻔﻴﺰﻳﺎء ﺍﻟﻜﻼﺳﻴﻜﻴﺔ‬
‫‪THE LIMITS OF CLASSICAL PHYSICS‬‬
‫ﻤﻴﻼﺩ ﺍﻟﻨﻅﺭﻴﺔ ﺍﻟﻜﻤﻴﺔ ‪The Dawn of the Quantum Theory‬‬

‫ﻓﻲ ﻨﻬﺎﻴﺔ ﺍﻟﻘﺭﻥ ﺍﻟﺘﺎﺴﻊ ﻋﺸﺭ‪ ،‬ﺍﻋﺘﻘﺩ ﻜﺜﻴﺭ ﻤﻥ ﺍﻟﻌﻠﻤﺎﺀ ﺃﻥ ﻜل ﺍﻻﻜﺘـﺸﺎﻓﺎﺕ‬


‫ﺍﻟﻌﻠﻤﻴﺔ ﻗﺩ ﺘﻡ ﺇﻨﺠﺎﺯﻫﺎ ﻭﻓﻬﻤﻬﺎ ﻭﺃﻨﻪ ﻟﻡ ﻴﺒﻘﻰ ﺇﻻ ﺒﻌﺽ ﺍﻟﻤـﺴﺎﺌل ﺍﻟﺒـﺴﻴﻁﺔ ﺍﻟﺘـﻲ‬
‫ﺘﺤﺘﺎﺝ ﻟﻤﺯﻴﺩ ﻤﻥ ﺍﻹﻴﻀﺎﺡ‪ .‬ﺇﻥ ﻫﺫﻩ ﺍﻟﻘﻨﺎﻋﺔ ﻭﻜﺎﻨﺕ ﻨﺎﺸﺌﺔ ﻤﻥ ﺍﻟﺘﻘﺩﻡ ﺍﻟﻌﻠﻤﻲ ﻓـﻲ‬
‫ﻤﺠﺎﻻﺕ ﺸﺘﻰ ﻭﺍﻟﺫﻱ ﺘﻤﺜل – ﻋﻠﻰ ﺴﺒﻴل ﺍﻟﻤﺜﺎل – ﻓﻲ ﻤﻴﻜﺎﻨﻴﻜﺎ ﻨﻴـﻭﺘﻥ ‪Newton‬‬
‫ﻭﺍﻟﺘﻲ ﻁﹸﻭﺭﺕ ﺒﻭﺍﺴﻁﺔ ﺍﻟﻌﺎﻟﻤﺎﻥ ﻻﺠـﺭﺍﻨﺞ ﻭﻫـﺎﻤﻠﺘﻭﻥ ‪J. LaGrange and W.‬‬
‫‪ .Hamilton‬ﺤﻴﺙ ﺘﻡ ﺍﺴﺘﺨﺩﺍﻡ ﻫﺫﻩ ﺍﻟﻨﻅﺭﻴﺔ ﻟﻭﺼﻑ ﺤﺭﻜﺔ ﺍﻟﻜﻭﺍﻜﺏ ﻭﻜﺫﻟﻙ ﻓﻬـﻡ‬
‫ﻜﺜﻴﺭ ﻤﻥ ﺍﻟﻅﻭﺍﻫﺭ ﺍﻟﻤﻌﻘﺩﺓ ﻤﺜل ﻨﻅﺭﻴﺔ ﺍﻟﻤﺭﻭﻨـﺔ ‪ elasticity theory‬ﻭﺩﻴﻨﺎﻤﻴﻜـﺎ‬
‫ﺍﻟﻤﻭﺍﺌﻊ ‪ hydrodynamic‬ﺇﻨﺠﺎﺯﺍﺕ ﺍﻟﻌﺎﻟﻡ ﺠﻭل ﻭﺒﻴﺎﻥ ﺘﻜﺎﻓﺅ ﺍﻟـﺸﻐل ﻭﺍﻟﺤـﺭﺍﺭﺓ‪،‬‬
‫ﺃﺒﺤﺎﺙ ﻜﺎﺭﻨﻭﺕ ‪ Carnot‬ﻭﺍﻟﺘﻲ ﺃﺩﺕ ﻟﻔﻬﻡ ﺍﻹﻨﺘﺭﻭﺒﻲ ﻭﺍﻟﻘﺎﻨﻭﻥ ﺍﻟﺜـﺎﻨﻲ ﻟﻠـﺩﻴﻨﺎﻤﻴﻜﺎ‬
‫ﺍﻟﺤﺭﺍﺭﻴﺔ‪ ،‬ﻭﻤﺎ ﻴﺘﺒﻊ ﻫﺫﻩ ﺍﻷﺒﺤﺎﺙ ﻤﻥ ﺘﻁﻭﻴﺭ ﻋﻠﻰ ﻴﺩ ﺍﻟﻌـﺎﻟﻡ ﺠـﺒﺱ ‪J. Gibbs‬‬
‫ﻹﺭﺴﺎﺀ ﺃﺴﺱ ﻋﻠﻡ ﺍﻟﺩﻴﻨﺎﻤﻴﻜﺎ ﺍﻟﺤﺭﺍﺭﻴﺔ‪.‬‬
‫ﻭﺸﻬﺩﺕ ﻤﺠـﺎﻻﺕ ﺃﺨـﺭﻯ ﻤـﻥ ﺍﻟﻔﻴﺯﻴـﺎﺀ )ﻤﺜـل ﺍﻟـﻀﻭﺀ ﻭﺍﻟﻨﻅﺭﻴـﺔ‬
‫ﺍﻟﻜﻬﺭﻭﻤﻐﻨﺎﻁﻴﺴﻴﺔ ‪ (optics and electromagnetic theory‬ﺇﻨﺠﺎﺯﺍﺕ ﻤﻠﺤﻭﻅـﺔ‪.‬‬
‫ﻓﻤﺜﻼﹰ‪ ،‬ﺍﻻﺴﺘﻨﺘﺎﺠﺎﺕ ﺍﻟﻬﺎﻤﺔ ﺍﻟﺘﻲ ﺘﻭﺼل ﺇﻟﻴﻬﺎ ﺍﻟﻌﺎﻟﻡ ﻤﺎﻜﺴﻭﻴل ‪ J. Maxwell‬ﻤﺘﻤﺜﻠﺔ‬
‫ﺒﻤﻌﺎﺩﻻﺘﻪ ﺍﻟﺸﻬﻴﺭﺓ "ﻭﺍﻟﺒﺴﻴﻁﺔ" ﻭﺍﻟﺘـﻲ ﻭ‪‬ﺤ‪‬ـﺩﺕ ﻤﺠـﺎﻻﺕ ﺍﻟـﻀﻭﺀ ﻭﺍﻟﻜﻬﺭﺒﻴـﺔ‬
‫ﻭﺍﻟﻤﻐﻨﺎﻁﻴﺴﻴﺔ ﻭﻤﺎ ﻴﺘﺒﻊ ﻫﺫﻩ ﺍﻷﺒﺤﺎﺙ ﻤﻥ ﺍﻟﺘﺠﺎﺭﺏ ﺍﻟﻤﻌﻤﻠﻴﺔ ﺒﻭﺍﺴﻁﺔ ﺍﻟﻌﺎﻟﻡ ﻫﻴﺭﺘﺯ‬
‫‪ H. Hertz‬ﻓﻲ ﻋﺎﻡ ‪ 1887‬ﻭﺍﻟﺘﻲ ﺃﺩﺕ ﺇﻟﻰ ﺇﺜﺒﺎﺕ ﺍﻟﻁﺒﻴﻌﺔ ﺍﻟﻤﻭﺠﺒﺔ ﻟﻠﻀﻭﺀ‪.‬‬

‫‪-٢-‬‬
‫‪PDF created with pdfFactory Pro trial version www.pdffactory.com‬‬
‫ﻜل ﻫﺫﻩ ﺍﻹﻨﺠﺎﺯﺍﺕ ﻓﻲ ﺍﻟﻤﺠﺎﻻﺕ ﺍﻟﻤﺨﺘﻠﻔﺔ ﻟﻠﻔﻴﺯﻴﺎﺀ ﻜﻭ‪‬ﻨﺕ ﻤﺎ ﻴ‪‬ﻌـﺭﻑ ﺍﻵﻥ‬
‫ﺒﺎﻟﻔﻴﺯﻴﺎﺀ ﺍﻟﺘﻘﻠﻴﺩﻴﺔ ‪ Classical physics‬ﻭﻤﻊ ﺒﺩﺍﻴﺔ ﺍﻟﻘﺭﻥ ﺍﻟﻌﺸﺭﻴﻥ‪ ،‬ﻭ‪‬ﺠﺩﺕ ﺒﻌﺽ‬
‫ﺍﻟﻨﺘﺎﺌﺞ ﺍﻟﺘﺠﺭﻴﺒﻴﺔ ﺍﻟﺠﺩﻴﺩﺓ ﻭﺍﻟﺘﻲ ﺍﺴﺘﻠﺯﻡ ﺘﻔﺴﻴﺭﻫﺎ ﻤﻔﺎﻫﻴﻡ ﻓﻴﺯﻴﺎﺌﻴﺔ ﺠﺩﻴﺩﺓ ﺘﺘﻨـﺎﻗﺽ‬
‫ﻤﻊ ﻤﺒﺎﺩﺉ ﺍﻟﻔﻴﺯﻴﺎﺀ ﺍﻟﺠﺩﻴﺩﺓ ﻭﻟﺩ ﻤﺎ ﻴﺴﻤﻰ ﺍﻵﻥ ﺒﺎﻟﻨﻅﺭﻴﺔ ﺍﻟﻜﻤﻴﺔ ‪quantum theory‬‬
‫ﻭﺴﻨﺤﺎﻭل ﻓﻲ ﻫﺫﺍ ﺍﻟﻔﺼل ﺃﻥ ﻨﺼﻑ ﺨﻠﻔﻴﺔ ﻫﺫﻩ ﺍﻷﺯﻤﺎﺕ ﻟﻨـﺼل ﻤـﻥ ﺨﻼﻟﻬـﺎ‬
‫ﻟﻤﻌﺭﻓﺔ ﺍﻟﻨﻅﺭﻴﺔ ﺍﻟﻜﻤﻴﺔ‪.‬‬
‫ﻭﻴﻤﻜﻨﻨﺎ ﺘﻠﺨﻴﺹ ﺍﻟﻤﻔﺎﻫﻴﻡ ﺍﻟﻔﻴﺯﻴﺎﺌﻴﺔ ﺍﻟﺠﺩﻴـﺩﺓ ﻓـﻲ‪ :‬ﺍﻟﺨـﻭﺍﺹ ﺍﻟﺠـﺴﻴﻤﻴﺔ‬
‫ﻟﻺﺸﻌﺎﻉ ‪ ،the particle properties of radiation‬ﺍﻟﺨﻭﺍﺹ ﺍﻟﻤﻭﺠﻴﺔ ﻟﻠﻤﺎﺩﺓ ‪the‬‬
‫‪ ،wave properties of matter‬ﻭﺘﻜﻤﻴﻡ ﺍﻟﻜﻤﻴﺎﺕ ﺍﻟﻔﻴﺯﻴﺎﺌﻴﺔ ‪the quantization of‬‬
‫‪ .physical quantities‬ﻭﺴﻨﻘﻭﻡ ﻓﻲ ﻫﺫﺍ ﺍﻟﻔﺼل ﺒﻤﻨﺎﻗﺸﺔ ﻫﺫﻩ ﺍﻟﻤﻔﺎﻫﻴﻡ‪.‬‬

‫‪ .1-1‬ﺇﺸﻌﺎﻉ ﺍﻟﺠﺴﻡ "ﺍﻷﺴﻭﺩ"‬


‫‪1-1 Blackbody Radiation‬‬
‫‪ -‬ﺍﻟﻔﻴﺯﻴﺎﺀ ﺍﻟﺘﻘﻠﻴﺩﻴﺔ ﻟﻡ ﺘﺘﻤﻜﻥ ﻤﻥ ﺸﺭﺡ ﺇﺸﻌﺎﻉ ﺍﻟﺠﺴﻡ ﺍﻷﺴﻭﺩ‬
‫‪- Blackbody Radiation could not be explained by classical physics‬‬
‫ﻤﻥ ﺃﻫﻡ ﺍﻟﻨﺘﺎﺌﺞ ﺍﻟﺘﺠﺭﻴﺒﻴﺔ ﺍﻟﺘﻲ ﺃﺤﺩﺜﺕ ﺜﻭﺭﺓ ﻓﻲ ﺍﻟﻤﻔﺎﻫﻴﻡ ﺍﻟﻔﻴﺯﻴﺎﺌﻴﺔ ﺍﻟﺘﻘﻠﻴﺩﻴﺔ‬
‫ﺘﻠﻙ ﺍﻟﻤﺘﻌﻠﻘﺔ ﺒﺎﻹﺸﻌﺎﻉ ﺍﻟﺼﺎﺩﺭ ﻤﻥ ﺍﻷﺠﺴﺎﻡ ﻋﻨﺩ ﺘﺴﺨﻴﻨﻬﺎ‪ .‬ﻓﻤـﻥ ﺍﻟﻤﻌﻠـﻭﻡ ﻋﻨـﺩ‬
‫ﺘﺴﺨﻴﻥ ﺠﺴﻡ ﻤﺎ‪ ،‬ﻨﺠﺩ ﺃﻥ ﻟﻭﻨﻪ ﻴﺘﻐﻴﺭ ﻤﻊ ﺯﻴﺎﺩﺓ ﺩﺭﺠﺔ ﺍﻟﺤﺭﺍﺭﺓ ﺤﻴﺙ ﻴﺒﺩﺃ ﺒﺎﻷﺤﻤﺭ‬
‫ﺜﻡ ﺍﻷﺒﻴﺽ ﺜﻡ ﺍﻷﺯﺭﻕ‪ .‬ﻭﺒﺩﻻﻟﺔ ﺍﻟﺘﺭﺩﺩ‪ ،‬ﻨﻘﻭل ﺃﻥ ﺍﻹﺸﻌﺎﻉ ﺍﻟﻤﻨﺒﻌﺙ ﻤﻥ ﻫﺫﺍ ﺍﻟﺠﺴﻡ‬
‫ﻴﺒﺩﺃ ﺒﺘﺭﺩﺩﺍﺕ ﻤﻨﺨﻔﻀﺔ‪ ،‬ﻭﻋﻨﺩ ﺍﺭﺘﻔﺎﻉ ﺩﺭﺠﺔ ﺍﻟﺤﺭﺍﺭﺓ‪ ،‬ﺘﺯﺩﺍﺩ ﺍﻟﺘﺭﺩﺩﺍﺕ‪ ،‬ﺤﻴﺙ ﺃﻥ‬
‫ﺍﻟﻠﻭﻥ ﺍﻷﺤﻤﺭ ﺫﻭ ﺘﺭﺩﺩ ﻗﻠﻴل ﻓﻲ ﻤﻨﻁﻘﺔ ﻁﻴﻑ ﺍﻹﺸﻌﺎﻉ ﻭﺫﻟـﻙ ﻤﻘﺎﺭﻨـﺔ ﺒـﺎﻟﻠﻭﻥ‬
‫ﺍﻷﺯﺭﻕ‪ .‬ﺇﻥ ﻁﻴﻑ ﺍﻟﺘﺭﺩﺩ ﻟﻺﺸﻌﺎﻉ ﺍﻟﻤﻨﺒﻌﺙ ﻤﻥ ﺠﺴﻡ ﻤﺎ ﻴﻌﺘﻤﺩ ﻋﻠﻰ ﻁﺒﻴﻌﺔ ﺍﻟﺠﺴﻡ‬
‫ﻨﻔﺴﻪ‪ ،‬ﻭﻟﻜﻥ ﺍﻟﺠﺴﻡ ﺍﻟﻤﺜﺎﻟﻲ ‪ ،deal body‬ﻭﺍﻟﺫﻱ ﻴﻤﺘﺹ ﺃﻭ ﻴﺒﻌﺙ ﻜل ﺍﻟﺘـﺭﺩﺩﺍﺕ‬

‫‪-٣-‬‬
‫‪PDF created with pdfFactory Pro trial version www.pdffactory.com‬‬
‫ﻴ‪‬ﺴﻤﻰ ﺒﺎﻟﺠﺴﻡ ﺍﻷﺴﻭﺩ ﻭﻴﻌﺘﺒﺭ ﺤﺎﻟﺔ ﻤﺜﺎﻟﻴﺔ ﻷﻱ ﻤﺎﺩﺓ ﺘﹸـﺼﺩﺭ ﺇﺸـﻌﺎﻉ‪ .‬ﺍﻹﺸـﻌﺎﻉ‬
‫ﺍﻟﻤﻨﺒﻌﺙ ﻤﻥ "ﺠﺴﻡ ﺃﺴﻭﺩ" ﻴﺴﻤﻰ ﺇﺸﻌﺎﻉ ﺍﻟﺠﺴﻡ ﺍﻷﺴﻭﺩ‪.‬‬
‫‪an ideal body, which absorbs and emits all frequencies, is called‬‬
‫‪a blackbody and serves as an idealization for any radiating material,‬‬
‫‪the radiation emitted by a blackbody is called blackbody radiation.‬‬
‫ﺸﻜل ‪ 1-1‬ﻴﻭﻀﺢ ﺘﻐﻴﺭ ﺸﺩﺓ ﺍﻹﺸﻌﺎﻉ ﺍﻟﺼﺎﺩﺭ ﻤﻥ ﺠﺴﻡ ﺃﺴﻭﺩ ﻤﻊ ﺍﻟﺘـﺭﺩﺩ‬
‫ﻭﺫﻟﻙ ﻋﻨﺩ ﺩﺭﺠﺎﺕ ﺤﺭﺍﺭﺓ ﻤﺨﺘﻠﻔﺔ‪ .‬ﻭﻗﺩ ﺤﺎﻭل ﺍﻟﻌﺩﻴﺩ ﻤﻥ ﺍﻟﻔﻴﺯﻴـﺎﺌﻴﻴﻥ ﺍﺴـﺘﻨﺘﺎﺝ‬
‫ﻤﻌﺎﺩﻟﺔ ﺭﻴﺎﻀﻴﺔ ﺘﺸﺭﺡ ﺍﻟﻨﺘﺎﺌﺞ ﺍﻟﺘﺠﺭﻴﺒﻴﺔ )ﻓﻲ ﺸﻜل ‪ (1-1‬ﻭﻟﻜﻥ ﺒﺩﻭﻥ ﺘﻭﺍﻓﻕ ﻜﺎﻤل‬
‫ﻭﺃﻭﻟﻰ ﺍﻟﻤﺤﺎﻭﻻﺕ ﻟﻭﺼﻑ ﻫﺫﻩ ﺍﻟﻨﺘﺎﺌﺞ ﻗﺎﻡ ﺒﻬﺎ ﻜل ﻤﻥ ﺍﻟﻌﺎﻟﻤﻴﻥ ﺭﺍﻟـﻲ ﻭﺠﻴﻨـﺯ‬
‫‪ Rayleigh and Jeans‬ﻭﺍﻟﺘﻲ ﺍﺸﺘﻘﺕ ﺒﻨﺎﺀﺍﹰ ﻋﻠﻰ ﻗﻭﺍﻨﻴﻥ ﺍﻟﻘﺭﻥ ﺍﻟﺘﺎﺴﻊ ﻋﺸﺭ ﻭﻴﻤﻜﻥ‬
‫ﻜﺘﺎﺒﺔ ﻫﺫﻩ ﺍﻟﻌﻼﻗﺔ ﺒﺎﻟﺼﻴﻐﺔ‪:‬‬
‫‪8D K B T‬‬
‫= )‪u (ννT‬‬ ‫‪3‬‬
‫‪ν2‬‬ ‫)‪(1-1‬‬
‫‪C‬‬

‫ﻜﺜﺎﻓﺔ ﺍﻟﻁﺎﻗﺔ ‪ energy density‬ﻭﻭﺤﺩﺘﻬﺎ ﺠـﻭل ﻟﻜـل ﻤﺘـﺭ‬ ‫)‪u (ν , T‬‬ ‫ﺤﻴﺙ‬
‫ﻤﻜﻌﺏ )‪ .(J/m3‬ﻓﻲ ﺍﻟﻤﻌﺎﺩﻟﺔ )‪T ،(1-1‬ﺩﺭﺠﺔ ﺍﻟﺤﺭﺍﺭﺓ ﺒﺎﻟﻜﻴﻠـﭭﻥ‪ c ،‬ﺴﺭﻋﺔ ﺍﻟﻀﻭﺀ‪،‬‬
‫‪ KB‬ﺜﺎﺒﺕ ﺒﻭﻟﺘﺯﻤﺎﻥ‪ .‬ﺍﻟﺨﻁ ﺍﻟﻤﺘﻘﻁﻊ ﻓﻲ ﺸﻜل ‪ 1-1‬ﻴﺒﻴﻥ ﺍﻟﻌﻼﻗﺔ ﺤـﺴﺏ ﻤﻌﺎﺩﻟـﺔ‬
‫ﺭﺍﻟﻲ‪ -‬ﺠﻴﻨﺯ‪ .‬ﻻﺤﻅ ﺍﻟﺘﻭﺍﻓﻕ ﺒﻴﻥ ﻫﺫﻩ ﺍﻟﻌﻼﻗﺔ ﻭﺍﻟﻨﺘﺎﺌﺞ ﺍﻟﺘﺠﺭﻴﺒﻴﺔ ﻋﻨﺩ ﺍﻟﺘـﺭﺩﺩﺍﺕ‬
‫ﺍﻟﻤﻨﺨﻔﻀﺔ‪ .‬ﻋﻨﺩ ﺍﻟﺘﺭﺩﺩﺍﺕ ﺍﻟﻌﺎﻟﻴﺔ‪ ،‬ﻭﺤﺴﺏ ﻤﻌﺎﺩﻟﺔ ﺭﺍﻟـﻲ‪-‬ﺠﻴﻨـﺯ‪ ،‬ﻓـﺈﻥ ﻜﺜﺎﻓـﺔ‬
‫ﻭﺘﺼل ﺇﻟﻰ ﻤﺎﻻ ﻨﻬﺎﻴﺔ ﻭﺫﻟﻙ ﻋﻨﺩﻤﺎ ﺘﺼل ﺍﻟﺘـﺭﺩﺩﺍﺕ‬ ‫‪ν2‬‬ ‫ﺍﻹﺸﻌﺎﻉ ﺘﺯﺩﺍﺩ ﻁﺒﻘﺎﹰ ﻟـ‬
‫ﺇﻟﻰ ﻤﺎﻻ ﻨﻬﺎﻴﺔ ﻭﻫﺫﺍ ﻴﺤﺩﺙ ﻓﻲ ﻤﻨﻁﻘﺔ ﺍﻷﺸﻌﺔ ﻓﻭﻕ ﺍﻟﺒﻨﻔﺴﺠﻴﺔ ‪ ultraviolet‬ﻭﻫـﺫﺍ‬
‫ﻤﺎ ﻴﻌﺭﻑ ﺒﺎﻻﻨﻬﻴﺎﺭ ﻓﻭﻕ ﺒﻨﻔﺴﺠﻲ ‪ .ultraviolet catastrophe‬ﻤﻥ ﺍﻟﻤﻌﺎﺩﻟﺔ )‪(1-1‬‬
‫ﻭﻟﻜل ﺍﻟﺘﺭﺩﺩﺍﺕ ﻤﻥ ﺼﻔﺭ ﺇﻟﻰ ﻤﺎﻻ ﻨﻬﺎﻴﺔ‪.‬‬
‫∞‬ ‫∞‬
‫‪8 Dk B T 2‬‬
‫∫ = ‪∫ u (ν1T) dν‬‬
‫‪°‬‬ ‫‪°‬‬
‫‪C3‬‬
‫∞ → ‪ν dν‬‬

‫‪-٤-‬‬
‫‪PDF created with pdfFactory Pro trial version www.pdffactory.com‬‬
‫ﻭﻫﺫﻩ ﺍﻟﻨﺘﻴﺠﺔ ﺘﺘﻨﺎﻗﺽ ﻤﻊ ﺍﻟﻨﺘﺎﺌﺞ ﺍﻟﺘﺠﺭﻴﺒﻴﺔ ﺤﻴﺙ ﺃﻥ ﺸﺩﺓ ﺍﻹﺸﻌﺎﻉ ﺘﺯﺩﺍﺩ ﻤﻊ‬
‫ﺜﻡ ﺘﻘل ﺇﻟﻰ‬ ν max ‫ﺯﻴﺎﺩﺓ ﺍﻟﺘﺭﺩﺩ ﻟﺘﺼل ﺇﻟﻰ ﺃﻗﺼﻰ ﻗﻴﻤﺔ ﻋﻨﺩ ﺘﺭﺩﺩ ﻤﻌﻴﻥ ﻨﺭﻤﺯ ﻟﻪ ﺒـ‬
‫ ﻭﻤﻥ ﺍﻟﺠـﺩﻴﺭ ﺒﺎﻟـﺫﻜﺭ‬.‫ﺍﻟﺼﻔﺭﺓ ﻫﺫﺍ ﻴﻌﻨﻲ ﺃﻥ ﻗﻴﻤﺔ ﺍﻟﺘﻜﺎﻤل ﻻ ﺘﺴﺎﻭﻱ ﻤﺎﻻ ﻨﻬﺎﻴﺔ‬
.‫ﺘﺘﻐﻴﺭ ﻤﻊ ﺘﻐﻴﺭ ﺩﺭﺠﺔ ﺍﻟﺤﺭﺍﺭﺓ ﻜﻤﺎ ﻫﻭ ﻤﻭﻀﺢ ﺒﺎﻟﺸﻜل‬ ν max ‫ﻜﺫﻟﻙ ﻤﻼﺤﻅﺔ ﺃﻥ‬

FIGURE 1.1
Spectral distribution of the intensity of blackbody radiation as a
function of frequency for several temperatures. The intensity is given
in arbitrary units. The dashed line is the prediction of classical
physics. As the temperature increases. The maximum shifts to higher
frequencies and the total radiated energy (the area under each curve)
increases significantly. Note that the horizontal axis is labeled by
14 -1
ν /10 s . This notation means that the dimensionless numbers on that
axis re frequencies divided by 1014 s-1. We shall use this notation to
label columns in tables and axes in figures because of its unambiguous
nature and algebraic convenience.

-٥-
PDF created with pdfFactory Pro trial version www.pdffactory.com
‫ﻜﻴﻑ ﺘﻡ ﺤل ﻫﺫﺍ ﺍﻹﺸﻜﺎل ﺒﻴﻥ ﺍﻟﻨﻅﺭﻴﺔ ﻭﺍﻟﺘﺠﺎﺭﺏ ﺍﻟﻌﻤﻠﻴﺔ؟‬
‫‪ 2-1‬ﺘﻭﺯﻴﻊ ﺒﻼﻨﻙ ﻭﺘﻜﻤﻴﻡ ﺍﻟﻁﺎﻗﺔ‬
‫‪1-2 The Planck Distribution and the Quantum of Energy‬‬
‫ﺇﻥ ﺃﻭل ﻤﻥ ﻗﺩﻡ ﺘﻔﺴﻴﺭ ﺼﺤﻴﺢ ﻹﺸﻌﺎﻉ ﺍﻟﺠﺴﻡ ﺍﻷﺴﻭﺩ ﻫﻭ ﺍﻟﻌﺎﻟﻡ ﺍﻷﻟﻤـﺎﻨﻲ‬
‫ﻤﺎﻜﺱ ﺒﻼﻨﻙ ‪ Max Planck‬ﻓﻲ ﻋﺎﻡ ‪ .1900‬ﻭﻓﻲ ﻨﻅﺭﻴﺘﻪ‪ ،‬ﺍﻓﺘـﺭﺽ ﺒﻼﻨـﻙ ﺃﻥ‬
‫ﺍﻹﺸﻌﺎﻉ ﺍﻟﻤﻨﺒﻌﺙ ﻤﻥ ﺍﻟﺠﺴﻡ ﺍﻷﺴﻭﺩ ﻤﻥ ﺍﻫﺘﺯﺍﺯ ﺍﻹﻟﻜﺘﺭﻭﻨـﺎﺕ ﺍﻟﻤﻜﻭﻨـﺔ ﻟﻤـﺎﺩﺓ‬
‫ﺍﻟﺠﺴﻡ‪ .‬ﻭﻟﻜﻭﻥ ﻫﺫﻩ ﺍﻻﻫﺘﺯﺍﺯﺍﺕ ﺫﺍﺕ ﺘﺭﺩﺩﺍﺕ ﻋﺎﻟﻴﺔ ﻓﺈﻨﻨﺎ ﻨﺠﺩ ﻓﻲ ﻁﻴﻑ ﺍﻹﺸﻌﺎﻉ‬
‫ﺍﻟﻤﻨﺒﻌﺙ ﺘﺭﺩﺩﺍﺕ ﻓﻲ ﻤﻨﻁﻘﺔ ﺍﻟﻀﻭﺀ ﺍﻟﻤﺭﺌﻲ ﻭﺍﻷﺸﻌﺔ ﺘﺤـﺕ ﺍﻟﺤﻤـﺭﺍﺀ ﻭﻓـﻭﻕ‬
‫ﺍﻟﺒﻨﻔﺴﺠﻴﺔ ﺒﻴﻨﻤﺎ ﻻ ﻨﺠﺩ ﺃﻱ ﻤﻥ ﺘﺭﺩﺩﺍﺕ ﺍﻟﺭﺍﺩﻴﻭ ﻓﻲ ﻫﺫﺍ ﺍﻟﻁﻴﻑ‪ .‬ﻁﺒﻘـﺎﹰ ﻟﻨﻅﺭﻴـﺔ‬
‫ﺭﺍﻟﻲ‪-‬ﺠﻴﻨﺯ‪ ،‬ﻓﺈﻨﻪ ﻤﻔﻬﻭﻡ ﻅﻤﻨﺎﹰ ﺃﻥ ﻁﺎﻗﺔ ﺍﻹﻟﻜﺘﺭﻭﻨﺎﺕ ﺍﻟﻤﻬﺘﺯﺓ‪ ،‬ﻭﺍﻟﺘﻲ ﻫﻲ ﺴـﺒﺏ‬
‫ﺍﻨﺒﻌﺎﺙ ﺍﻹﺸﻌﺎﻉ ﻤﻥ ﺍﻟﻤﺎﺩﺓ‪ -‬ﻤﺴﻤﻭﺡ ﻟﻬﺎ ﺃﻥ ﺘﺄﺨﺫ ﺃﻱ ﻗﻴﻤﺔ ﻤﻥ ﺍﻟﻁﺎﻗـﺔ‪ .‬ﻭﻫـﺫﻩ‬
‫ﺍﻟﻔﺭﻀﻴﺔ ﻫﻲ ﺇﺤﺩﻯ ﺍﻷﺴﺎﺴﻴﺎﺕ ﺍﻟﻔﺭﻀﻴﺔ ﻓﻲ ﺍﻟﻔﻴﺯﻴﺎﺀ ﺍﻟﺘﻘﻠﻴﺩﻴـﺔ‪ .‬ﻓـﻲ ﺍﻟﻔﻴﺯﻴـﺎﺀ‬
‫ﺍﻟﺘﻘﻠﻴﺩﻴﺔ‪ ،‬ﺍﻟﻜﻤﻴﺎﺕ ﺍﻟﻔﻴﺯﻴﺎﺌﻴﺔ ﺍﻟﻤﺘﻐﻴﺭﺓ ﻭﺍﻟﺘﻲ ﺘﹸﻤﺜـل ﻤـﺸﺎﻫﺩﺍﺕ )ﻤﺜـل ﺍﻟﻤﻭﻗـﻊ‬
‫‪ ،Position‬ﻜﻤﻴﺔ ﺍﻟﺤﺭﻜﺔ ‪ Momentum‬ﻭﺍﻟﻁﺎﻗﺔ ‪ (energy‬ﻴﻤﻜﻥ ﺘﻤﻠﻙ ﻗﻴﻡ ﻤﺘﺼﻠﺔ‪.‬‬
‫‪In classical physics, the variables that represent observables‬‬
‫‪(such as position, momentum and energy) can take on a continuum of‬‬
‫‪values.‬‬
‫ﻭﻟﻘﺩ ﺃﺩﺭﻙ ﺍﻟﻌﺎﻟﻡ ﺒﻼﻨﻙ – ﺒﻌﻤﻕ ﺘﻔﻜﻴﺭﻩ – ﺒﻀﺭﻭﺭﺓ ﺇﺤﺩﺍﺙ ﺘﻐﻴﻴﺭ ﺠﺫﺭﻱ‬
‫ﻭﺠﻭﻫﺭﻱ ﻓﻲ ﻫﺫﺍ ﺍﻟﻤﻔﻬﻭﻡ ﺍﻟﻔﻴﺯﻴﺎﺌﻲ ﻓﻜﺎﻨﺕ ﻓﺭﻀﻴﺘﻪ ﺍﻻﻨﻘﻼﺒﻴـﺔ ﻓـﻲ ﺍﻟﻔﻴﺯﻴـﺎﺀ‬
‫ﺍﻟﺤﺩﻴﺜﺔ‪ :‬ﻁﺎﻗﺔ ﺍﻹﻟﻜﺘﺭﻭﻨﺎﺕ ﺍﻟﻤﻬﺘﺯﺓ ﻤﻜﻤﻤﺔ ﻭﻗﻴﻤﻬﺎ ﻏﻴﺭ ﻤﺘﺼﻠﺔ ﻭﺘﺘﻨﺎﺴﺏ ﺒـﺭﻗﻡ‬
‫‪E = nhν‬‬ ‫ﻜﻤﻲ ﺼﺤﻴﺢ ﻤﻊ ﺍﻟﺘﺭﺩﺩ ﻭﺫﻟﻙ ﻤﻥ ﺨﻼل ﺍﻟﻤﻌﺎﺩﻟﺔ‬

‫‪-٦-‬‬
‫‪PDF created with pdfFactory Pro trial version www.pdffactory.com‬‬
‫‪Energies of the oscillators were discrete and had to be‬‬
‫‪proportional to an integral multiple of the frequently in an equation of‬‬
‫‪the form E = nhv‬‬
‫ﺤﻴﺙ ‪ E‬ﻫﻲ ﻁﺎﻗﺔ ﺍﻟﻤﺘﺫﺒﺫﺏ‪ n ،‬ﻫﻭ ﺭﻗﻡ ﺼﺤﻴﺢ‪ h ،‬ﺜﺎﺒﺕ ﺍﻟﺘﻨﺎﺴﺏ ﻭﻴ‪‬ﻌﺭﻑ‬
‫ﺒﺜﺎﺒﺕ ﺒﻼﻨﻙ‪ ،‬ﻭ‪ v‬ﻫﻭ ﺘﺭﺩﺩ ﺍﻟﻤﺘﺫﺒﺫﺏ‪.‬‬
‫ﻭﺒﻨﺎﺀﺍﹰ ﻋﻠﻰ ﻤﺒﺩﺃ ﺘﻜﻤﻴﻡ ﺍﻟﻁﺎﻗﺔ ﻭﻤﻔﺎﻫﻴﻡ ﺩﻴﻨﺎﻤﻴﻜﺎ ﺤﺭﺍﺭﻴﺔ ﺇﺤﺼﺎﺌﻴﺔ‪ ،‬ﺘﻤﻜـﻥ‬
‫ﺒﻼﻨﻙ ﻤﻥ ﺍﺴﺘﻨﺘﺎﺝ ﺍﻟﻌﻼﻗﺔ ﺍﻟﺭﻴﺎﻀﻴﺔ ﺍﻟﺘﺎﻟﻴﺔ‪:‬‬
‫‪8 Dh‬‬ ‫‪ν3‬‬
‫‪u (ν1 T) = 3 hv/k T‬‬ ‫)‪(1-2‬‬
‫‪c e B −1‬‬

‫ﻭﻫﺫﻩ ﺍﻟﻌﻼﻗﺔ ﺘﺘﻔﻕ ﺘﻤﺎﻤﺎﹰ ﻤﻊ ﺍﻟﻨﺘﺎﺌﺞ ﺍﻟﺘﺠﺭﻴﺒﻴﺔ ﻋﻨﺩ ﻜل ﺍﻟﺘﺭﺩﺩﺍﺕ ﻭﺩﺭﺠﺎﺕ‬


‫ﺍﻟﺤﺭﺍﺭﺓ ﺍﻟﻤﻌﺎﺩﻟﺔ )‪ (1-2‬ﺘﹸﻌﺭﻑ ﺒﺘﻭﺯﻴﻊ ﺒﻼﻨﻙ ﻹﺸﻌﺎﻉ ﺍﻟﺠﺴﻡ ﺍﻷﺴﻭﺩ‬
‫‪Planck distribution lance for black body radiation.‬‬
‫)‪ (i‬ﻋﻨﺩ ﺍﻟﺘﺭﺩﺩﺍﺕ ﺍﻟﻤﻨﺨﻔﻀﺔ‪ ،‬ﺘﺅﻭل ﻤﻌﺎﺩﻟﺔ ﺒﻼﻨﻙ ﺇﻟﻰ ﻤﻌﺎﺩﻟﺔ ﺭﺍﻟﻲ‪-‬ﺠﻴﻨﺯ ﺒﺎﻟﻨﻅﺭ‬
‫‪hv‬‬
‫‪〈〈 1‬‬ ‫ﻓﻌﻨﺩﻤﺎ ﺘﻜﻭﻥ ‪ v‬ﺼﻐﻴﺭﺓ ﻓﺈﻥ‬ ‫‪hv‬‬
‫ﺇﻟﻰ ﺍﻟﻨﺴﺒﺔ‬
‫‪k BT‬‬ ‫‪k BT‬‬

‫‪e v =1+ v +‬‬


‫‪v2‬‬
‫‪+ ...‬‬ ‫ﻤﻥ ﻤﻜﻨﻭﻥ ﺘﺎﻴﻠﻭﺭ ﻟﻠﺩﺍﻟﺔ ﺍﻷﺴﻴﺔ‬
‫‪2i‬‬

‫ﺇﺫﺍ ﻜﺎﻨﺕ ‪ v‬ﺼﻐﻴﺭﺓ‪ ،‬ﻴﻤﻜﻨﻨﺎ ﺇﻫﻤﺎل ﺍﻟﺤﺩﻭﺩ ﺫﺍﺕ ﺍﻷﺴـﺱ ﺍﻟﻌﻠﻴـﺎ ﻭﻋﻠﻴـﻪ‬
‫ﻓﻲ ﻫﺫﻩ ﺍﻟﺤﺎﻟﺔ ﻴﻤﻜﻨﻨﺎ ﻜﺘﺎﺒﺔ ﻤﻌﺎﺩﻟﺔ ﺒﻼﻨﻙ‬ ‫‪v 〈〈 1‬‬ ‫ﻭﺫﻟﻙ ﺇﺫﺍ ﻜﺎﻨﺕ‬ ‫‪e v ≈1+ v‬‬

‫‪8Dh‬‬ ‫‪v3‬‬ ‫‪8Dh v 3‬‬


‫‪u (v1 T) = 3‬‬ ‫=‬
‫‪c 1 + hν − 1 c 3 hv‬‬
‫‪vT‬‬ ‫‪k BT‬‬

‫‪8D‬‬
‫= )‪u (v1 T‬‬ ‫‪K T v3‬‬ ‫)‪(1-3‬‬
‫‪c3 B‬‬

‫ﻭﻫﺫﻩ ﻫﻲ ﻤﻌﺎﺩﻟﺔ ﺭﺍﻟﻲ‪ -‬ﺠﻴﻨﺯ‪.‬‬


‫)‪ (ii‬ﺍﻟﻁﺎﻗﺔ ﺍﻟﻜﻠﻴﺔ‬

‫‪-٧-‬‬
‫‪PDF created with pdfFactory Pro trial version www.pdffactory.com‬‬
‫∞‬ ‫∞‬
‫‪8Dh‬‬ ‫‪v3‬‬
‫‪u (t) = ∫ u (v1T) dv = 3 ∫ dv hv/k T‬‬
‫‪°‬‬ ‫‪c ° e B −1‬‬

‫= ‪dv‬‬
‫‪h‬‬
‫‪dv‬‬ ‫= ‪ ، v‬ﻫـﺫﺍ ﻴﻌﻨـﻲ ﺃﻥ‬ ‫‪hν‬‬
‫ﻭﻹﺠﺭﺍﺀ ﻫﺫﺍ ﺍﻟﺘﻜﺎﻤل ﻨﻌﻭﺽ ﻋـﻥ‬
‫‪k BT‬‬ ‫‪k BT‬‬

‫= ‪ . dv‬ﺤﺩﻭﺩ ﺍﻟﺘﻜﺎﻤل ﻟﻡ ﺘﺘﻐﻴﺭ‪ ،‬ﺒـﺎﻟﺘﻌﻭﻴﺽ‬ ‫ﻭﻴﻤﻜﻨﻨﺎ ﻜﺘﺎﺒﺔ ‪ dv‬ﺒﺩﻻﻟﺔ ‪ dv‬ﻭ‬


‫‪k BT‬‬
‫‪dv‬‬
‫‪h‬‬
‫ﻭﻜﺫﻟﻙ ‪ dv‬ﺒﺩﻻﻟﺔ ‪ dv‬ﻨﺘﺤﺼل ﻋﻠﻰ‬ ‫ﻋﻥ ‪ v‬ﺒـ‬
‫‪k BT‬‬
‫‪v‬‬
‫‪h‬‬
‫∞‬
‫‪8Dh kT‬‬ ‫‪v 3 kT‬‬
‫‪u (v, T) = 3 ∫ ( ) 3 v ( ) dv‬‬
‫‪c ° h e −1 h‬‬
‫∞‬ ‫∞‬
‫‪8Dh kT 4 v 3‬‬ ‫‪8Dh k B T 4 v 3‬‬
‫‪c 3 ∫° h e v − 1‬‬ ‫‪∫° e v −1 dv‬‬
‫=‬ ‫(‬ ‫)‬ ‫‪dv‬‬ ‫=‬ ‫(‬ ‫)‬
‫‪c3‬‬ ‫‪D‬‬

‫ﺍﻟﺘﻜﺎﻤل ‪ ∫ v dv‬ﻗﻴﻤﺘﻪ ﺘﺴﺎﻭﻱ‬


‫∞‬
‫‪D4‬‬ ‫‪3‬‬

‫‪15‬‬ ‫‪e −1‬‬


‫‪°‬‬
‫‪v‬‬

‫‪8Dh k B T 4 D 4 8D 5 k B 4 4‬‬
‫= )‪∴u (T‬‬ ‫(‬ ‫)‬ ‫=‬ ‫‪( ) T‬‬
‫‪c3‬‬ ‫‪h‬‬ ‫‪15 15 c 3 h‬‬
‫‪= aT 4 .............‬‬ ‫)‪(1 − 4‬‬

‫=‪a‬‬
‫‪8D 5 k B 4‬‬
‫) (‬ ‫ﺤﻴﺙ‪:‬‬
‫‪15c 3 h‬‬

‫ﺍﻟﻤﻌﺎﺩﻟﺔ )‪ (1-4‬ﺘﻤﺜل ﻤﺎ ﻴﻌـﺭﻑ ﺒﻘـﺎﻨﻭﻥ ﺴـﺘﻴﻔﺎﻥ‪ -‬ﺒﻭﻟﺘﺯﻤـﺎﻥ ‪Stefan-‬‬


‫‪ Boltzman‬ﻟﻠﻁﺎﻗﺔ ﺍﻟﻜﻠﻴﺔ ﻟﻜل ﻭﺤﺩﺓ ﺍﻟﺤﺠﻭﻡ‪.‬‬

‫)‪ (iii‬ﻤﻌﺎﺩﻟﺔ ﺒﻼﻨﻙ ﺘﻤﻜﻨﻨﺎ ﻤﻥ ﺇ – ﺝ ﻗﺎﻨﻭﻥ ﻭﺍﻴﻥ ‪ Wien‬ﻭﺍﻟﺫﻱ ﻴﻨﺹ ﻋﻠﻰ ﺃﻨﻪ ﺇﺫﺍ‬
‫ﺘﻤﺜل ﺍﻟﻁﻭل ﺍﻟﻤﻭﺠﻲ ﺍﻟﻤﻘﺎﺒل ﻷﻗﺼﻰ ﻗﻴﻤﺔ ﻟﻜﺜﺎﻓﺔ ﺍﻟﻁﺎﻗﺔ ‪ u‬ﻓﺈﻥ ﺤﺎﺼل‬ ‫‪λ max‬‬ ‫ﻜﺎﻨﺕ‬
‫ﻓﻲ ﺩﺭﺠﺔ ﺍﻟﺤﺭﺍﺭﺓ ‪ T‬ﻴﻌﻁﻴﻨﺎ ﻗﻴﻤﺔ ﺜﺎﺒﺘﺔ‪ ،‬ﺃﻱ‪:‬‬ ‫‪λ max‬‬ ‫ﻀﺭﺏ‬

‫‪λ max T = 2.90 × 10 −3 m.K‬‬ ‫)‪..(1-5‬‬


‫ﻭﻟﻠﺤﺼﻭل ﻋﻠﻰ ﻤﻌﺎﺩﻟﺔ )‪ (1-5‬ﻤﻥ ﻤﻌﺎﺩﻟﺔ ﺒﻼﻨﻙ‪.‬‬
‫ﺒﺎﻟﻨﺴﺒﺔ ﻟـ‬ ‫) ‪u (ν‬‬ ‫ﻴﻠﺯﻡ ﺃﻥ ﺘﻔﺎﻀل ﺍﻟﺩﺍﻟﺔ‬ ‫) ‪u (ν‬‬ ‫ﻟﻜﻲ ﻨﻭﺠﺩ ﺍﻟﻘﻴﻤﺔ ﺍﻟﻘﺼﻭﻯ ﻟـ‬
‫) ‪dv (ν‬‬
‫‪=0‬‬ ‫‪ v‬ﻭﺘﺴﺎﻭﻱ ﻫﺫﺍ ﺍﻟﺘﻔﺎﻀل ﺒﺎﻟﺼﻔﺭ‪ ،‬ﺃﻱ‬
‫‪dv‬‬

‫‪-٨-‬‬
‫‪PDF created with pdfFactory Pro trial version www.pdffactory.com‬‬
‫‪or‬‬
‫‪d  8Dh‬‬ ‫‪v3‬‬ ‫‪‬‬
‫‪‬‬ ‫‪ =0‬‬
‫‪− 1‬‬
‫‪3‬‬ ‫‪hv/k BT‬‬
‫‪dv  c e‬‬

‫‪vν‬‬
‫‪k BT‬‬
‫= ‪dv , v‬‬
‫‪hv‬‬
‫ﻭﻤﺭﺓ ﺃﺨﺭﻯ ﻨﻌﻭﺽ ﻋﻥ‬
‫‪h‬‬ ‫‪k BT‬‬

‫ﻭﺒﻘﺴﻤﺔ ﻁﺭﻓﻲ ﺍﻟﻤﻌﺎﺩﻟﺔ ﻋﻠﻰ ﻜل ﺍﻟﺜﻭﺍﺒﺕ ﻨﺘﺤﺼل ﻋﻠﻰ‪.‬‬


‫‪3 (e v − 1) − ve v = o‬‬
‫‪3 e v − 3 = ve v‬‬

‫ﺒﺎﻟﻘﺴﻤﺔ ﻋﻠﻰ ‪ev‬‬


‫‪3 − 3 e −v = v‬‬
‫‪or‬‬
‫‪3 - v = 3e - v‬‬

‫ﻭﻴﻤﻜﻥ ﺤل ﻫﺫﻩ ﺍﻟﻤﻌﺎﺩﻟﺔ ﻤﻥ ﺨﻼل ﻨﻘﻁﺔ ﺍﻟﺘﻘﺎﻁﻊ ﺒﻴﻥ ﺍﻟﺩﺍﻟﺘﻴﻥ‬


‫)‪ (3-v‬ﻭ )‪ (3e-v‬ﻜﻤﺎ ﻫﻭ ﻤﻭﻀﺢ ﺒﺎﻟﺭﺴﻡ ﻓﻲ ﺸﻜل )‪(١-٤‬‬

‫ﺸﻜل ‪ :1-2‬ﺤل ﺍﻟﻤﻌﺎﺩﻟﺔ ‪ 3 – x = 3 e-x‬ﺒﺎﻟﺭﺴﻡ‬

‫‪-٩-‬‬
‫‪PDF created with pdfFactory Pro trial version www.pdffactory.com‬‬
‫ﻤﻥ ﺍﻟﺸﻜل‪ ،‬ﻨﺠﺩ ﺃﻥ‬
‫‪v = 2.82‬‬
‫‪or‬‬
‫‪hv mar‬‬
‫‪= 2.82‬‬
‫‪k BT‬‬
‫‪or‬‬
‫‪k BT‬‬
‫= ‪v max‬‬ ‫)‪(2.82‬‬
‫‪h‬‬
‫‪c‬‬ ‫‪h T‬‬
‫∴‬ ‫)‪= b (2.82‬‬
‫‪λ max‬‬ ‫‪h‬‬
‫‪hc‬‬ ‫)‪(6.63 × 10 −34 J.s) (3.0 × 10 8 m/s‬‬
‫= ‪or λ max T‬‬ ‫=‬ ‫‪− 23‬‬ ‫‪−1‬‬
‫‪= 2.9 × 10 −3 m.K‬‬
‫‪2.82K B‬‬ ‫) ‪(2.82) (1.38× 10 J.v‬‬

‫ﻭﻤﻥ ﺍﻟﺠﺩﻴﺭ ﺒﺎﻟﺫﻜﺭ‪ ،‬ﺃﻥ ﻨﻅﺭﻴﺔ ﺇﺸﻌﺎﻉ ﺍﻟﺠﺴﻡ ﺍﻷﺴﻭﺩ ﺘﺴﺘﺨﺩﻡ ﻓﻲ ﻋﻠﻡ ﺍﻟﻔﻠﻙ‬
‫ﻭﺫﻟﻙ ﻟﺘﺤﺩﻴﺩ ﺩﺭﺠﺔ ﺤﺭﺍﺭﺓ ﺍﻟﻨﺠﻭﻡ‪ .‬ﺒﺎﻟﻨﻅﺭ ﻟﺸﻜل ﻭﺍﻟﺫﻱ ﻴﻭﻀﺢ ﻁﻴﻑ ﺇﺸـﻌﺎﻉ‬
‫)ﺤﻴﺙ‬ ‫‪λ max = 500nm‬‬ ‫ﺍﻟﺸﻤﺱ ﻤﻘﺎﺱ ﻤﻥ ﻁﺒﻘﺎﺕ ﺍﻟﺠﻭ ﺍﻟﻌﻠﻴﺎ ﻟﻸﺭﺽ‪ ،‬ﻨﺠﺩ ﺃﻥ ﻗﻴﻤﺔ‬
‫‪ ،(1nm = 10-9 m‬ﻭﺒﺎﺴﺘﺨﺩﺍﻡ ﻗﺎﻨﻭﻥ ﻭﺍﻴﻥ ﻴﺘﺒﻴﻥ ﺃﻥ ﺩﺭﺠﺔ ﺤﺭﺍﺭﺓ ﺍﻟﺸﻤﺱ‪.‬‬
‫‪2.9 ×10 −3 m.k 2.9 × 10 −3 m.k‬‬
‫=‪T‬‬ ‫=‬ ‫‪= 5800k‬‬
‫‪λ max‬‬ ‫‪500 × 10 −3 m‬‬

‫ﺍﻟﻁﻴﻑ ﺍﻟﻜﻬﺭﻭﻤﻐﻨﺎﻁﻴﺴﻲ ﻷﺸﻌﺔ ﺍﻟﺸﻤﺱ‪.‬‬

‫‪- ١٠ -‬‬
‫‪PDF created with pdfFactory Pro trial version www.pdffactory.com‬‬
‫‪FIGURE 1.3 The electromagnetic spectrum of the sun as measured in the upper‬‬
‫‪atmosphere of the earth. A comparison of this figure with Figure 1.2 shows that‬‬
‫‪the sun's surface radiates as a blackbody at a temperature of about 6000 k.‬‬

‫ﻭﺒﻌﺩﻤﺎ ﻋﺭﻓﻨﺎ ﺍﻟﻁﺒﻴﻌﺔ ﺍﻟﻜﻤﻴﺔ ﻟﻠﻁﺎﻗﺔ )ﻤﻤﺜﻠﺔ ‪ (quanta‬ﻭﻫﻲ ‪ nv‬ﻓﺈﻥ ﻫـﺫﺍ‬


‫‪ (1 A =10‬ﻁﺎﻗﺘﻪ ﺘﺴﺎﻭﻱ‪.‬‬ ‫)ﺤﻴﺙ‬ ‫ﻴﻌﻨﻲ ﺃﻥ ﺸﻌﺎﻉ ﻁﻭﻟﻪ ﺍﻟﻤﻭﺠﻲ‬
‫‪°‬‬ ‫‪°‬‬
‫‪−10‬‬
‫‪m‬‬ ‫‪λ = 6000 A‬‬

‫)‪hc (6.63 × 10 34 J.s) (3 × 10 8 m/s‬‬


‫= = ‪E = hv‬‬
‫‪λ‬‬ ‫‪(6000 ×10 −10 m‬‬
‫‪= 3.3 × 10 −19 J = 2.06 ev‬‬

‫ﻓﺈﺫﺍ ﻜﺎﻥ ﻫﺫﺍ ﺍﻟﺸﻌﺎﻉ ﺼﺎﺩﺭ ﻤﻥ ﻤﺼﺩﺭ ﻗﺩﺭﺘـﻪ ‪ 100‬ﻭﺍﻁ )‪) (W‬ﻭﺤـﺩﺓ‬
‫ﺍﻟﻭﺍﻁ ‪ Watt‬ﻫﻲ ﺠﻭل ﻟﻜل ﺜﺎﻨﻴﺔ ‪ ،(W: Joul/sec‬ﻫﺫﺍ ﻤﻌﻨﺎﻩ ﺃﻥ ﺇﺫﺍ ﻜﺎﻨﺕ ‪ N‬ﺘﺭﻤﺯ‬
‫ﻟﻌﺩﺩ ﺍﻟﻜﻤﺎﺕ ﻓﻲ ﺍﻟﺜﺎﻨﻴﺔ ﺍﻟﻭﺍﺤﺩﺓ‪ ،‬ﻓﺈﺫﺍ ﻜﺎﻨﺕ ﻜل ﻜﻡ ﻤﻥ ﺍﻟﻁﺎﻗﺔ ﺘـﺴﺎﻭﻱ ‪ hv‬ﻓـﺈﻥ‬
‫ﺍﻟﻘﺩﺭﺓ )‪.Power = N (hv‬‬
‫‪∴ P = 100 = W = N hv‬‬
‫‪P‬‬ ‫‪100W‬‬
‫= =‪∴N‬‬ ‫‪−19‬‬
‫‪= 3×10 20 guanta/sec‬‬
‫‪E 3.3× 10 J‬‬

‫ﻭﻫﺫﺍ ﺒﻼ ﺸﻙ ﻋﺩﺩ ﻜﺒﻴﺭ ﺠﺩﺍﹰ ﻤﻥ ﺍﻟﻜﻤﺎﺕ ﻤﻤﺎ ﻴﺅﺩﻱ ﺇﻟﻰ ﻋـﺩﻡ ﺇﺤـﺴﺎﺴﻨﺎ‬
‫ﺒﺎﻟﻁﺒﻴﻌﺔ ﺍﻟﺠﺴﻴﻤﻴﺔ ﻟﻠﻀﻭﺀ )ﻋﻠﻰ ﺍﻓﺘﺭﺍﺽ ﺃﻥ ﻜل ﻜﻡ ﻤﻥ ﺍﻟﻁﺎﻗﺔ ﻫﻭ ﺠﺴﻴﻡ(‪.‬‬
‫‪1-3 The Photoelectric Effect‬‬ ‫‪ 3-1‬ﺍﻟﺘﺄﺜﻴﺭ ﺍﻟﻜﻬﺭﻭﻀﻭﺌﻲ‬
‫ﻓﻲ ﻋﺎﻡ ‪ 1887, 1886‬ﻭﺒﻴﻨﻤﺎ ﻜﺎﻥ ﻴﺠﺭﻱ ﺘﺠﺎﺭﺒﻪ ﺍﻟﺘـﻲ ﺃﻜـﺩﺕ ﻨﻅﺭﻴـﺔ‬
‫ﻤﺎﻜﺴﻭﻴل ﺍﻟﺨﺎﺼﺔ ﺒﺎﻟﻁﺒﻴﻌﺔ ﺍﻟﻤﻭﺠﻴﺔ ﻟﻠﻀﻭﺀ‪ ،‬ﺍﻜﺘﺸﻑ ﺍﻟﻔﻴﺯﻴﺎﺌﻲ ﺍﻷﻟﻤﺎﻨﻲ ﻫﻴﻨـﺭﻙ‬
‫ﻫﻴﺭﺘﺯ ‪ Heinrich Hertz‬ﺃﻥ ﺍﻷﺸﻌﺔ ﺍﻟﻀﻭﺌﻴﺔ ﺍﻟﻔﻭﻕ ﺒﻨﻔـﺴﺠﻴﺔ ‪ultraviolet light‬‬
‫ﺘﺴﺒﺏ ﺍﻨﻁﻼﻕ )ﺍﻨﺒﻌﺎﺙ( ‪ emission‬ﺍﻹﻟﻜﺘﺭﻭﻨﺎﺕ ﻤﻥ ﺴﻁﺢ ﻤﻌﺩﻥ ﺒﺎﻹﺸﻌﺎﻉ ﻴﺴﻤﻰ‬
‫ﺒﺎﻟﺘﺄﺜﻴﺭ ﺍﻟﻜﻬﺭﻭﻀﻭﺌﻲ‪.‬‬
‫ﻭﻁﺒﻘﺎﹰ ﻟﻘﻭﺍﻨﻴﻥ ﺍﻟﻔﻴﺯﻴﺎﺀ ﺍﻟﺘﻘﻠﻴﺩﻴﺔ‪ ،‬ﻓﺈﻥ ﺍﻹﺸﻌﺎﻉ ﺍﻟﻜﻬﺭﻭﻤﻐﻨﺎﻁﻴﺴﻲ ﻋﺒﺎﺭﺓ ﻋﻥ‬
‫ﻤﺠﺎل ﻜﻬﺭﺒﻲ ﻴﺘﺫﺒﺫﺏ ﻋﻤﻭﺩﻴﺎﹰ ﻋﻠﻰ ﺍﺘﺠﺎﻩ ﺍﻨﺒﻌﺎﺙ ﺍﻹﺸﻌﺎﻉ )ﻫﻨﺎ ﺃﻫﻤﻠﻨـﺎ ﺍﻟﻤﺠـﺎل‬

‫‪- ١١ -‬‬
‫‪PDF created with pdfFactory Pro trial version www.pdffactory.com‬‬
‫ﺍﻟﻤﻐﻨﺎﻁﻴﺴﻲ(‪ .‬ﻭﺍﻟﺫﻱ ﻨﺭﻴﺩ ﺃﻥ ﻨﺭﻜﺯ ﻋﻠﻴﻪ ﻫﻨﺎ ﺃﻥ ﺸﺩﺓ ﺍﻷﺸﻌﺔ ‪intensity of the‬‬
‫‪ radiation‬ﺘﺘﻨﺎﺴﺏ ﻤﻊ ﻤﺭﺒﻊ ﺴﻌﺔ ﺍﻟﻤﺠﺎل ﺍﻟﻜﻬﺭﺒﻲ‪.‬‬
‫‪The intensity of the radiation is proportioned to the square of the‬‬
‫‪amplitude of the .oscillating electric field.‬‬
‫ﻭﻴﻤﻜﻥ ﻟﻼﻟﻜﺘﺭﻭﻨﺎﺕ ﺍﻟﺘﻲ ﻋﻠﻰ ﺴﻁﺢ ﺍﻟﻤﻌﺩﻥ ﺃﻥ ﺘﺘﺫﺒـﺫﺏ ﻤـﻊ ﺍﻟﻤﺠـﺎل‬
‫ﺍﻟﻜﻬﺭﺒﻲ ﺍﻟﺴﺎﻗﻁ ﻋﻠﻴﻬﺎ‪ ،‬ﻭﻜﻠﻤﺎ ﺍﺯﺩﺍﺩﺕ ﺸـﺩﺘﻪ )ﺴـﻌﺘﻪ( ﺘـﺯﺩﺍﺩ ﺴـﻌﺔ ﺘﺫﺒـﺫﺏ‬
‫ﺍﻹﻟﻜﺘﺭﻭﻨﺎﺕ ﻜﺜﻴﺭﺍﹰ ﻤﻤﺎ ﻴﺅﺩﻱ ﻓﻲ ﺍﻟﻨﻬﺎﻴﺔ ﺇﻟﻰ ﻜﺴﺭ ﺍﺭﺘﺒﺎﻁﻬﺎ ﺒﺎﻟﺴﻁﺢ ﻭﺍﻨﻁﻼﻗﻬـﺎ‬
‫ﺒﻁﺎﻗﺔ ﺤﺭﻜﻴﺔ ‪ kinetic energy‬ﻭﺍﻟﺘﻲ ﺴﺘﻌﺘﻤﺩ ﻋﻠﻰ ﺴﻌﺔ )ﺸﺩﺓ( ﺍﻟﻤﺠﺎل ﺍﻟﻜﻬﺭﺒﻲ‬
‫ﻟﻺﺸﻌﺎﻉ ﺍﻟﺴﺎﻗﻁ‪ .‬ﺇﻥ ﻫﺫﺍ ﺍﻟﺘﻔﺴﻴﺭ ﺍﻟﻔﻴﺯﻴﺎﺌﻲ )ﺍﻟﺘﻘﻠﻴـﺩﻱ( ﻴﺘﻌـﺎﺭﺽ ﺘﻤﺎﻤـﺎﹰ ﻤـﻊ‬
‫ﺍﻟﻤﺸﺎﻫﺩﺍﺕ ﺍﻟﻔﻴﺯﻴﺎﺌﻴﺔ ﻟﻬﺫﻩ ﺍﻟﻅﺎﻫﺭﺓ ﻭﺍﻟﺘﻲ ﺘﻤﺜﻠﺕ ﻓﻲ‪:‬‬
‫‪ -1‬ﻭ‪‬ﺠﺩ ﺃﻥ ﺍﻟﻁﺎﻗﺔ ﺍﻟﺤﺭﻜﻴﺔ ﻟﻺﻟﻜﺘﺭﻭﻨﺎﺕ ﺍﻟﻤﻨﺒﻌﺜﺔ ﻤﻥ ﺍﻟﺴﻁﺢ ﻻ ﺘﻌﺘﻤﺩ ﻋﻠﻰ ﺸﺩﺓ‬
‫ﺍﻻﻨﺒﻌﺎﺙ ‪.independent of the intensity of incidental‬‬
‫‪ -2‬ﻭﺠﺩ ﺘﺠﺭﻴﺒﻴﺎﹰ ﺃﻥ ﺍﻹﻟﻜﺘﺭﻭﻨﺎﺕ ﻻ ﺘﻨﺒﻌﺙ ﻤﻥ ﺍﻟﺴﻁﺢ ﺇﻻ ﺇﺫﺍ ﻜﺎﻥ ﺘﺭﺩﺩ ﺍﻹﺸﻌﺎﻉ‬
‫ﺒﻐﺽ ﺍﻟﻨﻅﺭ ﻋﻥ ﺸﺩﺓ ﺍﻹﺸـﻌﺎﻉ ﺴﻨـﺴﻤﻲ ﻫـﺫﺍ‬ ‫‪ν°‬‬ ‫ﺍﻟﺴﺎﻗﻁ ﺃﻜﺒﺭ ﻤﻥ ﺘﺭﺩﺩ ﻤﻌﻴﻥ‬
‫ﺘﻌﺘﻤﺩ ﻋﻠﻰ ﻨﻭﻉ ﺍﻟﻤﻌﺩﻥ‪.‬‬ ‫‪ν°‬‬ ‫ﺒﻌﺘﺒﺔ ﺍﻟﺘﺭﺩﺩ ‪ threshold frequency‬ﻭﻗﻴﻤﺔ‬ ‫‪ν°‬‬ ‫ﺍﻟﺘﺭﺩﺩ‬
‫‪ν‬‬ ‫ﻭﻜﺫﻟﻙ ﻭﺠﺩ ﺃﻥ ﺍﻟﻁﺎﻗﺔ ﺍﻟﺤﺭﻜﻴﺔ ﻟﻼﻟﻜﺘﺭﻭﻨﺎﺕ ﺍﻟﻤﻨﺒﻌﺜﺔ ﺘﺘﻨﺎﺴﺏ ﺨﻁﻴﺎﹰ ﻤﻊ ﺍﻟﺘﺭﺩﺩ‬
‫ﻜﻤﺎ ﻫﻭ ﻤﻭﻀﺢ ﻓﻲ ﺸﻜل ‪.1-4‬‬ ‫ﻤﻥ ‪ν °‬‬ ‫ﺃﻜﺒﺭ‬ ‫‪ν‬‬ ‫ﻭﺫﻟﻙ ﺇﺫﺍ ﻜﺎﻨﺕ‬

‫ﺸﻜل ‪ :1-4‬ﺘﻐﻴﺭ ﻗﻴﻡ ﺍﻟﻁﺎﻗﺔ ﺍﻟﺤﺭﻜﻴﺔ ﻟﻺﻟﻜﺘﺭﻭﻨﺎﺕ ﻤﻊ ﺍﻟﺘﺭﺩﺩ‪.‬‬

‫‪- ١٢ -‬‬
‫‪PDF created with pdfFactory Pro trial version www.pdffactory.com‬‬
‫‪FIGURE 1.4 The kinetic energy of electrons ejected from the surface of‬‬
‫‪sodium metal versus the frequency of the incident ultraviolet radiation. The‬‬
‫)‪threshold frequency here is 4.40 × 1014 Hz (1 Hz = 1 s-1‬‬

‫ﻭﻟﺘﻔﺴﻴﺭ ﻫﺫﻩ ﺍﻟﻨﺘﺎﺌﺞ‪ ،‬ﻁﻭﺭ ﺍﻟﻌﺎﻟﻡ ﺍﻴﻨﺸﺘﺎﻴﻥ ﻓﺭﻀﻴﺔ ﺒﻼﻨﻙ ﻤﻥ ﺘﻜﻤﻴﻡ ﺍﻟﻁﺎﻗﺔ‪.‬‬
‫ﻭﻫﻨﺎ ﻨﺫﻜﺭ ﺃﻥ ﺒﻼﻨﻙ ﻁﺒﻕ ﻤﻔﻬﻭﻡ ﺘﻜﻤﻴﻡ ﺍﻟﻁﺎﻗﺔ ﻓﻘﻁ ﻋﻠﻰ ﺍﻟﻤﺘﺫﺒﺫﺏ ﺍﻟﺫﻱ ﻴﻤـﺘﺹ‬
‫( ﺃﻤﺎ ﺇﺫﺍ ﻤﺎ ﺍﻨﺒﻌﺙ‬ ‫‪E = nhν‬‬ ‫ﻓﺈﻨﻪ ﻴﻤﻠﻙ ﻁﺎﻗﺔ‬ ‫‪ν‬‬ ‫ﺃﻭ ﻴﺒﻌﺙ ﺍﻟﻁﺎﻗﺔ )ﻓﺈﺫﺍ ﻤﺼﺩﺭ ﺘﺭﺩﺩﻩ‬
‫ﺍﻹﺸﻌﺎﻉ ﻤﻥ ﻫﺫﺍ ﺍﻟﻤﺘﺫﺒﺫﺏ ﻓﺈﻨﻪ ﻴﺴﻠﻙ )ﻜﻤﺎ ﺍﻗﺘﺭﺡ ﺒﻼﻨﻙ( ﻜﻤﻭﺠﺔ ﻋﺎﺩﻴﺔ ﺘﺨـﻀﻊ‬
‫ﻟﻠﻤﻔﺎﻫﻴﻡ ﺍﻟﻔﻴﺯﻴﺎﺌﻴﺔ ﺍﻟﺘﻘﻠﻴﺩﻴﺔ‪ .‬ﺃﻤﺎ ﺍﻟﻌﺎﻟﻡ ﺍﻴﻨﺸﺘﺎﻴﻥ ﻓﻘﺩ ﺫﻫﺏ ﺇﻟﻰ ﻤﺎ ﻫﻭ ﺃﺒﻌﺩ ﻤﻥ ﻫﺫﺍ‪،‬‬
‫ﻓﻘﺎل ﺇﻥ ﺍﻟﺘﻜﻤﻴﻡ ﻻ ﻴﻨﻁﺒﻕ ﻓﻘﻁ ﻋﻠﻰ ﺍﻟﻤﺘﺫﺒﺫﺏ ﺒل ﺇﻥ ﺍﻹﺸﻌﺎﻉ ﺍﻟﻤﻨﺒﻌﺙ ﻤﻨﻪ ﻴﻨﺒﻌﺙ‬
‫ﻜﻤﺎﺕ ﻤﻥ ﺍﻟﻁﺎﻗﺔ ) ﺃﻭ ﺤ‪‬ﺯ‪‬ﻡ‪ ‬ﻤﻥ ﺍﻟﻁﺎﻗﺔ( ﻤﻨﻔﺼﻠﺔ ﻋﻥ ﺒﻌﻀﻬﺎ ﻭﻁﺎﻗـﺔ ﻜـل ﻜـﻡ‬
‫ﻭﺃﻁﻠﻕ ﺍﺴﻡ ﺍﻟﻔﻭﺘﻭﻥ ‪ photon‬ﻋﻠﻴﻪ‪.‬‬ ‫‪E = hν‬‬

‫‪Einstein proposed that the radiation itself existed as smell‬‬


‫‪packers of energy, E = hν , now known as photons.‬‬

‫ﻭﺒﻨﺎﺀﺍﹰ ﻋﻠﻰ ﻤﺒﺩﺃ ﺤﻔﻅ ﺍﻟﻁﺎﻗﺔ ‪ ،Conservation of energy‬ﻭﻀﺢ ﺍﻴﻨﺸﺘﺎﻴﻥ‬


‫)ﺍﻟﺘﻲ ﻴﻨﺒﻌـﺙ ﺒﻬـﺎ ﺍﻹﻟﻜﺘـﺭﻭﻥ ﺫﻭ ﺍﻟﻜﺘﻠـﺔ ‪m‬‬ ‫= ‪K. E‬‬
‫‪1‬‬
‫‪mv 2‬‬ ‫ﺃﻥ ﺍﻟﻁﺎﻗﺔ ﺍﻟﺤﺭﻜﻴﺔ‬
‫‪2‬‬
‫ﻭﺍﻟﺴﺭﻋﺔ ‪ (v‬ﻤﻥ ﺴﻁﺢ ﺍﻟﻤﻌﺩﻥ ﻋﺒﺎﺭﺓ ﻋﻥ ﺍﻟﻔﺭﻕ ﺒﻴﻥ ﺍﻟﻁﺎﻗﺔ ﺍﻟـﺴﺎﻗﻁﺔ ﻟﻠﻔﻭﺘـﻭﻥ‬
‫ﻭﺃﻗل ﻁﺎﻗﺔ ﻻﺯﻤﺔ ﻟﻨﺯﻉ ﺍﻹﻟﻜﺘﺭﻭﻥ )ﻭﺘﺤﺭﻴﺭﻩ( ﻤﻥ ﻁﺎﻗﺔ ﺭﺒﻁﺔ ﺒﺎﻟﻤﻌـﺩﻥ ‪W‬‬ ‫‪hν‬‬

‫)ﻭﻴﺭﻤﺯ ﻟﻬﺎ ﺃﺤﻴﺎﻨﺎﹰ ﺒـ ‪ ( ο/‬ﻭﺍﻟﺘﻲ ﺘﺴﻤﻰ ﺩﺍﻟﺔ ﺍﻟﺸﻐل ‪.Work function‬‬


‫ﻭﻴﻤﻜﻨﻨﺎ ﺼﻴﺎﻏﺔ ﻤﺎ ﺴﺒﻕ ﺫﻜﺭﻩ ﺒﺎﻟﻤﻌﺎﺩﻟﺔ ﺍﻟﺘﺎﻟﻴﺔ‪:‬‬
‫‪1‬‬
‫= ‪K. E‬‬ ‫‪mv 2 = hv − φ‬‬ ‫)‪(1-5‬‬
‫‪2‬‬

‫ﻻ ﻴﻤﻜـﻥ ﺃﻥ ﻴﻜـﻭﻥ‬ ‫‪hν − φ‬‬ ‫ﻤﻭﺠﺒﺔ ﻭﻋﻠﻴﻪ ﻓـﺎﻟﻔﺭﻕ‬ ‫‪1 2‬‬


‫‪mc‬‬ ‫ﻻﺤﻅ ﺃﻥ ﻗﻴﻤﺔ‬
‫‪2‬‬
‫ﻫـﻲ ﺍﻟﻁﺎﻗـﺔ ﺍﻟﻼﺯﻤـﺔ ﻟﺘﺤﺭﻴـﺭ‬ ‫‪hν‬‬ ‫ﺴﺎﻟﺏ‪ .‬ﺃﻱ ﺃﻨﻪ ‪ . hν ≥ φ‬ﺇﻥ ﺍﻗل ﻗﻴﻤﺔ ﻟـ‬
‫ﺃﻱ ﺃﻥ‪:‬‬ ‫‪ν°‬‬ ‫ﺍﻹﻟﻜﺘﺭﻭﻥ ﻤﻥ ﺭﺒﻁ ﺍﻟﻨﻭﺍﺓ ﻭﻫﺫﺍ ﻴﺤﺩﺙ ﻋﻥ ﻋﺘﺒﺔ ﺍﻟﺘﺭﺩﺩ‬
‫‪hν ° = φ‬‬ ‫)‪(1-6‬‬

‫‪- ١٣ -‬‬
‫‪PDF created with pdfFactory Pro trial version www.pdffactory.com‬‬
‫ﻭﻴﻤﻜﻨﻨﺎ ﻜﺘﺎﺒﺔ ﻤﻌﺎﺩﻟﺔ )‪ (1-5‬ﻋﻠﻰ ﺍﻟﺼﻭﺭﺓ‪.‬‬
‫‪1‬‬
‫‪mc 2 = hv − hv °‬‬ ‫)‪(1-7‬‬
‫‪2‬‬

‫ﻭﻫﺫﻩ ﻤﻌﺎﺩﻟﺔ ﺨﻁ ﻤﺴﺘﻘﻴﻡ )ﻋﻠﻰ ﺍﻟﺼﻭﺭﺓ ‪ (y = mx-c‬ﻭﻫﻭ ﺘﻤﺎﻤﺎﹰ ﻤﺎ ﻴﺸﺎﻫﺩ‬


‫ﻤﻥ ﺍﻟﻨﺘﺎﺌﺞ ﺍﻟﺘﺠﺭﻴﺒﻴﺔ ﺍﻟﻤﻭﻀﺤﺔ ﻓﻲ ﺸﻜل ‪.1-5‬‬

‫‪Figure 1-5. Photoelectric effect data showing a plot of retarding‬‬


‫‪potential necessary to stop electron flow from a metal (lithium), or‬‬
‫‪equivalently, electron kinetic energy, as a function of frequency of the‬‬
‫‪incident light, the slope of the line is h/e.‬‬
‫ﻭﻗﺒل ﺃﻥ ﻨﻨﺘﻘل ﻟﻤﻭﻀﻭﻉ ﺁﺨﺭ‪ ،‬ﻤﻥ ﺍﻟﻤﻬﻡ ﺃﻥ ﻨﻭﻀﺢ ﺍﻟﻭﺤﺩﺍﺕ ﺍﻟﻤـﺴﺘﺨﺩﻤﺔ ﻓـﻲ‬
‫ﻋﺎﺩﺓ ﺘﻌﻁﻲ ﺒﻭﺤﺩﺓ ﺍﻹﻟﻜﺘﺭﻭﻥ ﻓﻭﻟﺕ ‪ ،eV‬ﻭﻫـﺫﻩ ﻭﺤـﺩﺓ‬ ‫‪φ‬‬ ‫ﺍﻟﻤﻌﺎﺩﻟﺔ‪ .‬ﺒﺎﻟﻨﺴﺒﺔ ﻟـ‬
‫ﻁﺎﻗﺔ‪ ،‬ﻭﻟﺤﺴﺎﺏ )ﻤﺜﻼﹰ( ﺴﺭﻋﺔ ﺍﻹﻟﻜﺘﺭﻭﻥ ﺍﻟﻤﻨﺒﻌﺙ ﺒﻭﺤﺩﺓ ‪ m/s‬ﻓﻤـﻥ ﺍﻟﻤﻬـﻡ ﺃﻥ‬
‫ﻴﻌﺭﻑ ﺍﻟﻁﺎﻟﺏ ﻜﻴﻑ ﻴﻘﻭﻡ ﺒﻌﻤﻠﻴﺔ ﺍﻟﺘﺤﻭﻴل ﺒﻁﺭﻴﻘﺔ ﺼﺤﻴﺤﺔ‪.‬‬

‫‪- ١٤ -‬‬
‫‪PDF created with pdfFactory Pro trial version www.pdffactory.com‬‬
‫ﺃﻭﻻﹰ‪ :‬ﻤﻥ ﺘﻌﺭﻴﻑ ﺍﻹﻟﻜﺘﺭﻭﻥ ﻓﻭﻟﺕ‬
‫‪(1 coulomb) (1 volt) = 1 Joule‬‬
‫ﻭﺍﻟﺸﺤﻨﺔ ﺍﻷﻭﻟﻴﺔ ﻟﻠﺒﺭﻭﺘﻭﻥ ﺘﺴﺎﻭﻱ‬
‫ﻋﻨﺩﺌﺫ‬ ‫‪1.6 ×10 −19 C‬‬ ‫ﻭﺍﻟﺸﺤﻨﺔ ﺍﻷﻭﻟﻴﺔ ﻟﻠﺒﺭﻭﺘﻭﻥ ﺘﺴﺎﻭﻱ‬

‫) ‪ J‬ﺘﺭﻤﺯ ﻟﻠﺠﻭل (‬
‫)‪1 eV = (1.6 × 10 −19 C) (1 V‬‬
‫‪= 1.6 ×10 −19 J‬‬

‫‪ν°‬‬ ‫ﻤﺜﺎل‪ :2‬ﺩﺍﻟﺔ ﺍﻟﺸﻐل ﻟﻠﺼﻭﺩﻴﻭﻡ ﺘﺴﺎﻭﻱ ‪ ،1.82 en‬ﺍﺤﺴﺏ ﻋﺘﺒﺔ ﺍﻟﺘـﺭﺩﺩ‬
‫ﻟﻠﺼﻭﺩﻴﻭﻡ؟‬
‫ﻤﻥ ‪ eV‬ﺇﻟﻰ ﺠﻭل‪:‬‬ ‫‪φ‬‬ ‫ﺍﻟﺤل‪ :‬ﺃﻭﻻﹰ‪ :‬ﻴﻠﺯﻡ ﺘﺤﻭﻴل‬
‫‪φ = 1.82 eV‬‬
‫‪J‬‬
‫‪= (1.82 eV) (1.6 × 10 −19‬‬ ‫)‬
‫‪eV‬‬
‫‪= 2.92 ×10 −19 J‬‬

‫ﺤﺴﺎﺏ ‪ν °‬‬ ‫‪ [hν‬ﻴﻤﻜﻨﻨﺎ‬ ‫‪°‬‬


‫]‪= φ‬‬ ‫ﻭﻤﻥ ﻤﻌﺎﺩﻟﺔ )‪(1-6‬‬
‫)‪φ (2.92 ×10 −19 J‬‬
‫= ‪ν°‬‬ ‫=‬
‫)‪h (6.63×10 −3 J.S‬‬
‫‪1‬‬
‫‪= 4.40 ×1014‬‬ ‫‪= 4.40 ×1014 H 2‬‬
‫‪5‬‬

‫‪1 ec‬‬
‫‪5‬‬
‫ﺤﻴﺙ ‪ Hz‬ﺘﺭﻤﺯ ﻟﻠﻬﺭﺘﺯ ﻭﻫﻭ‬

‫ﻤﺜﺎل‪:3‬‬
‫ﺃﺸﻌﺔ ﻓﻭﻕ ﺒﻨﻔﺴﺠﻴﺔ ﻁﻭﻟﻬﺎ ﺍﻟﻤﻭﺠﻲ ‪ 3500 Å‬ﺘﺴﻘﻁ ﻋﻠﻰ ﺴﻁﺢ ﺒﻭﺘﺎﺴﻴﻭﻡ‪.‬‬
‫ﺃﻗﺼﻰ ﻁﺎﻗﺔ ﻟﻺﻟﻜﺘﺭﻭﻨﺎﺕ ﺍﻟﻀﻭﺌﻴﺔ ﺘﺴﺎﻭﻱ ‪.1.6 eV‬‬

‫‪- ١٥ -‬‬
‫‪PDF created with pdfFactory Pro trial version www.pdffactory.com‬‬
‫ﺍﺤﺴﺏ ﺩﺍﻟﺔ ﺍﻟﺸﻐل ﻟﻠﺒﻭﺘﺎﺴﻴﻭﻡ؟‬
‫ﺍﻟﺤل‪:‬‬
‫ﻤﻥ ﻤﻌﺎﺩﻟﺔ )‪(1-5‬‬
‫‪K.E = hν − ο/‬‬

‫‪∴φ = hν − k.E‬‬

‫‪hν‬‬ ‫ﻨﺤﺴﺏ ﺃﻭﻻﹰ ﻗﻴﻤﺔ‬


‫)‪hc (6.63×10 −34 J.s) (3 × 10 8 m/s‬‬
‫= = ‪hν‬‬
‫‪λ‬‬ ‫)‪(3500 × 10 −10 m‬‬

‫ﻭﻟﺘﺤﻭﻴل ﻫﺫﻩ ﺍﻟﻘﻴﻤﺔ ﻤﻥ ﺍﻟﺠﻭل ﻟﻺﻟﻜﺘﺭﻭﻥ ﻓﻭﻟﺕ‬

‫‪∴φ = 3. eV − 1.6 eV = 1.95 eV‬‬

‫ﻤﺜﺎل‪ :‬ﻋﻨﺩ ﺘﻌﺭﺽ ﺴﻁﺢ ﻤﺎﺩﺓ ﺍﻟﻠﻴﺜﻴﻭﻡ ﻟﻺﺸﻌﺎﻉ‪ ،‬ﻓﺈﻥ ﺍﻟﻁﺎﻗﺔ ﺍﻟﺤﺭﻜﻴﺔ ﻟﻺﻟﻜﺘﺭﻭﻨﺎﺕ‬
‫ﻭﺫﻟﻙ ﺇﺫﺍ ﻜﺎﻨﺕ ‪. λ = 300.0nm‬‬ ‫‪2.935 ×10 −19 J‬‬ ‫ﺍﻟﻤﻨﺒﻌﺜﺔ‬

‫‪1.28 × 10 −19 J‬‬ ‫ﻓﺈﻥ ﺍﻟﻁﺎﻗﺔ ﺍﻟﺤﺭﻜﻴﺔ ﺘﺴﺎﻭﻱ‬ ‫‪λ‬‬ ‫=‬ ‫ﺇﻤﺎ ﺇﺫﺍ ﻜﺎﻨﺕ‬
‫ﺍﺤﺴﺏ )‪ (a‬ﺜﺎﺒﺕ ﺒﻼﻨﻙ؟‬
‫)‪ (b‬ﻋﺘﺒﺔ ﺍﻟﺘﺭﺩﺩ؟‬

‫)‪ (c‬ﺍﻟﺸﻐل؟‬
‫ﺍﻟﺤل‪ :‬ﻤﻥ ﻤﻌﺎﺩﻟﺔ )‪ (1-5‬ﻓﻲ ﺤﺎﻟﺔ ﺍﻟﻁﻭل ﺍﻟﻤﻭﺠﻲ ﺍﻷﻭل ﻭﺍﻟﻁﻭل ﺍﻟﻤﻭﺠﻲ‬
‫( ‪(K.E)1 − (K.E) 2 = h (ν1 − ν 2 ) = hc‬‬
‫‪1 1‬‬
‫) ‪−‬‬ ‫ﺍﻟﺜﺎﻨﻲ ﻨﺘﺤﺼل ﻋﻠﻰ‬
‫‪λ1 λ 2‬‬

‫‪‬‬
‫‪2.935×10 −19 J − 1.280 × 10 −19 J = h (3×10 8 m/s) ‬‬
‫‪1‬‬
‫‪−9‬‬
‫‪−‬‬
‫‪1‬‬
‫‪−9‬‬
‫‪‬‬
‫‪‬‬ ‫ﺒﺎﻟﺘﻌﻭﻴﺽ‪:‬‬
‫‪ 3 ×10 m 400 ×10 m ‬‬

‫‪- ١٦ -‬‬
‫‪PDF created with pdfFactory Pro trial version www.pdffactory.com‬‬
‫‪1.655 ×1019 J‬‬
‫‪∴h‬‬ ‫‪14 −1‬‬
‫‪= 6.625 ×10 −34 J.S‬‬
‫‪2.498 ×10 s‬‬

‫‪λ = 300 nm‬‬ ‫)‪ (b‬ﻟﺤﺴﺎﺏ ﻋﺘﺒﺔ ﺍﻟﺘﺭﺩﺩ‪ ،‬ﻨﺄﺨﺫ ﺍﻟﻁﻭل ﺍﻟﻤﻭﺠﻲ )ﻤﺜﻼﹰ(‬

‫‪∴ = hv − hv °‬‬

‫‪hc‬‬
‫= ‪∴ 2.935 ×10 −19 J‬‬ ‫‪− hv °‬‬
‫‪300 ×10 −9 m‬‬

‫‪v°‬‬ ‫ﻭﺒﺎﻟﺘﻌﻭﻴﺽ ﻋﻥ ﻗﻴﻡ ‪ c,h‬ﻨﺠﺩ ﺃﻥ ﻗﻴﻤﺔ‬

‫‪v ° = 5.564 × 1014 Hz‬‬

‫‪φ = hv°‬‬ ‫)‪ (٢‬ﺩﺍﻟﺔ ﺍﻟﺸﻐل ﺘﺤﺴﺏ ﻤﺒﺎﺸﺭﺓ ﻤﻥ ﺍﻟﻌﻼﻗﺔ‬

‫‪= 3.687 ×10 −19 J‬‬

‫‪3.687 ×1019 J/‬‬


‫=‬ ‫‪= 2.30/eV‬‬
‫‪J/‬‬
‫‪1.6 ± ×10 −19‬‬
‫‪eV‬‬

‫‪ 4-1‬ﺘﺄﺜﻴﺭ ﻜﻭﻤﺒﺘﻭﻥ‪:‬‬
‫‪1-4 The Compton Effect‬‬
‫ﺍﻟﺘﺠﺭﺒﺔ ﺍﻟﺘﻲ ﺘﺅﻜﺩ ﺒﻭﻀﻭﺡ ﺍﻟﻁﺒﻴﻌﺔ ﺍﻟﺠـﺴﻴﻤﻴﺔ ﻟﻺﺸـﻌﺎﻉ ‪the particle‬‬
‫‪ nature of radiation‬ﺘﺴﻤﻰ ﺒﺘﺄﺜﻴﺭ )ﻅﺎﻫﺭﺓ( ﻜﻭﻤﺒﺘﻭﻥ ﻨﺴﺒﺔ ﻟﻠﻌـﺎﻟﻡ ﻜﻭﻤﺒﺘـﻭﻥ‬
‫‪ Arthur H. Compton‬ﻟﻘﺩ ﺍﻜﺘﺸﻑ ﻜﻭﻤﺒﺘﻭﻥ ﺃﻨﻪ ﺇﺫﺍ ﺍﺨﺘﺭﻕ ﺇﺸـﻌﺎﻉ ﺫﻭ ﻁـﻭل‬
‫ﻤﻭﺠﻲ )ﻓﻲ ﻤﻨﻁﻘﺔ ﺍﻷﺸﻌﺔ ﺍﻟﺴﻴﻨﻴﺔ ‪ (X-ray‬ﺸﺭﻴﺤﺔ ﻤﻌﺩﻨﻴﺔ ﻓـﺴﻴﺘﺒﻌﺜﺭ ‪scattered‬‬
‫ﺒﻁﺭﻴﻘﺔ ﻻ ﻴﻤﻜﻥ ﺘﻔﺴﻴﺭﻫﺎ ﺤﺴﺏ ﺍﻟﻨﻅﺭﻴﺔ ﺍﻟﺘﻘﻠﻴﺩﻴﺔ ﻟﻺﺸﻌﺎﻉ‪.‬‬
‫ﺍﻟﺫﻱ ﺘﺨﺒﺭﻨﺎ ﺒﻪ ﻗﻭﺍﻨﻴﻥ ﺍﻟﻔﻴﺯﻴﺎﺀ ﺍﻟﺘﻘﻠﻴﺩﻴﺔ ﺃﻥ ﺸﺩﺓ ﺍﻹﺸﻌﺎﻉ ‪ I‬ﺍﻟﻤﻨﺒﻌﺙ ﻤـﻥ‬
‫ﻤﺎﺩﺓ ﻨﺘﻴﺠﺔ ﺘﺄﺜﺭﻫﺎ ﺒﺈﺸﻌﺎﻉ ﺴﻘﻁ ﻋﻠﻴﻬﺎ )ﻤﻤﺎ ﻴﺅﺩﻱ ﺇﻟﻰ ﺍﻫﺘﺯﺍﺯ ﺇﻟﻜﺘﺭﻭﻨﺎﺘﻬﺎ ﻭﺍﻟﺘـﻲ‬
‫)ﺒﺎﻟﻨـﺴﺒﺔ ﻻﺘﺠـﺎﻩ ﺍﻷﺸـﻌﺔ‬ ‫‪θ‬‬ ‫ﺒﺩﻭﺭﻫﺎ ﺴﺘﺒﻌﺙ ﺇﺸﻌﺎﻉ( ﻋﻨﺩﻤﺎ ﺘﻘﺎﺱ ﻋﻨﺩ ﺯﺍﻭﻴﺔ‬
‫ﺤﺴﺏ ﺍﻟﻌﻼﻗﺔ‬ ‫‪θ‬‬ ‫ﺍﻟﺴﺎﻗﻁﺔ( ﻓﺈﻥ ‪ I‬ﺘﺘﻐﻴﺭ ﻤﻊ‬

‫‪- ١٧ -‬‬
‫‪PDF created with pdfFactory Pro trial version www.pdffactory.com‬‬
‫)‪Ι ≈ (1 + cos 2 θ‬‬ ‫)‪(1-8‬‬
‫ﻭﻫﺫﺍ ﻴﻌﻨﻲ ﺃﻥ ‪ I‬ﻻ ﺘﻌﺘﻤﺩ ﻋﻠﻰ ﺍﻟﻁﻭل ﺍﻟﻤﻭﺠﻲ ﻟﻸﺸﻌﺔ ﺍﻟـﺴﺎﻗﻁﺔ‪ ،‬ﻭﻫـﺫﺍ‬
‫ﻴﺘﻌﺎﺭﺽ ﺒﻭﻀﻭﺡ ﻤﻊ ﺍﻟﻨﺘﺎﺌﺞ ﺍﻟﺘﺠﺭﻴﺒﻴﺔ )ﺸﻜل ‪ (1-6‬ﻭﺍﻟﺘﻲ ﻴﺘﺒﻴﻥ ﺒﻭﻀﻭﺡ ﺘﻐﻴﺭ ‪I‬‬
‫ﺒﺘﻐﻴﺭ ‪. λ‬‬

‫‪Figure 1-6. The spectrum of radiation scattered by carbon,‬‬


‫‪showing the unmodified line at 0.7078 Å on the left and the shifted‬‬
‫‪line at 0.7314 Å on the right. The former is the wave-length of the‬‬
‫‪primary radiation.‬‬
‫ﻨﺘﺎﺌﺞ ﺘﺠﺭﺒﺔ ﻜﻭﻤﺒﺘﻭﻥ‪:‬‬

‫ﻟﻘﺩ ﻭﺠﺩ ﻜﻭﻤﺒﺘﻭﻥ ﺃﻥ ﺍﻹﺸﻌﺎﻉ ﺍﻟﻤﺘﺒﻌﺜﺭ ﻟﻪ ﻤﺭﻜﺒﺘﻴﻥ؛ ﻤﺭﻜﺒﺔ ﻁﻭﻟﻬﺎ ﺍﻟﻤﻭﺠﻲ‬


‫ﻤﺴﺎﻭٍ ﻟﻁﻭل ﻤﻭﺠﺔ ﺍﻹﺸﻌﺎﻉ ﺍﻟﺴﺎﻗﻁ ﻭﻤﺭﻜﺒﺔ ﺃﺨﺭﻯ ﺘﺨﺘﻠﻑ ﻓﻲ ﻁﻭﻟﻬﺎ ﺍﻟﻤـﻭﺠﻲ‬
‫ﻋﻥ ﺍﻟﻁﻭل ﺍﻟﻤﻭﺠﻲ ﻟﻺﺸﻌﺎﻉ ﺍﻟﺴﺎﻗﻁ ﻭﺘﻌﺘﻤﺩ ﻋﻠﻰ ﺯﺍﻭﻴﺔ ﺍﻟﺒﻌﺜـﺭﺓ ﻭﻗـﺩ ﺘﻤﻜـﻥ‬
‫ﻜﻭﻤﺒﺘﻭﻥ ﻤﻥ ﺸﺭﺡ ﻭﺠﻭﺩ ﻤﺭﻜﺒﺔ ﺍﻹﺸـﻌﺎﻉ ﺍﻟﻤﺘﺒﻌﺜـﺭﺓ ﺫﺍﺕ ﺍﻟﻁـﻭل ﺍﻟﻤـﻭﺠﻲ‬
‫ﺍﻟﻤﺨﺘﻠﻑ ﻋﻥ ﺍﻟﻁﻭل ﺍﻟﻤﻭﺠﻲ ﺍﻟﺴﺎﻗﻁ ﻭﺫﻟﻙ ﺒﺎﻋﺘﺒﺎﺭ ﺍﻟﺸﻌﺎﻉ ﺍﻟﺴﺎﻗﻁ ﻋﺒﺎﺭﺓ ﻋـﻥ‬
‫ﺤﻴﺙ ﻴﻌﺎﻨﻲ ﻜل ﻓﻭﺘﻭﻥ ﻤﻥ ﺘﺒﻌﺜﺭ )ﺘﺸﺘﺕ( ﻤـﺭﻥ‬ ‫‪hv‬‬ ‫ﺸﻌﺎﻉ ﻤﻥ ﺍﻟﻔﻭﺘﻭﻨﺎﺕ ﺒﻁﺎﻗﺔ‬
‫‪ elastic scattering‬ﻤﻊ ﻜل ﺇﻟﻜﺘﺭﻭﻥ‪.‬‬
‫ﻭﻜﻤﺎ ﻫﻭ ﻤﻌﻠﻭﻡ‪ ،‬ﻓﻲ ﺤﺎﻟﺔ ﺍﻟﺘﺸﺘﺕ ﺍﻟﻤﺭﻥ ﻓﺈﻥ ﻜﻤﻴﺔ ﺍﻟﺤﺭﻜـﺔ ‪momentum‬‬
‫ﻭﺍﻟﻁﺎﻗﺔ ‪ energy‬ﻜﻤﻴﺎﺕ ﺘﺨﻀﻊ ﻟﻘﺎﻨﻭﻥ ﺍﻟﺤﻔﻅ )ﺍﻟﺒﻘﺎﺀ( ‪momentum and energy‬‬
‫‪.must be conserved‬‬

‫‪- ١٨ -‬‬
‫‪PDF created with pdfFactory Pro trial version www.pdffactory.com‬‬
‫ﻭﻟﺘﻔﺴﻴﺭ ﻫﺫﻩ ﺍﻟﻅﺎﻫﺭﺓ ﺭﻴﺎﻀﻴﺎﹰ‪ ،‬ﺍﻓﺘﺭﺽ ﻜﻭﻤﺒﺘﻭﻥ ﺃﻥ ﺍﻟﻔﻭﺘﻭﻥ ﻟـﻪ ﻜﻤﻴـﺔ‬
‫ﺤﺭﻜﺔ ‪ p‬ﺘﻌﻁﻲ ﺒﺎﻟﻌﻼﻗﺔ‬
‫‪hv‬‬
‫=‪p‬‬ ‫)‪(1-9‬‬
‫‪c‬‬

‫ﺤﻴﺙ ﺘﻡ ﺍﻋﺘﺒﺎﺭ ﺍﻟﺤﺭﻜﺔ ﺍﻟﺩﻴﻨﺎﻤﻴﻜﻴﺔ ﻟﻠﻔﻭﺘﻭﻥ ﻜﺠﺴﻴﻡ ﻴﺨﻀﻊ ﻟﻘﻭﺍﻨﻴﻥ ﺍﻟﻨﻅﺭﻴﺔ‬


‫ﺍﻟﻨﺴﺒﻴﺔ ﻭﺍﻟﺘﻲ ﺘﻭﻀﺢ ﺍﻟﻌﻼﻗﺔ ﺒﻴﻥ ﺍﻟﻁﺎﻗﺔ ﻭﻜﻤﻴﺔ ﺍﻟﺤﺭﻜﺔ‪.‬‬
‫[‬
‫‪E = (m o c 2 ) 2 + (pc) 2‬‬ ‫)‪] (1-10‬‬
‫‪1‬‬
‫‪2‬‬

‫ﺤﻴﺙ ‪ mo‬ﻫﻲ ﺍﻟﻜﺘﻠﺔ ﺍﻟﺴﻜﻭﻨﻴﺔ ‪ rest mass‬ﻟﻠﺠﺴﻴﻡ‪ ،‬ﻭﺴﺭﻋﺔ ﺍﻟﺠﺴﻴﻡ ‪ v‬ﻋﻨﺩ‬


‫ﻜﻤﻴﺔ ﺍﻟﺤﺭﻜﺔ ‪ p‬ﺘﻌﻁﻰ ﺒﺎﻟﻌﻼﻗﺔ‬
‫=‪v‬‬
‫‪aE d‬‬
‫=‬
‫‪ap dp‬‬
‫[‬
‫‪(m ο c 2 ) 2 + (pc) 2‬‬ ‫]‬‫‪1‬‬
‫‪2‬‬

‫[‬ ‫]‬
‫‪1‬‬
‫‪1‬‬ ‫‪−‬‬ ‫‪d‬‬
‫‪= = (m ο c 2 ) 2 + (pc) 2‬‬ ‫‪2‬‬ ‫)‪(pc‬‬
‫‪2‬‬ ‫‪ap‬‬
‫‪1‬‬ ‫‪1‬‬
‫=‬ ‫‪(pc)c‬‬
‫[‬
‫‪2 (m c 2 ) 2 + (pc) 2‬‬
‫‪°‬‬
‫]‬
‫‪1‬‬
‫‪2‬‬

‫‪pc 2‬‬ ‫‪pc 2‬‬


‫= ‪∴v‬‬ ‫=‬ ‫)‪(1-11‬‬
‫‪E (m 2 c 4 + p 2 C 2 ) 12‬‬
‫‪°‬‬

‫ﻓﻲ ﻤﻌﺎﺩﻟﺔ )‪(1-11‬‬ ‫‪v= c‬‬ ‫ﻭ‬ ‫‪m° = o‬‬ ‫ﻓﻲ ﺤﺎﻟﺔ ﺍﻟﻔﻭﺘﻭﻥ‪ ،‬ﻨﻌﻭﺽ ﻋﻥ‬
‫‪pc 2‬‬
‫= ‪∴C/‬‬ ‫‪⇒E=pc‬‬ ‫)‪(1-12‬‬
‫‪E‬‬

‫‪E = hv‬‬ ‫ﻤﻥ ﻤﻌﺎﺩﻟﺔ )‪ (1-12‬ﻨﺘﺤﺼل ﻋﻠﻰ ﻤﻌﺎﺩﻟﺔ )‪ (1-9‬ﺤﻴﺙ‬


‫‪E hv‬‬
‫=‪p‬‬ ‫=‬
‫‪C C‬‬

‫ﺴـﺎﻗﻁ ﻋﻠـﻰ‬ ‫‪p°‬‬ ‫ﺩﻋﻨﺎ ﻨﻔﺘﺭﺽ ﺍﻵﻥ ﻭﺠﻭﺩ ﻓﻭﺘﻭﻥ ﺒﻜﻤﻴﺔ ﺤﺭﻜﺔ ﺍﺒﺘﺩﺍﺌﻴـﺔ‬
‫ﺃﻤـﺎ‬ ‫‪p‬‬ ‫ﺇﻟﻜﺘﺭﻭﻥ ﺴﺎﻜﻥ‪ .‬ﻭﺒﻌﺩ ﺍﻟﺘﺼﺎﺩﻡ‪ ،‬ﻨﻔﺘﺭﺽ ﺃﻥ ﻜﻤﻴـﺔ ﺍﻟﺤﺭﻜـﺔ ﻟﻠﻔﻭﺘـﻭﻥ‬

‫‪- ١٩ -‬‬
‫‪PDF created with pdfFactory Pro trial version www.pdffactory.com‬‬
‫ﺍﻹﻟﻜﺘﺭﻭﻥ ﻓﻴﺤﺩﺙ ﻟﻪ ﺍﺭﺘﺩﺍﺩ ‪ recoil‬ﺒﻜﻤﻴﺔ ﺤﺭﻜﺔ ‪ . p‬ﻭﺒﺘﻁﺒﻴﻕ ﻗﺎﻨﻭﻥ ﺒﻘﺎﺀ ﻜﻤﻴـﺔ‬
‫‪e‬‬

‫ﺍﻟﺤﺭﻜﺔ )ﺍﻨﻅﺭ ﺸﻜل ‪.(1-7‬‬


‫‪pe = p ° + p‬‬ ‫)‪(1-13‬‬

‫ﺒﺘﺭﺒﻴﻊ ﻁﺭﻓﻲ ﺍﻟﻤﻌﺎﺩﻟﺔ‪.‬‬

‫‪2‬‬ ‫‪2‬‬ ‫‪r r‬‬


‫‪p e = p ° − p − 2p ° p‬‬
‫‪2‬‬
‫)‪(1-14‬‬

‫ﻨﻁﺒﻕ ﺍﻵﻥ ﻗﺎﻨﻭﻥ ﺒﻘﺎﺀ ﺍﻟﻁﺎﻗﺔ‬


‫[‬
‫‪E = (m ° c 2 ) 2 + (pc) 2‬‬ ‫ﺃﻭﻻﹰ‪ ،‬ﻁﺎﻗﺔ ﺍﻹﻟﻜﺘﺭﻭﻥ ‪ E‬ﻗﺒل ﺍﻟﺘﺼﺎﺩﻡ ]‬
‫‪1‬‬
‫‪2‬‬

‫ﻷﻥ ﺍﻹﻟﻜﺘﺭﻭﻥ ﺴـﺎﻜﻥ ﻗﺒـل ﺍﻟﺘـﺼﺎﺩﻡ )ﻷﻥ ﺃﻥ ‪P‬‬ ‫‪m° = 0‬‬ ‫ﺒﺎﻟﺘﻌﻭﻴﺽ ﻋﻥ‬
‫ﺘﺴﺎﻭﻱ ﺼﻔﺭ(‪.‬‬
‫‪∴E = m ° c = mc 2‬‬ ‫)‪(1-15‬‬

‫ﻗﺎﻨﻭﻥ ﺒﻘﺎﺀ ﺍﻟﻁﺎﻗﺔ‪ :‬ﻁﺎﻗﺔ ﺍﻹﻟﻜﺘﺭﻭﻥ ﻭﺍﻟﻔﻭﺘﻭﻥ ﺍﻟﺴﺎﻗﻁ ﻗﺒل ﺍﻟﺘﺼﺎﺩﻡ = ﻁﺎﻗﺔ‬
‫ﺍﻹﻟﻜﺘﺭﻭﻥ ﻭﺍﻟﻔﻭﺘﻭﻥ ﺒﻌﺩ ﺍﻟﺘﺼﺎﺩﻡ‬
‫‪1‬‬
‫) ‪hv ° + mc 2 = hv + (m 2 c 4 + p e c 2‬‬
‫‪2‬‬ ‫‪2‬‬

‫‪- ٢٠ -‬‬
‫‪PDF created with pdfFactory Pro trial version www.pdffactory.com‬‬
‫ﺒﻨﻘل ‪ hv‬ﻟﻠﻁﺭﻑ ﺍﻷﻴﺴﺭ ﻭﺘﺭﺒﻴﻊ ﻁﺭﻓﻲ ﺍﻟﻤﻌﺎﺩﻟﺔ‬
‫) ‪(hv ° + mc 2 − hv) 2 = (m 2c 4 + pe c 2‬‬
‫‪2‬‬

‫‪or‬‬
‫‪m 2c 4 + pe c 2 = (hv ° − hv + mc 2 )2‬‬
‫‪2‬‬

‫‪= (hν ° − hv) 2 + 2m c 2 (hv ° − hv) + m 2c 4‬‬

‫)‪(1-16‬‬
‫ﻤﻥ ﻤﻌﺎﺩﻟﺔ )‪ ،(1-14‬ﺒﺎﻟﺘﻌﻭﻴﺽ ﻋﻥ‬
‫‪hv‬‬ ‫‪hv‬‬
‫=‪p‬‬ ‫‪, p° = °‬‬
‫‪c‬‬ ‫‪c‬‬
‫‪hv ° 2 hv 2‬‬ ‫‪hv hv‬‬
‫( = ‪∴p e‬‬ ‫‪) + ( ) − 2 ( ° ) ( ) cosθ‬‬
‫‪2‬‬

‫‪c‬‬ ‫‪c‬‬ ‫‪c‬‬ ‫‪c‬‬

‫ﻫﻲ ﺍﻟﺯﺍﻭﻴﺔ ﺍﻟﻤﺤﺼﻭﺭﺓ ﺒﻴﻥ ﺍﺘﺠﺎﻩ ﺍﻟـﺸﻌﺎﻉ ﺍﻟﻤـﺸﺘﺕ ﻭﺍﻟـﺸﻌﺎﻉ‬ ‫‪θ‬‬ ‫ﺤﻴﺙ‬
‫ﺍﻟﺴﺎﻗﻁ‪ .‬ﺒﻀﺭﺏ ﻁﺭﻓﻲ ﺍﻟﻤﻌﺎﺩﻟﺔ ﻓﻲ ‪C2‬‬
‫‪pe c 2 = (hv° )2 + (hv)2 − 2 (hv ° ) (hv) cosθ‬‬
‫‪2‬‬
‫)‪(1-17‬‬

‫ﻭﻟﻠﺤﺼﻭل ﻋﻠﻰ ﻤﺭﺒﻊ ﻜﺎﻤل‪ ،‬ﻨﻀﻴﻑ ﻭﻨﻁﺭﺡ ]‪ [2hv° hv‬ﻟﻤﻌﺎﺩﻟﺔ )‪(1-17‬‬


‫‪pe c 2 (hv ° ) 2 + (hv) 2 − 2hv °hv + 2(hv ° ) (hv) − 2(hv° ) (hv) cosθ‬‬
‫‪2‬‬

‫‪1444‬‬ ‫‪424444‬‬ ‫‪3 14444‬‬ ‫‪4244444‬‬ ‫‪3‬‬


‫‪(hv ° − hv) 2‬‬ ‫)‪2hv ° hv (1− cos θ‬‬

‫)‪∴pe c 2 = (hv° − hv) 2 + 2hv °hv (1 − cos θ‬‬


‫‪2‬‬
‫)‪(1-18‬‬

‫ﻤﻥ ﻤﻌﺎﺩﻟﺔ )‪ (1-18‬ﻓﻲ ﻤﻌﺎﺩﻟﺔ )‪ (1-18‬ﻨﺘﺤـﺼل‬ ‫‪(hv ° − hv) 2‬‬ ‫ﺒﺎﻟﺘﻌﻭﻴﺽ ﻋﻥ ﻗﻴﻤﺔ‬
‫ﻋﻠﻰ‬

‫‪m 2 c 4 + pe c 2 = p e c 2 − 2hv °hv (1 − cos θ) + 2mc 2 (hv ° − hv) + m 2c 4‬‬


‫‪2‬‬ ‫‪2‬‬

‫)‪ ..(1-19‬ﺒﺤﺫﻑ ﺍﻟﺤﺩﻭﺩ ﺍﻟﻤﺘﺸﺎﺒﻬﺔ ﻤﻥ ﻁﺭﻓﻲ ﺍﻟﻤﻌﺎﺩﻟﺔ‪:‬‬

‫‪- ٢١ -‬‬
‫‪PDF created with pdfFactory Pro trial version www.pdffactory.com‬‬
‫‪2hv °hv (1 − cos θ) = 2mc 2 (hv ° − hv) −‬‬
‫)‪= 2mc 2 h (v° − v‬‬
‫‪c c‬‬ ‫‪1 1‬‬
‫( ‪= 2mc 2 h‬‬ ‫) ‪− ) 2mc 2 hc ( −‬‬
‫‪λ° λ‬‬ ‫‪λ° λ‬‬
‫‪c c‬‬ ‫‪λ − λ°‬‬
‫‪2h‬‬ ‫‪h (1 − cos θ) = 2mc 2 h‬‬
‫‪λ° λ‬‬ ‫‪λ°λ‬‬
‫‪or‬‬
‫‪2/‬‬ ‫‪c/ 2‬‬ ‫‪λ − λ°‬‬
‫‪2/ h‬‬ ‫‪(1 − cos θ) 2/ mc 3/ h/‬‬
‫‪λ/ °λ/‬‬ ‫‪λ °λ‬‬
‫) ‪h (1 − cos θ) = mc (λ − λ °‬‬

‫= ‪or λ − λ °‬‬
‫‪h‬‬
‫)‪(1 − cos θ‬‬ ‫)‪...(1-20‬‬
‫‪cm‬‬

‫ﻓﻲ ﻤﻌﺎﺩﻟﺔ )‪ (1-20‬ﻟﻪ ﺒ‪‬ﻌﺩ ﺍﻟﻁﻭل ﻭﻫﺫﺍ ﺍﻟﺤﺩ ﻴ‪‬ـﺴﻤﻰ ﻁـﻭل‬ ‫‪h‬‬
‫ﻻﺤﻅ ﺃﻥ‬
‫‪mc‬‬

‫ﻤﻭﺠﺔ ﻜﻭﻤﺒﺘﻭﻥ ‪ Compton wavelength‬ﻟﻼﻟﻜﺘﺭﻭﻥ ﻭﻤﻘﺩﺍﺭﻩ‬


‫‪h‬‬
‫‪≅ 2.4 × 10 −10 cm‬‬ ‫)‪(1-21‬‬
‫‪mc‬‬

‫‪λ‬‬ ‫ﻭﻗﺩ ﺘﺒﻴﻥ ﻤﻥ ﺍﻟﻘﻴﺎﺴﺎﺕ ﺍﻟﻤﻌﻤﻠﻴﺔ ﺃﻥ ﻁﻭل ﻤﻭﺠﺔ ﺍﻟﻔﻭﺘـﻭﻥ ﻭﺍﻟﻤﺘـﺸﺘﺕ‬


‫ﻭﺍﻟـﺫﻱ‬ ‫)ﺍﻨﻅﺭ ﺸﻜل‬ ‫‪λ‬‬ ‫ﺘﺘﻁﺎﺒﻕ ﻤﻊ ﺍﻟﻘﻴﻤﺔ ﺍﻟﻨﻅﺭﻴﺔ‪ .‬ﺃﻤ‪‬ﺎ ﺍﻟﻤﺭﻜﺒﺔ ﺍﻟﺜﺎﻨﻴﺔ ﻟـ‬
‫ﻭﺍﻷﺨﺭﻯ ﻤﺴﺎﻭﻴﺔ ﻟـ ‪ ( λ‬ﻭﺍﻟﺘﻲ ﻻ‬
‫‪°‬‬
‫‪λ°‬‬ ‫ﺇﺤﺩﺍﻫﻤﺎ ﺘﺨﺘﻠﻑ ﻋﻥ‬ ‫‪λ‬‬ ‫ﻴﺒﻴﻥ ﻤﺭﻜﺒﺘﻴﻥ ﻟـ‬
‫ﺘﺘﻐﻴﺭ ﺒﺎﻟﻨﺴﺒﺔ ﻓﺈﻥ ﻤﻨﺸﺄﻫﺎ ﻫﻭ ﺘﺼﺎﺩﻡ ﺍﻟﻔﻭﺘﻭﻥ ﺍﻟﺴﺎﻗﻁ ﻤﻊ ﺍﻟﺫﺭﺓ ﻜﻜل‪ ،‬ﻓﻠﻭ ﻋﻭﻀﻨﺎ‬
‫ﻋﻥ ‪ m‬ﺒﻜﺘﻠﺔ ﺍﻟﺫﺭﺓ )ﺒﺩﻻﹰ ﻤﻥ ﻜﺘﻠﺔ ﺍﻹﻟﻜﺘﺭﻭﻥ( ﻭﺤﻴﺙ ﺃﻥ ﻫﺫﻩ ﺍﻟﻘﻴﻤﺔ ﻓـﻲ ﺍﻟﻤﻘـﺎﻡ‬
‫ﺴﺘﻜﻭﻥ ﻗﻴﻤﺘﻪ ﺼـﻐﻴﺭﺓ‬ ‫‪h‬‬
‫)ﻭﻫﻲ ﻜﺒﻴﺭﺓ ﺠﺩﺍﹰ ﺒﺎﻟﻨﺴﺒﺔ ﻟﻜﺘﻠﺔ ﺍﻹﻟﻜﺘﺭﻭﻥ( ﻓﺈﻥ ﺍﻟﺤﺩ‬
‫‪mc‬‬
‫ﻭﺃﺨﻴـﺭﺍﹰ ﻤـﺎ ﺍﻟـﺫﻱ‬ ‫‪λ ≈ λ°‬‬ ‫ﻭﻫﺫﺍ ﻴﻌﻨﻲ‬ ‫‪λ − λ° ≅ ο‬‬ ‫ﺠﺩﺍﹰ ﻗﺭﻴﺒﺔ ﻤﻥ ﺍﻟﺼﻔﺭ‪ ،‬ﺃﻱ ﺃﻥ‬
‫ﻨﺴﺘﻨﺘﺠﻪ ﻤﻥ ﺘﺄﺜﻴﺭ ﻜﻭﻤﺒﺘﻭﻥ؟‬
‫ﺇﻥ ﺍﻟﻘﻴﺎﺴﺎﺕ ﺍﻟﺘﻲ ﺃﺠﺭﻴﺕ ﻋﻠﻰ ﺍﻹﻟﻜﺘﺭﻭﻥ ﺍﻟﻤﺭﺘﺩ ﻭﺍﻟﻔﻭﺘﻭﻥ ﺍﻟﻤﺘﺒﻌﺜﺭ ﻤﻨـﻪ‬
‫ﺘﺅﻜﺩ‪ -‬ﺒﻤﺎ ﻻ ﻴﺩﻉ ﻤﺠﺎﻻﹰ ﻟﻠﺸﻙ – ﺒﺄﻥ ﻫﺫﺍ ﺍﻟﺘﺼﺎﺩﻡ ﻤﻤﺎﺜل ﻟﻠﺘﺼﺎﺩﻡ ﺍﻟﺫﻱ ﻴﺤـﺩﺙ‬

‫‪- ٢٢ -‬‬
‫‪PDF created with pdfFactory Pro trial version www.pdffactory.com‬‬
‫ﺒﻴﻥ ﻜﹸﺭﺘﻲ ﺒﻠﻴﺎﺭﺩﻭ‪ ،‬ﺃﻱ ﺃﻥ ﺍﻟﻔﻭﺘﻭﻥ )ﺃﻭ ﺍﻟﺸﻌﺎﻉ ﺍﻟﺴﺎﻗﻁ( ﻴﺠﺏ ﺃﻥ ﺘﺘﻌﺎﻤـل ﻤﻌـﻪ‬
‫ﻋﻠﻰ ﺃﺴﺎﺱ ﺃﻨﻪ ﺠﺴﻴﻡ‪ ،‬ﻭﻫﺫﺍ ﻴﺅﻜﺩ ﺍﻟﻁﺒﻴﻌﺔ ﺍﻟﺠﺴﻴﻤﻴﺔ ﻟﻺﺸﻌﺎﻉ‪.‬‬
‫‪ 1-5‬ﺍﻟﺨﺼﺎﺌﺹ ﺍﻟﻤﻭﺠﻴﺔ ﻟﻠﻤﺎﺩﺓ ﻭﺤﻴﻭﺩ ﺍﻹﻟﻜﺘﺭﻭﻥ‬
‫‪1-5 Wave Prosperities and Electron Diffraction‬‬
‫ﻭﺍﺠﻪ ﺍﻟﻌﻠﻤﺎﺀ ﻜﺜﻴﺭﺍﹰ ﻤﻥ ﺍﻟﺼﻌﻭﺒﺎﺕ ﻓﻲ ﻭﺼﻑ ﻁﺒﻴﻌـﺔ ﺍﻟـﻀﻭﺀ‪ .‬ﺒﻌـﺽ‬
‫ﺍﻟﺘﺠﺎﺭﺏ ﺘﺒﻴﻥ ﺍﻟﻁﺒﻴﻌﺔ ﺍﻟﻤﻭﺠﻴﺔ ﻟﻠﻀﻭﺀ )ﻤﺜـل؟ ‪ dispersion‬ﺍﻟـﻀﻭﺀ ﺍﻷﺒـﻴﺽ‬
‫ﻟﻤﺭﻜﺒﺎﺕ ﻁﻴﻔﻪ ﻋﻨﺩ ﻤﺭﻭﺭﻩ ﺩﺍﺨل ﻤﻨﺸﻭﺭ ‪ (prism‬ﻭﺍﻟﺒﻌﺽ ﺍﻵﺨﺭ ﻤﻥ ﺍﻟﺘﺠـﺎﺭﺏ‬
‫ﻴﺜﺒﺕ ﺍﻟﻁﺒﻴﻌﺔ ﺍﻟﺠﺴﻴﻤﻴﺔ ﻟﻠﻀﻭﺀ )ﻤﺜل ﺍﻟﺘﺄﺜﻴﺭ ﺍﻟﻜﻬﺭﻭﻀﻭﺌﻲ(‪..‬‬

‫ﻭﻫﺫﺍ ﻤﺎ ﻴ‪‬ﻌﺭﻑ ﺒﺎﻟﻁﺒﻴﻌﺔ ﺍﻟﻤﺯﺩﻭﺠﺔ ﻟﻠـﻀﻭﺀ ‪ware- particle duality of‬‬


‫‪.lights‬‬
‫ﻓﻲ ﻋﺎﻡ ‪ ،1924‬ﻗﺩ‪‬ﻡ ﻋﺎﻟﻡ ﻓﺭﻨﺴﻲ ﻴﺩﻋﻰ ﺩﻴﺒﺭﻭﻟـﻲ ‪– Louis de Broglie‬‬
‫ﻭﺒﻨﺎﺀﺍﹰ ﻋﻠﻰ ﻤﺎ ﺴﺒﻕ ﺫﻜﺭﻩ ﺒﺎﻟﻨﺴﺒﺔ ﻟﻠﻀﻭﺀ‪ -‬ﻨﻤﻭﺫﺠﺎﹰ ﻴﺒﻴﻥ ﻓﻴﻪ ﺍﻟﻁﺒﻴﻌـﺔ ﺍﻟﻤﻭﺠﻴـﺔ‬
‫ﻟﻠﻤﺎﺩﺓ؛ ﻓﺈﺫﺍ ﻜﺎﻥ ﺍﻟﻀﻭﺀ‪ -‬ﺫﻭ ﺍﻟﻁﺒﻴﻌﺔ ﺍﻟﻤﻭﺠﻴﺔ – ﻴﺴﻠﻙ ﺃﺤﻴﺎﻨﺎﹰ ﻜﻤـﺎ ﻟـﻭ ﻜـﺎﻥ‬
‫ﺠﺴﻴﻤﺎﺕ‪ ،‬ﻓﻠﻤﺎﺫﺍ ﻻ ﻴﻜﻭﻥ ﻟﻠﻤﺎﺩﺓ ﻁﺒﻴﻌﺔ ﻤﻭﺠﻴﺔ؟!! ﻭﻗﺩ ﺼﺎﻍ ﺩﻴﺒﺭﻭﻟﻲ ﻓﻜﺭﺘﻪ ﻫﺫﻩ‬
‫ﺒﺼﻴﻐﺔ ﺭﻴﺎﻀﻴﺔ‪ -‬ﺍﻗﺘﺒﺴﻬﺎ ﻤﻥ ﻋﻼﻗﺔ ﺍﻴﻨﺸﺘﺎﻴﻥ ﺍﻟﺘﻲ ﺘﺭﺘﺒﻁ ﺒـﻴﻥ ﻁـﻭل ﻤﻭﺠـﺔ‬
‫‪ ،‬ﻓﺈﺫﺍ ﻜﺎﻥ ﺠﺴﻴﻡ ﻜﺘﻠﺘﻪ ‪ m‬ﻭﺴﺭﻋﺘﻪ ‪ v‬ﻓﺈﻥ ﻜﻤﻴﺔ‬ ‫)‪(١‬‬
‫ﻭﻜﻤﻴﺔ ﺤﺭﻜﺘﻪ ‪P‬‬ ‫‪λ‬‬ ‫ﺍﻟﻔﻭﺘﻭﻥ‬
‫ﻋﻨﺩﺌﺫ ﻓﺈﻥ ﺍﻟﺠﺴﻴﻡ – ﻭﺤﺴﺏ ﻨﻤﻭﺫﺝ ﺩﻴﺒﺭﻭﻟﻲ – ﺴﻴﻜﻭﻥ ﻟﻪ ﻁـﻭل‬ ‫‪p = mv‬‬ ‫ﺤﺭﻜﺘﻪ‬
‫ﻴﻌﻁﻲ ﺒﺎﻟﻌﻼﻗﺔ‪.‬‬ ‫‪λ‬‬ ‫ﻤﻭﺠﻲ‬
‫‪λ‬‬
‫‪h‬‬
‫)‪...(1-22‬‬
‫‪p‬‬

‫)‪ (١‬ﺍﻟﻌﻼﻗﺔ ﺍﻟﺘﻲ ﻗﺩﻤﻬﺎ ﺍﻴﻨﺸﺘﺎﻴﻥ ﻟﻠﻔﻭﺘﻭﻥ ﺍﻟﺫﻱ ﺘﺭﺩﺩﻩ ‪ v‬ﻭﻁﻭل ﻤﻭﺠﺘﻪ ‪ λ‬ﻭﻜﻤﻴﺔ ﺤﺭﻜﺘﻪ ‪p‬‬
‫‪c hc hc h h‬‬
‫=‪λ‬‬ ‫= = =‬ ‫=‬
‫‪v hv E E p‬‬
‫) (‬
‫‪c‬‬

‫‪- ٢٣ -‬‬
‫‪PDF created with pdfFactory Pro trial version www.pdffactory.com‬‬
‫ﺤﻴﺙ ‪ h‬ﻫﻭ ﺜﺎﺒﺕ ﺒﻼﻨﻙ‪.‬‬

‫ﻤﺜﺎل ‪:5‬‬
‫ﺍﺤﺴﺏ ﺍﻟﻁﻭل ﺍﻟﻤﻭﺠﻲ ﻟﻜﺭﺓ ﻜﺘﻠﺘﻬﺎ ‪ 0.14 kg‬ﻭﺴﺭﻋﺘﻬﺎ ‪ 40 m/s‬ﻭﻗﺎﺭﻥ ﻫﺫﺍ‬
‫ﺍﻟﻁﻭل ﺍﻟﻤﻭﺠﻲ ﻤﻊ ﻁﻭل ﻤﻭﺠﺔ ﺇﻟﻜﺘﺭﻭﻥ ﺴﺭﻋﺘﻪ ‪ 1.00%‬ﻤﻥ ﺴﺭﻋﺔ ﺍﻟﻀﻭﺀ؟‬
‫ﺍﻟﺤل‪:‬‬

‫ﻨﻭﺠﺩ ﺃﻭﻻﹰ ﻜﻤﻴﺔ ﺍﻟﺤﺭﻜﺔ ﻟﻠﻜﺭﺓ‬


‫)‪p = mco = (0.14kg) (40m/s‬‬
‫‪= 5.6 kg.m.s -1‬‬

‫ﺤﺴﺏ ﻤﻌﺎﺩﻟﺔ ﺩﻴﺒﺭﻭﻟﻲ‬ ‫‪λ‬‬ ‫ﺍﻟﻁﻭل ﺍﻟﻤﻭﺠﻲ‬


‫‪h 6.63×10 −34 J.s‬‬
‫= =‪λ‬‬ ‫‪−1‬‬
‫‪= 1.2 × 10 −34 m‬‬ ‫!!!‬
‫‪p‬‬ ‫‪5.6 kg ms‬‬

‫ﻻﺤﻅ ﺃﻥ ﻫﺫﺍ ﺍﻟﻁﻭل ﺍﻟﻤﻭﺠﻲ ﻤﺘﻨﺎﻫﻲ ﻓﻲ ﺍﻟﺼﻐﺭ‬

‫ﻨﻭﺠﺩ ﺍﻵﻥ ﻜﻤﻴﺔ ﺤﺭﻜﺔ ﺍﻹﻟﻜﺘﺭﻭﻥ‬

‫) ‪p = m e v = (9.1× 10 −31 kg) (2.998× 10 6 ms −1‬‬

‫)ﻻﺤﻅ ﺃﻥ ﺍﻟﺴﺭﻋﺔ ‪ v‬ﻟﻺﻟﻜﺘﺭﻭﻥ ﻫﻲ ‪ 1%‬ﻤﻥ ﺴﺭﻋﺔ ﺍﻟـﻀﻭﺀ ﻜﻤـﺎ ﻫـﻭ‬


‫ﻤﻌﻁﻰ ﻓﻲ ﺍﻟﺴﺅﺍل(‪.‬‬

‫‪∴ p = 2.73× 10 −24 kg.m.s −1‬‬

‫ﻁﻭل ﻤﻭﺠﺔ ﺩﻴﺒﺭﻭﻟﻲ ﻟﻺﻟﻜﺘﺭﻭﻥ‪.‬‬


‫‪h‬‬ ‫‪6.63 ×10 −34 J.s‬‬
‫= =‪λ‬‬ ‫‪= 2.43× 10 −10 m‬‬
‫‪p 2.73× 10 − 24 kg.m.s −1‬‬
‫‪= 243 pm‬‬

‫‪ ..(10‬ﻭﻫﺫﻩ ﺍﻟﻘﻴﻤﺔ ‪ 243‬ﻤﻘﺎﺭﻨﺔ ﻟﻸﺒﻌﺎﺩ ﺍﻟﺫﺭﻴﺔ‪..‬‬ ‫‪−12‬‬


‫‪m‬‬ ‫)ﺤﻴﺙ ‪ pm‬ﻴﺴﺎﻭﻱ‬

‫‪- ٢٤ -‬‬
‫‪PDF created with pdfFactory Pro trial version www.pdffactory.com‬‬
‫ﻤﻥ ﺨﻼل ﻫﺫﻩ ﺍﻟﻘﻴﻡ‪ ،‬ﻴﺘﻀﺢ ﻟﻨﺎ ﺃﻥ ﻁﻭل ﻤﻭﺠﺔ ﺩﻴﺒﺭﻭﻟﻲ ﻟﻺﻟﻜﺘﺭﻭﻥ ﻤﻘﺎﺭﺒﺔ‬
‫ﻟﻁﻭل ﻤﻭﺠﺔ ﺍﻷﺸﻌﺔ ﺍﻟﺴﻴﻨﻴﺔ‪ .‬ﻭﻫﺫﺍ ﻴﻌﻨﻲ ﺃﻥ ﺍﻹﻟﻜﺘﺭﻭﻥ ﺴﻴﺴﻠﻙ ﻜﻤﺎ ﻟﻭ ﻜﺎﻥ ﺃﺸﻌﺔ‬
‫ﺴﻴﻨﻴﺔ!!‪.‬‬
‫ﺃﻤﺎ ﺒﺎﻟﻨﺴﺒﺔ ﻟﻠﻜﺭﺓ ﻓﺈﻥ ﻁﻭل ﻤﻭﺠﺔ ﺩﻴﺒﺭﻭﻟﻲ ﻟﻬﺎ ﻗﺼﻴﺭ ﺠﺩﺍﹰ ﻤﻘﺎﺭﻨﺔ ﺒﺎﻷﺒﻌﺎﺩ‬
‫ﺍﻟﺫﺭﻴﺔ‪...‬‬
‫ﻤﻭﺠﺎﺕ ﺩﻴﺒﺭﻭﻟﻲ ﻴﻤﻜﻥ ﻤﺸﺎﻫﺩﺘﻬﺎ ﺘﺠﺭﻴﺒﻴﺎﹰ‬
‫‪de Broglie Waves are observed Experimentally‬‬
‫ﻋﻨﺩ ﺍﺼﻁﺩﺍﻡ ﺃﺸﻌﺔ ‪ X‬ﺒﻤﺎﺩﺓ ﺒﻠﻭﺭﻴﺔ ﻓﺈﻨﻪ ﻴﺘﺸﺘﺕ ﺒﻁﺭﻴﻘـﺔ ﻤﻌﻴﻨـﺔ ﺘﻌﻜـﺱ‬
‫ﻁﺒﻴﻌﺔ ﺍﻟﺘﺭﺘﻴﺏ ﺍﻟﺩﻭﺭﻱ ﺍﻟﺒﻠﻭﺭﻱ ﻟﻠﻤﺎﺩﺓ ﻫﺫﻩ ﺍﻟﻅﺎﻫﺭﺓ ﺘﹲﻌﺭﻑ ﺒﺤﻴـﻭﺩ ﺃﺸـﻌﺔ ‪.X‬‬
‫‪ X-ray diffraction‬ﻭﺴﺒﺏ ﺤﺩﻭﺙ ﻫﺫﻩ ﺍﻟﻅﺎﻫﺭﺓ ﻫﻭ ﺃﻥ ﺍﻷﺒﻌﺎﺩ ﺍﻟﺒﻠﻭﺭﻴﺔ )ﺍﻟﻤﺴﺎﻓﺔ‬
‫ﺒﻴﻥ ﺍﻟﻤﺴﺘﻭﻴﺎﺕ ﺍﻟﺒﻠﻭﺭﻴﺔ( ﻤﺘﻘﺎﺭﺒﺔ ﻤﻊ ﺍﻟﻁﻭل ﺍﻟﻤﻭﺠﻲ ﻷﺸﻌﺔ ‪.X‬‬

‫ﻭﻤﻥ ﺍﻟﻨﺎﺤﻴﺔ ﺍﻟﺘﺠﺭﻴﺒﻴﺔ‪ ،‬ﻓﻘﺩ ﻭ‪‬ﺠﺩ ﺃﻥ ﺘﺒﻌﺜﺭ ﺸﻌﺎﻉ ﻤﻥ ﺍﻹﻟﻜﺘﺭﻭﻨﺎﺕ ﺒﻤـﺎﺩﺓ‬


‫ﺒﻠﻭﺭﻴﺔ ﻴﻨﺸﺄ ﻋﻨﻪ ﺤﻴﻭﺩ ﻤﺸﺎﺒﻪ ﻟﺤﻴﻭﺩ ﺃﺸﻌﺔ ‪ X‬ﻤﻤﺎ ﺒﻴﻥ‪ -‬ﺒﻤﺎ ﻻ ﻴﺩﻉ ﻤﺠﺎﻻﹰ ﻟﻠﺸﻙ‪-‬‬
‫ﺍﻟﻁﺒﻴﻌﺔ ﺍﻟﻤﻭﺠﻴﺔ ﻟﻠﺠﺴﻴﻤﺎﺕ‪.‬‬
‫ﺸﻜل ‪ 1-8‬ﻴﻭﻀﺢ ﺘﺸﺘﺕ ﻤﻭﺠـﺎﺕ ﻨﺘﻴﺠـﺔ ﺍﺼـﻁﺩﺍﻤﻬﺎ ﺒﺘﺭﻜﻴـﺏ ﺩﻭﺭﻱ‬
‫‪) periodic Structure‬ﻤﺜﻼﹰ ﺸﻜل ﺒﻠﻭﺭﻱ(‪ .‬ﻭﻤﻥ ﺍﻟﺸﻜل ﻴﺘﻀﺢ ﺃﻨﻪ ﺴﻴﻜﻭﻥ ﻓـﺭﻕ‬
‫ﻓﻲ ﺍﻟﻁﻭﺭ ‪ phase difference‬ﺒﻴﻥ ﺍﻟﻤﻭﺠﺎﺕ ﺍﻟﻤﻨﻌﻜﺴﺔ ﻤﻥ ﻤـﺴﺘﻭﻴﺎﺕ ﺒﻠﻭﺭﻴـﺔ‬
‫ﻤﺘﺠﺎﻭﺭﺓ ﻭﻤﻘﺩﺍﺭ ﻓﺭﻕ ﺍﻟﻁﻭﺭ ‪. ( 2D ) 2a sin θ‬‬
‫‪λ‬‬

‫)ﺤﻴﺙ ‪ n‬ﻫﻭ ﺭﻗﻡ ﺼـﺤﻴﺢ( ﻓـﺴﻴﺘﺤﻘﻕ‬ ‫‪2 Dn‬‬ ‫ﻭﻋﻨﺩﻤﺎ ﻴﺴﺎﻭﻱ ﻓﺭﻕ ﺍﻟﻁﻭﺭ‬
‫ﺸﺭﻁ ﺍﻟﺘﺩﺍﺨل ﺍﻟﺒ‪‬ﻨﱠﺎﺀ‪ ،‬ﺃﻱ ﺃﻥ‪.‬‬
‫‪2D‬‬
‫‪2a sinθ = 2D n‬‬
‫‪λ‬‬

‫‪- ٢٥ -‬‬
‫‪PDF created with pdfFactory Pro trial version www.pdffactory.com‬‬
Figure 1-8 the scattering of X-rays by crystal planes when the
angle 0 between the scattered X-rays and the crystal plane equals the
angle between the incident X-rays and the plane.
λ=
2a sin θ
... (1-23)
n

‫ﺇﻥ ﺸﻜل ﺍﻟﺘﺩﺍﺨل ﺍﻟﻤﺸﺎﻫﺩ ﺘﺠﺭﻴﺒﻴﺎﹰ ﻤﻥ ﺘﺸﺘﺕ ﺍﻹﻟﻜﺘﺭﻭﻨﺎﺕ ﻨﺘﻴﺠﺔ ﺍﺴـﺘﺩﺍﻤﻬﺎ‬


(Davisson and ‫ﺒﺒﻠﻭﺭﺓ )ﻜﻤﺎ ﺘﺤﻘﻕ ﻋﻠﻰ ﻴﺩﻱ ﺍﻟﻌـﺎﻟﻤﻴﻥ ﺩﺍﻓﻴـﺴﻭﻥ ﻭ ﺠﻴﺭﻤـﺭ‬
(1- ‫ﺘﻌﻁﻲ ﺒﺎﻟﻤﻌﺎﺩﻟﺔ‬ λ ‫( ﻭﺫﻟﻙ ﺒﺸﺭﻁ ﺃﻥ‬1-23) ‫ ﻴﻤﻜﻥ ﺘﻔﺴﻴﺭﻩ ﺒﺎﻟﻤﻌﺎﺩﻟﺔ‬Germer
‫ ﺇﻥ ﻫﺫﻩ ﺍﻟﻨﺘﻴﺠﺔ ﺍﻟﺘﺠﺭﻴﺒﻴﺔ ﺴﺎﻫﻤﺕ ﺒﺨﻁـﻭﺓ‬.1-9 ‫ ﻜﻤﺎ ﻫﻭ ﻤﻭﻀﺢ ﻓﻲ ﺸﻜل‬22)
.Wave mechanics ‫ﻜﺒﻴﺭﺓ ﻓﻲ ﺘﻁﻭﻴﺭ ﻋﻠﻡ ﻤﻴﻜﺎﻨﻴﻜﺎ ﺍﻟﻤﻭﺠﺎﺕ‬

FIGURE 1.9
(a) the X-ray diffraction pattern of aluminum foil. (b) the electron
diffraction pattern of aluminum foil. The similarity of these two patterns
shows that electrons can behave like X-rays and display wavelike
properties.

- ٢٦ -
PDF created with pdfFactory Pro trial version www.pdffactory.com
‫‪ 1-6‬ﺫﺭﺓ ﺒﻭﺭ‬
‫‪1-6 The Bohr Atom‬‬
‫‪"The Bohr Theory of the Hydrogen Atom Can be Used‬‬
‫"‪Drive the Rydberg For mole‬‬
‫ﻓﻲ ﻋﺎﻡ ‪ ،1911‬ﻗﺩﻡ ﺍﻟﻌﺎﻟﻡ ﺍﻟﺩﻨﻤﺎﺭﻜﻲ ﻨﻴﻠﺱ ﺒـﻭﺭ ‪ Niels Bohr‬ﻨﻅﺭﻴﺘـﻪ‬
‫ﺍﻟﺸﻬﻴﺭﺓ ﻟﺫﺭﺓ ﺍﻟﻬﻴﺩﺭﻭﺠﻴﻥ ﻭﺍﻟﺘﻲ ﺘﺸﺭﺡ ﻭﺘﻔﺴﺭ ﺒﺒﺴﺎﻁﺔ ﺍﻟﻁﻴـﻑ ﺍﻟﻤﻨﺒﻌـﺙ ﻤـﻥ‬
‫ﺍﻟﺫﺭﺓ‪.‬‬
‫ﻁﺒﻘﺎﹰ ﻟﻠﻨﻤﻭﺫﺝ ﺍﻟﻨﻭﻭﻱ ﻟﻠﺫﺭﺓ‪ ،‬ﻭﺍﻟﺫﻱ ﻴﻘﻭﻡ ﻋﻠﻰ ﺍﻟﻨﺘﺎﺌﺞ ﺍﻟﺘﺠﺭﻴﺒﻴﺔ ﻟﺘﻁـﺎﻴﺭ‬
‫ﺠﺴﻴﻤﺎﺕ ‪ ، α‬ﻴﻤﻜﻥ ﺍﻋﺘﺒﺎﺭ ﺃﻥ ﻜﺘﻠﺔ ﺍﻟﺫﺭﺓ ﻤﺘﺭﻜﺯﺓ ﻓﻲ ﺍﻟﻨﻭﺍﺓ ﻭﺍﻟﺘﻲ ﺘﻌﺘﺒـﺭ ﺜﺎﺒﺘـﺔ‬
‫ﻭﻴﺩﻭﺭ ﺤﻭﻟﻬﺎ ﺇﻟﻜﺘﺭﻭﻥ‪ .‬ﺍﻟﻘﻭﺓ ‪ F‬ﺍﻟﺘﻲ ﻴﺭﺘﺒﻁ ﺒﻬﺎ ﺍﻹﻟﻜﺘﺭﻭﻥ ﻓﻲ ﻤﺩﺍﺭ ﺩﺍﺌﺭﻱ ﻫـﻲ‬
‫ﻗﻭﺓ ﻜﻭﻟﻭﻡ ﺤﺴﺏ ﻗﺎﻨﻭﻥ ﻜﻭﻟﻭﻡ‪.‬‬
‫=‪F‬‬
‫)‪1 (Ze) (e‬‬
‫)‪... (1-24‬‬
‫‪4Dε ° r 2‬‬

‫ﺤﻴﺙ )‪ (Ze‬ﻫﻲ ﺸﺤﻨﺔ ﺍﻟﻨﻭﺍﺓ )ﻟﺫﺭﺓ ﺍﻟﻬﻴﺩﺭﻭﺠﻴﻥ ‪ ( Z =1‬ﻭ )‪ (e‬ﻫﻲ ﺸﺤﻨﺔ‬


‫ﺍﻹﻟﻜﺘﺭﻭﻥ‪.‬‬
‫ﻨﺼﻑ ﻗﻁﺭ ﺍﻟﺫﺭﺓ‪ .‬ﺘﺘﻭﺍﺯﻥ ﻗﻭﺓ ﻜﻭﻟﻭﻡ ﻤﻊ ﺍﻟﻘﻭﺓ‬ ‫‪r‬‬ ‫‪,‬‬ ‫‪ε ° = 8.85× 10 −12 c 2 N −1 m −2‬‬

‫ﺍﻟﻁﺎﺭﻗﺔ ﺍﻟﻤﺭﻜﺯﻴﺔ‪.‬‬
‫=‪F‬‬
‫‪mv 2‬‬
‫)‪...(1-25‬‬
‫‪r‬‬

‫ﺤﻴﺙ ‪ v‬ﻫﻲ ﺍﻟﺴﺭﻋﺔ ﺍﻟﺨﻁﻴﺔ ﻟﻺﻟﻜﺘﺭﻭﻥ‪.‬‬

‫ﺒﻤﺴﺎﻭﺍﺓ ﺍﻟﻘﻭﺘﺎﻥ ﻨﺠﺩ‪:‬‬


‫‪1 e 2 mv 2‬‬
‫=‬ ‫)‪...(1-26‬‬
‫‪4Dε ° r 2‬‬ ‫‪r‬‬

‫‪- ٢٧ -‬‬
‫‪PDF created with pdfFactory Pro trial version www.pdffactory.com‬‬
‫ﻁﺒﻘﺎﹰ ﻟﻘﻭﺍﻨﻴﻥ ﺍﻟﻔﻴﺯﻴﺎﺀ ﺍﻟﻜﻼﺴﻴﻜﻴﺔ‪ ،‬ﻓﺈﻥ ﺍﻟﺠﺴﻡ ﺍﻟﻤﺸﺤﻭﻥ ﺍﻟﻤﺘﺴﺎﺭﻉ‪ .‬ﻴ‪‬ﺼﺩﺭ‬
‫ﺇﺸﻌﺎﻉ ﻤﻤﺎ ﻴﺅﺩﻱ ﺇﻟﻰ ﻓﻘﺩﺍﻥ ﻟﻁﺎﻗﺘﻪ‪ ،‬ﻭﻟﻬﺫﺍ ﺍﻟﺴﺒﺏ ﻓﺈﻥ ﺍﻹﻟﻜﺘﺭﻭﻥ ﺴـﻴﻔﻘﺩ ﻁﺎﻗﺘـﻪ‬
‫ﺨﻼل ﺩﻭﺭﺍﻨﻪ ﺤﻭل ﺍﻟﻨﻭﺍﺓ ﻭﺴﻴﺩﻭﺭ ﻓﻲ ﺸﻜل ﺤﻠﺯﻭﻨﻲ ﻭﻴﺘﻼﺸﻰ ﺩﺍﺨـل ﺍﻟﻨـﻭﺍﺓ‪،‬‬
‫ﻭﻋﻠﻴﻪ ﻓﻼ ﻴﻤﻜﻥ ﺃﻥ ﻴﻭﺠﺩ ﻤﺩﺍﺭ ﻤﺴﺘﻘﺭ ‪ stable orbit‬ﻟﻼﻟﻜﺘﺭﻭﻥ‪ .‬ﻭﻟﻠﺨﺭﻭﺝ ﻤـﻥ‬
‫ﻫﺫﺍ ﺍﻹﺸﻜﺎل ﺍﻗﺘﺭﺡ ﺒﻭﺭ ﻓﺭﻀﻴﺎﺘﻪ ﺍﻟﺘﻲ ﺘﺨﺎﻟﻑ ﻗﻭﺍﻨﻴﻥ ﺍﻟﻔﻴﺯﻴﺎﺀ ﺍﻟﺘﻘﻠﻴﺩﻴﺔ‪.‬‬
‫ﺍﻟﻔﺭﻀﻴﺔ ﺍﻷﻭﻟﻰ‪:‬‬

‫ﺍﻟﻤﺩﺍﺭﺍﺕ ﺍﻹﻟﻜﺘﺭﻭﻨﻴﺔ ﺍﻟﻤﺴﺘﻘﺭﺓ )ﺍﻟﺜﺎﺒﺘﺔ(‬


‫‪Stationary electron orbits‬‬
‫ﻭﻫﺫﻩ ﺍﻟﻔﺭﻀﻴﺔ ﻫﻲ ﺘﹶﺤﺩٍ ﺒﺎﻟﻤﻔﺎﻫﻴﻡ ﺍﻟﺘﻘﻠﻴﺩﻴﺔ ﻟﻠﻔﻴﺯﻴﺎﺀ‪ .‬ﻭﻟﻘﺩ ﺤﺩﺩ ﺒـﻭﺭ ﻫـﺫﻩ‬
‫ﺍﻟﻤﺩﺍﺭﺍﺕ ﺒﺎﺴﺘﺤﺩﺍﺙ ﺸﺭﻁ ﺍﻟﺘﻜﻤﻴﻡ ‪ quantization condition‬ﻭﺍﻓﺘﺭﺽ ﺃﻥ ﻜﻤﻴﺔ‬
‫ﺍﻟﺤﺭﻜﺔ ﺍﻟﺯﺍﻭﻴﺔ ‪ angular momentum L‬ﻤﻜﻤﻠﺔ ﺤﺴﺏ ﺍﻟﻌﻼﻗﺔ‪.‬‬

‫‪L = mvr = nh , n = 1, 2, ........‬‬

‫)‪...(1-27‬‬
‫=‪v‬‬
‫‪hh‬‬
‫ﻤﻥ ﻫﺫﻩ ﺍﻟﻌﻼﻗﺔ ﻨﺠﺩ ﺃﻥ‬
‫‪mr‬‬

‫ﺒﺎﻟﺘﻌﻭﻴﺽ ﻓﻲ )‪(1-26‬‬
‫‪1 e 2 m nh 2 m n 2 h 2‬‬
‫) ( =‬
‫‪4Dε ° r 2 r mr‬‬ ‫‪m2 r3‬‬

‫ﻭﻓﻴﻬﺎ ﻨﺠﺩ ﺃﻥ‪.‬‬

‫)‪...(1-28‬‬
‫‪1‬‬ ‫‪n2 h2‬‬ ‫‪4D ε ° h 2 2‬‬
‫= ‪e‬‬
‫‪2‬‬
‫=‪⇒ r‬‬ ‫‪.n‬‬
‫‪4Dε °‬‬ ‫‪mr‬‬ ‫‪me 2‬‬

‫ﻭﻤﻥ ﻫﺫﻩ ﺍﻟﻌﻼﻗﺔ ﻨﺠﺩ ﺃﻥ ﺃﻨﺼﺎﻑ ﺃﻗﻁﺎﺭ ﺍﻟﻤﺩﺍﺭﺍﺕ ﻟﻬﺎ ﻤﻘﺎﺩﻴﺭ ﻤﺤـﺩﺩﺓ ﺃﻭ‬
‫ﻤﻜﻤﻤﺔ‪ .‬ﺇﻥ ﺃﻗل ﻨﺼﻑ ﻗﻁﺭ ﻟﻺﻟﻜﺘﺭﻭﻥ ﻴﻭﺠﺩ ﺒﺎﻟﺘﻌﻭﻴﺽ ﻋﻥ ‪.n = 1‬‬

‫‪- ٢٨ -‬‬
‫‪PDF created with pdfFactory Pro trial version www.pdffactory.com‬‬
‫‪( u D ) ( 8.85×10−12 c 2 N −1 m −2 ) (1.055× 10−34 J.s) 2‬‬
‫=‪r‬‬
‫‪(9.11× 10 − 31 kg) (1.602 × 10 −19 c) 2‬‬

‫‪°‬‬
‫‪= 5.29 ×10 −11 m = 52.9 pm ≈ 0.53 A‬‬

‫‪a°‬‬ ‫ﻭﻫﺫﻩ ﺍﻟﻘﻴﻤﺔ ﻴﺭﻤﺯ ﻟﻬﺎ ﻋﺎﺩﺓ ﺒـ‬


‫ﺍﻟﻁﺎﻗﺔ ﺍﻟﻜﻠﻴﺔ ﻟﻺﻟﻜﺘﺭﻭﻥ ‪ E‬ﻫﻲ ﻋﺒﺎﺭﺓ ﻋﻥ ﻤﺠﻤﻭﻉ ﻁـﺎﻗﺘﻲ ﺍﻟﺤﺭﻜـﺔ ‪k.E‬‬
‫ﻭﻁﺎﻗﺔ ﺍﻟﻭﻀﻊ )‪ V(r‬ﻭﺼﻴﻐﺘﻬﺎ‪.‬‬ ‫‪1‬‬
‫‪mv 2‬‬ ‫ﻭﺍﻟﺘﻲ ﺘﺴﺎﻭﻱ‬
‫‪2‬‬

‫‪V (r) = −‬‬


‫‪e2 1‬‬
‫)‪...(1-29‬‬
‫‪4D ε ° r‬‬

‫ﺍﻹﺸﺎﺭﺓ ﺍﻟﺴﺎﻟﺒﺔ ﻓﻲ )‪ (1-29‬ﺘﻌﻨﻲ ﺃﻥ ﺍﻟﺒﺭﻭﺘﻭﻥ ﻭﺍﻹﻟﻜﺘﺭﻭﻥ ﻴﺠـﺫﺏ ﻜـل‬


‫ﻤﻨﻬﻤﺎ ﺍﻵﺨﺭ‪.‬‬

‫ﻻﺤﻅ ﺃﻥ ﻁﺎﻗﺔ ﺍﻟﺘﺠﺎﺫﺏ ﺒﻴﻥ ﺍﻟﺒﺭﻭﺘﻭﻥ ﻭﺍﻹﻟﻜﺘﺭﻭﻥ ﺘﻘل ﻜﻠﻤﺎ ﺯﺍﺩﺕ ﺍﻟﻤﺴﺎﻓﺔ‬
‫‪V (∞ ) = 0‬‬ ‫ﻓﺈﻥ‬ ‫∞= ‪r‬‬ ‫ﺒﻴﻨﻬﻤﺎ ﻭﻋﻨﺩ‬

‫= )‪E = KE + V (r‬‬
‫‪1‬‬
‫‪mv 2 + (−‬‬
‫‪e2 1‬‬
‫)‬ ‫)‪...(1-30‬‬
‫‪2‬‬ ‫‪4Dε ° r‬‬

‫ﻴﻤﻜﻥ ﻜﺘﺎﺒﺔ ‪ mv2‬ﺒﺎﻟﺼﻴﻐﺔ‬


‫‪ 1 e 2 mv 2 ‬‬
‫= ‪mv 2‬‬
‫‪1 e2‬‬
‫‪‬‬ ‫=‬ ‫‪‬‬ ‫ﻤﻥ ﺍﻟﻌﻼﻗﺔ )‪(1-26‬‬
‫‪4Dε ° r‬‬ ‫‪ 4Dε ° r‬‬
‫‪2‬‬
‫‪r ‬‬

‫ﺒﺎﻟﺘﻌﻭﻴﺽ ﻓﻲ )‪(1-30‬‬
‫‪1 1 e2‬‬ ‫‪1 e2‬‬
‫( = ‪∴E‬‬ ‫‪)−‬‬
‫‪2 4D ε ° r‬‬ ‫‪4Dε ° r‬‬
‫‪1 1 e2‬‬ ‫‪1 e2‬‬
‫‪=−‬‬ ‫‪=−‬‬ ‫)‪......(1 − 3‬‬
‫‪2 4Dε ° r‬‬ ‫‪8Dε ° r‬‬

‫ﺒﺎﻟﺘﻌﻭﻴﺽ ﻋﻥ ‪ r‬ﻤﻥ )‪ (1-28‬ﻓﻲ )‪ (1-31‬ﻨﺠﺩ‪:‬‬


‫‪h‬‬
‫=‪h‬‬
‫‪2D‬‬
‫‪1‬‬ ‫‪e2‬‬ ‫‪me 4‬‬ ‫‪1‬‬
‫‪E=−‬‬ ‫‪=−‬‬
‫‪8Dε ° 4Dε ° h 2‬‬ ‫‪2 2‬‬
‫‪32 D ε ° h n‬‬
‫‪2‬‬ ‫‪2‬‬

‫‪2‬‬
‫‪n2‬‬
‫‪me‬‬

‫‪- ٢٩ -‬‬
‫‪PDF created with pdfFactory Pro trial version www.pdffactory.com‬‬
‫=‪h‬‬
‫‪h‬‬
‫ﺍﻻﺨﺘﺼﺎﺭ‬
‫‪2h‬‬
‫‪me 4‬‬ ‫‪1‬‬
‫‪E=−‬‬ ‫‪2‬‬
‫‪h‬‬ ‫‪2‬‬ ‫‪n2‬‬
‫‪(32)D 2 ε °‬‬
‫‪4D 2‬‬
‫‪me 4 1‬‬
‫‪∴E n = − 2 2 2‬‬ ‫‪n =1,2,.............‬‬
‫‪8ε ° h n‬‬

‫)‪...(1-32‬‬

‫ﺍﻹﺸﺎﺭﺓ ﺍﻟﺴﺎﻟﺒﺔ ﻓﻲ ﻫﺫﻩ ﺍﻟﻤﻌﺎﺩﻟﺔ ﺘﺩل ﻋﻠﻰ ﺃﻥ ﺤﺎﻻﺕ ﺍﻟﻁﺎﻗﺔ ﺤﺎﻻﺕ ﻤﻘﻴﺩﺓ‬
‫‪.bound states‬‬
‫ﻻﺤﻅ ﺃﻥ ﻓﻲ ﺤﺎﻟﺔ ‪ n = 1‬ﻓﺈﻥ ﻤﻌﺎﺩﻟﺔ )‪ (1- 32‬ﺘﻌﻁﻴﻨﺎ ﺍﻟﺤﺎﻟﺔ ﺫﺍﺕ ﺃﻗل ﻁﺎﻗﺔ‬
‫‪ state of lowest energy‬ﻭﺘﺴﻤﻰ ﻫﺫﻩ ﺍﻟﻁﺎﻗﺔ ﺒﺎﻟﻁﺎﻗﺔ ﺍﻷﺭﻀﻴﺔ ﺃﻭ ﻁﺎﻗﺔ ﺍﻟﺤﺎﻟـﺔ‬
‫ﺍﻷﺭﻀﻴﺔ ‪ .ground-state energy‬ﺃﻤﺎ ﺤﺎﻻﺕ ﺍﻟﻁﺎﻗﺔ ﺍﻷﻋﻠﻰ ﺘـﺴﻤﻰ ﺍﻟﺤـﺎﻻﺕ‬
‫ﺍﻟﻤﺴﺘﺜﺎﺭﺓ ‪ excited states‬ﻭﻋﺎﺩﺓ ﻤﺎ ﺘﻜﻭﻥ ﻏﻴﺭ ﻤﺴﺘﻘﺭﺓ ﺒﺎﻟﻨﺴﺒﺔ ﻟﻠﺤﺎﻟﺔ ﺍﻷﺭﻀﻴﺔ‪،‬‬
‫ﻭﻋﻨﺩﻤﺎ ﺘﻜﻭﻥ ﺍﻟﺫﺭﺓ )ﺃﻭ ﺍﻟﺠﺯﻱﺀ( ﻓﻲ ﺍﻟﺤﺎﻟﺔ ﺍﻟﻤﺴﺘﺜﺎﺭﺓ ﻓﺈﻨﻬﺎ ﺘـﺴﺘﺭﺨﻲ ﻭﺘﺭﺠـﻊ‬
‫ﻟﻠﺤﺎﻟﺔ ﺍﻷﺭﻀﻴﺔ ﻭﺘﻌﻁـﻲ ﺃﻭ ﺘـﺘﺨﻠﺹ ﻤـﻥ ﻁﺎﻗﺘﻬـﺎ ﻓـﻲ ﺼـﻭﺭﺓ ﻤﻭﺠـﺎﺕ‬
‫ﻜﻬﺭﻭﻤﻐﻨﺎﻁﻴﺴﻴﺔ ﻜﻤﺎ ﻫﻭ ﻤﻭﻀﺢ ﻓﻲ ﺍﻟﺸﻜل )‪.(1-10‬‬
‫ﺍﻟﻔﺭﻀﻴﺔ ﺍﻟﺜﺎﻨﻴﺔ‪ :‬ﻴﻤﻜﻥ ﻟﻼﻟﻜﺘﺭﻭﻨﺎﺕ ﺃﻥ ﺘﻨﺘﻘل )ﺃﻭ ﺘﻘﻔـﺯ( ﻤـﻥ ﻤـﺩﺍﺭﺍﺘﻬﺎ‬
‫ﺒﻁﺭﻴﻘﺔ ﻏﻴﺭ ﻤﺘﺼﻠﺔ ‪ discontinuous transitions‬ﻭﺃﻥ ﺍﻟﺘﻐﻴﺭ ﻓـﻲ ﺍﻟﻁﺎﻗـﺔ ‪ΔE‬‬
‫ﻴﺅﺩﻱ ﻻﻨﺒﻌﺎﺙ ﺇﺸﻌﺎﻉ ﻟﻪ ﺘﺭﺩﺩ ‪ . hv‬ﻭﻟﻬﺫﺍ ﺍﻟﺴﺒﺏ ﻟﻭ ﺍﻨﺘﻘل ﺇﻟﻜﺘﺭﻭﻥ ﻤـﻥ ﺍﻟﻤـﺩﺍﺭ‬
‫ﺍﻟﺫﻱ ﻟﻪ ‪ n2 =1‬ﺇﻟﻰ ﺍﻟﻤﺩﺍﺭ ﺫﻭ ‪ n1 = 2‬ﻓﺈﻥ ﺍﻟﻔﺭﻕ ﻓﻲ ﺍﻟﻁﺎﻗﺔ‪.‬‬
‫‪me 4 1‬‬ ‫‪me 4 1‬‬
‫‪ΔE = E 2 − E 1 = −‬‬ ‫‪2‬‬ ‫‪2‬‬
‫‪−‬‬ ‫(‬ ‫‪−‬‬ ‫‪2‬‬ ‫‪2‬‬
‫)‬
‫‪8ε ° h 2 n 2‬‬ ‫‪8 ε ° h 2 n1‬‬
‫‪me 4‬‬ ‫‪1‬‬ ‫‪1‬‬
‫‪ΔE = 2 2 ( 2 − 2 ) = hv‬‬ ‫)‪...............(1 − 33‬‬
‫‪8ε ° h n 1 n 2‬‬

‫‪- ٣٠ -‬‬
‫‪PDF created with pdfFactory Pro trial version www.pdffactory.com‬‬
FIGURE 1.10
The energy level diagram for the hydrogen atom, showing how
transitions from higher states into some particular state lead to the
observed spectral series for hydrogen.
‫( ﻴﻤﻜﻨﻨﺎ ﺍﻟﺤﺼﻭل ﻋﻠﻰ ﺍﻟﺼﻴﻐﺔ ﺍﻟﺭﻴﺎﻀﻴﺔ ﻟﻠﻌﻼﻗﺔ ﺍﻟﻌﺩﺩﻴﺔ‬1-33) ‫ﻤﻥ ﻤﻌﺎﺩﻟﺔ‬
.‫ﺍﻟﻤﻌﺭﻭﻓﺔ ﺒﻤﻌﺎﺩﻟﺔ ﺭﻴﺩﺒﺭﺝ ﻭﺍﻟﺘﻲ ﺘﺼﻑ ﺠﻤﻴﻊ ﺨﻁﻭﻁ ﺍﻟﻁﻴﻑ ﻟﺫﺭﺓ ﺍﻟﻬﻴﺩﺭﻭﺠﻴﻥ‬
.‫( ﻨﺠﺩ‬1-33) ‫ﻓﻲ‬ hc
‫ ﺒـ‬hv ‫ﺒﺎﻟﺘﻌﻭﻴﺽ ﻋﻥ‬
λ
hc me 4 1 1
= 2 2 ( 2 − 2)
λ 8ε ° h n 1 n 2

1
‫ﻭﻓﻴﻬﺎ ﻨﺠﺩ‬
λ
1 me 4 1 1
= 2 3 ( 2 − 2)
λ 8ε ° h c n 1 n 2
1 1 1
= R H ( 2 − 2 ) ..............(1 − 34)
λ n1 n2

- ٣١ -
PDF created with pdfFactory Pro trial version www.pdffactory.com
‫ﺤﻴﺙ ‪ RH‬ﻫﻭ ﺜﺎﺒﺕ ﺭﻴﺩﺒﺭﺝ ‪Rydberg constant‬‬
‫‪me 4‬‬ ‫‪(9.11× 10 −31 kg) (1.60× 10−19 c)4‬‬
‫= ‪RH‬‬ ‫=‬
‫)‪8ε ° h 3c (8) (8.85× 10 −12 c 2 N −1m − 2 ) (6.63 × 10 − 34 J.s)3 (2.0 × 108 m/s‬‬
‫‪2‬‬

‫‪R H =1.089 × 107 m −1‬‬


‫)‪(1‬‬

‫ﺒﺎﻟﻨﻅﺭ ﻟﺨﻁﻭﻁ ﺍﻟﻁﻴﻑ ﻓﻲ ﺸﻜل ‪ 1-10‬ﻴﻤﻜﻥ ﺃﻥ ﻨﻼﺤﻅ ﺍﻟﺘﻭﺍﻓﻕ ﺒﻴﻥ ﻨﻤﻭﺫﺝ‬


‫ﺒﻭﺭ ﻭﻫﺫﻩ ﺍﻟﺨﻁﻭﻁ ﺍﻟﻤﺘﻔﻘﺔ ﻤﻊ ﺍﻟﻨﺘﺎﺌﺞ ﺍﻟﺘﺠﺭﻴﺒﻴﺔ‪ .‬ﻓﻤﺜﻼﹰ ﺨﻁﻭﻁ ﺴﻠـﺴﻠﺔ ﻟﻴﻤـﺎﻥ‬
‫‪ Lyman‬ﺘﻨﺸﺄ ﻤﻥ ﺭﺠﻭﻉ )ﺍﺴﺘﺭﺨﺎﺀ( ﺍﻹﻟﻜﺘﺭﻭﻨﺎﺕ ﺍﻟﻤﺴﺘﺜﺎﺭﺓ ﻤﻥ ﺍﻟﻤﺴﺘﻭﻴﺎﺕ ﺍﻟﻌﻠﻴﺎ‬
‫ﻟﻠﻤﺩﺍﺭ ﺍﻷﻭل )‪ ،(n = 1‬ﻭﻜﺫﻟﻙ ﺨﻁﻭﻁ ﺴﻠﺴﻠﺔ ﺒﺎﻟﻤﺭ ‪ Balmer series‬ﺘﺤﺩﺙ ﻤـﻥ‬
‫ﺍﺴﺘﺭﺨﺎﺀ ﺍﻹﻟﻜﺘﺭﻭﻨﺎﺕ ﺍﻟﻤﺴﺘﺜﺎﺭﺓ ﻤﻥ ﻜل ﺍﻟﻤﺴﺘﻭﻴﺎﺕ ﺍﻟﻌﻠﻴـﺎ ﻟﻠﻤـﺴﺘﻭﻯ ﺍﻟﺜـﺎﻨﻲ‬
‫‪٢‬‬
‫)‪.(n=2‬‬
‫ﻤﺜﺎل ‪6‬‬

‫ﺍﺤﺴﺏ ﻁﺎﻗﺔ ﺍﻟﺘﺄﻴﻥ ﻟﺫﺭﺓ ﺍﻟﻬﻴﺩﺭﻭﺠﻴﻥ؟‬


‫?‪Calculate the ionization energy of the hydrogen atom‬‬
‫ﺍﻟﺤل‪:‬‬

‫ﻁﺎﻗﺔ ﺍﻟﺘﺄﻴﻥ ﻫﻲ ﺍﻟﻁﺎﻗﺔ ﺍﻟﻼﺯﻤﺔ ﻹﺯﺍﻟﺔ ﺍﻹﻟﻜﺘﺭﻭﻥ ﻤـﻥ ﻤـﺴﺘﻭﻯ ﺍﻟﺤﺎﻟـﺔ‬


‫∞= ‪n2‬‬ ‫ﺍﻷﺭﻀﻴﺔ ‪ n1 = 1‬ﺇﻟﻰ ﺍﻟﺤﺎﻟﺔ ﻏﻴﺭ ﺍﻟﻤﻘﻴﺩﺓ ﺃﻱ‬
‫‪me 4‬‬ ‫‪1‬‬ ‫‪1‬‬
‫= ‪ΔE‬‬ ‫‪2 2‬‬
‫)‪( 2 − 2‬‬
‫‪8ε ° h n 1 n 2‬‬
‫‪(9.1 × 10 - 34 kg) (1.9 × 10 −19 c) 4‬‬ ‫‪1‬‬ ‫‪1‬‬
‫=‬ ‫‪− 12 2‬‬ ‫‪−1‬‬ ‫‪−2‬‬ ‫‪− 34‬‬
‫)‪( 2 − 2‬‬
‫∞ ‪(8) (8.85 × 10 c N m ) (6.63 × 10 J.s) 1‬‬ ‫‪2‬‬

‫‪= 2.18 × 10 -18 J = 13.6 eV‬‬

‫ﻓﻲ ﻋﺎﻡ ‪ ،1925‬ﻅﻬﺭﺕ ﺍﻟﻨﻅﺭﻴﺔ ﺍﻟﺤﺩﻴﺜﺔ ﻟﻤﻴﻜﺎﻨﻴﻜﺎ ﺍﻟﻜﻡ ﻭﺫﻟﻙ ﺒﻨﺎﺀﺍﹰ ﻋﻠـﻰ‬
‫ﺍﻷﺒﺤﺎﺙ ﺍﻟﺘﻲ ﺭﺴﺦ ﺩﻋﺎﺌﻤﻬﺎ ﻜل ﻤﻥ ﺍﻟﻌﺎﻟﻡ ﻫﻴﺯﻨﺒﺭﺝ ‪ W. Heisenberg‬ﻭﺍﻟﻌـﺎﻟﻡ‬

‫‪(2 ) The exact value of RH = 1.09736 × 107 m-1 = 109.736 cm-1‬‬

‫‪- ٣٢ -‬‬
‫‪PDF created with pdfFactory Pro trial version www.pdffactory.com‬‬
‫ﻤﺎﻜﺱ ﺒﻭﺭﻥ ‪ ،M. Born‬ﻭﺍﻟﻌﺎﻟﻡ ﺸـﺭﻭﺩﻨﺠﺭ ‪ E. Shrödinger‬ﻭﺍﻟﻌـﺎﻟﻡ ﺩﻴـﺭﺍﻙ‬
‫‪.P. Dirac‬‬
‫ﻭﺘﺸﺭﺡ ﺍﻟﻨﻅﺭﻴﺔ ﺍﻟﻜﻤﻴﺔ ﺍﻟﺤﺩﻴﺜﺔ ﺍﻟﻤﻔﺎﻫﻴﻡ ﺍﻟﻤﺤﻴﺭﺓ ﺒﺸﺭﻁ ﺃﻥ ﻨﺘﺨﻠـﻰ ﻋـﻥ‬
‫ﺒﻌﺽ ﺍﻟﻤﻔﺎﻫﻴﻡ ﺍﻟﺘﻲ ﺘﺭﺴﺨﺕ ﻓﻲ ﺃﺫﻫﺎﻨﻨﺎ ﻤﻥ ﻋﻠﻡ ﺍﻟﻔﻴﺯﻴﺎﺀ ﺍﻟﺘﻘﻠﻴﺩﻴﺔ!‪.‬‬

‫= )‪u (v, T‬‬


‫‪8Dh‬‬ ‫‪v3‬‬
‫‪ (Q1‬ﺇﺫﺍ ﻋﻠﻤﺕ ﺃﻥ ﻜﺜﺎﻓﺔ ﺍﻹﺸﻌﺎﻉ ﺘﻌﻁﻰ ﺒﺎﻟﻤﻌﺎﺩﻟﺔ‬
‫‪c 3 e hv/KT − 1‬‬

‫‪ u (λ‬؟‬ ‫‪, T)dλ‬‬ ‫)‪ (a‬ﺍﺤﺴﺏ ﻜﺜﺎﻓﺔ ﺍﻟﻁﺎﻗﺔ ﻓﻲ ﻤﺩﻯ ﻁﻭل ﻤﻭﺠﻲ ‪ ، Δλ‬ﺃﻱ‬
‫ﻭﺍﻟﺘﻲ ﻋﻨﺩﻫﺎ ﺘﻜﻭﻥ ﻜﺜﺎﻓﺔ‬ ‫‪λ =λ‬‬ ‫‪max‬‬
‫)‪ (b‬ﺍﺴﺘﺨﺩﻡ ﺍﻟﻨﺘﻴﺠﺔ ﻓﻲ ﺠﺯﺀ )‪ (a‬ﻹﻴﺠﺎﺩ ﻗﻴﻤﺔ‬
‫ﺍﻹﺸﻌﺎﻉ ﺃﻗﺼﻰ ﻤﺎ ﻴﻤﻜﻥ؟‬
‫‪ ، λ‬ﻭﺍﺤﺴﺏ ﻗﻴﻤـﺔ ‪ b‬ﻓـﻲ‬ ‫‪max‬‬
‫‪b‬‬
‫ﻴﻤﻜﻨﻨﺎ ﻜﺘﺎﺒﺘﻬﺎ ﻋﻠﻰ ﺍﻟﺼﻴﻐﺔ‬ ‫‪λ max‬‬ ‫)‪ (c‬ﻭﻀﺢ ﺃﻥ‬
‫‪T‬‬

‫ﺤﺎﻟﺔ ﺴﻁﺢ ﺍﻟﺸﻤﺱ ﻋﻠﻤﺎﹰ ﺒﺄﻥ ﺩﺭﺠﺔ ﺤﺭﺍﺭﺓ ﺴﻁﺢ ﺍﻟﺸﻤﺱ ﺘﺴﺎﻭﻱ ‪5620k‬؟‬
‫)ﺘﻨﺒﻴﻪ‪ :‬ﺤل ﺍﻟﻤﻌﺎﺩﻟﺔ ‪ s-x = se-x‬ﺒﺎﻟﺭﺴﻡ ﺍﻟﺒﻴﺎﻨﻲ(‪.‬‬
‫)‪ (d‬ﻴﻨﺒﻌﺙ ﻤﻥ ﺃﺸﺩ ﺍﻟﻨﺠﻭﻡ ﺤﺭﺍﺭﺓ )ﻨﺠﻡ ﺍﻟﺸﹼﻌﺭﻱ ‪ (Sirius‬ﻁﻴﻑ ﺇﺸﻌﺎﻉ ﺍﻟﺠـﺴﻡ‬
‫‪ . λ‬ﺍﺤﺴﺏ ﺩﺭﺠﺔ ﺤﺭﺍﺭﺓ ﺴﻁﺢ ﻫﺫﺍ ﺍﻟﻨﺠﻡ؟‬ ‫‪max‬‬
‫‪= 260nm‬‬ ‫ﺍﻷﺴﻭﺩ ﻭﺍﻟﺫﻱ ﻟﻪ‬
‫)‪ (e‬ﺇﺫﺍ ﻋﻠﻤﺕ ﺃﻥ ﻤﺘﻭﺴﻁ ﺩﺭﺠﺔ ﺤﺭﺍﺭﺓ ﺴﻁﺢ ﺍﻷﺭﺽ ‪ .288 k‬ﺍﺴﺤﺏ ﺍﻟﻁـﻭل‬
‫ﺍﻟﻤﻭﺠﻲ ﻷﻗﺼﻰ ﻜﺜﺎﻓﺔ ﺇﺸﻌﺎﻉ ﻟﻠﺠﺴﻡ ﺍﻷﺴﻭﺩ ﻟﻸﺭﺽ‪ .‬ﺤﺩﺩ ﻷﻱ ﺠﺯﺉ ﻤـﻥ‬
‫ﺍﻟﻁﻴﻑ ﻴﻘﺎﺒل ﻫﺫﺍ ﺍﻟﻁﻭل ﺍﻟﻤﻭﺠﻲ؟‬
‫‪ (Q2‬ﺃﻗﺼﻰ ﻁﺎﻗﺔ ﺤﺭﻜﻴﺔ ﻹﻟﻜﺘﺭﻭﻨﺎﺕ ﻤﻨﺒﻌﺜﺔ ﻤﻥ ﺴﻁﺢ ﺃﻟﻭﻤﻨﻴﻭﻡ ﺘﺴﺎﻭﻱ ‪2.3 eV‬‬
‫ﻭﺫﻟﻙ ﻋﻨﺩ ﺘﻌﺭﺽ ﻫﺫﺍ ﺍﻟﺴﻁﺢ ﻷﺸﻌﺔ ﺫﺍﺕ ﻁﻭل ﻤﻭﺠﻲ ‪ .2000Å‬ﺃﻤﺎ ﻋﻨـﺩ‬
‫ﺘﻌﺭﺽ ﺍﻟﺴﻁﺢ ﻷﺸﻌﺔ ﻁﻭﻟﻬﺎ ﺍﻟﻤﻭﺠﻲ ‪ 2580 Å‬ﻓـﺈﻥ ﺍﻟﻁﺎﻗـﺔ ﺍﻟﺤﺭﻜﻴـﺔ‬
‫ﻟﻺﻟﻜﺘﺭﻭﻨﺎﺕ ﺍﻟﻤﻨﺒﻌﺜﺔ ﺘﺴﺎﻭﻱ ‪ .0.9 eV‬ﺍﺤﺴﺏ ﻗﻴﻤﺔ ﺜﺎﺒـﺕ ﺒﻼﻨـﻙ ﻭﺩﺍﻟـﺔ‬
‫ﺍﻟﺸﻐل ﻟﻸﻟﻤﻭﻨﻴﻭﻡ؟‬

‫‪- ٣٣ -‬‬
‫‪PDF created with pdfFactory Pro trial version www.pdffactory.com‬‬
‫‪ (Q3‬ﺍﺤﺴﺏ )‪ (a‬ﺍﻟﻁﻭل ﺍﻟﻤﻭﺠﻲ ﻭﺍﻟﻁﺎﻗﺔ ﺍﻟﺤﺭﻜﻴﺔ ﻹﻟﻜﺘﺭﻭﻥ ﻤ‪‬ﻌ‪‬ﺠ‪‬ل ﺘﺤﺕ ﺘـﺄﺜﻴﺭ‬
‫ﻓﺭﻕ ﺠﻬﺩ ‪ 100 V‬ﻭ )‪ (b‬ﻁﺎﻗﺔ ﺍﻟﺤﺭﻜﺔ ﺍﻹﻟﻜﺘﺭﻭﻥ ﻟﻪ ﻁﻭل ﻤﻭﺠﻲ ﺩﻴﺒﺭﻭﻟﻲ‬
‫‪) 200 pm‬ﺤﻴﺙ ‪(1 pm = 10-12 m‬؟‬
‫‪ (Q4‬ﺃﺸﻌﺔ ‪ X‬ﺍﺴﺘﻁﺎﺭﺕ ﺒﺈﻟﻜﺘﺭﻭﻥ ﺴﺎﻜﻥ‪ .‬ﺍﺤﺴﺏ ﻁﺎﻗﺔ ﺃﺸـﻌﺔ ‪ X‬ﺍﻟـﺴﺎﻗﻁﺔ ﺇﺫﺍ‬
‫ﺘﺴﺎﻭﻱ ‪ 0.035 A‬؟‬ ‫ﻋﻠﻤﺕ ﺃﻥ ﻁﻭل ﻤﻭﺠﺔ ﺍﻷﺸﻌﺔ ﺍﻟﻤﺴﺘﻁﺎﺭﺓ ﻋﻨﺩ ﺯﺍﻭﻴﺔ‬
‫‪°‬‬
‫‪60°‬‬

‫‪ (Q5‬ﺍﻟﻤﺴﺎﻓﺔ ﺒﻴﻥ ﻤﺴﺘﻭﻴﻴﻥ ﻤﺘﺠﺎﻭﺭﻴﻥ ﻤﻥ ﺍﻟﻤﺴﺘﻭﻴﺎﺕ ﺍﻟﺒﻠﻭﺭﻴـﺔ ﻴـﺭﺍﺩ ﻗﻴﺎﺴـﻬﺎ‬


‫ﺒﺎﺴﺘﺨﺩﺍﻡ ﺃﺸﻌﺔ ‪ X‬ﻁﻭﻟﻬﺎ ﺍﻟﻤﻭﺠﻲ ‪ 0.5 Å‬ﻭﺍﻟﺘﻲ ﺘﻡ ﻗﻴﺎﺴﻬﺎ ﻋﻨﺩ ﺯﺍﻭﻴـﺔ ‪. 5‬‬
‫‪°‬‬

‫ﺍﺤﺴﺏ ﻗﻴﻤﺔ ﺍﻟﻤﺴﺎﻓﺔ ﺒﻴﻥ ﻫﺫﻴﻥ ﺍﻟﻤﺴﺘﻭﻴﻴﻥ؟ ﻋﻨﺩ ﺃﻱ ﺯﺍﻭﻴﺔ ﻴﻤﻜﻨﻨـﺎ ﻗﻴـﺎﺱ‬
‫ﺍﻟﻘﻴﻤﺔ ﺍﻟﺜﺎﻨﻴﺔ؟‬
‫ﺇﺠﺎﺒﺎﺕ ﺍﻷﺴﺌﻠﺔ‪ :‬ﺍﻹﺠﺎﺒﺔ ﺍﻷﺨﻴﺭﺓ ﻤﻥ ﻜل ﺴﺅﺍل‪:‬‬
‫‪8Dhc hc/λc/‬‬
‫)‪Q1) (a‬‬ ‫= )‪u (λ , T‬‬ ‫‪(e‬‬ ‫‪− T) −1‬‬
‫‪λs‬‬

‫ﻨﻅﺭﻱ ‪ λ max = b / T‬ﺍﻟﻤﻁﻠﻭﺏ ﺇﺜﺒﺎﺕ ﺍﻟﻘﺎﻨﻭﻥ )‪(b‬‬


‫‪°‬‬
‫)‪(c‬‬ ‫‪λ mex = 5160 A‬‬

‫)‪(d‬‬ ‫‪T = 1.12 ×10 4 k 1‬‬

‫)‪(e‬‬ ‫‪λ = 1.01× 10 −5 m‬‬

‫)‪(Q2‬‬ ‫‪W = 3.92 eV‬‬

‫)‪(Q3‬‬ ‫‪K.E = 6.02 × 10-18 J‬‬

‫)‪(Q4‬‬ ‫‪E = 5.4× 105 eV‬‬

‫)‪(Q5‬‬ ‫‪θ =10 °‬‬

‫‪- ٣٤ -‬‬
‫‪PDF created with pdfFactory Pro trial version www.pdffactory.com‬‬
‫ﺍﻟﺤﺯﻡ ﺍﻟﻤﻭﺠﻴﺔ ﻭﻋﻼﻗﺎﺕ ﺍﻟﻼﺘﺤﺩﻴﺩ‬
‫‪WAVE PACKETS AND THE UNCERTAINTY RELATIONS‬‬
‫ﺇﻥ ﻋﻠﻡ ﻤﻴﻜﺎﻨﻴﻜﺎ ﺍﻟﻜﻡ ﻴﺯﻭﺩﻨﺎ ﺒﻔﻬﻡ ﺼﺤﻴﺢ ﻟﻜل ﺍﻟﻅﻭﺍﻫﺭ ﺍﻟﻔﻴﺯﻴﺎﺌﻴﺔ ﺍﻟﺘﻲ ﺘـﻡ‬
‫ﻤﻨﺎﻗﺸﺘﻬﺎ ﻓﻲ ﺍﻟﻔﺼل ﺍﻷﻭل‪ .‬ﻭﻫﺫﺍ ﺍﻟﻌﻠﻡ ﻀﺭﻭﺭﻱ ﻟﻔﻬﻡ ﺴﻠﻭﻙ ﺍﻟﺫﺭﺍﺕ‪ ،‬ﺍﻟﺠﺯﻴﺌﺎﺕ‪،‬‬
‫ﺃﻨﻭﻴﺔ ﺍﻟﺫﺭﺍﺕ ﻭﺘﺠﻤﻌﺎﺕ ﻤﻥ ﻫﺫﻩ ﻭﺘﻠﻙ‪ .‬ﻭﻋﺎﺩﺓ ﻤﺎ ﺘﺘﻡ ﺍﻟﺩﺭﺍﺴﺔ ﺒﺎﺴﺘﺨﺩﺍﻡ ﻤﻌﺎﺩﻟـﺔ‬
‫ﺸﺭﻭﺩﻨﺠﺭ ‪ Shrodinger equation‬ﻭﺍﻟﺘﺄﻭﻴﻼﺕ ﺍﻟﺼﺤﻴﺤﺔ ﻟﺤﻠﻭﻟﻬﺎ‪ .‬ﻭﻫﺫﻩ ﺍﻟﻤﻌﺎﺩﻟﺔ‬
‫ﻻ ﻴﻭﺠﺩ ﻟﻬﺎ ﺇﺜﺒﺎﺕ! ﻭﻟﻜﻥ ﺍﺴﺘﻁﺎﻉ ﺸﺭﻭﺩﻨﺠﺭ ﺍﻟﺘﻭﺼل ﺇﻟﻴﻬﺎ ﻋﻥ ﻁﺭﻴـﻕ ﺇﺘﺒـﺎﻉ‬
‫ﺘﻭﺠﻴﻬﺎﺕ )ﺃﻭ ﻓﺭﺍﺴﺎﺕ ‪ (insight‬ﺍﻟﻌﺎﻟﻡ ﺍﻟﻔﺭﻨﺴﻲ ﺩﻴﺒﺭﻭﻟﻲ‪ .‬ﻭﻤﻥ ﺍﻟﺠﺩﻴﺭ ﺒﺎﻟﺫﻜﺭ ﺃﻥ‬
‫ﻫﺫﻩ ﺍﻟﻤﻌﺎﺩﻟﺔ ﺘﻭﺠﺩ ﺨﺎﺭﺝ ﻨﻁﺎﻕ ﺍﻟﻔﻴﺯﻴﺎﺀ ﺍﻟﺘﻘﻠﻴﺩﻴﺔ‪ ،‬ﺍﻷﻤﺭ ﺍﻟﺫﻱ ﻴﺠﻌل ﺍﺴـﺘﻨﺒﺎﻁﻬﺎ‬
‫ﺼﻌﺏ‪ .‬ﻭﺨﻼل ﺍﻟﻤﻨﺎﻗﺸﺔ ﻓﻲ ﻫﺫﺍ ﺍﻟﻔﺼل ﺴﻨﺤﺎﻭل ﺍﻟﺘﻭﻓﻴﻕ ﺒﻴﻥ ﺍﻟﺨﻭﺍﺹ ﺍﻟﺠﺴﻴﻤﻴﺔ‬
‫ﻭﺍﻟﻤﻭﺠﻴﺔ ﻟﻼﻟﻜﺘﺭﻭﻨﺎﺕ‪.‬‬

‫ﺇﻨﻪ ﻤﻥ ﺍﻟﺼﻌﺏ ﺘﺨﻴل ﺍﻹﻟﻜﺘﺭﻭﻨﺎﺕ ﻋﻠﻰ ﺃﺴﺎﺱ ﺃﻥ ﻟﻬﺎ ﺴـﻠﻭﻙ ﺍﻟﻤﻭﺠـﺎﺕ‬


‫ﻭﻟﻜﻥ ﺘﺠﺎﺭﺏ ﺍﻟﺤﻴﻭﺩ ﺍﻟﺘﻲ ﺃﺠﺭﺍﻫﺎ ﻜـل ﻤـﻥ ‪ Freshet and Young‬ﺃﺩ‪‬ﺕ ﺇﻟـﻰ‬
‫ﺍﻹﺠﻤﺎﻉ )ﺃﻭ ﺍﺘﺤﺎﺩ ﺍﻵﺭﺍﺀ( ﺒﻘﺒﻭل ﺍﻟﻨﻅﺭﻴﺔ ﺍﻟﻤﻭﺠﻴﺔ ﻟﻠﻀﻭﺀ‪ .‬ﻭﻤﻥ ﺠﺎﻨـﺏ ﺁﺨـﺭ‬
‫ﺒﺎﻹﻤﻜﺎﻥ ﺃﻥ ﻨﺘﺼﻭﺭ ﺘﺠﻤﻌﺎﺕ )ﺃﻭ ﻫﻴﺌﺎﺕ( ﻟﻠﻤﻭﺠﺎﺕ ﺍﻟﻤﺤﺼﻭﺭﺓ ﺃﻭ ﺍﻟﻤﺘﺤﻴﺯﺓ )ﺃﻭ‬
‫ﺍﻟﻤﺘﻤﺭﻜﺯﺓ ﺠﺩﺍﹰ( ‪ Very localized‬ﻓﻌﻠﻰ ﺴﺒﻴل ﺍﻟﻤﺜﺎل ﺘﻌﺘﺒﺭ ﻗﺭﻗﻌﺔ ﺍﻟﺭﻋﺩ ‪a clap‬‬
‫‪ of thunder‬ﻤﺜﺎل ﻟﺘﺩﺍﺨل ﺍﻟﻤﻭﺠﺎﺕ ﻭﺘﺭﺍﻜﺒﻬﺎ ﻤﻤﺎ ﻴﺅﺩﻱ ﺇﻟﻰ ﺘﺄﺜﻴﺭ ﻤﺘﻤﺭﻜﺯ ﻤـﻊ‬
‫ﺍﻟﺯﻤﻥ ‪ .localized in time‬ﻤﺜل ﻫﺫﻩ ﺍﻟﺤ‪‬ﺯﻡ ﺍﻟﻤﻭﺠﻴـﺔ ﺍﻟﻤﺘﻤﺭﻜـﺯﺓ ‪localized‬‬
‫"‪ "wane packets‬ﻴﻤﻜﻥ ﺍﻟﺤﺼﻭل ﻋﻠﻴﻬـﺎ ﻋـﻥ ﻁﺭﻴـﻕ ﺘﺭﺍﻜـﺏ ﺍﻟﻤﻭﺠـﺎﺕ‬
‫‪ superposing waves‬ﺒﺘﺭﺩﺩﺍﺕ ﻤﺨﺘﻠﻔﺔ ﺒﺤﻴﺙ ﻴﺘﻡ ﺘﺩﺍﺨل ﻜل ﻤﻭﺠﺔ ﻤﻊ ﺍﻷﺨـﺭﻯ‬
‫ﺨﺎﺭﺝ ﻤﻨﻁﻘﺔ ﻤﻜﺎﻨﻴﺔ‪.‬‬
‫‪They interfere with each other almost completely outside of a‬‬
‫‪given spatial region.‬‬

‫‪- ٣٥ -‬‬
‫‪PDF created with pdfFactory Pro trial version www.pdffactory.com‬‬
‫ﻭﺍﻟﻁﺭﻴﻘﺔ ﺍﻟﺘﻲ ﻴﻤﻜﻥ ﺇﺘﺒﺎﻋﻬﺎ )ﺃﻭ ﺍﻷﺩﻭﺍﺕ ﺍﻟﺭﻴﺎﻀﻴﺔ ﺍﻟﻤﺴﺘﺨﺩﻤﺔ( ﻫﻲ ﻋﻤل‬
‫ﺘﻜﺎﻤﻼﺕ ﻓﻭﺭﻴﺭ ‪.Fourier integrals‬‬
‫ﻭﻗﺒل ﺍﻟﺒﺩﺀ ﻓﻲ ﺍﻟﻤﻌﺎﻟﺠﺔ ﺍﻟﺭﻴﺎﻀﻴﺔ ﻟﻠﺤﺯﻡ ﺍﻟﻤﻭﺠﻴﺔ‪ ،‬ﻟﻌﻠﻪ ﻤﻥ ﺍﻟﻤﻔﻴﺩ ﻤﺭﺍﺠﻌﺔ‬
‫ﺒﻌﺽ ﺃﺴﺎﺴﻴﺎﺕ ﺍﻟﺩﻭﺍل ﺍﻟﻤﺭﻜﺒﺔ ﻭﺘﻜﺎﻤﻼﺕ ﻓﻭﺭﻴﺭ ﻭﻜﻴﻔﻴﺔ ﺇﻴﺠﺎﺩ ﻋﺭﺽ ﺍﻟـﺩﻭﺍل‬
‫ﻭﻜﺫﻟﻙ ﻤﺭﺍﺠﻌﺔ ﺒﻌﺽ ﺍﻟﺘﻜﺎﻤﻼﺕ‪.‬‬
‫ﻴﻤﻜﻥ ﻜﺘﺎﺒﺔ ﺃﻱ ﺭﻗﻡ ﻤﺭﻜﺏ ‪ z‬ﺒﺎﻟﺼﻴﻐﺔ‪.‬‬

‫‪z= x+i y‬‬ ‫)‪......(A : 2 − 1‬‬

‫ﻭ ‪ y‬ﺘﻤﺜل ﺍﻟﺠﺯﺀ ﺍﻟﺘﺨﻴﻠﻲ ‪Im‬‬ ‫) ‪Re ( z‬‬ ‫ﺤﻴﺙ ‪ x‬ﺘﻤﺜل ﺍﻟﺠﺯﺀ ﺍﻟﺤﻘﻴﻘﻲ ﻟـ ‪ z‬ﺃﻱ‬
‫ﺍﻟﻭﺤﺩﺓ ﺍﻟﺘﺨﻴﻠﻴﺔ ‪ I‬ﻭﻫﻲ‪:‬‬ ‫‪z‬‬

‫‪i = −1 ⇒ i2 = −1‬‬ ‫)‪(A : 2 − 2‬‬

‫ﻟﻘﺴﻤﺔ ﺩﻭﺍل ﺘﺨﻴﻠﻴﺔ )ﺃﻭ ﺃﺭﻗﺎﻡ ﺘﺨﻴﻠﻴﺔ( ﻴﺤﺴﻥ ﺒﻨﺎ ﺍﺴﺘﺤﺩﺍﺙ ﺍﻟﻤﺭﺍﻓﻕ ﺍﻟﻤﺭﻜﺏ‬
‫‪ complex conjugate‬ﻭﻴﺭﻤﺯ ﻟﻪ *‪ z‬ﺤﻴﺙ‪:‬‬

‫‪z * = v − iy‬‬ ‫)‪(A : 2 − 3‬‬

‫ﻻﺤﻅ ﺃﻨﻨﺎ ﺍﺴﺘﺒﺩﻟﻨﺎ ‪ i‬ﺏ ‪ –i‬ﺍﻟﺭﻗﻡ ﺍﻟﺘﺨﻴﻠﻲ ﻤﻀﺭﻭﺏ ﻓﻲ ﻤﺭﺍﻓﻘﻪ ﺃﻱ *‪ zz‬ﻴﻌﻁﻴﻨﺎ‬

‫‪zz * = (x + iy) (x − iy) = x 2 − i 2 y 2 = x 2 + y 2‬‬ ‫)‪........... (A : 2 − 4‬‬

‫‪z‬‬
‫‪2‬‬
‫ﺘﺴﺎﻭﻱ ﻤﺭﺒﻊ ﺍﻟﻘﻴﻤﺔ ﺍﻟﻤﻁﻠﻘﺔ‬ ‫*‪z z‬‬ ‫ﻻﺤﻅ ﺃﻥ‬
‫‪1‬‬
‫=‬
‫‪1‬‬
‫ﻭﺍﻵﻥ ﻹﻴﺠﺎﺩ ﺍﻟﻘﻴﻤﺔ ﺍﻟﺤﻘﻴﻘﻴﺔ ﻭﺍﻟﺘﺨﻴﻠﻴﺔ ﻟـ‬
‫‪z x + iy‬‬
‫‪1‬‬ ‫‪1 x − iy x − iy‬‬ ‫‪x‬‬ ‫‪y‬‬
‫=‬ ‫‪= 2 2 = 2 2 −i 2 2‬‬
‫‪z x + iy x − iy x + y x + y‬‬ ‫‪x +y‬‬
‫‪1‬‬ ‫‪x‬‬ ‫‪1‬‬ ‫‪y‬‬
‫‪⇒ Re ( ) = 2 2 and In ( ) = 2 2‬‬
‫‪z x +y‬‬ ‫‪z x +y‬‬

‫ﺒﺎﺴﺘﺨﺩﺍﻡ ﻤﻌﺎﺩﻟﺔ ‪Euler‬‬ ‫‪θ, r‬‬ ‫ﺒﺩﻻﻟﺔ‬ ‫‪z‬‬ ‫ﻤﻥ ﺍﻟﻤﻔﻴﺩ ﻜﺘﺎﺒﺔ‬

‫‪- ٣٦ -‬‬
‫‪PDF created with pdfFactory Pro trial version www.pdffactory.com‬‬
eiθ = cos θ + i sin θ (A : 1 − 5)
x = r cos θ and y = r sin θ
⇒ z = x + iy = r cos θ + i r sin θ = r (cos θ + i sin θ ) = reiθ

e iθ . e − iθ = e ° = 1⇐ e − iθ ‫ﻫﻭ‬ e iθ ‫ﻻﺤﻅ ﺃﻥ ﺍﻟﻤﺭﺍﻓﻕ ﻟـ‬


z* = r e − iθ ‫ﻭﻜﺫﻟﻙ‬
zz * = (re−iθ ) (re−iθ ) = r 2 ‫ﻭﻤﻨﻬﺎ ﻨﺠﺩ‬

‫ﻭﻤﻥ ﺍﻟﻤﻔﻴﺩ ﺘﺫﻜﺭ ﺃﻥ‬


e iθ + e − iθ
cos θ = (A : 2 − 6)
2
and
eiθ − e − iθ
sinθ = ............ (A : 2 − 7)
2i

ϕ (θ ) =
1 imθ
e ϕ ( θ) ‫ﺇﺫﺍ ﻜﺎﻨﺕ ﻟﺩﻴﻨﺎ ﺩﺍﻟﺔ‬
2D

0 ≤ θ ≤ 2D ‫ﻫﻲ‬ θ ‫ﻭ ﺤﺩﻭﺩ‬ m = 0 ± 1 ± 2 ....... ‫ﺤﻴﺙ‬


:‫ﻓﺈﻥ‬
2D
1  eimθ 
2D 2D
1 imθ
∫° dθ ϕ (θ ) = ∫° 2D
dθ e =  
2D  im  °
let m ≠ o

=
1 1 i2 Dm °
e −e [ ]
2D im
1 1  
= cos 2Dm + i sin 2Dm − 1 
2D im  1
424
1
3 1 424
0
3 

=
1 1
[1 − 1]= 0
2D im
let m = 0
2D 2D 2D 2D

∫ dθ ϕ (θ ) = ∫ dθ
1 i(0) θ 1 1
[ ]
2D ∫°
e = dθ = θ °
° ° 2D 2D
1 2D
= . (2D − 0) = = 2D
2D 2D

- ٣٧ -
PDF created with pdfFactory Pro trial version www.pdffactory.com
‫ﻴﻤﻜﻥ ﺇﺠﺭﺍﺀ ﺍﻟﺘﻜﺎﻤل ﺍﻷﻭل ﺒﺼﻭﺭﺓ ﺃﺴﻬل ﻜﺎﻟﺘﺎﻟﻲ‪:‬‬
‫‪2D‬‬ ‫‪2D‬‬ ‫‪2D‬‬
‫‪1 imθ‬‬ ‫‪1‬‬ ‫‪i‬‬
‫∫‬
‫‪°‬‬ ‫‪2D‬‬
‫∫ = ‪e dθ‬‬
‫‪°‬‬ ‫‪2D‬‬
‫∫ ‪cos m θ dθ +‬‬
‫‪°‬‬ ‫‪2D‬‬
‫‪sin mθ dθ = 0‬‬

‫ﻜل ﺤﺩ ﻤﻥ ﺍﻟﺘﻜﺎﻤل ﻴﺴﺎﻭﻱ ﺼﻔﺭ ﻷﻨﻨﺎ ﻨﺠﺭﻱ ﺤﺩﻭﺩ ﺍﻟﺘﻜﺎﻤـل ﻋﺒـﺭ ﺩﻭﺭﺓ‬
‫ﻜﺎﻤﻠﺔ ﻭﻨﺭﺠﻊ ﻟﻨﻔﺱ ﺍﻟﺒﺩﺍﻴﺔ ﻭﻟﻬﺫﺍ ﻨﺴﺘﺨﻠﺹ ﺃﻨﻪ ﺇﺫﺍ ﻜﺎﻨﺕ ‪ m = 0‬ﻓـﺈﻥ ﺍﻟﺘﻜﺎﻤـل‬
‫ﺃﻤﺎ ﻟﻠﻘﻴﻡ ﺍﻷﺨﺭﻯ ﻟـ ‪ m‬ﻓﺈﻥ ﺍﻟﺘﻜﺎﻤل ﻴﺴﺎﻭﻱ ﺼﻔﺭ ﻭﻴﻤﻜﻥ ﺼـﻴﺎﻏﺔ‬ ‫‪2D‬‬ ‫ﻴﺴﺎﻭﻱ‬
‫ﺍﻟﻨﺘﻴﺠﺔ ﺒﻬﺫﻩ ﺍﻟﺼﻭﺭﺓ‪.‬‬
‫‪2D‬‬

‫‪∫ dθϕ (θ ) = 0‬‬


‫‪°‬‬
‫)‪for all values of m ≠ 0 ..................(A : 2 - 8‬‬

‫‪= 2D‬‬ ‫‪for m = 0‬‬ ‫)‪(A : 2 - 9‬‬

‫ﻭﺒﻨﻔﺱ ﺍﻟﻁﺭﻴﻘﺔ ﻴﻤﻜﻨﻨﺎ ﺍﺴﺘﻨﺘﺎﺝ ﺍﻟﻌﻼﻗﺔ‬


‫‪2D‬‬

‫‪∫ dθϕ‬‬ ‫‪(θ )ϕ n (θ ) = 0‬‬ ‫‪m≠n‬‬


‫*‬
‫‪m‬‬
‫)‪..................(A : 2 - 10‬‬
‫‪°‬‬

‫‪=1‬‬ ‫‪m= n‬‬ ‫)‪(A : 2 -11‬‬


‫‪2D‬‬
‫‪1 − imθ 1 inθ‬‬
‫‪∫ dθ‬‬
‫‪°‬‬ ‫‪2D‬‬
‫‪e .‬‬
‫‪2D‬‬
‫‪e‬‬

‫‪2D‬‬ ‫‪2D‬‬ ‫‪2D‬‬


‫‪1‬‬ ‫‪1‬‬ ‫‪1‬‬
‫[‬ ‫]‬
‫‪2D ∫°‬‬ ‫‪2D ∫°‬‬
‫‪i (n − m)θ‬‬ ‫‪°‬‬
‫‪Now, Let‬‬ ‫⇒‪m=n‬‬ ‫‪dθ e‬‬ ‫=‬ ‫= ‪dθ e‬‬ ‫‪θ°‬‬
‫‪2D‬‬
‫‪1‬‬
‫=‬ ‫‪(2D − 0) = 1‬‬
‫‪2D‬‬
‫‪2D‬‬
‫‪1‬‬
‫‪2D ∫°‬‬
‫⇒ ‪Let m ≠ n‬‬ ‫‪dθ ei (n − m) θ‬‬

‫‪2D‬‬
‫] ‪dθ [cos ( n − m ) θ + i sin ( n − m ) θ‬‬
‫‪1‬‬
‫‪2D ∫°‬‬
‫=‬

‫ﻗﻴﻤﺔ ﻫﺫﺍ ﺍﻟﺘﻜﺎﻤل ﺘﺴﺎﻭﻱ ﺼﻔﺭ ﻭﺫﻟﻙ ﻷﻥ ﺍﻟﺘﻜﺎﻤل ﻴﺠﺭﻱ ﻋﻠﻰ ﺩﻭﺭﺓ ﻜﺎﻤﻠﺔ‬
‫ﻟﻠﺩﺍﻟﺔ )ﺃﻱ ﺃﻨﻪ ﻴﺒﺩﺃ ﺜﻡ ﻴﻨﺘﻬﻲ ﻋﻨﺩ ﻨﻔﺱ ﺍﻟﻘﻴﻤﺔ(‪.‬‬
‫ﻭﻤﻥ ﺍﻟﺩﻭﺍل ﺍﻟﻤﺭﻜﺒﺔ ﺍﻟﻤﻬﻤﺔ‪ sin h x :‬ﻭ ‪ cos h x‬ﻭﺘﻌﺭﻑ ﺒـ‬
‫‪ex − e− x‬‬
‫= ‪sin h x‬‬ ‫‪⇒ sin h i x = i sin x‬‬ ‫)‪..................(A : 2 - 12‬‬
‫‪2‬‬
‫‪ex + e− x‬‬
‫‪cos h x‬‬ ‫‪⇒ cos h i x = cos x‬‬ ‫)‪(A : 2 - 13‬‬
‫‪2‬‬

‫‪- ٣٨ -‬‬
‫‪PDF created with pdfFactory Pro trial version www.pdffactory.com‬‬
‫ﻨﺒﺫﺓ ﻤﺨﺘﺼﺭﺓ ﻋﻥ ﺘﺤﻠﻴﻼﺕ ﻓﻭﺭﻴﺭ ﻭﺘﻜﺎﻤﻼﺘﻪ‬
‫‪Short Introduction about Fourier Series and its integral.‬‬
‫ﺇﻥ ﺃﻱ ﺩﺍﻟﺔ ﻟﻬﺎ ﺼﻭﺭﺓ ﻤﻭﺠﻴﺔ ﺃﻭ ﺘﻜﺭﺭ ﻨﻔﺴﻬﺎ ﺒﺩﻭﺭﺓ ﻤﻌﻴﻨﺔ‪ ،‬ﻤﺜل ﺍﻟـﺩﻭﺍل‬
‫ﺍﻟﻤﻭﺠﻴﺔ ﺍﻟﻤﻭﻀﺤﺔ ﻓﻲ ﺸﻜل ‪ ،A:2.1‬ﻴﻤﻜﻥ ﻭﺼﻔﻬﺎ ﺭﻴﺎﻀـﻴﺎﹰ ﺒﺎﺴـﺘﺨﺩﺍﻡ ﻤﺒـﺩﺃ‬
‫ﺍﻟﺘﺭﺍﻜﺏ ‪ ،Principle of super position‬ﺃﻱ ﻴﻤﻜﻥ ﺍﻟﺘﻌﺒﻴﺭ ﻋﻨﻬﺎ ﻋﻥ ﻁﺭﻴﻕ ﺠﻤﻊ‬
‫)ﺃﻭ ﺘﺭﺍﻜﺏ( ﻤﻭﺠﺎﺕ ﺘﻜﺘﺏ ﺭﻴﺎﻀﻴﺎﹰ ﺒﺩﻻﻟﺔ ﺍﻟـ‪ :‬ﺠﺎ ‪ sin‬ﻭﺍﻟـ ﺠﺘـﺎ ‪ .cos‬ﻫـﺫﻩ‬
‫ﺍﻟﻁﺭﻴﻘﺔ ﺍﻟﺭﻴﺎﻀﻴﺔ ﺘﹸﻌﺭﻑ ﺒﺘﺤﻠﻴل ﻓﻭﺭﻴﺭ ‪ Fourier analysis‬ﻨﺴﺒﺔ ﻟﻠﻌﺎﻟﻡ ﻓـﻭﺭﻴﺭ‪.‬‬
‫ﻭﺘﻌﺘﻤﺩ ﻫﺫﻩ ﺍﻟﻁﺭﻴﻘﺔ ﻋﻠﻰ ﺃﺴﺎﺱ ﺃﻨﻬﺎ ﻴﻤﻜﻥ ﺼﻴﺎﻏﺔ ﺃﻱ ﺩﺍﻟﺔ )‪) f (x‬ﺤﻴﺙ ‪ x‬ﻴﻤﻜﻥ‬
‫ﺃﻥ ﺘﻤﺜل ﺍﻟﻤﻭﻗﻊ ﺃﻭ ﺍﻟﺯﻤﻥ( ﻴﻤﻜﻥ ﺃﻥ ﺘﻜﺘﺏ ﻓﻲ ﺼﻭﺭﺓ ﻤﺘﺴﻠﺴﻠﺔ ﻤﻥ ﺩﻭﺍل ﺩﻭﺭﻴﺔ‬
‫)ﺘﻌﺭﻑ ﺒﺴﻠﺴﻠﺔ ﻓﻭﺭﻴﺭ( ‪.Fourier series‬‬
‫‪1‬‬
‫‪f (x) = a ° + a 1 cos x + a 2 cos 2x + .....+ a n cos n x + b1 sin x + b 2 sin 2x + .....+ b n sin n x + ........‬‬
‫‪2‬‬
‫∞‬ ‫∞‬
‫‪1‬‬
‫)‪= a ° + ∑ a u cos n x + ∑ b n sin v x ..............(A : 2 − 14‬‬
‫‪2‬‬ ‫‪n =1‬‬ ‫‪n =1‬‬

‫ﺒﺎﻹﻀﺎﻓﺔ ﺇﻟﻰ ﺤﺩﻭﺩ ﻤﻥ ﺍﻟﺠﻴـﺏ‬ ‫‪1‬‬


‫‪a‬‬ ‫ﺘﺘﻜﻭﻥ ﻫﺫﻩ ﺍﻟﻤﺘﺴﻠﺴﻠﺔ ﻤﻥ ﺤﺩ ﺜﺎﺒﺕ‬
‫‪2 °‬‬
‫‪ (sine and cosine terms‬ﻟﻬـﺎ ﺴـﻌﺎﺕ ﻤﺨﺘﻠﻔـﺔ ‪a1, a2, ….‬‬ ‫ﻭﺠﻴﻭﺏ ﺍﻟﺘﻤﺎﻡ‬
‫‪ .b1, b2, ……… Different amplitudes‬ﺘﺭﺩﺩﺍﺕ ﺍﻟﺠﺎ ﻭﺍﻟﺠﺘﺎ ﻓﻲ ﻜل ﺤﺩ ﻤـﻥ‬
‫ﺍﻟﺤﺩﻭﺩ ﻋﺒﺎﺭﺓ ﻋﻥ ‪ harmonics‬ﻭ ﻤﻀﺭﻭﺒﺎﺕ ‪ multiples‬ﺍﻟﺘﺭﺩﺩ ﺍﻷﺴﺎﺴﻲ ‪.x‬‬
‫‪ .Fundamental frequency x‬ﻟﻠﺘﻌﺒﻴﺭ ﻋﻥ ﻤﻌﻴﻨﺔ ﻤﻥ ﺨﻼل ﺘﺤﻠﻴل ﻓﻭﺭﻴﺭ ﻓـﺈﻥ‬
‫ﺍﻟﻤﻌﺎﻤﻼﺕ ……… ‪ a1, a2,‬ﻭ ‪ b1, b2, ……..‬ﻴﻠﺯﻡ ﺘﺤﺩﻴﺩﻫﺎ ﻭﺫﻟﻙ ﺒﺘﻜﺎﻤل ﺍﻟﺩﺍﻟﺔ‬
‫ﻜﻤﺎ ﻴﻠﻲ‪:‬‬ ‫‪2D‬‬ ‫)‪ f(x‬ﺨﻼل ﺩﻭﺭﺓ ﻜﺎﻤﻠﺔ ﺃﻱ ﻤﻥ ﺼﻔﺭ ﺇﻟﻰ‬
‫‪2D‬‬ ‫‪2D‬‬
‫‪1‬‬ ‫‪1‬‬
‫‪a n = ∫ f (x) cos n x dx‬‬ ‫‪,‬‬ ‫)‪b n = ∫ f (x) sin n x dx .....................(A : 2 − 15‬‬
‫‪D °‬‬ ‫‪D °‬‬

‫ﺍﻟﺸﻜل ﻴﻭﻀﺢ ﺃﻥ ﺘﻭﻟﻴﺩ ﺩﺍﻟﺔ ﻤﻭﺠﻴﺔ ﻤﺭﺒﻌﺔ‪:‬‬

‫‪- ٣٩ -‬‬
‫‪PDF created with pdfFactory Pro trial version www.pdffactory.com‬‬
‫‪1‬‬ ‫‪1‬‬
‫‪sin 2x , sin 3x , sinx‬‬ ‫ﻴﺘﻡ ﻋﻥ ﻁﺭﻴﻕ ﺘﺩﺍﺨل )ﺃﻭ ﺘﺭﺍﻜﺏ( ﺜﻼﺙ ﻤﻭﺠﺎﺕ ﺠﻴﺒﻴﺔ ﻫﻲ‬
‫‪2‬‬ ‫‪3‬‬

‫ﻴﻤﻜﻥ ﻜﺘﺎﺒﺘﻪ )ﻤﻥ ﻤﻌﺎﺩﻟـﺔ ‪(A: 2-15‬‬ ‫‪1‬‬


‫‪a‬‬ ‫ﺃﻭل ﺍﻟﺤﺩﻭﺩ ﻓﻲ ﻤﺘﺴﻠﺴﻠﺔ ﻓﻭﺭﻴﺭ‬
‫‪2 °‬‬
‫ﺒﺎﻟﺼﻭﺭﺓ‪.‬‬
‫‪2D‬‬
‫‪1‬‬
‫‪2D ∫°‬‬
‫‪f (x) dx‬‬

‫ﻭﻫﺫﻩ ﺍﻟﻤﻌﺎﺩﻟﺔ ﺘﻤﺜل ﺒﺒﺴﺎﻁﺔ ﻤﺘﻭﺴﻁ ﺍﻟﺩﺍﻟﺔ )‪ f(x‬ﻋﺒﺭ ﺍﻟﻔﺘﺭﺓ ‪. 2D‬‬

‫ﺘﺤﻠﻴل ﻓﻭﺭﻴﺭ ﻭﺍﻟﺤ‪‬ﺯ‪‬ﻡ ﺍﻟﻤﻭﺠﻴﺔ‪:‬‬

‫ﻴﻤﻜﻥ ﺍﻻﺴﺘﻔﺎﺩﺓ ﻤﻥ ﺘﺤﻠﻴل ﻓﻭﺭﻴﺭ ﻟﺘﻤﺜﻴل ﺍﻟﺤﺯﻡ ﺍﻟﻤﻭﺠﻴـﺔ ‪Wave packets‬‬


‫ﻭﻫﻲ ﻋﺒﺎﺭﺓ ﻋﻥ ﻨﺒﻀﺎﺕ ﻤﻭﺠﻴﺔ ‪ Wave pulses‬ﻋﻨﺩ ﺩﺭﺍﺴﺔ ﺍﻟﺤﺭﻜـﺔ ﺍﻟﻤﻭﺠﻴـﺔ‬
‫ﻟﻨﺘﺤﺼل ﻋﻠﻰ ﻨﺒﻀﺔ ﻤﻭﺠﻴﺔ ﻴﻠﺯﻡ ﺘﺭﺍﻜﺏ ﻤﻭﺠﺎﺕ ﺠﻴﺒﻴﺔ ﻋﻠﻰ ﻤﺩﻯ ﻤﺘﺼل ﻤـﻥ‬
‫ﺍﻷﺭﻗﺎﻡ ﺍﻟﻤﻭﺠﻴﺔ ‪ k‬ﻓﻲ ﻤﺩﻯ ﻗﺩﺭﺘﻪ ‪ . Δk‬ﻴﻤﻜﻨﻨﺎ ﻓﻬﻡ ﻜﻴﻔﻴﺔ ﺘﻜﻭﻥ ﺤﺯﻤﺔ ﻤﻭﺠﻴـﺔ‪،‬‬

‫‪- ٤٠ -‬‬
‫‪PDF created with pdfFactory Pro trial version www.pdffactory.com‬‬
‫ﻋﻠﻰ ﺴﺒﻴل ﺍﻟﻤﺜﺎل‪ ،‬ﺨﺫ ﺘﺭﺍﻜﺏ ﻋﺩﺩ ﺴﺒﻌﺔ ﻤﻥ ﺍﻟﻤﻭﺠﺎﺕ ﺍﻟﺠﻴﺒﻴﺔ ﺍﻟﺘﻲ ﻟﻜـل ﻤﻨﻬـﺎ‬
‫ﺍﻟﺼﻴﻐﺔ ﺍﻟﺭﻴﺎﻀﻴﺔ‪.‬‬

‫)‪y = A k cos ( k x − ω t‬‬ ‫)‪.......... .......... .(A : 2 − 16‬‬

‫ﻭﻋﻨــﺩ ‪ ،t = 0‬ﺍﻷﺭﻗــﺎﻡ ﺍﻟﻤﻭﺠﻴــﺔ )‪ k (Wave umbers‬ﺘــﺴﺎﻭﻱ‬


‫‪ 45, 42, 39, 36, 33, 30 , 27‬ﻭﻜل ﻤﻥ ﻫﺫﻩ ﺍﻷﺭﻗﺎﻡ ﻴﻘﺎﺒﻠﻬﺎ ﺴـﻌﺔ ‪ Ak‬ﺘـﺴﺎﻭﻱ‬
‫‪ 0.25 , 0.33 , 0.5 , 1.0, 0.5 , 0.33 , 0. 25‬ﺸﻜل ‪ A: 2-3‬ﻴﻭﻀﺢ ﺘﻐﻴﺭ ﺍﻟﺴﻌﺔ‬
‫‪ Ak‬ﻤﻊ ‪ k‬ﻭﻫﻲ ﻤﺎ ﻴﺴﻤﻰ ﺒﻁﻴﻑ ﺍﻟﺭﻗﻡ ﺍﻟﻤﻭﺠﻲ ‪.Wave umber spectrum‬‬

‫‪Figure A:2-3 the frequency spectrum used to generate the we‬‬


‫‪packet shown in Figure 12.84.‬‬
‫ﺍﻟﺸﻜل ﺍﻟﻤﻭﺠﻲ ﻟﻜل ﻤﻭﺠﺔ ﻤﻥ ﺍﻟﻤﻭﺠﺎﺕ ﺍﻟﺴﺒﻌﺔ ﻭﺤﺎﺼـل ﺘﺭﺍﻜﺒﻬـﺎ )ﺃﻭ‬
‫ﻤﺤﺼﻠﺔ ﺘﺭﺍﻜﺒﻬﺎ( ﻤﻭﻀﺤﺔ ﻓﻲ ﺸﻜل )‪.(A: 2-4‬‬

‫‪- ٤١ -‬‬
‫‪PDF created with pdfFactory Pro trial version www.pdffactory.com‬‬
‫ﺸﻜل )‪ (A: 2-4‬ﻜﻴﻔﻴﺔ ﺘﺅﻜﺩ ﺤﺯﻤﺔ ﻤﻭﺠﻴﺔ )ﺃﺴﻔل ﺍﻟﺸﻜل( ﻤﻥ ﺘﺩﺍﺨل ﺴـﺒﻌﺔ‬
‫ﻤﻭﺠﺎﺕ ﻜﺎﻟﻤﻭﻀﺤﺔ ﻓﻲ ﺃﻋﻠﻰ ﺍﻟﺤﺯﻤﺔ ﺍﻟﻤﻭﺠﻴﺔ‪.‬‬
‫ﺃﻤﺎ ﻓﻲ ﺤﺎﻟﺔ ﺘﺭﺍﻜﺏ ﻋﺩﺩ ﻜﺒﻴﺭ ﺠﺩﺍﹰ ﻤﻥ ﺍﻟﻤﻭﺠﺎﺕ ﺒﺤﻴﺙ ﻴﻤﺘﺩ ﺘﻐﻴﺭ ﺍﻷﺭﻗﺎﻡ‬
‫ﺴﻨﺘﺤﺼل ﻋﻠﻰ ﺤﺯﻤﺔ ﻤﻭﺠﻴﺔ ﻭﺍﺤﺩﺓ ﻜﻤـﺎ ﻫـﻭ‬ ‫‪ΔK‬‬ ‫ﺍﻟﻤﻭﺠﻴﺔ ﻋﺒﺭ ﻋﺭﺽ ﻗﺩﺭﻩ‬
‫ﻤﻭﻀﺢ ﻓﻲ ﺍﻟﺸﻜل ‪.A: 2-5‬‬
‫ﻻﺤﻅ ﺃﻥ ﻋﺭﺽ ﺍﻟﺤﺯﻤﺔ ﺍﻟﻤﻭﺠﻴﺔ ‪...... ΔK‬‬

‫‪- ٤٢ -‬‬
‫‪PDF created with pdfFactory Pro trial version www.pdffactory.com‬‬
‫ﻓﺈﻥ ﻋﺭﺽ‬ ‫‪ΔK‬‬ ‫ﻭﻴﻤﻜﻥ ﺃﻥ ﻨﻼﺤﻅ ﻨﻘﻁﺔ ﻫﺎﻤﺔ ﺠﺩﺍﹰ ﻭﻫﻲ ﺃﻨﻪ ﻜﻠﻤﺎ ﺍﺯﺩﺍﺩﺕ‬
‫ﻴﻘل‪ .‬ﻭﻫﺫﻩ ﺍﻟﻨﺘﻴﺠﺔ ﻫﺎﻤﺔ ﻷﻨﻬﺎ ﺍﻷﺴﺎﺱ ﺍﻟﺫﻱ ﻴﺒﻨﻰ ﻋﻠﻴﻪ ﻤﺒﺩﺃ ﺍﻟﻼﺘﺤﺩﻴﺩ‬ ‫‪ΔK‬‬ ‫ﺍﻟﺤﺯﻤﺔ‬
‫ﻓﻲ ﻋﻠﻡ ﻤﻴﻜﺎﻨﻴﻜﺎ ﺍﻟﻜﻡ‪.‬‬

‫‪Figure A: 2-5 (a) the wave number spectrum and (b) the wave‬‬
‫‪packet. Produced when ∆k the wave number spread shown in Figure‬‬
‫‪A:2-4 becomes broader.‬‬
‫ﺇﻥ ﺘﺭﺍﻜﺏ ﻋﺩﺩ ﻻﻨﻬﺎﺌﻲ ‪ infinite number‬ﻤﻥ ﺍﻟﻤﻭﺠﺎﺕ ﻴﻤﻜﻥ ﺼـﻴﺎﻏﺘﻪ‬
‫ﺭﻴﺎﻀﻴﺎﹰ ﺒﺘﻜﺎﻤل ﻓﻭﺭﻴﺭ ‪ Fourier integral‬ﺒﺩﻻﹰ ﻤﻥ ﻤﺘﺴﻠﺴﻠﺔ ﻓﻭﺭﻴﺭ‪ .‬ﻭﻫﻜﺫﺍ‪ ،‬ﻓﻌﻨﺩ‬
‫ﺯﻤﻥ ﺜﺎﺒﺕ‪ ،‬ﻓﺈﻥ ﻤﺠﻤﻭﻋﺔ ﻤﻥ ﺍﻟﻤﻭﺠﺎﺕ ﺘﻌﻁﻲ ﺒـ‪:‬‬
‫∞‬
‫‪y (x) ∫ A (k) cos k x dk‬‬ ‫)‪..........(A : 2 − 17‬‬
‫‪°‬‬

‫ﺤﻴﺙ )‪ A(k‬ﻫﻲ ﺍﻟﺴﻌﺔ ﻭﺍﻟﺘﻲ ﺘﺘﻐﻴﺭ ﻤﻊ ‪ .k‬ﻭﺴﻨﻨﺎﻗﺵ ﻓﻲ ﻫﺫﺍ ﺍﻟﻔﺼل ﺒﻌـﺩ‬


‫‪Δx , Δk‬‬ ‫ﻫﺫﻩ ﺍﻟﻤﻘﺩﻤﺔ‪ ،‬ﻜﻴﻑ ﻴﻤﻜﻨﻨﺎ‪ ،‬ﺒﺎﺴﺘﺨﺩﺍﻡ ﺘﻜﺎﻤﻼﺕ ﻓـﻭﺭﻴﺭ‪ ،‬ﺃﻥ ﻨﻭﻀـﺢ ﺃﻥ‬
‫ﻤﺭﺘﺒﻁﺘﺎﻥ ﺒﺎﻟﺩﺍﻟﺔ‪.‬‬

‫‪(Δ x) (Δ k) = constant‬‬ ‫)‪....................(A : 2 − 18‬‬

‫‪- ٤٣ -‬‬
‫‪PDF created with pdfFactory Pro trial version www.pdffactory.com‬‬
‫ﻭﻟﻌﻠﻪ ﻤﻥ ﺍﻟﻤﻔﻴﺩ ﺼﻴﺎﻏﺔ ﺍﻟﻤﻌﺎﺩﻟﺔ )‪ (A: 2-17‬ﻟﻠﺤﺯﻤﺔ ﺍﻟﻤﻭﺠﻴﺔ ﺒﺩﻻﻟﺔ ﺍﻟﺩﺍﻟﺔ‬
‫ﺍﻟﻤﺭﻜﺒﺔ ﻋﻠﻰ ﺍﻟﺼﻭﺭﺓ ﺍﻟﻌﺎﻤﺔ‪.‬‬
‫∞‬
‫‪1‬‬
‫=)‪f (x‬‬ ‫‪∫ dk A (k) e‬‬ ‫)‪.............(A : 2 − 19‬‬
‫‪ikx‬‬

‫‪2D‬‬ ‫∞‪−‬‬

‫ﻭﺇﺫﺍ ﺃﺩﺨﻠﻨﺎ ﺍﻟﺯﻤﻥ ﻜﻤﺘﻐﻴﺭ ﻟﺩﺭﺍﺴﺔ ﺘﺄﺜﻴﺭ ﺍﻟﺯﻤﻥ ﻋﻠـﻰ ﺍﻨﺘـﺸﺎﺭ ﺍﻟﺤﺯﻤـﺔ‬
‫ﺍﻟﻤﻭﺠﻴﺔ ﻴﻤﻜﻥ ﺼﻴﺎﻏﺔ ﺍﻟﻤﻌﺎﺩﻟﺔ )‪ (A:2-19‬ﺒﺎﻟﺼﻭﺭﺓ‪.‬‬
‫∞‬
‫= ) ‪f ( x, t‬‬ ‫‪∫ dk A (k) e‬‬ ‫)‪.............(A : 2 − 20‬‬
‫‪ikx - ω (k) t‬‬

‫∞‪−‬‬

‫ﻋﻨﺩ ﺘﺩﺍﺨل ﻤﻭﺠﺎﺕ ﺴﺭﻋﺔ ﻜل ﻤﻨﻬﺎ ‪ v‬ﻓﺈﻥ ﺍﻟﺤﺯﻤﺔ ﺍﻟﻤﻭﺠﻴﺔ ﺍﻟﻤﺘﻜﻭﻨﺔ ﺘﻜﻭﻥ‬
‫ﺴﺭﻋﺘﻬﺎ ﺘﻤﺜل ﺴﺭﻋﺔ ﻤﺠﻤﻭﻋﺔ ﻤﻥ ﺍﻟﻤﻭﺠﺎﺕ ﺍﻟﻤﻜﻭﻨﺔ ﻟﻠﺤﺯﻤﺔ ﻭﺘـﺴﻤﻰ ﺒـﺴﺭﻋﺔ‬
‫ﺍﻟﻤﺠﻤﻭﻋﺔ ‪ group velocity‬ﻭﻴﺭﻤﺯ ﻟﻬﺎ ﺒـ ‪ vg‬ﻭﺘﻌﻁﻰ ﺒﺎﻟﻌﻼﻗﺔ‪.‬‬
‫‪dω‬‬
‫= ‪vg‬‬ ‫)‪...................... (A : 2 − 21‬‬
‫‪dk‬‬

‫ﻜﻴﻔﻴﺔ ﺇﻴﺠﺎﺩ ﻋﺭﺽ ﺩﺍﻟﺔ‪:‬‬

‫ﻤﻥ ﺍﻟﻨﻘﺎﻁ ﺍﻟﺘﻲ ﺘﻬﻤﻨﺎ ﻓﻲ ﻫﺫﺍ ﺍﻟﻔﺼل ﺇﻴﺠﺎﺩ ﻋﺭﺽ ﺩﺍﻟـﺔ ﻭﺫﻟـﻙ ﻷﻫﻤﻴـﺔ‬
‫ﺍﻟﻤﻭﻀﻭﻉ ﺒﻤﺒﺩﺃ ﺍﻟﻼﺘﺤﺩﻴﺩ ﻓﻲ ﻤﻴﻜﺎﻨﻴﻜﺎ ﺍﻟﻜﻡ‪.‬‬
‫ﺨﺫ ﻋﻠﻰ ﺴﺒﻴل ﺍﻟﻤﺜﺎل ﺍﻟﺩﺍﻟﺔ ﺍﻟﻤﻭﻀﺤﺔ ﻓﻲ ﺍﻟﺸﻜل ‪A: 2-6‬‬

‫‪- ٤٤ -‬‬
‫‪PDF created with pdfFactory Pro trial version www.pdffactory.com‬‬
‫ﻜﻠﺘﺎ ﺍﻟﺩﺍﻟﺘﺎﻥ ﻟﻬﻤﺎ ﻨﻔﺱ ﺍﻟﻌﺭﺽ‪ ،‬ﻭﻟﻜﻥ ﺇﺤﺩﺍﻫﻤﺎ ﻤﺘﻤﺎﺜﻠﺔ ﺤﻭل ‪ x = o‬ﻭﺍﻷﺨـﺭﻯ‬
‫‪y = A e− α x‬‬
‫‪2‬‬
‫ﻤﺘﻤﺎﺜﻠﺔ ﺤﻭل ‪ .xo‬ﺍﻷﻭل ﻓﻲ ﺍﻟﺸﻜل )‪ (a‬ﻴﻤﻜﻥ ﻜﺘﺎﺒﺘﻬﺎ ﺭﻴﺎﻀﻴﺎﹰ ﺒﺎﻟﺼﻴﻐﺔ‬
‫ﺤﻴﺙ ‪ α‬ﺜﺎﺒﺕ ﻭﻤﺭﺘﺒﻁ ﺒﻌﺭﺽ ﺍﻟﺩﺍﻟﺔ ﻭ ‪ A‬ﻴﻤﺜل ﺴﻌﺔ ﺍﻟﺩﺍﻟـﺔ ﺃﻱ ﻗﻴﻤـﺔ ‪y‬‬
‫ﻋﻨﺩﻤﺎ ‪.x = o‬‬

‫ﺇﺤﺩﻯ ﺍﻟﻁﺭﻕ ﻹﻴﺠﺎﺩ ﻋﺭﺽ ﺍﻟﺩﺍﻟﺔ ﺃﻥ ﻨﺄﺨﺫ ﺍﻟﻌﺭﺽ ﻋﻨﺩﻤﺎ ﺘﻘل ﺍﻟﺩﺍﻟﺔ )ﺃﻱ‬
‫ﻋﻨﺩﻤﺎ ﺘﻘل ﻗﻴﻤﺔ ‪ .y‬ﻤﻥ ﺃﻗﺼﻰ ﻗﻴﻤﺔ )ﻋﻨﺩﻤﺎ ‪ (y = A‬ﺇﻟﻰ ﺃﻥ ﺘـﺼل ﺇﻟـﻰ ‪، y = A‬‬
‫‪2‬‬

‫ﻋﻨﺩﻫﺎ ﻨﺄﺨﺫ ﻋﺭﺽ ﺍﻟﺩﺍﻟﺔ ‪. ∆x‬‬


‫ﻜﻤﺎ ﻫﻭ ﻤﻭﻀﺢ ﺒﺎﻟﺸﻜل‪.‬‬
‫‪ln y = ln A − α x 2‬‬
‫‪⇒ α x 2 = ln A − ln y‬‬
‫‪1‬‬ ‫‪1‬‬
‫‪α x 2 = ln A − ln A‬‬ ‫=‪y‬‬ ‫‪A‬‬ ‫ﻧﻌﻮض ﻋﻦ‬
‫‪2‬‬ ‫‪2‬‬
‫‪A‬‬
‫‪= ln‬‬ ‫‪= ln 2 = 0.693‬‬
‫‪A/2‬‬
‫‪1‬‬ ‫‪0.693‬‬ ‫‪0.693‬‬
‫‪⇒ x 2 = 0.693‬‬ ‫= ‪⇒ Δx °‬‬ ‫‪⇒ Δx = 2‬‬
‫‪α‬‬ ‫‪α‬‬ ‫‪α‬‬

‫‪- ٤٥ -‬‬
‫‪PDF created with pdfFactory Pro trial version www.pdffactory.com‬‬
‫ﻴﻤﻜﻥ ﺇﻴﺠﺎﺩ ﺘﻌﺭﻴﻑ ﺁﺨﺭ ﻟﻌﺭﺽ ﺍﻟﺩﺍﻟﺔ ﻭﻫﻭ ﺃﺴﻬل ﻤﻥ ﺍﻟﻁﺭﻴﻘﺔ ﺍﻟـﺴﺎﺒﻘﺔ‪.‬‬
‫‪1‬‬
‫ﻨﺄﺨﺫ ﻋﺭﺽ ﺍﻟﺩﺍﻟﺔ ﻋﻨﺩﻤﺎ ﺘﻘل ﻗﻴﻤﺔ ‪ y‬ﻤﻥ ﺍﻟﻘﻴﻤﺔ ﺍﻟﻘﺼﻭﻯ ﺇﻟﻰ ﺃﻥ ﺘﺼل ﺇﻟـﻰ‬
‫‪e‬‬
‫ﺃﻱ ﺒﺩل ﺃﻥ ﻨﺄﺨﺫ ﺍﻟﻌﺭﺽ ﻋﻨﺩﻤﺎ ﺘﻘـل ‪ y‬ﺇﻟـﻰ‬ ‫‪1‬‬
‫‪= 0.368‬‬ ‫ﺤﻴﺙ ‪ e = 2.718‬ﻭﻋﻠﻴﻪ‬
‫‪e‬‬

‫ﺍﻟﻨﺼﻑ )ﻜﻤﺎ ﻓﻲ ﺍﻟﻁﺭﻴﻘﺔ ﺍﻟﺴﺎﺒﻘﺔ( ﻨﺄﺨﺫ ﺍﻟﻌﺭﺽ ﻋﻨﺩﻤﺎ ﺘﻘل ‪ y‬ﺇﻟﻰ ‪ 0.368‬ﻤـﻥ‬
‫‪y = A e− α x‬‬
‫‪2‬‬
‫ﻗﻴﻤﺘﻬﺎ ﺍﻟﻘﺼﻭﻯ‪ ..‬ﻭﺴﻬﻭﻟﺔ ﻫﺫﻩ ﺍﻟﻁﺭﻴﻘﺔ ﻤﻥ ﺍﻟﻨﺎﺤﻴﺔ ﺍﻟﺭﻴﺎﻀﻴﺔ‪.‬‬
‫ﺨﺫ ﻤﺜﻼﹰ ﺍﻟﺩﺍﻟﺔ ﺍﻟﺴﺎﺒﻘﺔ‬

‫ﻤﻥ ﻗﻴﻤﺘﻬﺎ ﺍﻟﻘﺼﻭﻯ‪ ،‬ﺃﻱ ﻤﺎ‬ ‫‪1‬‬


‫ﻗﻴﻤﺔ ‪ y‬ﺍﻟﻘﺼﻭﻯ ﻫﻲ ‪ y = A‬ﻟﻜﻲ ﺘﻘل ‪ y‬ﺇﻟﻰ‬
‫‪e‬‬

‫ﻴﺴﺎﻭﻱ ﺍﻟﻭﺍﺤﺩ‪ .‬ﻋﻨﺩﻤﺎ ﻴﺴﺎﻭﻱ ﺍﻷﺱ ﻗﻴﻤـﺔ‬ ‫) ‪(α x 2‬‬ ‫ﻫﻲ ﻗﻴﻤﺔ ‪ x2‬ﺍﻟﺘﻲ ﺘﺠﻌل ﺍﻷﺱ‬
‫‪1‬‬
‫‪y = ( ) A ⇐ y = A e −1‬‬ ‫ﻭﺍﺤﺩ ﻓﺈﻥ‬
‫‪e‬‬
‫‪1‬‬ ‫‪2‬‬
‫= ‪⇒ α x 2 =1 ⇒ Δ x °‬‬ ‫= ‪⇒ Δx = 2Δ x °‬‬
‫‪α‬‬ ‫‪α‬‬

‫ﻭﻫﺫﻩ ﻫﻲ ﺍﻟﻁﺭﻴﻘﺔ ﺍﻟﺘﻲ ﺴﻨﺴﺘﺨﺩﻤﻬﺎ ﻹﻴﺠﺎﺩ ﻋﺭﺽ ﺍﻟﺩﻭﺍل ﺍﻟﺘﻲ ﻟﻬﺎ ﺍﻟﺼﻴﻐﺔ‬
‫ﺍﻷﺴﻴﺔ‪ .‬ﻭﻫﻨﺎﻙ ﻁﺭﻕ ﺃﺨﺭﻯ ﻹﻴﺠﺎﺩ ﻋﺭﺽ ﺍﻟﺩﻭﺍل ﺍﻟﺠﻴﺒﻴﺔ‪.‬‬

‫ﻭﺒﻌﺩ ﻫﺫﺍ ﺍﻻﺴﺘﻁﺭﺍﺩ ﻨﻌﻭﺩ ﻟﻤﻨﺎﻗﺸﺔ ﻤﻭﻀﻭﻉ ﻫﺫﺍ ﺍﻟﻔـﺼل ﻭﻫـﻭ ﺍﻟﺤـﺯﻡ‬
‫ﺍﻟﻤﻭﺠﻴﺔ ﻭﻋﻼﻗﺎﺕ ﺍﻟﻼﺘﺤﺩﻴﺩ‪..‬‬
‫ﺨﺫ ﻋﻠﻰ ﺴﺒﻴل ﺍﻟﻤﺜﺎل ﺍﻟﺩﺍﻟﺔ )‪ f (x‬ﺍﻟﻤﻌﺭﻓﺔ ﺒﺎﻟﻌﻼﻗﺔ‪.‬‬

‫‪- ٤٦ -‬‬
‫‪PDF created with pdfFactory Pro trial version www.pdffactory.com‬‬
‫∞‬
‫‪f (x) = ∫ dk g (k) e ikv‬‬ ‫)‪....(2 − 1‬‬
‫∞‪−‬‬

‫ﻫﺫﻩ ﺍﻟﻌﻼﻗﺔ ﺘﹸﻌﺒ‪‬ﺭ ﻋﻥ ﺘﺭﺍﻜﺏ ﺨﻁﻲ ﻤﻥ ﻤﻭﺠﺎﺕ ﺫﺍﺕ ﻁﻭل ﻤﻭﺠﻲ ‪، λ = 2D‬‬
‫‪k‬‬
‫ﻭﻟﻜل ﻗﻴﻤﺔ ﻟـ ‪ k‬ﻓﻜل ﻤﻭﺠﺔ ﺘﻜﺭﺭ ﻨﻔﺴﻬﺎ ﻋﻨﺩﻤﺎ ﺘﺘﻐﻴﺭ ‪ x‬ﺇﻟﻰ ‪. v + 2D‬‬
‫‪k‬‬

‫ﻭﻟﺘﻭﻀﻴﺢ ﻁﺒﻴﻌﺔ ﻫﺫﻩ ﺍﻟﺤﺯﻤﺔ ﺍﻟﻤﻭﺠﻴﺔ‪ ،‬ﺩﻋﻨﺎ ﻨﺨﺘﺎﺭ ﺼـﻴﻐﺔ ﻟﻠﺩﺍﻟـﺔ )‪g (k‬‬
‫ﺤﻴﺙ‪.‬‬
‫‪2‬‬
‫) ‪g (k) = e −α (k − k °‬‬

‫)‪...(2-2‬‬
‫‪Let k 1 = k − k °‬‬
‫‪⇒ k = k1 + k °‬‬
‫‪⇒ dk = dk 1‬‬
‫‪12‬‬ ‫‪i (u1 + k ° )v‬‬
‫‪⇒ g (k) → g (k 1 ) = e −αk and e ikv = e‬‬

‫ﺒﻌﺩ ﻫﺫﻩ ﺍﻟﺘﻌﻭﻴﻀﺎﺕ ﻴﻤﻜﻥ ﻜﺘﺎﺒﺔ )‪ (2-1‬ﺒﺎﻟﺼﻴﻐﺔ‬


‫∞‬
‫‪i (k1 + k ° )v‬‬
‫‪⇒ f (x) = ∫ dk 1 e −αk . e‬‬
‫‪12‬‬

‫∞‪−‬‬
‫∞‬ ‫‪1‬‬

‫{‪= ∫ dk e − αk e ik v . e‬‬
‫‪12‬‬ ‫‪1‬‬ ‫‪ik v‬‬
‫‪°‬‬

‫∞‪−‬‬ ‫ﺛﺎﺑﺖ‬
‫∞‬

‫‪∫ dk‬‬
‫‪12‬‬ ‫‪1‬‬
‫‪= e ikov‬‬ ‫‪1‬‬
‫‪e − αk e ik v‬‬
‫∞‪−‬‬

‫ﻟﺤﺴﺎﺏ ﻗﻴﻤﺔ ﺍﻟﺘﻜﺎﻤل ﻴﺠﺏ ﺘﺤﻭﻴﺭ ﺼﻴﻐﺘﻪ ﺍﻟﺭﻴﺎﻀﻴﺔ ﻟﻠـﺼﻭﺭﺓ‬


‫∞‬
‫‪−αk − 2‬‬
‫‪∫e‬‬
‫∞‪−‬‬
‫‪dk 1‬‬

‫ﻭﺫﻟﻙ ﻷﻥ ﻗﻴﻤﺔ ﺍﻟﺘﻜﺎﻤل‬


‫∞‬ ‫‪1‬‬
‫‪D‬‬
‫‪∫ e dk = 2‬‬
‫‪− αk‬‬‫‪12‬‬
‫‪1‬‬
‫)‪.................(2 − 3‬‬
‫‪°‬‬
‫‪α‬‬

‫ﻭﻟﻠﻭﺼﻭل ﻟﻬﺫﻩ ﺍﻟﺼﻭﺭﺓ ﻴﻠﺯﻡ ﺃﻥ ﻨﺠﻌل ﺍﻷﺱ ﻤﺭﺒﻊ ﻜﺎﻤل ﺤﺴﺏ ﺍﻟﺨﻁﻭﺍﺕ‬
‫ﺍﻟﺘﺎﻟﻴﺔ‪.‬‬
‫∞‬ ‫∞‬

‫‪∫ dk e . e = ∫ dk e‬‬
‫‪1 − αk‬‬ ‫‪12‬‬
‫‪ik v‬‬ ‫‪1 − αk‬‬ ‫‪1‬‬ ‫‪12 + ik iv‬‬

‫∞‪−‬‬ ‫∞‪−‬‬

‫ﻋﺎﻤل ﻤﺸﺘﺭﻙ ﻓﻲ ﺍﻷﺱ‪.‬‬ ‫) ‪( −α‬‬ ‫ﺨﺫ‬

‫‪- ٤٧ -‬‬
‫‪PDF created with pdfFactory Pro trial version www.pdffactory.com‬‬
‫∞‬ ‫∞‬ ‫‪i‬‬
‫)‪− α(k12 − k1v‬‬
‫‪∫ dk‬‬ ‫‪= ∫ dk e‬‬
‫‪1‬‬ ‫‪− αk12 + ik1v‬‬ ‫‪1‬‬ ‫‪α‬‬
‫‪e‬‬
‫∞‪−‬‬ ‫∞‪−‬‬

‫ﻨﻁﺭﺡ ﻭﻨـﻀﻴﻑ ﺍﻟﺤـﺩ‬ ‫‪i‬‬


‫)‪(k 12 − u 1 v‬‬ ‫ﻟﻜﻲ ﻨﺘﺤﺼل ﻋﻠﻰ ﻤﺭﺒﻊ ﻜﺎﻤل ﻟﻸﺱ‬
‫‪α‬‬
‫‪v2‬‬
‫‪4α 2‬‬
‫‪i‬‬ ‫‪v2‬‬ ‫‪i‬‬ ‫‪v2‬‬
‫‪u 12 − k 1 v = k 12 − 2 − k 1 v + 2‬‬
‫‪α‬‬ ‫‪1‬‬‫‪23 α‬‬
‫‪4α‬‬ ‫‪4α‬‬
‫{‬

‫‪v2 i 1‬‬ ‫‪v2‬‬


‫‪= (k 12 −‬‬ ‫‪−‬‬ ‫‪k‬‬ ‫)‪v‬‬ ‫‪+‬‬
‫‪4α 2 α‬‬ ‫‪4α 2‬‬
‫‪2‬‬
‫‪‬‬ ‫‪iv ‬‬ ‫‪v2‬‬
‫‪= k 1 − ( )  + 2‬‬
‫‪‬‬ ‫‪2α  4α‬‬

‫ﻓﻴﺼﺒﺢ ﺍﻟﺘﻜﺎﻤل ﻋﻠﻰ ﺍﻟﺼﻭﺭﺓ‬


‫‪‬‬ ‫‪‬‬
‫‪2‬‬ ‫‪‬‬
‫‪‬‬ ‫‪‬‬ ‫‪‬‬
‫‪  1 iv ‬‬ ‫‪v 2 ‬‬
‫‪−α  k − ( ) +‬‬ ‫‪‬‬
‫‪‬‬ ‫‪‬‬
‫‪  1422α‬‬
‫‪4‬‬
‫‪3‬‬ ‫‪4α 2 ‬‬
‫∞‬ ‫‪ ‬‬ ‫‪‬‬
‫‪q‬‬ ‫‪‬‬
‫‪‬‬
‫‪∫ dk‬‬
‫‪1‬‬ ‫‪‬‬ ‫‪‬‬
‫‪e‬‬
‫∞‪−‬‬

‫∞‬ ‫‪v2‬‬ ‫∞ ‪v2‬‬


‫‪−α‬‬ ‫‪−‬‬
‫‪= ∫ dk e‬‬ ‫‪− αq 2‬‬
‫‪∫−∞dk e‬‬
‫‪1 − αq‬‬ ‫‪2‬‬
‫‪1‬‬
‫‪.e‬‬ ‫‪4α 2‬‬
‫‪=e‬‬ ‫‪4α‬‬

‫∞‪−‬‬
‫‪14243‬‬

‫ﺍﻟﺩﺍﻟﺔ ﺯﻭﺠﻴﺔ ﻭﻋﻠﻴﻪ ﻴﻤﻜﻥ ﻜﺘﺎﺒﺔ ﺍﻟﺘﻜﺎﻤل‬


‫∞‬
‫‪2∫ du 1 e −αq‬‬
‫‪2‬‬

‫‪°‬‬

‫∫ ‪1‬‬
‫ﻭﻗﻴﻤﺔ ﺍﻟﺘﻜﺎﻤل ﻴﺴﺎﻭﻱ‬
‫‪2 α‬‬
‫‪v2‬‬
‫‪D‬‬ ‫‪−‬‬
‫‪∴f (x) = e{ .‬‬ ‫‪ikov‬‬
‫‪.e‬‬ ‫‪4α‬‬
‫)‪..............(2 − 4‬‬
‫‪phase factor‬‬ ‫‪α‬‬

‫ﻻﺤﻅ ﺃﻥ ﺍﻟﺩﺍﻟﺔ )‪ f (x‬ﻓﻲ )‪ (2-4‬ﺩﺍﻟﺔ ﺘﺨﻴﻠﻴﺔ‪ .‬ﻭﻟﻜﻲ ﺘﺒﻌﺩ ﻫﺫﻩ ﺍﻟﺩﺍﻟﺔ ﻋﻠـﻰ‬
‫ﺠﺴﻡ ﻴﻠﺯﻡ ﺃﻥ ﺘﻜﻭﻥ ﺤﻘﻴﻘﻴﺔ‪ .‬ﻭﻟﻬﺫﺍ ﺍﻟﺴﺒﺏ ﻓﺎﻟﺫﻱ ﻴﻬﻤﻨﺎ ﻟﻴﺱ )‪ f (x‬ﺒل ﻤﺭﺒﻊ ﺍﻟﻘﻴﻤﺔ‬
‫ﺍﻟﻤﻁﻠﻘﺔ ﻷﻨﻬﺎ ﺤﻘﻴﻘﻴﺔ‪.‬‬

‫‪- ٤٨ -‬‬
‫‪PDF created with pdfFactory Pro trial version www.pdffactory.com‬‬
‫)‪f (x) = f * (x) f (x‬‬
‫‪2‬‬

‫‪x2‬‬ ‫‪x2‬‬
‫‪− kox‬‬ ‫‪D − 4α‬‬ ‫‪D − 4α‬‬
‫‪= (e‬‬ ‫‪e ) (e +ikox‬‬ ‫) ‪e‬‬
‫‪α‬‬ ‫‪α‬‬
‫‪v2‬‬
‫‪D −‬‬
‫‪f ( x ) = e 2α‬‬ ‫)‪........... (2 − 5‬‬
‫‪2‬‬

‫‪α‬‬

‫ﻫﺫﻩ ﺍﻟﺩﺍﻟﺔ ﺘﻤﺜل ﺠﺴﻴﻡ‪.‬‬


‫ﺍﻟﺩﺍﻟﺔ )‪ (2-5‬ﻟﻬﺎ ﻗﻴﻤﺔ ﻗﺼﻭﻯ )ﺃﻭ ﻗﻴﻤﺔ( ﻋﻨﺩ ‪ v = 0‬ﻭﺘﻌﺘﻤﺩ ﻋﻠـﻰ ﻗﻴﻤـﺔ‬
‫‪α‬‬ ‫ﻜﺒﻴﺭﺓ ﺘﻜﻭﻥ ﺍﻟﺩﺍﻟﺔ ﻋﺭﻴﻀﺔ ﻭﺍﻟﻌﻜﺱ ﺼﺤﻴﺢ ﺇﺫﺍ ﻜﺎﻨﺕ‬ ‫‪α‬‬ ‫ﺍﻟﺜﺎﺒﺕ ‪ ، α‬ﺇﺫﺍ ﻜﺎﻨﺕ‬
‫ﺼﻐﻴﺭﺓ ﻓﺈﻥ ﺍﻟﺩﺍﻟﺔ ﺘﻌﺒﺭ ﻋﻥ ﺤﺯﻤﺔ ﻤﻭﺠﻴﺔ ﻀـﻴﻘﺔ ‪ harrow‬ﻭﻜﻤـﺎ ﻗﻠﻨـﺎ ﻓـﺈﻥ‬
‫ﺘﻌﺒﺭ ﻋﻥ ﺠﺴﻴﻡ‪.‬‬ ‫)‪f (x‬‬
‫‪2‬‬

‫‪ ،‬ﻜﻤﺎ ﺸﺭﺤﻨﺎ ﻓﻲ ﺍﻟﺠـﺯﺀ ﺍﻟـﺴﺎﺒﻕ‬ ‫)‪f (x‬‬


‫‪2‬‬
‫ﺒﺈﻤﻜﺎﻨﻨﺎ ﺃﻥ ﻨﻭﺠﺩ ﻋﺭﺽ ﺍﻟﺩﺍﻟﺔ‬
‫ﻋﻥ ﺃﻗﺼﻰ ﻗﻴﻤﺔ ﻟﻬﺎ‬ ‫‪1‬‬
‫‪e‬‬
‫ﻋﻨﺩﻤﺎ ﺘﻘل ﻗﻴﻤﺘﻬﺎ ﺇﻟﻰ‬ ‫‪∆v‬‬ ‫)ﻤﻠﺤﻕ ‪ (A‬ﻨﺄﺨﺫ ﻋﺭﺽ ﺍﻟﺩﺍﻟﺔ‬
‫ﻫﺫﺍ ﻴﺤﺩﺙ ﻋﻨـﺩﻤﺎ ﻴـﺴﺎﻭﻱ ﺍﻷﺱ ﻭﺍﺤـﺩ ﺃﻱ‬ ‫‪1‬‬
‫‪e‬‬
‫ﻟﻜﻲ ﺘﻨﺫل ﺍﻟﺩﺍﻟﺔ ﻤﻥ ﺍﻟﻘﻴﻤﺔ ﺇﻟﻰ‬
‫ﻋﻨﺩﻤﺎ ‪. =1‬‬
‫‪v2‬‬
‫‪2α‬‬

‫‪⇒ v 2 = 2α ⇒ Δv ° = ± 2α‬‬
‫‪⇒ Δv = 2 2α‬‬ ‫)‪................(2 − 6‬‬

‫ﻭﺍﻟـﺫﻱ‬ ‫ﻨﻭﺠﺩ ﺍﻵﻥ ﻋﺭﺽ ﺍﻟﺩﺍﻟﺔ )‪ g (k‬ﻭﺍﻟﺘﻲ ﺘﻌﻁﻲ ﺒــ‬


‫‪2‬‬
‫) ‪g (k) = e − α (k − k °‬‬

‫ﻫـﺫﻩ ﺍﻟﺩﺍﻟـﺔ‬ ‫ﻴﻬﻤﻨﺎ ﻫﻭ ﻋﺭﺽ ﺍﻟﺩﺍﻟﺔ )‪ g2 (k‬ﻭﻟـﻴﺱ ﺍﻟﺩﺍﻟـﺔ‬


‫)‪2 g (k‬‬
‫) ‪g 2 (k) = e − 2 α (k − k °‬‬

‫‪- ٤٩ -‬‬
‫‪PDF created with pdfFactory Pro trial version www.pdffactory.com‬‬
‫ﻋﺭﻀﻬﺎ ﻴﻭﺠـﺩ ﺒـﻨﻔﺱ‬ ‫‪k =k°‬‬ ‫ﻭﻟﻴﺱ ﺤﻭل ‪ k = 0‬ﺒل ﺤﻭل‬ ‫‪k°‬‬ ‫ﻤﺘﻤﺭﻜﺯﺓ ﺤﻭل‬
‫ﻤﻥ ﻗﺴﻤﺘﻬﺎ ﺍﻟﻘﺼﻭﻯ‪.‬‬ ‫‪1‬‬
‫‪e‬‬
‫ﺍﻟﻁﺭﻴﻘﺔ‪ ،‬ﻨﻭﺠﺩ ﺍﻟﻌﺭﺽ ﻋﻨﺩﻤﺎ ﺘﻘل ﺍﻟﺩﺍﻟﺔ ﺇﻟﻰ‬
‫‪1‬‬
‫( )‪Δk = 2ΔΔ° = (2‬‬ ‫)‬ ‫)‪...................(2 − 7‬‬
‫‪2α‬‬

‫‪∆k‬‬ ‫ﻓﻲ )‪ (2-6‬ﻭﺍﻟﻌﺭﺽ ﻓﻲ‬ ‫‪∆v‬‬ ‫ﻻﺤﻅ ﺍﻟﺘﻨﺎﺴﺏ ﺍﻟﻌﻜﺴﻲ ﻓﻲ ﻋﺭﺽ ﺍﻟﺩﺍﻟﺔ‬
‫ﻓـﻲ ﻓـﻀﺎﺀ ‪ x‬ﻜﻠﻤـﺎ‬ ‫)‪f (x‬‬ ‫ﻓﻲ ﺍﻟﻌﻼﻗﺔ )‪ (2-7‬ﻫﺫﺍ ﻴﻌﻨﻲ ﺃﻨﻪ ﻜﻠﻤﺎ ﺘﻤﻴﺯﺕ ﺍﻟﺩﺍﻟﺔ‬
‫ﺍﻨﺒﺴﻁﺕ ﻭﺍﺴﺘﻌﺭﻀﺕ ﻓﻲ ﻓﻀﺎﺀ ‪ k‬ﻭﺍﻟﻌﻜﺱ ﺼﺤﻴﺢ‪.‬‬
‫‪∆ u , ∆v‬‬ ‫ﻭﻤﻥ ﺍﻟﻤﻬﻡ ﺃﻥ ﻨﻨﻅﺭ ﺇﻟﻰ ﺤﺎﺼل ﻀﺭﺏ ﺍﻟﻌﺭﻀﻴﻥ‬
‫‪1‬‬
‫‪Δ u . Δv = 2‬‬ ‫‪.‬‬ ‫‪2 2α = 4‬‬ ‫)‪...............(2 − 8‬‬
‫‪2α‬‬

‫ﺍﻟﺫﻱ ﻴﻬﻤﻨﺎ ﻓﻲ ﺍﻟﻌﻼﻗﺔ )‪ (2-8‬ﻟﻴﺱ ﺍﻟﻘﻴﻤﺔ ﺍﻟﻌﺩﺩﻴﺔ‪ ،‬ﺍﻟﺫﻱ ﻴﻬﻡ ﻫﻭ ﺃﻥ ﺤﺎﺼل‬


‫ﻻ ﻴﻌﺘﻤﺩ ﻋﻠﻰ ﻗﻴﻤﺔ ﺍﻟﺜﺎﺒﺕ ‪ .α‬ﻫﺫﻩ ﺨﺎﺼﻴﺔ ﻋﺎﻤﺔ ﻟﻜـل ﺍﻟـﺩﻭﺍل‬ ‫‪∆ u , ∆v‬‬ ‫ﺍﻟﻀﺭﺏ‬
‫ﺘﺭﺘﺒﻁ ﺒﺒﻌﻀﻬﺎ ﺒﺘﺤﻭﻴﻼﺕ ﻓﻭﺭﻴﺭ ﻜﻤﺎ ﻫﻭ ﻤﻭﻀﺢ ﻓﻲ ﺸﻜل ‪.2-1‬‬
‫‪This is a general property of functions that are Fourier trans forms of‬‬
‫‪each other.‬‬

‫‪- ٥٠ -‬‬
‫‪PDF created with pdfFactory Pro trial version www.pdffactory.com‬‬
‫ﻭﻨﺨﻠﺹ ﻤﻥ ﻫﺫﺍ ﺍﻟﺠﺯﺀ ﺒﺎﻻﺴﺘﻨﺘﺎﺝ ﺃﻨﻪ ﻤﻥ ﺍﻟﻤﺴﺘﺤﻴل ﺃﻥ ﻨﺠﻌل ﻜـﻼﹰ ﻤـﻥ‬
‫ ﺍﻷﻤﺭ ﺍﻟـﺫﻱ‬،‫ ﻭﻫﺫﻩ ﺨﺎﺼﻴﺔ ﻋﺎﻤﺔ ﻟﻠﺤﺯﻡ ﺍﻟﻤﻭﺠﻴﺔ‬.‫ﺫﺍﺕ ﻗﻴﻡ ﺼﻐﻴﺭﺓ‬ ∆k ‫ﻭ‬ ∆v

.‫ﻴﺘﺭﺘﺏ ﻋﻠﻴﻪ ﻨﺘﺎﺌﺞ ﻫﺎﻤﺔ ﺠﺩﺍﹰ ﻓﻲ ﻤﻴﻜﺎﻨﻴﻜﺎ ﺍﻟﻜﻡ‬


It is impossible to make both Δ v and Δk small.
This is a general feature of wouae packets, but we shall soon see
rat it has some very deep implications for quantum mechanics.

Figure 2-4 Relation between wave packet and its Fourier trans-
form for a square-shaped wave packet.

- ٥١ -
PDF created with pdfFactory Pro trial version www.pdffactory.com
‫ﺍﻨﺘﺸﺎﺭ ﺍﻟﺤﺯﻡ ﺍﻟﻤﻭﺠﻴﺔ‬
‫‪THE PROPAGATION OF WAVE PACKETS‬‬
‫ﻟﻨﺘﺤﺼل ﻋﻠﻰ ﺩﺍﻟﺔ )‪ ،f (x‬ﻨـﻭﺩ‬ ‫‪e ikx‬‬ ‫ﺒﻌﺩﻤﺎ ﺩﺭﺴﻨﺎ ﺘﺭﺍﻜﺏ ﻤﻭﺠﺎﺕ ﻤﻥ ﻨﻭﻉ‬
‫ﺍﻵﻥ ﻤﻨﺎﻗﺸﺔ ﻜﻴﻔﻴﺔ ﺇﻨﺘﺸﺎﺭ ﻫﺫﻩ ﺍﻟﻤﻭﺠﺎﺕ؟ ﺒﻼ ﺸﻙ ﺇﻥ ﺍﻨﺘﺸﺎﺭ ﺤﺯﻤﺔ ﻤﻭﺠﻴﺔ ﻴﻌﺘﻤﺩ‬
‫ﻋﻠﻰ ﺇﻨﺘﺸﺎﺭ ﻜل ﻤﻥ ﺍﻟﻤﻭﺠﺎﺕ ﺍﻟﻤﻜﻭﻨﺔ ﻟﻬﺎ‪ .‬ﻓﻠﻭ ﺃﺨﺫﻨﺎ ﻤﻭﺠﺔ ﺘﺘﻐﻴﺭ ﻤﻜﺎﻨﻴﺎﹰ ﻋﻠـﻰ‬
‫ﻤﺤﻭﺭ ‪) X‬ﺒﻴﻨﻤﺎ ﻻ ﺘﻌﺘﻤﺩ ﺍﻟﻤﻭﺠﺔ ﻤﻥ ﺤﻴﺙ ﺍﻹﺤﺩﺍﺜﻴﺎﺕ ‪ t , y‬ﻋﻠﻰ ﺍﻟﺯﻤﻥ(‪ .‬ﻫـﺫﻩ‬
‫ﺍﻟﻤﻭﺠﺎﺕ ﺘﺴﻤﻰ ﻤﻭﺠﺎﺕ ﻤﺴﺘﻭﻴﺔ ‪ ،plane wanes‬ﻴﻤﻜﻥ ﻜﺘﺎﺒﺔ ﻫﺫﻩ ﺍﻟﻤﻭﺠﺎﺕ ﻜﺩﺍﻟﺔ‬
‫ﻓﻲ ﺍﻟﺯﻤﻥ ‪ t‬ﻭﺍﻟﻤﻭﺠﺔ ‪ X‬ﺒﺎﻟﺼﻴﻐﺔ‪.‬‬

‫‪e iuv − iwt‬‬ ‫)‪...... (2-9‬‬


‫‪w = 2Dv‬‬
‫‪2D‬‬ ‫ﺤﻴﺙ‬
‫=‪k‬‬
‫‪λ‬‬

‫ﺒﺎﻟﺘﻌﻭﻴﺽ ﻓﻲ )‪(2-9‬‬
‫‪2D‬‬ ‫‪v‬‬
‫‪v − i 2 Dvt‬‬ ‫) ‪2 Di ( − vt‬‬
‫‪ei‬‬ ‫‪λ‬‬
‫‪=e‬‬ ‫‪D‬‬
‫)‪................(2 − 10‬‬

‫=‪v‬‬
‫‪c‬‬
‫ﻟﻭ ﺃﺨﺫﻨﺎ ﻜﺤﺎﻟﺔ ﺨﺎﺼﺔ؛ ﺇﻨﺘﺸﺎﺭ ﻤﻭﺠﺎﺕ ﺍﻟﻀﻭﺀ ﻓﻲ ﺍﻟﻔـﻀﺎﺀ ﺤﻴـﺙ‬
‫‪λ‬‬
‫ﻟﻠﻀﻭﺀ‪ ،‬ﺒﺎﻟﺘﻌﻭﻴﺽ ﻓﻲ )‪.(2-10‬‬
‫‪v c‬‬ ‫‪2D‬‬
‫( ‪2 Di‬‬ ‫)‪− t‬‬ ‫) ‪i ( v − ct‬‬
‫‪e‬‬ ‫‪λ λ‬‬
‫‪=e‬‬ ‫‪λ‬‬
‫) ‪= e ik ( v − ct‬‬ ‫)‪.................(2 − 12‬‬

‫ﺨﺫ ﺍﻵﻥ ﺘﺭﺍﻜﺏ ﺍﻟﻤﻭﺠﺎﺕ ﺴﻌﺘﻬﺎ )‪ g (k‬ﻤﻥ ﻫﺫﺍ ﺍﻟﻨﻭﻉ ﻤﻥ ﺍﻟﻤﻭﺠﺎﺕ‪ ،‬ﻓﺒﻌﺩ‬
‫ﻤﺭﻭﺭ ﺯﻤﻥ ﻗﺩﺭﻩ ‪t‬‬
‫∞‬
‫)‪f (x 1 t) = ∫ du g (u) e iu (v −ct) = eik (v − ct‬‬
‫∞‪−‬‬

‫‪- ٥٢ -‬‬
‫‪PDF created with pdfFactory Pro trial version www.pdffactory.com‬‬
‫ﺇﻥ ﻫﺫﻩ ﺍﻟﺩﺍﻟﺔ )‪ f (x – ct‬ﻟﻬﺎ ﻨﻔﺱ ﺍﻟﺸﻜل ﺍﻟﺫﻱ ﺒﺩﺃﺕ ﺒﻪ ﻗﺒل ﺍﻻﻨﺘﺸﺎﺭ ﻤـﺎ‬
‫ﻋﺩﺍ ﺃﻥ ﻤﻭﻗﻌﻬﺎ ﺒﺩﻻﹰ ﻤﻥ ﺃﻥ ﻴﻜﻭﻥ ‪ X‬ﺃﺼـﺒﺢ ﺍﻵﻥ ﻤﺘﺭﻜـﺯ )ﺃﻭ ﻤﺘﺤﻴـﺯ( ﻋﻨـﺩ‬
‫‪v = ct ⇐ v − ct = o‬‬

‫ﻫﺫﺍ ﻴﻌﻨﻲ ﺃﻥ ﺤﺯﻡ ﺍﻟﻀﻭﺀ ﺍﻟﻤﻭﺠﻴﺔ ﺘﻨﺘﺸﺭ ﻓﻲ ﺍﻟﻔﻀﺎﺀ ﺒﻼ ﺘﺸﻭﻴﻪ‪.‬‬


‫‪Thus the wave packet go light waves propagates, without any‬‬
‫‪distortion, with velocity, the velsal'y of light.‬‬
‫ﻭﻟﻜﻥ ﺍﻟﺫﻱ ﻴﻌﻨﻴﻨﺎ ﻫﻨﺎ ﺍﻨﺘﺸﺎﺭ ﻤﻭﺠﺎﺕ‪ ،‬ﻴﻔﺘﺭﺽ ﺃﻨﻬﺎ ﺘﹸﻌﺒﺭ ﻋﻥ ﺠﺴﻴﻤﺎﺕ‪ ،‬ﻓﻲ‬
‫ﻫﺫﻩ ﺍﻟﺤﺎﻟﺔ ﻟﻥ ﻨﺴﺘﻁﻴﻊ ﺍﺴﺘﺨﺩﺍﻡ ﺍﻟﻌﻼﻗﺔ‪.‬‬

‫‪w = kc‬‬ ‫)‪... (2-13‬‬


‫‪W (k) = kc‬‬
‫ﻻﺤﻅ ﻫﺫﻩ ﺍﻟﻌﻼﻗﺔ ﺼﺤﻴﺤﺔ ﻓﻘﻁ ﻓﻲ ﺤﺎﻟﺔ ﺍﻨﺘﺸﺎﺭ ﺤﺯﻤﺔ ﻀﻭﺀ ﻤﻭﺠﻴﺔ ﻓـﻲ‬
‫ﺍﻟﻔﻀﺎﺀ‪ .‬ﻭﻟﻜﻲ ﺘﻨﻁﺒﻕ ﺩﺭﺍﺴﺘﻨﺎ ﻋﻠﻰ ﺍﻨﺘﺸﺎﺭ ﺤﺯﻤﺔ ﻤﻭﺠﻴﺔ )ﺠﺴﻴﻡ( ﻴﻠﺯﻡ ﺍﺴـﺘﺨﺩﺍﻡ‬
‫‪w = (k) ≠ kc or kv‬‬ ‫ﺍﻟﻌﻼﻗﺔ ﺍﻟﻌﻼﻤﺔ ﻟـ )‪ w (k‬ﺤﻴﺙ‬

‫ﻓﻲ ﻫﺫﻩ ﺍﻟﺤﺎﻟﺔ ﻨﻌﻴﺩ ﻜﺘﺎﺒﺔ ﻤﻌﺎﺩﻟﺔ )‪ (2-11‬ﺒﺎﻟﺼﻴﻐﺔ‪.‬‬


‫)‪ik (v − ct‬‬
‫)‪f (x 1 t) = ∫ dk g (k) e ik (v − ct‬‬

‫ﻟﺘﺼﺒﺢ ﻓﻲ ﺤﺎﻟﺔ ﺠﺴﻡ ﺤﺭ ﺍﻟﺤﺭﻜﺔ‬

‫‪= dk g(k) e ikv − iw (k)t‬‬ ‫)‪..........(2 − 14‬‬

‫ﻻﺤﻅ ﺃﻨﻨﺎ ﻟﻡ ﻨﻌﺭﻑ ﺒﻌﺩ ﺍﻟﺼﻴﻐﺔ ﺍﻟﺤﻘﻴﻘﻴﺔ ﻟﻁﺒﻴﻌﺔ ﺍﻟﺩﺍﻟﺔ )‪ ،w (k‬ﻭﻟﻜﻥ ﺨـﺫ‬
‫ﻓﻲ ﻓﻀﺎﺀ ‪.u‬‬ ‫‪k°‬‬ ‫ﺍﻵﻥ ﺤﺯﻤﺔ ﻤﻭﺠﻴﺔ ﻤﺘﻤﺭﻜﺯﺓ ﻭﻤﺘﺤﺒﺯﺓ ﺠﺩﺍﹰ ﺤﻭل‬
‫ﺴﺘﻜﻭﻥ ﻜﺒﻴﺭﺓ‪ .‬ﻫﺫﺍ ﻗﺩ ﻻ ﻴﻌﻨﻲ ﺃﻥ‬ ‫ﻫﺫﺍ ﻴﻌﻨﻲ ﺃﻥ‪ α‬ﻓﻲ ﺍﻟﻌﻼﻗﺔ‬
‫‪2‬‬
‫) ‪g (k) = e −α(k − k °‬‬

‫)‪ f (x‬ﺴﺘﻜﻭﻥ ﻤﺘﺤﺒﺯﺓ ﻓﻲ ﻓﻀﺎﺀ ‪ v‬ﻭﻟﻜﻥ ﻫﺫﺍ ﺍﻟﺘﻘﺭﻴﺏ ﺴﻴﺴﻬل ﺤﺴﺎﺒﺎﺘﻨﺎ‪.‬‬

‫‪- ٥٣ -‬‬
‫‪PDF created with pdfFactory Pro trial version www.pdffactory.com‬‬
‫ﻻ‬ ‫)‪w (k‬‬ ‫ﻤﻊ ﺍﻓﺘـﺭﺍﺽ ﺃﻥ‬ ‫‪k°‬‬ ‫ﺒﻤﺎ ﺃﻥ ﺍﻟﺘﻜﺎﻤل ﻓﻲ )‪ (2- 14‬ﻤﺘﻤﺭﻜﺯ ﺤﻭل‬
‫ﺘﺘﻐﻴﺭﻩ ﺒﺸﺩﺓ ﻤﻊ ﺘﻐﻴﺭ ‪.k‬‬
‫)‪dw(k‬‬ ‫‪1‬‬ ‫)‪d 2 w (k‬‬
‫( ) ‪w (k) ≅ w (k ° ) + (k − k °‬‬ ‫( ‪) k + (k − k ° ) 2‬‬ ‫)‬ ‫)‪............(2 − 15‬‬
‫‪dk‬‬ ‫‪°‬‬ ‫‪2‬‬ ‫‪dk 2 k°‬‬

‫ﻴﻤﺜل ﺴﺭﻋﺔ‬ ‫(‬


‫‪dw‬‬
‫)‬ ‫ﺜﺎﺒﺕ ﻷﻨﻪ ﻻ ﻴﻌﺘﻤﺩ ﻋﻠﻰ ‪ .k‬ﺍﻟﺤﺩ ﺍﻟﺜﺎﻨﻲ‬ ‫) ‪w (k °‬‬ ‫ﺍﻟﺤﺩ ﺍﻷﻭل‬
‫‪dk k = k °‬‬
‫ﺍﻟﻤﺠﻤﻭﻋﺔ ‪ -group velocity‬ﻜﻤﺎ ﺸﺭﺤﻨﺎ ﻓﻲ ﺍﻟﺘﻤﻬﻴﺩ‪ -‬ﻫﻲ ﺍﻟﺴﺭﻋﺔ ﺍﻟﺘﻲ ﺘـﺴﻴﺭ‬
‫ﺒﻬﺎ ﺍﻟﺤﺯﻤﺔ ﺍﻟﻤﻭﺠﻴﺔ‪.‬‬
‫‪dw‬‬
‫⇒‬ ‫(‬ ‫)‬ ‫‪= vg‬‬
‫‪dk k = k°‬‬
‫‪and‬‬ ‫)‪(2-16‬‬
‫‪1 d2w‬‬
‫(‬ ‫)‬ ‫‪=β‬‬
‫‪2 dk 2 k = k °‬‬

‫ﺒﺎﻓﺘﺭﺍﺽ ﺃﻥ‪:‬‬
‫‪k1 = k − k °‬‬
‫‪⇒ dk 1 = dk‬‬

‫ﻓﺈﻥ ﺍﻟﺤﺯﻤﺔ ﺍﻟﻤﻭﺠﻴﺔ ﺘﻌﺘﻤﺩ ﻋﻠﻰ ﺍﻟﺯﻤﻥ ﺒﺎﻟﺼﻴﻐﺔ‪.‬‬


‫∞‬
‫‪i (k1 + k ° ) v − i  w (k ° ) + k1vg + k12β  t‬‬
‫‪f (x 1 t) = ∫ dk 1 e −αk e‬‬
‫‪12‬‬ ‫‪‬‬ ‫‪‬‬

‫∞‪−‬‬

‫ﺨﺫ ﺍﻟﺤﺩﻭﺩ ﺍﻟﺜﺎﺒﺘﺔ ﺨﺎﺭﺝ ﺍﻟﺘﻜﺎﻤل‪.‬‬


‫∞‬
‫‪∫ dk‬‬
‫‪ik ° v − i w(k ° )t‬‬ ‫‪1‬‬
‫‪12‬‬ ‫‪1‬‬ ‫‪1‬‬ ‫‪12βt‬‬
‫‪=e‬‬ ‫∞‪−‬‬
‫‪e −αk . e ik v. e −ik vgt. e −ik‬‬
‫∞‬
‫‪ikov − iw(k ° )t‬‬ ‫)‪1 (v − vgt‬‬
‫‪∫ dk e‬‬
‫‪1 − αk12‬‬ ‫‪12βt‬‬
‫‪=e‬‬ ‫‪e ik‬‬ ‫‪e −ik‬‬
‫∞‪−‬‬

‫)‪........(2-17‬‬
‫ﻭﺒﻨﻔﺱ ﺍﻟﻁﺭﻴﻘﺔ ﺍﻟﺘﻲ ﺃﺠﺭﻴﻨﺎﻫﺎ ﻓﻲ ﺍﻟﺤﺎﻟﺔ ﺍﻷﻭﻟﻰ‪ ،‬ﻴﻤﻜﻥ ﻋﻤل ﺍﻟﺘﻜﺎﻤل ﻋﻥ‬
‫ﻁﺭﻴﻕ ﺇﻜﻤﺎل ﺍﻟﻤﺭﺒﻊ ﻓﻲ ﺍﻷﺱ‪ ،‬ﺨﺫ ﺍﻷﺱ ﻓﻘﻁ‪.‬‬
‫‪12 + ik1 (v − μgt) − ik12 βt‬‬ ‫)‪− (α + iβ t ) k12 + ik1 (v − μ g t‬‬
‫‪e − αk‬‬ ‫‪=e‬‬

‫‪- ٥٤ -‬‬
‫‪PDF created with pdfFactory Pro trial version www.pdffactory.com‬‬
.‫ﻋﺎﻤل ﻤﺸﺘﺭﻙ‬ − (α + iβ t) ‫ﺨﺫ‬
 (v − ν g t) 
− (α + iβ t)  k12 − ik1 
 (α = iβ t ) 
=e

.‫ﻟﻠﺤﺼﻭل ﻋﻠﻰ ﻤﺭﺒﻊ ﻜﺎﻤل ﻨﻁﺭﺡ ﻭﻨﻀﻴﻑ ﺍﻟﺤﺩ‬


(v − ν g t)2
4(α + iβ t)
 iu1 (u − v g t)  (x − v g t) 2 ( x − v g t) 2
− (α + iβ t)  u12 − + −
α + iββ  4 (α + iββ) 4 (α + iβ t)
 
e

‫ﺨﺫ ﻫﺫﻩ ﺍﻹﺸﺎﺭﺓ ﻤﺸﺘﺭﻙ ﻤﻊ ﺍﻟﺤﺩ ﺍﻷﻭل‬


 ik 1 (u − v g t) (u − v g t) 2  ( x − v t) 2
− (α + iβ t)  k12 −  − g
−  4 (α + iβ t)
α + iβ t 4 (α + iββ)
 
e

i (u − v g t) 2 2
  1 ( x − v g t)
− (α + iβ t)  k1 −  +
 2 ( α + iβ t  4 (α + iβ t)
e ............(2 − 18)

:‫( ﻨﺘﺤﺼل ﻋﻠﻰ‬2-17) ‫( ﻓﻲ‬2-18) ‫ﺒﺘﻌﻭﻴﺽ‬


2
 (v − µ g t )  ( x − vg t ) 2
∞ − (α + i β t )  k1 −  −
2 (α = i β t ) 
∫ dk 
i ( k° x − w ( k° ) t )
f ( x1t ) = e 1
e e 4 ( α + i βt )
1424 3
−∞ ‫ﺛﺎﺑﺖ‬ ‫اﻟﺤﺪ‬ ‫ھﺬا‬ ‫اﺑﺖ‬

( x− y g t ) 2  i ( x −u gt)  2
− ∞ − ( α + i β t )  k1 − 
2 (α + iβ t ) 
= e i ( k° x − w ( k° ) t e ∫ dk e 
4 (α + iβ t ) 1

−∞
1444424444
3

− (x − u g t) 2
i (u ° x − w (k ° ) t 4 (α + iβ t) D
f (x 1 t) = e e
(α + iβ t)
(v − u g t) 2 ( v − u g t) 2
2 − − D D
f (x 1 t) = f (n 1 t) f * (x 1 t) = e ° . e 4 (α + iβ t) 4 (α − iβ t)

(α + i β t ) ( α − i β t)
 − (v − u g t) 2 ( 2α ) 
 2 α 2 + β 2t 2   D 
= 1. e   (α 2 + β 2 t 2 ) 
  
 
(v − u g t) 2 D

2 (α + β 2 t 2 /αα (α 2 + β 2 t 2 )
=e

- ٥٥ -
PDF created with pdfFactory Pro trial version www.pdffactory.com
‫ﺒﻘﺴﻤﺔ ﺍﻷﺱ ﻓﻲ ﺍﻟﺒﺴﻁ ﻭﺍﻟﻤﻘﺎﻡ ﻋﻠﻰ ‪. α‬‬
‫‪‬‬ ‫‪D − 2α ‬‬
‫‪v2‬‬

‫‪f (x) = e ‬‬


‫‪2‬‬
‫ﺒﻤﻘﺎﺭﻨﺔ ﺍﻟﺼﻴﻐﺔ ﻓﻲ )‪ (2-19‬ﺒـ )‪(2-5‬‬
‫‪‬‬ ‫‪α‬‬ ‫‪‬‬

‫ﻭ‪α‬‬ ‫)‪(v − u g t‬‬ ‫ﻨﺠﺩ ﺃﻥ ﻫﻨﺎﻙ ﺘﻤﺎﺜل ﻤﻊ ﻤﻼﺤﻅﺔ ﺃﻥ ‪ v‬ﻓﻲ )‪ (2-5‬ﺘﻐﻴﺭﺕ ﻟـ‬
‫‪α+‬‬
‫‪β2 t 2‬‬
‫ﺘﺤﻭﻟﺕ ﻟـ‬
‫‪α‬‬

‫ﻭﻟﻜـﻥ ﻋـﺭﺽ‬ ‫‪ug‬‬ ‫ﺘﻤﺜل ﺤﺯﻤﺔ ﻤﻭﺠﻴﺔ ﺘﺴﻴﺭ ﺒﺴﺭﻋﺔ‬ ‫) ‪f (x 1 t‬‬


‫‪2‬‬
‫ﻭﻟﻬﺫﺍ ﻓﺈﻥ‬
‫ﻫﺫﻩ ﺍﻟﺤﺯﻤﺔ ﻏﻴﺭ ﻤﺤﺩﺩ ﻭﻟﻜﻥ ﻴﺘﻐﻴﺭ ﻤﻊ ﺍﻟﺯﻤﻥ‪ ،‬ﻻﺤﻅ ﺃﻥ ﻋﺭﺽ ﺍﻟﺩﺍﻟﺔ )‪.(2-19‬‬
‫‪Δv = 2. Δ v ° = 2. 2 (α + β 2 t 2 /αα‬‬
‫‪or‬‬
‫‪β 2 t 2 12‬‬
‫‪Δv = 8α (1 +‬‬ ‫)‬ ‫)‪...............(2 − 20‬‬
‫‪α2‬‬

‫ﻤﻥ ﺍﻟﻤﻌﺎﺩﻟﺔ )‪ (2-20‬ﻴﺘﻀﺢ ﺃﻥ ﻋﺭﺽ ﺍﻟﺩﺍﻟﺔ ﻴﺯﺩﺍﺩ ﻤﻊ ﺍﻟﺯﻤﻥ ﺃﻱ ﺃﻥ ﺍﻟﺩﺍﻟﺔ‬


‫ﺘﻨﺒﺴﻁ ﻤﻊ ﻤﺭﻭﺭ ﺍﻟﺯﻤﻥ ﻭﺫﻟﻙ ﻤﻘﺎﺭﻨﺔ ﺒﻌﺭﻀﻬﺎ ﻋﻨﺩ ‪t = 0‬‬

‫‪Δv = ( t = 0 ) = 8α‬‬

‫ﺃﻤﺎ ﻋﻨﺩ ﺃﻱ ﺯﻤﻥ ﺁﺨﺭ ﻓﺈﻥ ﻋﺭﺽ ﺍﻟﺩﺍﻟﺔ ﻴﻌﻁﻲ ﺒﺎﻟﺼﻴﻐﺔ )‪ (2- 20‬ﺇﻥ ﻤﻌﺩل‬
‫ﺍﻻﻨﺒﺴﺎﻁ ﻓﻲ ﺍﻟﺩﺍﻟﺔ ﺴﻴﻜﻭﻥ ﺼﻐﻴﺭﺍﹰ ﺇﺫﺍ ﻜﺎﻨﺕ ‪ α‬ﻜﺒﻴﺭﺓ ﺃﻱ ﺇﺫﺍ ﻜﺎﻨﺕ ﺍﻟﺩﺍﻟﺔ ﻤﻨﺒﺴﻁﺔ‬
‫ﻜﺜﻴﺭﺍﹰ ﻋﻨﺩ ﺍﻟﺒﺩﺍﻴﺔ‪.‬‬

‫‪- ٥٦ -‬‬
‫‪PDF created with pdfFactory Pro trial version www.pdffactory.com‬‬
‫ﻤﻥ ﺍﻟﺤﺯﻡ ﺍﻟﻤﻭﺠﻴﺔ ﺇﻟﻰ ﻤﻌﺎﺩﻟﺔ ﺸﺭﻭﺩﻨﺠﺭ‪:‬‬
‫‪FROM WAVE PACKETS TO THE SCHRöDINGER EQUATION‬‬

‫ﺘﻤﺜل ﺠﺴﻴﻡ ﻜﻤﻴﺔ ﺍﻟﺤﺭﻜﺔ ﻟﻪ‬ ‫‪{f ( x t ) = ∫ dk g (u) e‬‬


‫‪1‬‬
‫‪iku −iw ( k ) t‬‬
‫ﺇﺫﺍ ﻜﺎﻨﺕ ﺍﻟﺩﺍﻟﺔ }‬
‫‪.........‬‬

‫ﺘﺴﺎﻭﻱ ‪ p‬ﻭﻁﺎﻗﺘﻪ ﺍﻟﺤﺭﻜﻴﺔ )‪ ،(p2 /2m‬ﻓﺈﻥ ﻫﺫﺍ ﻴﺴﺘﻠﺯﻡ ﺃﻥ ﺴﺭﻋﺔ ﺍﻟﻤﺠﻤﻭﻋـﺔ ‪ug‬‬
‫ﺘﻌﻁﻰ ﺒﺎﻟﻤﻌﺎﺩﻟﺔ‪.‬‬
‫‪dw‬‬ ‫‪p‬‬
‫= ‪ug‬‬ ‫)‪= particle velocity ⇒ = .................(2 − 21‬‬
‫‪dk‬‬ ‫‪m‬‬

‫ﻭﻤﻥ ﺍﻓﺘﺭﺍﻀﺎﺕ ﻤﻴﻜﺎﻨﻴﻜﺎ ﺍﻟﻜﻡ‪ ،‬ﻓﺈﻥ ﺃﻱ ﺇﺸﻌﺎﻉ ﻟﻪ ﺘﺭﺩﺩ ‪ w‬ﻓﺈﻨﻪ ﻴﻘﺎﺒﻠﻪ ﻁﺎﻗﺔ‬
‫‪ E‬ﺘﻌﻁﻲ ﺒﺎﻟﻌﻼﻗﺔ‪.‬‬
‫‪E=hw‬‬
‫‪or‬‬
‫‪E‬‬
‫=‪w‬‬
‫‪h‬‬

‫ﺇﺫﺍ ﻜﺎﻨﺕ ‪ E‬ﻫﻲ ﻁﺎﻗﺔ ﺠﺴﻴﻡ ﺤﺭ‪،‬ﻓﺈﻨﻬﺎ ﺘﻤﺜل ﻁﺎﻗﺘﻪ ﺍﻟﺤﺭﻜﻴﺔ ‪ p2/2m‬ﺃﻱ ﺃﻥ‪.‬‬
‫‪( p 2 / 2m) p 2‬‬
‫=‪w‬‬ ‫=‬ ‫)‪(2 − 23‬‬
‫‪h‬‬ ‫‪2mh‬‬

‫ﻭﻋﻠﻴﻪ ﻓﻤﻥ ﺍﻟﻤﻤﻜﻥ ﺃﻥ ﻨﻜﺘﺏ ﻤﺘﺠﻪ ﺍﻟﻤﻭﺠﺔ ‪) k‬ﺒﻨﻔﺱ ﺍﻟﻁﺭﻴﻘﺔ( ﻭﻤﻥ ﻤﻌﺎﺩﻟﺔ‬
‫ﺩﻴﺒﺭﻭﻟﻲ ‪. λ = n‬‬
‫‪p‬‬
‫‪2D 2 D‬‬ ‫‪p‬‬ ‫‪p‬‬
‫=‪k‬‬ ‫=‬ ‫=‬ ‫=‬ ‫)‪.................(2 − 24‬‬
‫‪λ (h ) ( h ) h‬‬
‫‪p‬‬ ‫‪2D‬‬

‫ﻭﻟﻬﺫﺍ ﺍﻟﺴﺒﺏ ﻴﻤﻜﻨﻨﺎ ﺇﻋﺎﺩﺓ ﻜﺘﺎﺒﺔ ﺍﻟﺼﻴﻐﺔ )‪ (2-14‬ﺒﺩﻻﻟﺔ ‪) p‬ﺒﺩﻻﹰ ﻤﻥ ‪(k‬‬


‫‪1‬‬
‫= )‪ψ (x 1 t‬‬
‫‪2Dh‬‬
‫‪∫ dp‬‬ ‫‪ψ (p ) e i (px −Et)/h‬‬ ‫)‪............(2 − 25‬‬

‫ﺘﻤﺜل ﺍﻟﺤل ﺍﻟﻌـﺎﻡ ﻟﻠﻤﻌﺎﺩﻟـﺔ ﺍﻟﺘﻔﺎﻀـﻠﻴﺔ‬ ‫)‪ψ (x 1 t‬‬ ‫ﺤﻴﺙ ﺃﻥ ﺍﻟﺤﺯﻤﺔ ﺍﻟﻤﻭﺠﻴﺔ‬
‫ﺍﻟﺠﺯﺌﻴﺔ ‪.partial differentiable.‬‬

‫‪- ٥٧ -‬‬
‫‪PDF created with pdfFactory Pro trial version www.pdffactory.com‬‬
∂ ψ (x 1 t) 1
∫ dp φ (p) E e
i (px − Et)/t
iD =
∂t 2Dh
1 p 2 i( px − Et) /D
=
2Dh
∫ dp φ (p) 2m e ............(2 − 26)

.x ‫ﻤﺭﺘﻴﻥ ﺒﺎﻟﻨﺴﺒﺔ ﻟـ‬ ψ (x 1 t) ‫( ﻴﻤﻜﻥ ﺍﻟﺤﺼﻭل ﻋﻠﻴﻬﺎ ﺒﺘﻔﺎﻀل‬2-26) ‫ﺤﻴﺙ ﺃﻥ‬


∂ ∂ψ ∂  1 ∂ 
( )=
∂x ∂x 
∂x  2Dh ∫ dp φ (p) e i (px − Et)/D 
∂x 
∂  1 ip 
= 
∂x  2Dh ∫ dp φ (p) . e i (px − Et)/h 
h 
1 ip ip i (px − Et)/t 
=
2Dh
∫ dp φ (p) h . h
e 
p 2 i (px − Et) /t
=
1
∫ dp φ ( p ) ( − )e ]................(2 − 27)
2Dh D2

‫ ﻴﻠﺯﻡ ﻀﺭﺏ‬،‫( ﻤﻥ ﺍﻟﻁﺭﻑ ﺍﻷﻴﻤﻥ‬2-26) ‫( ﺒـ‬2-27) ‫ﻭﻟﻜﻲ ﺘﺘﺴﺎﻭﻯ ﻤﻌﺎﺩﻟﺔ‬


‫ ﺃﻱ‬، − D ‫( ﻓﻲ‬2-27) ‫ﻁﺭﻓﻲ‬
2

2m
h2 ∂2ψ D2 p 2 i (px − Et)/D
− =
1
∫ dp φ (p) ( − ) ( − )e ]
2m ∂x 2 2Dh 2m D2
p 2 i (px − Et)/D
=
1
∫ dp φ (p) ( )e ]
2Dh 2m

:‫ﻭﻋﻠﻴﻪ ﻓﺈﻥ‬
∂ψ (x 1 t) h 2 ∂ ψ (v1 t)
2

ih =− ................(2 − 28)
∂t 2m ∂v 2

.‫ﻭﻫﺫﻩ ﺼﺤﻴﺤﺔ ﻟﺠﺴﻡ ﺤﺭ‬


The uncertainty Relations (‫ﻋﻼﻗﺎﺕ ﺍﻟﻼﺘﺤﺩﻴﺩ )ﺍﻟﻼﺘﻌﻴﻴﻥ‬

‫ﺃﺤﺩ ﺍﻟﻨﺘﺎﺌﺞ ﺍﻟﻤﻬﻤﺔ ﺍﻟﺘﻲ ﺘﻭﺼﻠﻨﺎ ﻟﻬﺎ ﻤﻥ ﻤﻨﺎﻗﺸﺘﻨﺎ ﻟﻠﺤﺯﻡ )ﺍﻟﺭﺯﻡ( ﺍﻟﻤﻭﺠﻴـﺔ‬
. k ‫ﻭ ﻓﺭﺍﻍ‬ v ‫ﻫﻲ ﺍﻟﻤﻌﻜﻭﺴﺔ ﻟﻠﻌﻼﻗﺔ ﺒﻴﻥ ﺍﻷﻋﺭﺍﺽ ﻓﻲ ﻓﺭﺍﻍ‬
Δ k Δ v ≥1

p=h k ‫ﻭﺒﺎﺴﺘﻌﻤﺎل ﺍﻟﻌﻼﻗﺔ‬ h ‫ﺒﻀﺭﺏ ﺍﻟﻁﺭﻓﻴﻥ ﺒـ‬

- ٥٨ -
PDF created with pdfFactory Pro trial version www.pdffactory.com
‫ﺴﻨﺘﺤﺼل ﻋﻠﻰ ﻋﻼﻗﺔ ﻫﺎﻴﺯﻨﺒﺭﺝ ‪Heisenberg ancernrainrg clarion‬‬

‫‪Δ pΔ v≥h‬‬ ‫)‪........(1‬‬

‫ﻭﺒﻤﺎ ﺃﻥ ﺍﻟﻌﺭﺽ ﻴﻤﺜل ﺍﻟﻤﻨﻁﻘﺔ ﺍﻟﺘﻲ ﻤﻥ ﺍﻟﻤﺤﺘﻤل ﺃﻥ ﻴﻜﻭﻥ ﻓـﻲ ﺍﻟﺠـﺴﻡ‬


‫)‪( p- , v - I‬ﻓﻀﺎﺀ ﻓﺈﻥ ﺍﻟﻤﻌﺎﺩﻟﺔ )‪ (1‬ﺘﺒﻴﻥ‪:‬‬

‫ﻟﻭ ﺃﺭﺩﻨﺎ ﺃﻥ ﻨﻌﻤل ﺤﺯﻤﺔ )ﺭﺯﻤﺔ( ﻤﺘﻤﺭﻜﺯﺓ )ﻤﺘﻤﻴﺯﺓ ﺠﺩﺍﹰ( ﻭﻤﻭﺠﻴﺔ ﻓـﻲ ‪x‬‬
‫ﻓﻀﺎﺀ‪ ،‬ﻓﺈﻨﻪ ﻤﻥ ﺍﻟﻤﺴﺘﺤﻴل ﻋﻠﻴﻨﺎ ﺃﻥ ﻨﺸﺎﺭﻙ ﺒﻬﺎ ﻜﻤﻴﺔ ﺤﺭﻜﺔ ﻤﻌﺭﻓﺔ ﺠﻴﺩﺍﹰ‪) .‬ﺍﻷﻤﺭ‬
‫ﺍﻟﺫﻱ ﻴﻌﺘﺒﺭ ﺒﺩﻴﻬﻲ ﻭﻋﻠﻴﻪ ﻓﺈﻥ ﺭﺯﻤﺔ ﻤﻭﺠﻴﺔ ﺫﺍﺕ ﻜﻤﻴﺔ ﻤﺤﺩﺩﺓ ﻤﻥ ﻜﻤﻴﺔ ﺍﻟﺤﺭﻜـﺔ‬
‫ﻴﺠﺏ ﺃﻥ ﺘﻜﻭﻥ ﻏﻴﺭ ﻤﺘﻤﻴﺯﺓ ﻓﻲ ﻓﻀﺎﺀ ‪ v-‬ﺃﻱ ﺃﻥ ﺘﻜﻭﻥ ﻋﺭﻴﻀﺔ ﺠﺩﺍﹰ‪.‬‬
‫ﻭﻫﺫﺍ ﺍﻟﺤﺩ )ﺍﻟﺤﺼﺭ( ﻭﺍﺤﺩ ﻤﻥ ﺇﻟﺯﺍﻤﺎﺕ ﻤﻴﻜﺎﻨﻴﻜﺎ ﺍﻟﻜﻡ ﻋﻠﻰ ﻭﺼﻔﻨﺎ ﻟﻸﻨﻅﻤﺔ‬
‫ﺍﻟﻔﻴﺯﻴﺎﺌﻴﺔ ﺒﺎﻟﻤﻔﺎﻫﻴﻡ ﺍﻟﺘﻘﻠﻴﺩﻴﺔ‪ .‬ﻓﻲ ﺍﻟﻔﻴﺯﻴﺎﺀ ﺍﻟﺘﻘﻠﻴﺩﻴﺔ ﻜﺎﻷﻤﺭ ‪) p, x‬ﺍﻟﻤﻭﻗـﻊ‪ ،‬ﻜﻤﻴـﺔ‬
‫ﺍﻟﺤﺭﻜﺔ( ﻤﺴﺘﻘﻼﺕ ﻋﻥ ﺒﻌﻀﻬﻤﺎ‪.‬‬
‫ﺒﻴﻨﻤﺎ ﻓﻲ ﻤﻴﻜﺎﻨﻴﻜﺎ ﺍﻟﻜﻡ ‪ p, x‬ﻤﻜﻤﻠﻴﻥ ﻟﺒﻌﻀﻬﻤﺎ‪.‬‬
‫ﺍﻟﺼﻐﻴﺭﺓ ﻴﺘﻀﺢ ﻟﻨﺎ ﺃﻥ ﻤﻴﻜﺎﻨﻴﻜﺎ ﺍﻟﺘﻘﻠﻴﺩﻴﺔ ﺘﻔﺸل ﻓـﻲ‬ ‫‪h‬‬ ‫ﻻﺤﻅ ﺃﻨﻪ ﻤﻥ ﻗﻴﻤﺔ‬
‫‪m = 10 −4 g‬‬
‫ﺤﺎﻟﺔ ﺍﻷﻨﻅﻤﺔ ﺍﻟﻤﺠﻬﺭﻴﺔ ﻓﻠﻭ ﺍﻋﺘﺒﺭﻨﺎ ﺃﻥ ﺤﺒﺔ ﻏﺒﺎﺭ‬
‫‪υ = 10 4 cn / s‬‬

‫ﻭﻤﻭﻗﻌﻪ ﻤﻌﺭﻭﻑ ﻟﺩﻗﺔ ‪ ، 1μ m‬ﺍﺤﺴﺏ ﺍﻟﻼﺘﻌﻴﻴﻥ ﻓـﻲ‬ ‫‪m 1μ g‬‬ ‫ﻤﺜﺎل‪ :‬ﺨﺫ ﺠﺴﻤﺎﹰ ﻜﺘﻠﺘﻪ‬
‫ﻤﻥ ﺍﻟﻤﻌﺎﺩﻟﺔ ﺍﻟﺴﺎﺒﻘﺔ‪.‬‬ ‫‪Δp‬‬ ‫ﻜﻤﻴﺔ ﺤﺭﻜﺘﻪ‬
‫‪D‬‬ ‫‪h‬‬ ‫‪m2‬‬
‫≥ ‪Δp Δv‬‬ ‫≥ ‪⇒ Δp‬‬ ‫‪J = kg.‬‬
‫‪2‬‬ ‫‪2Δ v‬‬ ‫‪s2‬‬
‫‪6.63×10 −34 J.S‬‬ ‫‪J. S‬‬ ‫‪m‬‬
‫≥‪Δ p‬‬ ‫‪−6‬‬
‫‪= 0.5× 10 −28‬‬ ‫‪= 5 × 10 −29 k g‬‬
‫‪4D ×1 ×10 m‬‬ ‫‪m‬‬ ‫‪s‬‬
‫‪−29‬‬
‫‪m 5 ×10 u g m/s‬‬
‫‪Δν = 5 ×10‬‬ ‫‪− 20‬‬
‫=‬ ‫ﻭﻫﺫﺍ ﻴﻁﺎﺒﻕ ﺴﺭﻋﺔ ﻤﻘﺩﺍﺭﻫﺎ‬
‫‪s‬‬ ‫‪1×10 −9 u g‬‬

‫‪- ٥٩ -‬‬
‫‪PDF created with pdfFactory Pro trial version www.pdffactory.com‬‬
‫ﻭﻫﺫﻩ ﺍﻟﻘﻴﻤﺔ ﻤﻬﻤﻠﺔ‪ .‬ﻭﻋﻠﻴﻪ ﻓﺈﻥ ﻟﻸﺠﺴﺎﻡ ﺍﻟﻜﺒﻴﺭﺓ ﻋﻼﻗﺔ ﺍﻟﻼﺘﻌﻴﻴﻥ ﻟﻴﺴﺕ ﺫﺍﺕ‬
‫ﺃﻫﻤﻴﺔ ﻭﻋﻠﻴﻪ ﻓﺈﻥ )ﻓﻲ ﻫﺫﻩ ﺍﻟﺤﺎﻟﺔ( ﺍﻟﻤﻴﻜﺎﻨﻴﻜﺎ ﺍﻟﺘﻘﻠﻴﺩﻴﺔ ﺘﻌﻁﻲ ﻨﻔﺱ ﺍﻟﻘﻴﻤﺔ‪.‬‬
‫ﻗﻴﺎﺱ ﻤﻜﺎﻥ ﺇﻟﻜﺘﺭﻭﻥ )ﻤﻴﻜﺭﻭﺴﻜﻭﺏ ﻫﻴﺯﺒﻨﺭﺝ(‬
‫)‪Measwrcmerg position of aneleiton (Hesibdoceg Microswgee‬‬
‫ﺍﺘﺠﺎﻩ ‪t x‬‬ ‫ﺘﺘﺤﺭﻙ ﻓﻲ‬ ‫‪px‬‬ ‫ﺸﻌﺎﻉ ﻤﻥ ﺍﻹﻟﻜﺘﺭﻭﻨﺎﺕ ﻟﻬﺎ ﻜﻤﻴﺔ ﺤﺭﻜﺔ ﻤﺤﺩﺩﺓ ﺠﻴﺩﺍﹰ‬
‫ﻭﺍﻟﻐﺭﺽ ﻤﻥ ﺍﻟﻤﻴﻜﺭﻭﺴﻜﻭﺏ ﺍﻟﺸﺎﺸﺔ ﻫﻲ ﺭﺅﻴﺔ ﻤﻭﻗﻊ ﺍﻹﻟﻜﺘﺭﻭﻥ ﻭﺫﻟﻙ ﺒﻤـﺸﺎﻫﺩﺓ‬
‫ﺍﻟﻀﻭﺀ ﺍﻟﻤﺘﺸﺘﺕ ﻤﻥ ﺍﻹﻟﻜﺘﺭﻭﻥ‪.‬‬
‫ﺇﻥ ﺍﺴﺘﻁﺎﻡ ﺍﻟﻔﻭﺘﻭﻨﺎﺕ ﺒﺎﻹﻟﻜﺘﺭﻭﻥ ﻴﺭﺘﺩ ﺨﻼل ﺍﻟﻤﻴﻜﺭﻭﺴﻜﻭﺏ‪ .‬ﻭﺇﻥ ﻗـﺩﺭﺓ‬
‫ﺘﺤﻠﻴل ﺍﻟﻤﻴﻜﺭﻭﺴﻜﻭﺏ ‪) bastion‬ﺃﻱ ﺍﻟﺩﻗﺔ ﺍﻟﺘﻲ ﺒﻬﺎ ﻴﻤﻜﻨﻨﺎ ﺘﺤﺩﻴﺩ ﻤﻭﻗﻊ ﺍﻹﻟﻜﺘﺭﻭﻥ‬
‫ﻤﻥ ﻤﻌﻠﻭﻤﺎﺕ ﻤﻥ ﺍﻟﻀﻭﺀ(‪.‬‬
‫~ ‪Δν‬‬
‫‪λ‬‬
‫ﺤﻴﺙ ‪ λ‬ﻁﻭل ﻤﻭﺠﺔ ﺍﻟﻀﻭﺀ‬
‫‪sin ϕ‬‬

‫ﻟﻠﺤﺩ ﺍﻟﻤﺭﻏﻭﺏ‬ ‫‪∆v‬‬ ‫ﻴﺠﻌل‪ λ‬ﺃﻗل ﻤﺎ ﻴﻤﻜﻥ ﻭ ‪ ϕ‬ﺃﻜﺒﺭ ﻤﺎ ﻴﻤﻜﻥ ﺒﺈﻤﻜﺎﻨﻨﺎ ﺘﺼﻐﻴﺭ‬
‫ﻫﺫﺍ ﻴﻤﻜﻨﻨﺎ ﻓﻌﻠﻪ ﻋﻠﻰ ﺤﺴﺎﺏ ﻤﻌﻠﻭﻤﺎﺘﻨﺎ ﻤﻥ ﺩﻗﺔ ﻜﻤﻴﺔ ﺍﻟﺤﺭﻜﺔ ﻓﻲ ﺍﺘﺠﺎﻩ ‪.νx‬‬
‫ﻤﻴﻜﺎﻨﻴﻜﺎ ﺍﻟﻜﻡ ﺘﺨﺒﺭﻨﺎ ﺒﺄﻥ ﺍﻹﺸﺎﺭﺍﺕ ﺍﻟﻤﺴﺠﻠﺔ ﻋﻠﻰ ﺍﻟﺸﺎﺸﺔ ﻫﻲ ﻋﺒﺎﺭﺓ ﻋـﻥ‬
‫ﻓﻭﺘﻭﻨﺎﺕ ﺘﺸﺘﺕ ﻤﻥ ﺍﻹﻟﻜﺘﺭﻭﻨﺎﺕ ﺍﺘﺠﺎﻩ ﺍﻟﻔﻭﺘﻭﻨﺎﺕ ﺒﻌﺩ ﺍﻟﺘﺸﺘﺕ ﻏﻴﺭ ﻤﺤﺩﺩ ﺨـﻼل‬
‫ﺍﻟﺯﺍﻭﻴﺔ ﺍﻟﻤﻘﺎﺒﻠﺔ ﻟﻠﻔﺘﺤﺔ ﻭﻋﻠﻴﻪ ﻜﻤﻴﺔ ﺍﻟﺤﺭﻜﺔ ﺍﻟﻤﺭﺘﺩﺓ ﻟﻺﻟﻜﺘﺭﻭﻥ ﻏﻴﺭ ﻤﺤﺩﺩﺓ ﺒـ‬
‫‪h‬‬
‫‪Δp (sin ϕ + sin ϕ ).‬‬
‫‪λ‬‬
‫‪c‬‬ ‫‪h‬‬ ‫‪hv‬‬
‫‪λ ⇒ ϕ = 2 sin ϕ . ⇒ Δ p x ~ 2 sin φ‬‬
‫‪v‬‬ ‫‪c‬‬ ‫‪c‬‬
‫‪v‬‬

‫‪Δ p v Δv ~ 2‬‬
‫‪Dv‬‬
‫‪sin ϕ .‬‬
‫‪λ‬‬
‫‪~ 4Dh‬‬ ‫ﻭﻋﻠﻴﻪ‬
‫‪c‬‬ ‫‪sin ϕ‬‬

‫‪- ٦٠ -‬‬
‫‪PDF created with pdfFactory Pro trial version www.pdffactory.com‬‬
‫ﻟﺘﺤﺴﻴﻥ ﺩﻗﺔ ﺍﻟﻤﻭﻗﻊ ﻟﻺﻟﻜﺘﺭﻭﻥ ﻴﺠﺏ ﺃﻥ ﻨﺴﺘﺨﺩﻡ ﻀﻭﺀ ﺫﻭ ‪ λ‬ﺼﻐﻴﺭﺓ ﻟﻜﻥ‬
‫ﻫﺫﺍ ﻴﻨﺘﺞ ﺨﻁﺄ ﻜﺒﻴﺭ ﺃﻭ ﺘﺸﻭﻴﺵ ﻜﺒﻴﺭ ﻓﻲ ﻜﻤﻴﺔ ﺍﻟﺤﺭﻜﺔ ﻭﺍﻟﻌﻜﺱ‪ :‬ﻟﻭ ﺃﺭﺩﻨﺎ ﺃﻥ ﻨﻘﻠل‬
‫ﺍﻟﺨﻁﺄ ﺒﻘﻴﺎﺴﻨﺎ ﺒﻜﻤﻴﺔ ﺍﻟﺤﺭﻜﺔ ﻴﺠﺏ ﺃﻥ ﻨﺴﺘﺨﺩﻡ ﺇﺸﻌﺎﻉ ﺫﻭ ﻁﻭل ﻤﻭﺠﻲ ﻜﺒﻴﺭ ﻭﻫﺫﺍ‬
‫ﺒﺩﻭﺭﻩ ﻴﺅﺩﻱ ﺇﻟﻰ ﺨﻁﺄ ﻜﺒﻴﺭ ﻓﻲ ﺍﻟﻤﻭﻗﻊ‪.‬‬

‫ﻭﻋﻠﻴﻪ ﻓﺈﻥ ﻤﺒﺩﺃ ﺍﻟﻼﺘﻌﻴﻴﻥ ﻫﻭ ﻨﺎﺸﺊ ﻤﻥ ﻁﺭﻴﻘﺔ ﺍﻟﻘﻴـﺎﺱ ﻨﻔـﺴﻬﺎ‪ .‬ﻭﻋﻠـﻰ‬


‫ﺍﻟﻤﺴﺘﻭﻯ ﺍﻟﺫﺭﻱ ﻓﺈﻥ ﺍﻟﻘﻴﺎﺱ ﻨﻔﺴﻪ ﻴﻨﺸﺄ ﻋﻨﻪ ﺨﻁﺄ ﻭﺘﺸﻭﻴﺵ ﻤﻠﺤﻭﻅ ﻟﻠﻨﻅﺎﻡ ﻭﺫﻟـﻙ‬
‫ﻟﻠﺘﻔﺎﻋل ﺒﻴﻥ ﺠﻬﺎﺯ ﺍﻟﻘﻴﺎﺱ ﺍﻟﻜﻤﻴﺔ ﺍﻟﻤﻘﺎﺴﺔ‪.‬‬

‫ﻭﻤﺒﺩﺃ ﺍﻟﻼﺘﻌﻴﻴﻥ ﻴﻘﺭﺭ ﺃﻨﻨﺎ ﻻ ﻨﺴﺘﻁﻴﻊ ﺘﺤﺩﻴﺩ ﻤﺴﺎﺭ ﺠﺴﻡ ﺒﺩﻗﺔ ﻤﺘﻨﺎﻫﻴﺔ ﻜﻤـﺎ‬
‫ﻫﻭ ﺍﻟﺤﺎل ﻓﻲ ﺍﻟﻤﻴﻜﺎﻨﻴﻜﺎ ﺍﻟﺘﻘﻠﻴﺩﻴﺔ‪.‬‬
‫‪( 2 ×10‬‬ ‫‪−27‬‬
‫‪kg‬‬ ‫ﻤﺜﺎل‪ :‬ﺒﺎﻓﺘﺭﺍﺽ ﺃﻥ ﺍﻟﻼﺘﻌﻴﻴﻥ ﻓﻲ ﻤﻭﻗﻊ ﺠﺯﺉ ﻫﻴﺩﺭﻭﺠﻴﻥ )ﺍﻟﺫﻱ ﻜﺘﻠﺘﻪ‬
‫ﻴﻘﺩﺭ ﺒﺤﻭﺍﻟﻲ ‪ 10-10 m‬ﺍﺤﺴﺏ ﻗﻴﻤﺔ ﺍﻟﻼﺘﻌﻴﻴﻥ ﻓﻲ ﻜﻤﻴﺔ ﺍﻟﺤﺭﻜﺔ؟ ﻭﻜﺫﻟﻙ ﺍﻟﻨـﺴﺒﺔ‬
‫‪Δ px‬‬
‫‪px‬‬

‫‪h‬‬ ‫‪6.6 ×10 −34‬‬


‫≥ ‪Δp x‬‬ ‫=‬ ‫‪−10‬‬
‫‪≥ 6.6 ×10 − 24 kg − m/s‬‬
‫‪Δx (2D )10‬‬

‫)ﺍﻟﺴﺭﻋﺔ ﺍﻟﺤﺭﺍﺭﻴﺔ ﻋﻨﺩ‬ ‫‪2000‬‬


‫‪m‬‬
‫ﻜﻤﻴﺔ ﺤﺭﻜﺔ ﺍﻟﺠﺯﺉ ﺍﻟﺫﻱ ﻴﺘﺤﺭﻙ ﺒﺴﺭﻋﺔ‬
‫‪s‬‬

‫ﺩﺭﺠﺔ ﺤﺭﺍﺭﺓ ﺍﻟﻐﺭﻓﺔ(‬


‫‪m‬‬
‫‪Px = mc = 2 ×10 − 27 kg × 2 ×10 3‬‬ ‫‪= 4 ×10 −24 kg. m/s‬‬
‫‪s‬‬

‫ﻭﻋﻠﻴﻪ ﻓﺈﻥ ﻨﺴﺒﺔ ﺍﻟﻼﺘﻌﻴﻴﻥ‪:‬‬


‫‪Δp x‬‬ ‫‪6.6 ×10 −24‬‬
‫=‬ ‫‪= 1.7‬‬ ‫‪0.25‬‬
‫‪px‬‬ ‫‪4 ×10 − 24‬‬

‫ﻭﻋﻠﻴﻪ ﻓﺈﻥ ﻜﻤﻴﺔ ﺤﺭﻜﺔ ﻫﺫﺍ ﺍﻟﺠﺯﺉ ﻻ ﻴﻤﻜﻥ ﺘﺤﺩﻴﺩﻫﺎ ﺒﺩﻗﺔ ﺃﻜﺜﺭ ﻤﻥ ﺍﻟﻘﻴﻤﺔ‬
‫ﺃﻷﺼﻠﻴﺔ‪.‬‬

‫‪- ٦١ -‬‬
‫‪PDF created with pdfFactory Pro trial version www.pdffactory.com‬‬
‫ﻟﻜﻥ‪ :‬ﻟﻭ ﺃﺨﺫﻨﺎ ﻜﺘﻠﺔ‪ ،‬ﻟﻬﺎ ﺴﺭﻋﺔ ‪ ،1000 m‬ﻤﻭﻗﻌﻬﺎ ﻤﺤﺩﺩ ﻟﺩﻗﺔ ‪ 1. mm‬ﻫـﺫﺍ‬
‫‪s‬‬

‫ﺍﻟﺠﺴﻡ ﻤﻘﺩﺍﺭ ﺍﻟﻼﺘﻌﻴﻴﻥ ﻓﻲ ﻜﻤﻴﺔ ﺤﺭﻜﺘﻪ‪.‬‬


‫‪6.63×10 −34 6.6‬‬
‫≥ ‪Δp‬‬ ‫‪−3‬‬
‫=‬ ‫‪×10 −31 kg − m/s‬‬
‫‪10 m‬‬ ‫‪2D‬‬
‫‪p = 0.005 ×1000 = 50 kg - m/s‬‬

‫ﻨﺴﺒﺔ ﺍﻟﻼﺘﻌﻴﻴﻥ‪.‬‬
‫‪Δp 6.6 ×10 −31 kg − k/s‬‬
‫=‬ ‫‪= 1.3×10 −32‬‬
‫‪p‬‬

‫ﻭﻫﺫﺍ ﺭﻗﻡ ﺼﻐﻴﺭ ﺠﺩﺍﹰ ﻭﻻ ﻴﻤﻜﻥ ﻗﻴﺎﺴﻪ ﺒﺄﻱ ﺠﻬﺎﺯ‪.‬‬


‫‪ .10‬ﻓﺈﻥ ﺍﻗل ﻗﻴﻤـﺔ‬ ‫‪−8‬‬
‫‪s‬‬ ‫ﻤﺜﺎل‪ :‬ﺯﻤﻥ ﺍﻟﺤﻴﺎﺓ ﻟﻠﺤﺎﻟﺔ ﺍﻟﻤﺜﺎﺭﺓ ﻟﺫﺭﺓ ﺍﻟﻬﻴﺩﺭﻭﺠﻴﻥ‬
‫ﻟﻼﺘﺤﺩﻴﺩ ﻟﻠﻁﺎﻗﺔ ﻫﻭ؟‬

‫‪ΔE. Δt ≥ h‬‬

‫‪h 6.6 ×10 −34‬‬


‫≥‪∆E‬‬ ‫=‬ ‫‪= 1.0 × − 26 J = 6.5 ×10 −8 ev.‬‬
‫‪∆t 2D ×10 −8‬‬

‫ﻭﻫﺫﺍ ﻴﺴﻤﻰ ﻋﺭﺽ ﺍﻟﻁﺎﻗﺔ ﻟﻠﺤﺎﻟﺔ ﺍﻟﻤﺜﺎﺭﺓ‪.‬‬

‫ﺍﻟﻌﻼﻗﺔ ﺒﻴﻥ ﻁﻭل ﺍﻟﻤﻭﺠﺔ ﻭﺍﻟﺘﺭﺩﺩ ﻓـﻲ ﻤﻭﺠـﺔ ﺍﻟﻤﻭﺠـﺎﺕ ‪wave Guide‬‬
‫ﺘﻌﻁﻲ ﺒﺎﻟﻤﻌﺎﺩﻟﺔ‪:‬‬
‫‪c‬‬
‫=‪λ‬‬
‫‪v 2 − v°‬‬
‫‪2‬‬

‫ﻟﻬﺫﻩ ﺍﻟﻤﻭﺠﺎﺕ‪.‬‬ ‫‪vg‬‬ ‫ﺃﻭﺠﺩ ﺴﺭﻋﺔ ﺍﻟﻤﺠﻤﻭﻋﺔ‬


‫‪c2‬‬
‫= ‪λ2‬‬
‫‪v 2 − v°‬‬
‫‪2‬‬

‫‪4D 2 (v − v° ) w − w°‬‬
‫‪2‬‬ ‫‪2‬‬ ‫‪2‬‬ ‫‪2‬‬

‫=‬ ‫=‬
‫‪D2‬‬ ‫‪c2‬‬ ‫‪c2‬‬

‫‪- ٦٢ -‬‬
‫‪PDF created with pdfFactory Pro trial version www.pdffactory.com‬‬
2D 4D2
k= ⇒ k2 = 2
λ λ
w
w = 2Dv ⇒ v =
2D
w2
v2 =
vD 2

w2 − w° w2 − w°
2 2

k = ⇒ c k = w − w° ⇒ k =
2 2 2 2 2

c2 c

w ‫ﺒﺎﻟﺘﻔﺎﻀل ﺒﺎﻟﻨﺴﺒﺔ ﻟـ‬


dw c 2 k
c 2 2kdk = 2 w dw ⇒ =
dk w
2
dw c 2 k c w
vg = = = w l − w ° = c 1 − °2
2

dk w w w
2

= c 1−
v2

.1 mev = ‫ ﺤﺯﻤﺔ ﻤﻥ ﺍﻟﻨﻴﻭﺘﺭﻭﻨﺎﺕ ﺒﻁﺎﻗﺔ ﻤﺘﻭﺴﻁﺔ ﻟﻠﻨﻴﺘﺭﻭﻥ ﺍﻟﻭﺍﺤﺩ‬:‫ﻤﺜﺎل‬

10 km ‫ ﺍﺤﺴﺏ ﺍﺘﺴﺎﻉ ﺍﻟﺤﺯﻤﺔ ﺒﻌﺩ ﻗﻁﻊ ﻤﺴﺎﻓﺔ‬،1 cu ‫ﺒﺩﺃﺕ ﺍﻟﺤﺯﻤﺔ ﺒﺎﺘﺴﺎﻉ‬


.‫ﻓﻲ ﺍﻟﻔﺭﺍﻏﺎﺕ‬
b b Δy ‫ﺍﻻﺘﺴﺎﻉ‬
Δy = 1com, ⇒ Δp y ? , Δp y . Δy ≥ h
D 6.6 ×10 −34
∴Δp y = = = 5.3×10 −33 kgm/sec
2DΔy 4D ×10 − 2 m

p x = 2m n E = 2 (1.67 ×10 −27 ) (10 6 en) (1.6 × 10 −19 )


= 2.3×10 − 20 kg . m / sec
−33
5.3 ×10
∴tomθ =
2.3 × 10 − 20
R
=
x
R 5.3× 10 −33
= ton θ ⇒ R = v ton θ = 10 m ×
9
− 20
= 2 ×10 −4 m
x 2.3×10

.2R ‫ ﺒﻤﻘﺩﺍﺭ‬1am ‫ﻭﻋﻠﻴﻪ ﻓﺈﻥ ﺍﻟﺤﺯﻤﺔ ﺴﺘﺯﺩﺍﺩ ﻤﻥ‬

- ٦٣ -
PDF created with pdfFactory Pro trial version www.pdffactory.com
‫‪2 R = 4 ×10 −4 m = 4 ×10 −2 con = 0.4‬‬

‫ﺴﻴﺼﺒﺢ ‪10.4 nm‬‬

‫ﺇﺫﺍ‬ ‫ﺱ‪ /‬ﻜﻡ ﻴﺒﻠﻎ ﻋﺩﻡ ﺍﻟﺘﺤﺩﻴﺩ )ﺍﻟﻼﺘﺤﺩﻴﺩ( ﻓﻲ ﻤﻭﻀﻊ ﻓﻭﺘﻭﻥ ﻁﻭل ﻤﻭﺠﺘﻪ‬
‫‪°‬‬
‫‪3000 A‬‬

‫ﻜﺎﻨﺕ ﺩﻗﺔ ﺘﺤﺩﻴﺩ ﻁﻭل ﺍﻟﻤﻭﺠﺔ ﻴﺼل ﺇﻟﻰ ﺠﺯﺀ ﻤﻥ ﺍﻟﻤﻠﻴﻭﻥ؟‬


‫‪∆λ‬‬
‫‪= 10‬‬
‫‪λ‬‬

‫ﺱ‪ /‬ﺇﺫﺍ ﻜﺎﻨﺕ ﺇﻟﻜﺘﺭﻭﻥ ﻓﻲ ﺤﺎﻟﺔ ﻤﺸﺘﺎﻗﺔ ﺃﻭﻟﻲ ﻤﺩﺓ ‪ 10-6 s‬ﺍﺤﺴﺏ ﻋﺩﺩ ﺍﻟﻠﻔﺎﺕ ﺍﻟﺘﻲ‬
‫ﺴﻴﻠﻔﻬﺎ ﻓﻲ ﻫﺫﺍ ﺍﻟﻤﺩﺍﺭ‪.‬‬
‫‪2D 2Dh‬‬ ‫‪13.6‬‬
‫=‪T‬‬ ‫=‬ ‫= ) ‪→ hw = (E 2‬‬ ‫=‬
‫‪w hw‬‬ ‫‪4‬‬
‫) ‪2D (1.05 × 10 −34‬‬
‫=‪T‬‬ ‫‪= 1.2 ×10 −15 sec‬‬
‫‪13.6‬‬
‫(‬ ‫) ‪) (1.6 ×10 −19‬‬
‫‪4‬‬
‫~‬
‫‪C 10 −8 sec‬‬
‫= =‪T‬‬ ‫‪−15‬‬
‫‪= 8 × 10 6 tw‬‬
‫‪T 1.2 × 10‬‬

‫ﺍﺤـﺴﺏ‬ ‫ﻫـﻭ‬ ‫ﺍﺘﺴﺎﻉ ﺍﻟﺨﻁ ﺍﻟﻁﻴﻔﻲ ﺍﻟﺫﻱ ﻟﻪ ﻁﻭل ﻤﻭﺠﺔ‬


‫‪°‬‬ ‫‪°‬‬
‫‪0.0001A‬‬ ‫‪4000 A‬‬

‫ﻤﺘﻭﺴﻁ ﺯﻤﻥ ﻤﻜﻭﺙ ﺍﻟﻨﻅﺎﻡ ﺍﻟﺫﺭﻱ ﻓﻲ ﺤﺎﻟﺔ ﺍﻟﻁﺎﻗﺔ ﺍﻟﻤﻨﺎﻅﺭﺓ‪.‬‬


‫‪°‬‬ ‫‪°‬‬
‫‪D = 4000 A , Δλ = 10 −4 A = 10 −14 m.‬‬
‫‪h c ΔD hc ΔD‬‬
‫=‪ΔE =hΔv‬‬ ‫‪= 2‬‬
‫‪D D‬‬ ‫‪D‬‬

‫ﻋﺩﻡ ﺍﻟﺘﺤﺩﻴﺩ ﻓﻲ ﺍﻟﺯﻤﻥ ‪ ، ∆t‬ﺯﻤﻥ ﻤﻜﻭﺙ ﺍﻟﺫﺭﺓ ﻓﻲ ﺍﻟﺤﺎﻟﺔ ﺍﻟﻤﺴﺘﺜﺎﺭﺓ‪.‬‬


‫`‬ ‫‪4Dh 4h D 2‬‬
‫= ‪C = Δt‬‬ ‫=‬
‫‪Δ E c D ΔD‬‬
‫) ‪(4) (16) (10 −14‬‬
‫=‬ ‫‪−14‬‬
‫‪= 2.1× 10 −7 sec.‬‬
‫) ‪(3×10 ) (10‬‬
‫‪8‬‬

‫‪= 0.21μ sec.‬‬

‫ﺱ‪ /‬ﺍﻟﻁﺎﻗﺔ ﺍﻟﻤﺴﺘﺜﺎﺭﺓ ﻟﻬﺎ ﺇﺸﻌﺎﻉ ﻴﺴﺎﻭﻱ ‪ ،1.1 ev‬ﺍﺤﺴﺏ ﻤﺘﻭﺴﻁ ﻋﻤﺭ ﻫﺫﻩ ﺍﻟﺤﺎﻟﺔ؟‬
‫= ‪Δt . ΔE‬‬
‫‪h‬‬
‫ﻤﻥ ﻋﻼﻗﺔ ﺍﻟﻼﺘﺤﺩﻴﺩ‬
‫‪2D‬‬

‫‪- ٦٤ -‬‬
‫‪PDF created with pdfFactory Pro trial version www.pdffactory.com‬‬
ΔE = DΔv
h 6.6 × 10 −34
Δt = = = ..............?
2D ΔE 2D ×1.1ev

‫ﺤﺯﻤﺔ ﻤﻭﺠﻴﺔ ﻤﻌﺭﻓﺔ ﺒـ‬


α∞
F (x) = ∫ dk g (k) e ikx
−∞

‫ ﺘﻌﻁﻲ ﺒـ‬g ( k) ‫ﺤﻴﺙ‬


g (k ) = o k 〈 − k/2
=N − k/2 〈 k 〈 k/2
4
=o 〈k
2

‫ ﺍﻟﺘﻲ ﻟﻬﺎ‬N ‫ﻗﻴﻤﺔ‬


+∞

∫ dx f ( x ) =1
2

−∞

2N kx
Qf( x ) = sin
x 2
+∞ 2
4N kx
⇒ ∫ dx 2 sin 2 =
−∞ x 2
+∞
2 4N 2 sin 2 n
∫ k 2u 2 = ∫−∞ u 2 = 2D N = 1
z 2 z
du . Sin u 2kN du

k2
1
⇒ N=
2Dk

‫ ﺍﻟﺘﻲ ﻟﻬﺎ‬N ‫ﻜﻴﻑ ﻫﺫﻩ ﻤﺭﺘﺒﻁﺔ ﺒﺎﺨﺘﻴﺎﺭ‬


+∞
1
∫ dk g ( u ) =
2

−∞
2D
+∞ u/z
1
∫ dk g ( u) = N ∫ dk = N k=
2
2 2

−∞ − k/z
2D
1
⇒N
2Dk

sin
kv
‫ ﺒﻴﻥ ﻨﻘﻁﺘﻴﻥ ﺍﻟﺘـﻲ ﻋﻨـﺩﻫﺎ ﺍﻟﺩﺍﻟـﺔ‬v ‫ﻤﻥ ﺍﻟﻤﻨﺎﺴﺏ ﺍﺨﺘﻴﺎﺭ ﻋﺭﺽ‬
2

Δu =
uD
⇐ ×=±
2D
‫ ﻭﻫﺫﺍ ﻴﺤﺩﺙ ﻋﻨﺩ‬.‫ﺘﺘﻼﺸﻰ‬
u k

- ٦٥ -
PDF created with pdfFactory Pro trial version www.pdffactory.com
‫‪uD‬‬
‫( = ‪∴Δ × Δk‬‬ ‫‪) u = uD‬‬
‫‪u‬‬

‫ﻻ ﺘﻌﺘﻤﺩ ﻋﻠﻰ ‪.u‬‬


‫‪dv‬‬ ‫‪2D dv‬‬ ‫‪dv‬‬ ‫‪dv‬‬
‫= ‪vg‬‬ ‫=‬ ‫=‬ ‫‪= −D2‬‬ ‫)‪(1‬‬
‫) ‪dk 2D d (1/D ) d (1/D‬‬ ‫‪dD‬‬
‫‪2‬‬ ‫‪C2‬‬
‫= ‪QD‬‬ ‫‪,‬‬ ‫= ‪D2‬‬
‫‪v2 − v°‬‬
‫‪2‬‬
‫‪v 2 − v°‬‬
‫‪2‬‬

‫‪2‬‬
‫‪v2 v°‬‬ ‫‪1‬‬ ‫‪2v dv‬‬ ‫‪dv dv c 2‬‬
‫⇒ ‪∴ 2 = 2 + 2 ⇒ 2 =− 2 2‬‬ ‫=‬
‫‪C‬‬ ‫‪c‬‬ ‫‪D‬‬ ‫‪c‬‬ ‫‪D‬‬ ‫‪dD D 3‬‬
‫‪dv‬‬ ‫‪c2‬‬ ‫‪c2‬‬
‫‪−D2‬‬ ‫‪= (−D 2 ) . ( 3 ) = = c 1 − v ° /v 2‬‬
‫‪2‬‬
‫)‪(1‬‬
‫‪dD‬‬ ‫‪D v Dv‬‬
‫‪c‬‬ ‫‪c‬‬ ‫‪c‬‬
‫=‬ ‫=‬ ‫=‬ ‫‪v2 − v°‬‬
‫‪2‬‬

‫‪c‬‬ ‫‪1‬‬ ‫‪v‬‬


‫‪v°‬‬ ‫(‪v‬‬ ‫)‬
‫‪v − v ° /v‬‬ ‫‪v − v°‬‬
‫‪2‬‬ ‫‪2‬‬ ‫‪2‬‬ ‫‪2‬‬ ‫‪2‬‬

‫‪u g = c 1 − v ° /v 2‬‬
‫‪2‬‬

‫ﺍﻟﻤﻌﺎﺩﻟﺔ ﺍﻟﻤﻭﺠﻴﺔ ﻟﺸﺭﻭﺩﻨﺠﺭ ﻭﺘﻔﺴﻴﺭ ﺍﻻﺤﺘﻤﺎل‬


‫‪The Schrödinger wave Eqn. nd the Probability Interpretation‬‬
‫ﻓﻲ ﻫﺫﺍ ﺍﻟﻔﺼل ﺴﻨﻨﺎﻗﺵ ﺒﻌﺽ ﺨﻭﺍﺹ ﻤﻌﺎﺩﻟﺔ ﺸﺭﻭﺩﻨﺠﺭ ﻟﻠﺠﺴﻡ ﺍﻟﺤﺭ‪ ،‬ﻫﺫﻩ‬
‫ﺍﻟﻤﻌﺎﺩﻟﺔ ﺍﻟﺘﻲ ﺍﺴﺘﻨﺘﺠﻨﺎﻫﺎ ﻓﻲ ﺍﻟﻔﺼل ﺍﻟﺜﺎﻨﻲ‪ .‬ﻭﻜﺫﻟﻙ ﺴﻨﺴﺘﻨﺘﺞ ﺘﻔـﺴﻴﺭ ﺍﻻﺤﺘﻤـﺎل‬
‫ﻟﻠﺩﺍﻟﺔ ﺍﻟﻤﻭﺠﻴﺔ ‪ Ψ‬ﻭﻫﺫﺍ ﺒﺩﻭﺭﻩ ﻴﺅﺩﻱ ﺇﻟﻰ ﺘﻌﺭﻴﻑ ﻜﻤﻴﺔ ﺍﻟﺤﺭﻜﺔ ‪ montentum‬ﻓﻲ‬
‫ﻤﻴﻜﺎﻨﻴﻜﺎ ﺍﻟﻜﻡ‪ ،‬ﻭﻜﺫﻟﻙ ﻟﻤﻌﺎﺩﻟﺔ ﺸﺭﻭﺩﻨﺠﺭ ﺍﻟﺘﻲ ﺘﺼﻑ ﺠﺴﻡ ﻓﻲ ﺠﻬﺩ )‪.V (x‬‬

‫ﻭﻨﻘﻁﺔ ﺍﻟﺒﺩﺍﻴﺔ ﻫﻲ ﺍﻟﻤﻌﺎﺩﻟﺔ ﺍﻟﺘﻔﺎﻀﻠﻴﺔ‪.‬‬


‫)‪∂ ψ (x, t‬‬ ‫)‪h 2 ∂ 2 ψ (x, t‬‬
‫‪ih‬‬ ‫‪=−‬‬ ‫)‪.............(1‬‬
‫‪∂t‬‬ ‫‪2m ∂x 2‬‬

‫ﻭﻫﺫﻩ ﻤﻌﺎﺩﻟﺔ ﺼﺤﻴﺤﺔ ﻟﻭﺼﻑ ﺤﺭﻜﺔ ﺠﺴﻡ ﺤﺭ‪ .‬ﻭﺤل ﻫﺫﻩ ﺍﻟﻤﻌﺎﺩﻟﺔ ﻴﻤﻜـﻥ‬
‫ﺍﻟﺤﺼﻭل ﻋﻠﻴﻪ )ﺒﻌﻜﺱ ﺍﻟﺨﻁﻭﻁ ﺍﻟﺘﻲ ﺃﺩﺕ ﺇﻟﻴﻬﺎ(‪ ..‬ﻭﻨﺘﺤﺼل ﻋﻠﻰ ﺍﻟﺤل ﺍﻟﻌﺎﻡ‪:‬‬
‫‪1‬‬
‫‪∫ ap ϕ (p) e‬‬
‫‪i(px − Et)/h‬‬
‫= )‪ψ (x, t‬‬
‫‪2Dh‬‬

‫‪- ٦٦ -‬‬
‫‪PDF created with pdfFactory Pro trial version www.pdffactory.com‬‬
‫ﻭﺤﻴﺙ ﺃﻥ ‪ E‬ﻟﻠﺠﺴﻡ ﺍﻟﺤﺭ ﻫﻲ ‪p2/2m‬‬
‫‪1‬‬
‫‪∫ ap ϕ (p) e‬‬
‫‪i(px − (p 2 /2m)t)lh‬‬
‫= )‪ψ (x, t‬‬ ‫)‪.............(2‬‬
‫‪2Dh‬‬

‫ﻭﻤﻥ ﺍﻟﺠﺩﻴﺭ ﺒﺎﻟﻤﻼﺤﻅﺔ‪ :‬ﺃﻥ ﺍﻟﻤﻌﺎﺩﻟﺔ )‪ (١‬ﻫﻲ ﻤﻌﺎﺩﻟﺔ ﺘﻔﺎﻀﻠﻴﺔ ﻤﻥ ﺍﻟﺩﺭﺠﺔ‬


‫ﻓـﺈﻥ ﻗﻴﻤـﺔ‬ ‫) )‪(ψ ( x, o‬‬ ‫ﺍﻷﻭﻟﻰ ﻭﻫﺫﺍ ﻴﻌﻨﻲ ﺃﻨﻨﺎ ﺒﻤﺠﺭﺩ ﺃﻥ ﻨﻌﺭﻑ ﺍﻟﻘﻴﻤﺔ ﺍﻷﻭﻟﻴﺔ ﻟـ‬
‫ﻋﻨﺩ ﻜل ﺍﻷﺯﻤﻨﺔ ﺍﻷﺨﺭﻯ ﻴﻤﻜﻥ ﻤﻌﺭﻓﺘﻬﺎ‪.‬‬ ‫) ‪(ψ‬‬

‫)‪∂ψ (x, t‬‬


‫‪ψ (x, t + Δt) = ψ (x, t) +‬‬ ‫‪Δt‬‬
‫‪∂t‬‬
‫‪∂ψ (x, t) i 2h 2 ∂ψ (x, t) ih ∂ψ‬‬
‫=‬ ‫=‬
‫‪∂t‬‬ ‫‪i2m h ∂xH2 2m xt 2‬‬
‫‪ih ∂ 2 ψ‬‬
‫‪ψ ( x, t + Δt) = ψ (x, t) +‬‬ ‫‪Δt‬‬
‫‪2m ∂x 2‬‬
‫)‪ih ∂ 2 ψ (x, t‬‬
‫‪ψ (x, t + Δt ) = ψ (x, t ) +‬‬ ‫)‪Δt .................(3‬‬
‫‪2m ∂x 2‬‬

‫ﻭﺒﺈﻋﻁﺎﺀ ﻗﻴﻤﺔ )‪ ψ (x, o‬ﺍﻟﺩﺍﻟﺔ ) ‪ ϕ (p‬ﻴﻤﻜﻥ ﺇﻴﺠﺎﺩﻫﺎ ﻤﻥ )‪ (2‬ﻀﻊ ‪ t = o‬ﻓـﻲ‬


‫ﺍﻟﻤﻌﺎﺩﻟﺔ )‪ (2‬ﺘﻜﺎﻤل ﻓﻭﺭﻴﺭ‪.‬‬
‫= ) ‪ψ (x, o‬‬
‫‪1‬‬
‫‪∫ dp φ (p) e‬‬
‫‪ipx/h‬‬
‫)‪.................(4‬‬
‫‪2xh‬‬

‫)‪ϕ ( p‬‬ ‫ﻴﻤﻜﻥ ﻋﻜﺴﻪ ﻟﻠﺤﺼﻭل ﻋﻠﻰ‬


‫ﺸﺭﻭﻁ ﺍﻟﻤﻌﺎﻴﺭﺓ‪:‬‬

‫‪ϕ (P ) = 2Dh ∫ ψ (x, o ) dx e ipx/ h‬‬

‫ﻭﺒﺎﻟﺘﻌﻭﻴﺽ ﻓﻲ ﺍﻟﻤﻌﺎﺩﻟﺔ )‪ (2‬ﻴﻤﻜﻨﻨﺎ ﺤﺴﺎﺏ ) ‪ ψ (x, t‬ﻋﻨﺩ ﻜل ﻗﻴﻡ ‪..t‬‬


‫ﻻﺤﻅ ﺃﻨﻪ ﻻ ﻴﻭﺠﺩ "ﻻ ﺘﻌﻴﻴﻥ" ﻓﻲ ﺍﻟﻤﻌﺎﺩﻟﺔ ﺍﻟﺘﻔﺎﻀﻠﻴﺔ‪:‬‬
‫ﻁﺎﻟﻤﺎ ‪ once‬ﻨﻌﺭﻑ ﺍﻟﻘﻴﻡ ﺍﻷﻭﻟﻰ ﻟﻠﺤﺎﻟﺔ )ﻟﻠﺤﺯﻤﺔ ﺍﻟﻤﻭﺠﺒﺔ( ﻻ ﺘﻭﺠﺩ ﻗﻴـﻭﺩ‬
‫ﻋﻠﻰ ﻗﻴﻡ )‪ ψ (x, o‬ﻋﻨﺩﺌﺫ ﺍﻟﺤﺯﻤﺔ ﺍﻟﻤﻭﺠﻴﺔ ﻜﻠﻬﺎ ﻤﻌﺭﻓﺔ ﻋﻨﺩ ﻜل ﺍﻷﺯﻤﻨﺔ ﺍﻟﺘﺎﻟﻴﺔ‪..‬‬
‫) ‪ψ ( x, t‬‬ ‫ﺘﻔﺴﻴﺭ ﺍﻻﺤﺘﻤﺎل ﻟﻠﺩﺍﻟﺔ ﺍﻟﻤﻭﺠﻴﺔ‬

‫‪- ٦٧ -‬‬
‫‪PDF created with pdfFactory Pro trial version www.pdffactory.com‬‬
‫ﻴﺠﺏ ﺃﻥ ﻴﻜﻭﻥ ﻓـﻲ‬ ‫) ‪ψ ( x, t‬‬ ‫ﻭﻨﺤﻥ ﺒﺼﺩﺩ ﺍﻟﺒﺤﺙ ﻋﻥ ﺘﺄﻭﻴل )ﺘﻔﺴﻴﺭ( ﻟﻠﺩﺍﻟﺔ‬
‫ﺘﻔﻜﻴﺭﻨﺎ‪.‬‬
‫)‪ (1‬ﺃﻥ ) ‪ ψ (x, t‬ﻫﻲ ﺒﺸﻜل ﻫﺎﻡ ﺩﺍﻟﺔ ﻤﻌﻘﺩﺓ ‪.conglex fetom‬‬
‫ﺘﻜﻭﻥ ﻜﺒﻴﺭﺓ ﺤﻴﺙ ﻴﻔﺘﺭﺽ ﺃﻥ ﻴﻜﻭﻥ ﺍﻟﺠﺴﻡ ﻤﻭﺠـﻭﺩ ﻭﺘﻜـﻭﻥ‬ ‫) ‪ψ (x, t‬‬
‫‪1‬‬
‫‪424‬‬ ‫‪3‬‬
‫)‪ (2‬ﺍﻟﺩﺍﻟﺔ‬
‫ﻛﻤﯿﺔﺣﻘﯿﻘﯿﺔ‬

‫ﺼﻐﻴﺭﺓ ﻓﻲ ﺃﻤﺎﻜﻥ ﻏﻴﺭ ﻫﺫﺍ ﺍﻟﻤﻜﺎﻥ‪.‬‬

‫)‪ (3‬ﻭﻜﺫﻟﻙ ﻤﺼﺎﺤﺏ ﻟﻠﺩﺍﻟﺔ ﻅﺎﻫﺭﺓ ﺍﻻﻨﺒﺴﺎﻁ )‪) (Spreading‬ﺍﻟﻼﺘﺤﻴﺯ(‪.‬‬


‫ﻭﻤﺒﺎﺸﺭﺓ ﺒﻌﺩ ﺍﻜﺘﺸﺎﻑ ﻤﻌﺎﺩﻟﺔ ﺸﺭﻭﺩﻨﺠﺭ )ﺍﻟﺘﻲ ﺘﺒﻌﺙ ﺍﻜﺘﺸﺎﻑ ﻤﻴﻜﺎﻨﻴﻜﺎ ﺍﻟﻜﻡ‬
‫ﺒﻭﺍﺴﻁﺔ ﻫﻴﺯﺒﻨﺭﺝ ﺴﻨﺔ ‪ (١٩٢٥‬ﻗﺎﻡ ‪ MaxBorn‬ﺒﺩﺭﺍﺴﺔ ﺘﺒﻌﺜﺭ )ﺘﻁﺎﻴﺭ( ﺸﻌﺎﻉ ﻤﻥ‬
‫ﺍﻹﻟﻜﺘﺭﻭﻨﺎﺕ ﺒﻬﺩﻑ ﺃﻭ ﺍﻟﺘﻲ ﺃﺭﺸﺩﺘﻪ ﺇﻟﻰ ﺍﻟﺘﺄﻭﻴل ﺍﻟﺼﺤﻴﺢ ﻟﻠﺩﺍﻟﺔ ﺍﻟﻤﻭﺠﻴﺔ‪ .‬ﻭﻋﻠﻴـﻪ‬
‫ﻓﻘﺩ ﺍﻗﺘﺭﺡ ﺃﻥ‪.‬‬
‫ﺩﺍﻟـﺔ ﺤﻘﻴﻘﻴـﺔ‬ ‫⇒) ‪⇐ p (x, t‬‬ ‫ﺘﻜﻭﻥ ﻜﺒﻴﺭﺓ ﺤﻴﻨﻤﺎ ﻴﻔﺘﺭﺽ ﺃﻥ ﻴﻜﻭﻥ ﺍﻟﺠـﺴﻡ‬
‫ﻭﺘﻔﻠﻁﺤﻬﺎ ﻻ ﻴﻌﻨﻲ ﺃﻥ ﺍﻟﺠﺴﻡ ﻤﺘﻔﻠﻁﺢ‪.‬‬
‫ﻭﻜل ﻤﺎ ﺘﻌﻨﻴﻪ ﻫﻭ ﺃﻨﻪ ﻋﻨﺩﻤﺎ ﻴﺘﻐﻴﺭ ﺍﻟﺯﻤﻥ ﻴﻜﻭﻥ ﻤﻥ ﺍﻷﻗل ﺍﺤﺘﻤﺎﻻﹰ ﺃﻥ ﻨﺠـﺩ‬
‫ﺍﻟﺠﺴﻡ ﺤﻴﻨﻤﺎ ﻜﺎﻥ ﻋﻨﺩ ‪.t = 0‬‬

‫ﻭﻟﻜﻲ ﻴﻜﻭﻥ ﺍﻟﺘﺄﻭﻴل ﺼﺤﻴﺤﺎﹰ ﻴﻠﺯﻤﻨﺎ‪:‬‬


‫∞‪x‬‬

‫‪∫ p (x, t ) dx =1‬‬


‫∞‬
‫)‪(6‬‬

‫ﺸﺭﻁ ﺍﻟﻤﻌﺎﻴﺭﺓ ‪vormalization‬‬

‫ﻭﺫﻟﻙ ﻷﻥ ﺍﻟﺠﺴﻡ ﻴﺠﺏ ﺃﻥ ﻴﻜﻭﻥ ﻓﻲ ﻤﻜﺎﻥ ﻤﺎ ﻭﺴﻴﺘﻀﺢ ﻟﻨﺎ ﻻﺤﻘﺎﹰ ﺃﻥ ﻴﻜﻔﻲ‬


‫ﺃﻥ ﻴﻠﺯﻡ ﺃﻥ‪:‬‬
‫∞‪x‬‬

‫) ‪∫ dx ψ (x, o‬‬ ‫∞〈‬


‫‪2‬‬
‫) ‪(7‬‬
‫∞‬

‫‪- ٦٨ -‬‬
‫‪PDF created with pdfFactory Pro trial version www.pdffactory.com‬‬
‫ـﺔ‬
‫ـﺔ ﻤﺘﻜﺎﻤﻠـ‬
‫ـﻭﻥ ﻤﺭﺒﻌـ‬
‫ـﺏ ﺃﻥ ﺘﻜـ‬
‫ـﺔ )‪ ψ (x, o‬ﻴﺠـ‬
‫ـﻲ ﺃﻥ ﺍﻟﺩﺍﻟـ‬
‫ـﺫﺍ ﻴﻌﻨـ‬
‫ﻭﻫـ‬
‫ﻴﻌﻨﻲ ﺃﻥ )‪.ψ (x, o‬‬ ‫∞ ‪+ ∞←−‬‬ ‫‪ squaredintegrabli‬ﻭﺒﻨﻁﺎﻕ ﺘﻜﺎﻤل‬
‫ﻭﻫﺫﺍ ﻴﻠـﺯﻡ ﺃﻥ ﺘﻜـﻭﻥ ﺍﻟﺩﺍﻟـﺔ‬ ‫‪x −1 / 2‬‬ ‫ﻴﺠﺏ ﺃﻥ ﺘﺅﻭل ﺇﻟﻰ ﺼﻔﺭ ﺃﺴﺭﻉ ﻤﻥ‬
‫‪.contineon sin x.‬‬ ‫ﻤﺘﺼﻠﺔ ﻓﻲ ‪x‬‬ ‫) ‪ψ (x, t‬‬

‫ﺃﻫﻤﻴﺔ ﺍﻷﻁﻭﺍﺭ‪Import on ce of the ases :‬‬

‫ﺫﺍﺕ ﻤﻌﻨﻰ ﻓﻴﺯﻴـﺎﺌﻲ‪ ،‬ﻴﺒـﺩﻭ ﺃﻥ‬ ‫) ‪ψ (x, t‬‬


‫‪2‬‬
‫ﻴﺒﺩﻭ ﻟﻠﻭﻫﻠﺔ ﺍﻷﻭﻟﻰ ﻜﻭﻥ ﺍﻟﺩﺍﻟﺔ‬
‫ﺍﻟﻁﻭل ﻟﻴﺱ ﻟﻪ ﺃﻫﻤﻴﺔ‪ .‬ﻭﻫﺫﺍ ﺨﻁﺄ‪.‬‬

‫ﻫﻲ‬ ‫)‪ψ 2 (x, t ), ψ1 (x, t‬‬ ‫ﻭﺒﻤﺎ ﺃﻥ ﺍﻟﻤﻌﺎﺩﻟﺔ )‪ (1‬ﻫﻲ ﻤﻌﺎﺩﻟﺔ ﺨﻁﻴﺔ‪) ،‬ﻭﺒﺎﻗﺘﺭﺍﻥ ﺃﻥ‬
‫ﺤل‪.‬‬
‫ﻫﺫﺍ ﻜﺫﻟﻙ ﺤل‪.‬‬ ‫←‬ ‫)‪ψ (x, t ) = ψ1 (x, t) + ψ 2 (x, t‬‬ ‫)‪(8‬‬

‫‪ψ 2 (x, t ) = R 2 e iθθ , ψ1 (x, t) = R 1 e‬‬


‫‪iθ1‬‬
‫ﻟﻭ ﺃﻥ‬
‫ﻫﻲ ﺤﻘﻴﻘﺔ‪ ،‬ﻋﻨﺩﻫﺎ‬ ‫‪R 1 , R 2 , θ1 , θ 2‬‬ ‫ﺤﻴﺙ ﺃﻥ‬
‫) ‪ψ (x, t‬‬
‫) ‪i (θ 2 − θ 1‬‬ ‫‪2‬‬
‫‪= e‬‬ ‫‪(R1 +R 2 e‬‬
‫‪2‬‬ ‫‪iθ 1‬‬
‫)‬

‫) ‪= R 1 + R 2 + 2R 1 cos (θ 1 − θ 2‬‬
‫‪2‬‬ ‫‪2‬‬
‫)‪(9‬‬

‫ﻭﻫﺫﺍ ﻴﻅﻬﺭ ﺃﻥ ﺍﻟﻁﻭﺭ ﻤﻬﻡ‪ ،‬ﺍﻟﻁﻭﺭ ﺍﻟﻜﻠﻲ ﻓﻲ ) ‪ ψ (x, t‬ﻴﻤﻜﻥ ﺇﻫﻤﺎﻟـﻪ‪ ،‬ﺃﻤـﺎ‬
‫‪.‬‬ ‫‪ψ‬‬
‫‪2‬‬
‫ﺘﻅﻬﺭ ﻓﻲ‬ ‫و ‪ψ2‬‬ ‫‪ψ1‬‬ ‫ﺒﻴﻥ ﺩﺍﻟﺘﻴﻥ ﻤﻭﺠﺒﺘﻴﻥ‬ ‫) ‪(θ 1 − θ 2‬‬ ‫ﺍﻟﻁﻭل ﺍﻟﻨﺴﺒﻲ‬
‫ﻻﺤﻅ ﺃﻥ ﺍﻟﻁﺭﻑ ﺍﻷﻴﻤﻥ ﻓﻲ ﺍﻟﻤﻌﺎﺩﻟﺔ )‪ (9‬ﻫﻭ ﺍﻟﺫﻱ ﻨـﺭﺍﻩ ﻋﻨـﺩ ﺘﺭﺍﻜـﺏ‬
‫ﺍﻟﻤﻭﺠﺎﺕ‪ .‬ﻭﻓﻲ ﺍﻟﺤﻘﻴﻘﺔ ﺃﻨﻬﺎ ﺍﻟﺨﻁﻭﺓ ‪ lineally‬ﻟﻠﺩﻭﺍل ﺍﻟﻤﻭﺠﺒﺔ ﺍﻟﺘﻲ ﺃﺩﺕ ﻟـ ﻟﺸﻜل‬
‫ﺍﻟﺘﺩﺍﺨل ﺍﻟﺫﻱ ﻴﻅﻬﺭ ﻤﻥ ﺨﻼل ‪ cosine‬ﻓﻲ ﻫﺫﻩ ﺍﻟﻤﻌﺎﺩﻟﺔ‪.‬‬
‫ﻭﻋﻨﺩﻤﺎ ﻨﺘﺤﺩﺙ ﻋﻥ ﺍﻟﺴﻠﻭﻙ ﺍﻟﻤﻭﺠﻲ ﻟﻺﻟﻜﺘﺭﻭﻨﺎﺕ‪ ،‬ﺍﻟﻔﻭﺘﻭﻨﺎﺕ‪ ،‬ﻓﺘﺤﺩﺙ ﻤﻥ‬
‫ﺍﻟﺨﻁﻴﺔ‪...‬‬

‫‪- ٦٩ -‬‬
‫‪PDF created with pdfFactory Pro trial version www.pdffactory.com‬‬
Peobalrility current ‫ﺍﺤﺘﻤﺎل ﺍﻟﺘﻴﺎﺭ‬

.(1) ‫ ﺍﻟﻤﺭﺍﻓﻕ ﺍﻟﻤﻌﻘﺩ ﻟﻠﻤﻌﺎﺩﻟﺔ‬complex conjugate ‫ﻟﻨﺄﺨﺫ‬


∂ψ h2 ∂2ψ ∂ψ ih ∂ 2 ψ
ih =− ⇒ =
∂t 2m ∂t 2 ∂t 2m ∂x 2
∂ψ * (x, t) h 2 ∂ 2 ψ * (x, t) ∂ψ * ih ∂ 2 ψ *
⇒ − ih =− ⇒ =−
∂t 2m ∂t 2 ∂t 2m ∂x 2
ψ 2
644744 8
∂ ∂
p (x, t) = ( ψ (x, t) ψ x (x, t) )
∂t ∂t
∂ψ * (x, t) ∂ψ
= ψ + ψ*
∂t ∂t
 ih ∂ψ * ih ∂ 2 ψ 
*
= − ψ 
 2m ∂x 2m ∂x 2 
2

1  h 2 ∂ 2ψ* * h
2
∂ 2ψ 
= ψ − ψ
ih  2m ∂x 2 2m ∂x t 
∂ ∂
p (x, t) = −
∂t ∂x
∂ ∂
⇒ t (x, t) = − j (x, t)
∂t ∂x
+∞ +∞
∂ ∂
∂x −∫∞
dx p (x, t) = − ∫ dx j (x, t) = o
−∞
∂x
= − [J (− ∞ ) − j (− ∞ ) ]= 0 , j → 0

‫ﻴﺘﻌـﻭﺽ ﺒـﺎﻟﺘﻐﻴﺭ‬ x ‫ ﺍﻟﺘﻐﻴﺭ ﻓﻲ ﺍﻟﻜﺜﺎﻓﺔ ﻓﻲ ﻤﻨﻁﻘﺔ ﻓـﻲ‬:‫ﺘﺒﻴﻥ‬ ⇐ (12) ‫ﻤﻌﺎﺩﻟﺔ‬


.‫ﺍﻟﺼﺎﻓﻲ ﻓﻲ ﺍﻟﻔﻴﺽ ﻓﻲ ﺘﻠﻙ ﺍﻟﻤﻨﻁﻘﺔ‬
∞ ‫ ﺘﺘﻼﺸﻰ ﻋﻨﺩ‬square integreble ‫ﺍﻟﺩﻭﺍل ﻤﺘﻜﺎﻤﻠﺔ ﺍﻟﺘﺭﺒﻴﻊ‬ Q

∂ ∂
∂t ∫ ax p(x, t) = − ∫
−∞
dx
∂x
j (x, t)

= j (a, t) − j (b, t) (14)

(1) ‫ﻭﻗﺎﻨﻭﻥ ﺍﻟﺒﻘﺎﺀ ﺘﻜﻭﻥ ﺼﺤﻴﺤﺔ ﻟـﻭ ﺍﻟﻤﻌﺎﺩﻟـﺔ‬ j (x, t) , p(x, t ) ‫ﻭﺘﻌﺭﻴﻑ ﻟـ‬
‫ﺘﺘﻐﻴﺭ ﻟـ‬

- ٧٠ -
PDF created with pdfFactory Pro trial version www.pdffactory.com
‫ﻭﻫﺫﻩ ﺍﻟﺘﻌﺭﻴﻔﺎﺕ ﻟـ ) ‪ d (x, t ), p (x, t‬ﻭﻗﺎﻨﻭﻥ ﺍﻟﺤﻔﻅ )ﺍﻟﺒﻘﺎﺀ( ﺘﺒﻘـﻰ ﺼـﺤﻴﺤﺔ‬
‫ﺤﺘﻰ ﻟﻭ ﻏﻴﺭﻨﺎ ﻤﻌﺎﺩﻟﺔ ﺍﻟﺠﺴﻡ ﺍﻟﺤﺭ ﻟﻠﻤﻌﺎﺩﻟﺔ ﺍﻟﺘﺎﻟﻴﺔ ﻤﻌﺎﺩﻟﺔ ﺸﺭﻭﺩﻨﺠﺭ‪.‬‬
‫) ‪∂ ψ (x, t‬‬ ‫)‪h 2 ∂ 2 ψ (x, t‬‬
‫‪it‬‬ ‫‪=−‬‬ ‫)‪+ ψ (x, t‬‬ ‫)‪(15‬‬
‫‪∂t‬‬ ‫‪2m‬‬ ‫‪∂x‬‬

‫ﻭﻫﺫﻩ ﻤﻬﻤﺔ ﻷﻨﻬﺎ ﺘﻤﺜل ﻤﻌﺎﺩﻟﺔ ﺸﺭﻭﺩﻨﺠﺭ ﻟﺠﺴﻡ ﻓﻲ ﺠﻬﺩ )‪ V(x‬ﻓﻲ ﺜﻼﺜـﺔ‬
‫ﺃﺒﻌﺎﺩ‪.‬‬
‫‪ψ (x, y, z, t),‬‬
‫) ‪V (x, y, z‬‬
‫‪∂2‬‬ ‫‪∂2‬‬ ‫‪∂2‬‬
‫‪+‬‬ ‫‪+‬‬ ‫‪⇒∇2‬‬
‫‪∂x 2 ∂y 2 ∂z 2‬‬

‫ﻜﺫﻟﻙ ﺘﻌﻤﻴﻡ )‪(12‬‬


‫∂‬
‫‪p (r, t ) + ∇ . J (r, t) = 0‬‬
‫‪∂t‬‬
‫) ‪p (r, t ) = ψ (r, t‬‬
‫‪2‬‬

‫)‪∂ψ (x, t‬‬ ‫‪h2 2‬‬


‫‪ih‬‬ ‫‪=−‬‬ ‫) ‪∇ ψ (r, t ) + v (r ) ψ (r, t‬‬
‫‪∂t‬‬ ‫‪2m‬‬
‫= ) ‪j (r, t‬‬
‫‪h‬‬
‫‪2im‬‬
‫[‬
‫) ‪ψ * (r, t ) ∇ψ (x, t ) − ∇ψ * (r, t ) ψ (r, t‬‬ ‫]‬
‫ﺍﻟﻘﻴﻡ ﺍﻟﻤﺘﻭﻗﻌﺔ ﻭﻜﻤﻴﺔ ﺤﺭﻜﺔ ﺍﻟﺠﺴﻡ‬
‫‪Expectation Values of Canticle nonctn‬‬
‫ﺒﺈﻋﻁﺎﺀ ﻜﺜﺎﻓﺔ ﺍﻻﺤﺘﻤﺎل‪.P (x,t) .‬‬
‫ﻓﺈﻥ ﺍﻟﻘﻴﻤﺔ ﺍﻟﻤﺘﻭﻗﻌﺔ ﻟـ )‪ f (x‬ﻴﻤﻜﻥ ﺤﺴﺎﺒﻬﺎ ﺒﺸﻜل ﻋﺎﻡ‪.‬‬
‫) ‪f (x ) = ∫ ax f (x ) p (x, t‬‬ ‫)‪(20‬‬

‫) ‪= ∫ ax ψ * (x, t ) f (x ) ψ (x, t‬‬

‫ﻻﺴﺘﻨﺘﺎﺝ ﻤﺅﺜﺭ ﻜﻤﻴﺔ ﺍﻟﺤﺭﻜﺔ ﻤﻥ ﺍﻟﻤﻴﻜﺎﻨﻴﻜﺎ‬

‫‪- ٧١ -‬‬
‫‪PDF created with pdfFactory Pro trial version www.pdffactory.com‬‬
dx
p = mv = m (21)
dt
x = m ∫ dx ψ * 9x, t) v ψ (x, t )
d d
p =m (22)
dt dt
+∞
∂ψ * ∂ψ
p (x ) = m ∫ dx ( xψ + ψ * x )
−∞
∂Z ∂Z
+∞
∂ψ * ∂ψ
p (x ) = m ∫ dx ( + ψ (x, t ) + ψ * x )
−∞
∂t ∂Z

،‫ﻫﻲ ﺍﻟﺘﻲ ﺘﺘﻐﻴﺭ ﺒﺎﻟﺯﻤﻥ‬ ψ (x, t ) ‫ ﻓﻘﻁ‬.‫ﺘﺤﺕ ﺍﻟﺘﻜﺎﻤل‬ dx


‫ﻻﺤﻅ ﺃﻨﻪ ﻻ ﻴﻭﺠﺩ‬
dt
.‫ﻤﻊ ﺍﻟﺯﻤﻥ‬ x ‫ﻭﻫﺫﺍ ﺍﻟﺘﻐﻴﺭ ﻫﻭ ﺍﻟﺫﻱ ﻴﺅﺩﻱ ﺇﻟﻰ ﺍﻟﺘﻐﻴﺭ ﻓﻲ‬

.‫ﻭﺒﻤﺎ ﺃﻥ ﺒﺎﻟﺘﻌﻭﻴﺽ ﻓﻲ ﺁﺨﺭ ﻤﻌﺎﺩﻟﺔ‬


∂ψ h ∂ψ
ih =−
∂t 2m ∂x 2
∂ψ h ∂ 2ψ
m =−
∂t 2i ∂x 2
∂ψ * h ∂ 2 ψ *
m =
∂t 2i ∂x 2
h +∞ ∂ 2 ψ * * ∂ ψ
2
p = ∫( xψ − ψ x 2 )dx
2i −∞ ∂x 2 ∂x
∂ 2 ψ* ∂ ∂ψ * ∂ψ * ∂ψ * ∂ψ
xψ = ( xψψ− ψ − x
∂x 2 ∂x ∂x ∂x 1∂4
x 24 ∂3
x
∂x * ∂ψ
ψ
∂x ∂x

∂ ∂ψ * ∂ ∂ψ
= ( xψψ− (ψ * ψ) + ψ *
∂x ∂x ∂x ∂x
∂ ∂ψ ∂ψ ∂ ψ2
− (ψ * x ) + ψ * + ψ*x 2
∂x ∂x ∂x ∂x

.‫ﻭﻋﻠﻴﻪ ﻓﺈﻥ ﺍﻟﺤﺩ ﺍﻟﺫﻱ ﻴﻘﻊ ﺩﺍﺨل ﺍﻟﺘﻜﺎﻤل ﻴﺼﺒﺢ‬


∂ 2 ψ* * ∂ ψ
2
∂ ∂ψ * ∂ ∂ψ
(xψx− ψ x = ( x ψ) − (ψ * ψ) + ψ *
∂x 2
∂x 2
∂x ∂x ∂x ∂x
∂ ∂ψ ∂ψ
− ( ψ* x ) + ψ*
∂x ∂x ∂x
∂  ∂ψ *
∂ψ  ∂ψ
=  xψ − ψ * ψ − ψ * x  + 2ψ *
∂x  ∂x ∂x  ∂x

- ٧٢ -
PDF created with pdfFactory Pro trial version www.pdffactory.com
+∞ +∞
h ∂
p = ∫ [ ]+ h ∫ 2ψ * (x, t) ∂ ψ (x, t) dx
2i − ∞ ∂x 2i −∞ ∂x
+∞
h ∂
p = ∫ dx ψ * (x, t) ψ (x, t)
−∞
i ∂x

.operator ‫ﻫﺫﻩ ﺍﻟﻤﻌﺎﺩﻟﺔ ﺘﻘﺘﺭﺡ ﺃﻥ ﻜﻤﻴﺔ ﺍﻟﺤﺭﻜﺔ ﻴﻤﻜﻥ ﺃﻥ ﺘﻤﺜل ﺒﺎﻟﻤﺅﺜﺭ‬


h ∂
p=
i ∂x

.‫ ﻋﻠﻰ ﺴﺒﻴل ﺍﻟﻤﺜﺎل‬،‫ﻭﻟﻬﺫﺍ‬


∂2
p 2 = ∫ dxψ * (x, t) (−h 2 ) ψ (x, t)
∂x 2

f ( p ) = ∫ dxψ * (x, t) f (
D ∂
) ψ (x, t) .‫ﻭﺒﺸﻜل ﻋﺎﻡ‬
t ∂x

φ ( p) ‫ﻭﺒﻬﺫﺍ ﺍﻟﺘﻤﺜﻴل ﻟﻜﻤﻴﺔ ﺍﻟﺤﺭﻜﺔ ﻴﻤﻜﻨﻨﺎ ﻤﻨﺎﻗﺸﺔ ﺍﻟﻤﺩﻟﻭل ﺍﻟﻔﻴﺯﻴـﺎﺌﻲ ﻟــ‬


.‫ﺍﻟﺘﻲ ﺘﻅﻬﺭ ﻓﻲ ﺍﻟﻤﻌﺎﺩﻟﺔ ﺍﻟﺩﺍﻟﺔ ﺍﻟﻤﻭﺠﻴﺔ ﻓﻲ ﻓﺭﺍﻍ ﻜﻤﻴﺔ ﺍﻟﺤﺭﻜﺔ‬
1
ψ(x, t )= ∫ ap φ ( p ) e i [px - (p/2m) t ] / h
2Dh

.t ‫ﻻ ﺘﻌﺘﻤﺩ ﻋﻠﻰ‬ φ (p ) ‫ ﻭﺫﻟﻙ ﻷﻥ‬t = 0 ‫ ﺩﻉ‬:‫ﺃﻭﻻﹰ‬


1 h
ψ (x ) =
2Dh
∫ ap φ ( p ) e ipn/ h =
1D ∫
ou φ (hD ) e ikv

‫ﻭﺒﺎﺴﺘﺨﺩﺍﻡ ﻤﻌﻜﻭﺱ ﺘﻜﺎﻤل ﻓﻭﺭﻴﺭ‬


φ ( hD ) = ∫ axψ ( x) e −ikx
or
1
⊗ φ (p) =
2Dh
∫ ax ψ (x) e - ipx/ h

1
⇒ ∫ ap φ * ( p) φ ( p) = ∫ ap φ * ( p) ∫ ax ψ (x) e - ipx/ h
2Dh
1
= ∫ ax ψ ∫
ap φ * e - ipx/ h
Dh4424443
124
ψ*

= ∫ ax ψψ * = 1

- ٧٣ -
PDF created with pdfFactory Pro trial version www.pdffactory.com
‫ﻭﻫﺫﻩ ﺍﻟﻨﺘﻴﺠﺔ ﺘﺴﻤﻰ ﺒﻨﻅﺭﻴﺔ ﺒﺎﺭﺴﻴﻔﺎل ‪Parsevali‬‬

‫ﻭﻤﻨﻁﻭﻗﻬﺎ‪ :‬ﻟﻭ ﺃﻥ ﺩﺍﻟﺔ ﻤﻌﺎﻴﺭﺓ ﻟـ ‪ :1‬ﻓﺈﻥ ﺘﺤﻭﻴل ﻓﻭﺭﻴﺭ ﻟﻠﺩﺍﻟﺔ ﻴﻜﻭﻥ ﻤﻌﺎﻴﺭ ﻜﺫﻟﻙ‬
‫‪If a function is normalized to 1, 50 is its Fourier Transform‬‬
‫) ﻋﻨﺩﻨﺎ ﺘﻤﺜﻴﻼﻥ ﻟﻠﺩﻭﺍل‪ :‬ﺘﻤﺜﻴل ‪ ،x‬ﺘﻤﺜﻴل ‪.(regasrauney p‬‬
‫ﺨﺫ‪:‬‬
‫‪h d‬‬
‫) ‪p = ∫ ax ψ * ( x‬‬ ‫)‪ψ (x‬‬
‫‪i dx‬‬
‫‪h d 1‬‬
‫) ‪= ∫ ax ψ * ( x‬‬ ‫∫‬ ‫‪ap φ (p) e ipx/h‬‬
‫‪i dx 2Dh‬‬
‫‪1‬‬
‫‪= ∫ ap φ ( p) p‬‬ ‫∫‬ ‫‪ax ψ * (x) e ipx/ h‬‬
‫‪2Dh‬‬
‫)‪p = ∫ ap φ ( p ) p φ * (p‬‬

‫ﺘﻔﺴﻴﺭ ﺒﺄﻨﻬﺎ ﺍﻟﺩﺍﻟﺔ ﺍﻟﻤﻭﺠﻴﺔ ﻓﻲ ﻓـﺭﺍﻍ ﻜﻤﻴـﺔ‬ ‫) ‪φ (p‬‬ ‫ﻭﻫﺫﻩ ﺍﻟﻨﺘﻴﺠﺔ ﺘﻌﻨﻲ ﺃﻥ‬
‫ﺘﻌﻁﻴﻨﺎ ﻜﺜﺎﻓﺔ ﺍﺤﺘﻤﺎل ﺇﻴﺠﺎﺩ ﺍﻟﺠﺴﻡ ﺍﻟﺫﻱ ﻜﻤﻴﺔ ﺤﺭﻜﺘﻪ ‪.p‬‬ ‫) ‪φ( p‬‬
‫‪2‬‬
‫ﺍﻟﺤﺭﻜﺔ ﺒـ‬

‫ﺴﺘﺠﺩ ﻓﻴﻤﺎ ﺒﻌﺩ ﺃﻥ ﺍﻟﻤﺅﺜﺭﺍﺕ‬ ‫‪x = ih‬‬


‫∂‬
‫ﻓﻲ ﻓﺭﺍﻍ ﻜﻤﻴﺔ ﺍﻟﺤﺭﻜﺔ ‪ x‬ﻟﻬﺎ ﺍﻟﺘﻤﺜﻴل‬
‫‪∂p‬‬

‫‪ ogenatrs‬ﺘﻠﻌﺏ ﺩﻭﺭﺍﹰ ﻤﻬﻤﺎﹰ ﻓﻲ ﻤﻴﻜﺎﻨﻴﻜﺎ ﺍﻟﻜﻡ‪.‬‬

‫ﻭﺍﻵﻥ ﺴﻨﻭﻀﺢ ﺒﻌﺽ ﺨﺼﺎﺌﺼﻬﺎ‪.‬‬


‫ﻋﻠﻰ ﻋﻜﺱ ﺍﻷﺭﻗﺎﻡ ﺍﻟﻌﺎﺩﻴﺔ ﺍﻟﻤﺅﺜﺭﺍﺕ ﻟﻴﺴﺕ ﺒﻨﺩ ﺩﺍﺌﻤﺎﹰ‪ ،‬ﻟﻭ ﻋﺭﻓﻨﺎ‬

‫‪[ A B ] = AB − BA‬‬
‫‪1‬‬

‫∂‬ ‫)‪D∂ ψ (x, t‬‬


‫‪[ p, x ] ψ (x, t) = h‬‬ ‫‪x ψ (x, t ) − x‬‬
‫‪i ∂x‬‬ ‫‪i ∂x‬‬
‫‪D ∂ψ‬‬
‫‪= (x‬‬ ‫‪+ψ ( x, t ) − x‬‬
‫‪∂ψ‬‬
‫)‬ ‫ﺤﻴﻨﺌﺫ‪:‬‬
‫‪i‬‬ ‫‪∂x‬‬ ‫‪∂x‬‬
‫) ‪[ p, x ] ψ (x, t) = h ψ (x, t‬‬
‫‪i‬‬

‫ﻭﻋﻠﻴﻪ ﻨﻘﻭل ﺃﻨﻨﺎ ﻋﻨﺩﻨﺎ ﻋﻼﻗﺔ ﺍﻟﺘﺒﺩﻴل ‪Commutation relatively‬‬

‫‪- ٧٤ -‬‬
‫‪PDF created with pdfFactory Pro trial version www.pdffactory.com‬‬
[ p, x ] = h
i

normalized ware fumets ‫ ﺨﺫ ﺠﺴﻡ ﺍﻟﺫﻱ ﺩﺍﻟﺘﻪ ﺍﻟﻤﻌﺎﻴﺭﺓ‬:‫ﻤﺜﺎل‬


ψ (x ) = 2 α α v e − αv v > 0
= v<0

‫ﻓﻲ ﺍﻟﻘﻴﻤﺔ؟‬ p(x )= ψ (x )


2
‫ﺘﻜﻭﻥ‬ ν ‫( ﻷﻱ ﻗﻴﻡ ﻤﻥ‬a)

v2 , v ‫( ﺍﺤﺴﺏ‬b)
v = 1 α ,v= 0 ‫ ( ﻤﺎ ﻗﻴﻤﺔ ﺍﺤﺘﻤﺎل ﺃﻥ ﺍﻟﺠﺴﻡ ﻴﻭﺠﺩ ﺒﻴﻥ‬c )
p2 , p ‫ﻭﺍﺴﺘﺨﺩﻡ ﺍﻟﻨﺘﻴﺠﺔ ﻟﺤﺴﺎﺏ‬ φ (p) ‫( ﺍﺤﺴﺏ‬b)
ap
=o ‫ ﺘﺤﺩﺙ ﻋﻨﺩﻤﺎ‬p (x) ‫ ﻗﻴﻤﺔ‬:‫ﺃﻭﻻﹰ‬
ax
a a
⇒= ( ψ ψ * ) = (2α α2 e −αv * 2α x α e − αv )
ax ax
a
= (4 α 3 x 2 e − 2αα ) = 0
ax
= ( − 2 α e −2αα v 2 + e − 2αα 2v) = 0
= 2 v (1 − 2 α v ) e − 2αα = 0
1
⇒ 1 = 2 xn ⇒ ν =
α
∞ ∞
(b) v = ∫ dxψ ( x) vψ ( x) = ∫ 2α α v e −αv . v . 2 α e −αx
*

0 0

∞ ∞
= ∫
0
4α 3 v(v 2 e − 2αv ) dx = ∫
0
ax.v (4α 3 v 2 e −2αx )

‫ﻋﻭﺽ ﻋﻥ‬ 1
y = 2α v , dy = 2 αv , dy = 2α ax ⇒ ax = dy , x =
1
y
2 2α

1 3i 3
v =
4α ∫
0
dy y 3 e − y = =
4α 2α
∞ ∞ ∞ ∞

∫ ∫ +n ∫ t = ∫ t n e − t at = Γ (n + 1) = n !
−t −t n −1 − tdt
t e dt = t e
n n
e
0 0 0 0

n is aminke Γ (n + 1) = n ( n − 1) (n − 2).........1 = n!

- ٧٥ -
PDF created with pdfFactory Pro trial version www.pdffactory.com
‫∞‬
‫‪4i‬‬ ‫‪3‬‬
‫=‬ ‫∫‬ ‫= ) ‪dy y 2 ( 4 α 3 x 2 e − 2αx‬‬ ‫‪= 2‬‬
‫‪2‬‬
‫‪a‬‬
‫‪0‬‬ ‫‪8α‬‬ ‫‪2‬‬
‫‪α‬‬
‫=‬

‫)ﻭﺍﺼل ﺍﻟﻤﺤﺎﻀﺭﺓ ( ﺍﻷﺨﻴﺭﺓ )ﺍﻟﺴﺅﺍل ﻓﻲ ﺍﻟﻤﻨﺘﺼﻑ(‪:‬‬


‫ﻭﺍﺼل ﺨﻭﺍﺹ ﺍﻟﻤﺅﺜﺭﺍﺕ‪:‬‬
‫ﻜﻤﻴـﺔ‬ ‫‪p‬‬ ‫ﻅﻬﻭﺭ ﺍﻟﻤﺅﺜﺭﺍﺕ ‪ p‬ﺒـ ‪ I‬ﻤﻤﻜﻨﻪ ﺃﻥ ﻴﺅﺩﻱ ﺇﻟﻰ ﺍﻟﺸﺩﺓ ﻓـﻲ ﺃﻥ‬
‫ﺤﻘﻴﻘﻴﺔ ﻭﻟﻜﻨﻨﺎ ﻴﻤﻜﻨﻨﺎ ﻤﺭﺍﺠﻌﺔ ﻫﺫﻩ ﺍﻟﺤﻘﻴﻘﺔ ﺃﻥ ‪ p‬ﻜﻤﻴﺔ ﺤﻘﻴﻘﻴﺔ‪.‬‬
‫‪h ∂ψ‬‬ ‫* ‪h ∂ψ‬‬
‫* ‪= ∫ ax ψ‬‬ ‫‪− ∫ ax ψ ( −‬‬
‫‪α‬‬
‫‪p − p‬‬ ‫)‬
‫‪i ∂x‬‬ ‫‪i ∂x‬‬
‫‪h‬‬ ‫* ‪∂ψ ∂ψ‬‬
‫∫ ‪i‬‬
‫=‬ ‫* ‪ax ( ψ‬‬ ‫‪+‬‬ ‫)‪ψ‬‬
‫‪∂x‬‬ ‫‪∂x‬‬
‫‪h‬‬ ‫∂‬
‫∫ =‬ ‫‪ax‬‬ ‫‪( ψ * ψ) = 0‬‬
‫‪i‬‬ ‫‪∂x‬‬

‫ﺒﺸﺭﻁ ﺃﻥ ﺍﻟﺩﺍﻟﺔ ﺘﺘﻼﺸﻰ ﻋﻨﺩ ∞ ‪ ،‬ﻭﻫﺫﺍ ﺍﻟﺸﺭﻁ ﻤﺘﺤﻘﻕ ﻓﻲ ﺍﻟﺩﻭﺍل ﻓﻲ ﻤﺭﺒﻌﺔ‬


‫ﺍﻟﺘﻜﺎﻤل ﻭﻟﻜﻥ ﺇﺫﺍ ﻟﻡ ﻴﺘﺤﻘﻕ ﻫﺫﺍ ﺍﻟﺸﺭﻁ‪ ،‬ﻤﺜﻼﹰ ﻓﻲ ﺤﺎﻟﺔ ﺍﻟﺩﻭﺍل ﺍﻟﺘﻲ ﺘﻐﻴﺭ ﻨﻔـﺴﻬﺎ‬
‫‪Periodical‬‬

‫)‪ψ(x )=ψ(x +L‬‬

‫ﻻﺯﺍل ﻴﻜـﻭﻥ ﻤـﺅﺜﺭ‬ ‫∂‪t‬‬


‫ﻋﻨـﺩ‬ ‫‪0≤ v < L‬‬ ‫ﻭﻟﻭ ﻗﺼﺭﻨﺎ ﺃﻨﻔﺴﻨﺎ ﻟﻠﻌﻤل ﻓـﻲ‬
‫‪lax‬‬

‫ﻫﻴﺭﻭﻤﺘﻲ ‪hermitical poetry‬‬


‫∂‬
‫‪L‬‬
‫‪h‬‬
‫= ‪p − p‬‬ ‫∫‬
‫*‬
‫‪ax‬‬ ‫) )‪(ψ * (x) ψ (x‬‬
‫‪i 0‬‬ ‫‪∂x‬‬
‫‪h‬‬ ‫‪h‬‬
‫=‬ ‫‪ψ(L) − ψ(o) =0‬‬
‫‪2‬‬ ‫‪2‬‬

‫‪i‬‬ ‫‪i‬‬

‫ﺍﻟﻤﺅﺜﺭ ﺍﻟﺫﻱ ﻗﻴﻤﺘﻪ ﺍﻟﻤﺘﻭﻗﻌﺔ ﻟﻜل ﺍﻟﻘﻴﻤﺔ ﺍﻟﻤﺴﻤﻭﺤﺔ ﻟﻠﺩﺍﻟﺔ ﺍﻟﻤﻭﺠﻴـﺔ ﺘﻜـﻭﻥ‬
‫ﺤﻘﻴﻘﻴﺔ ﻴﺴﻤﻰ ‪ ،lerimti of opener‬ﻭﻋﻠﻴﻪ ﻓﺈﻥ ‪ p‬ﻤﺜل ‪ x‬ﻫﻭ ﻤﺅﺜﺭ ﻫﻴﺭﻤﺘﻲ‪.‬‬
‫‪An operator whose expectation value for all admissible wave‬‬
‫‪functims is real is called a hermitical operator‬‬

‫‪- ٧٦ -‬‬
‫‪PDF created with pdfFactory Pro trial version www.pdffactory.com‬‬
‫ﻭﻟﻌﻠﻨﺎ ﻨﺴﺘﻨﺞ ﻫﺫﺍ ﺍﻟﻔﺼل ﺒﻤﻼﺤﻅﺔ ﺃﻥ ﺍﻟﻤﻌﺎﺩﻟﺔ‪:‬‬
‫)‪∂ψ (x, t‬‬ ‫)‪h 2 ∂ 2 ψ (x, t‬‬
‫= ‪ih‬‬ ‫‪= −‬‬
‫‪∂t‬‬ ‫‪2m‬‬ ‫‪∂x 2‬‬

‫ﻴﻤﻜﻥ ﻜﺘﺎﺒﺘﻬﺎ ﺒﺎﻟﺼﻭﺭﺓ ﺍﻟﺘﺎﻟﻴﺔ‪:‬‬ ‫= ‪p op‬‬


‫∂ ‪h‬‬
‫ﺒﺎﻟﺘﻌﺭﻴﻑ‪:‬‬
‫‪i ∂x‬‬
‫‪2‬‬
‫)‪∂ψ (x, t‬‬ ‫‪p op‬‬
‫= ‪ih‬‬ ‫‪= −‬‬ ‫)‪ψ (x, t‬‬
‫‪∂t‬‬ ‫‪2m‬‬

‫ﺍﻟﻤﺅﺜﺭ ﻋﻠﻰ ﺍﻟﺠﺎﻨﺏ ﺍﻷﻴﻤﻥ ﻫﻭ ﺍﻟﻁﺎﻗﺔ ﻟﺠﺴﻡ ﺤﺭ‪ .‬ﻭﻟﻭ ﻋﻤﻤﻨﺎ ﻫﺫﺍ ﺍﻟﺠـﺴﻡ‬
‫ﻓﻲ ﺠﻬﺩ‪ ،‬ﻨﺘﺤﺼل ﻋﻠﻰ‪:‬‬
‫‪∂ψ (x, t)  p op‬‬ ‫‪‬‬
‫‪2‬‬

‫= ‪ih‬‬ ‫‪=‬‬ ‫)‪+ V (x‬‬ ‫)‪ ψ (x, t‬‬


‫‪∂t‬‬ ‫‪ 2m‬‬ ‫‪‬‬
‫‪or‬‬
‫)‪∂ψ (x, t‬‬ ‫)‪h 2 ∂ 2 ψ (x, t‬‬
‫= ‪ih‬‬ ‫‪=−‬‬ ‫)‪+ V (x) ψ (x, t‬‬
‫‪∂t‬‬ ‫‪2m ∂x 2‬‬
‫‪or‬‬
‫)‪∂ψ (x, t‬‬
‫‪ih‬‬ ‫)‪= H ψ (x, t‬‬
‫‪∂t‬‬

‫ﺤﻴﺙ ‪ H‬ﻫﻭ ﻤﺅﺜﺭ ﺍﻟﻁﺎﻗﺔ‪ .‬ﻭﻴﺴﻤﻰ ﻋﺎﺩﺓ ﺍﻟﻬﺎﻤﻠﺘﻭﻨﻴﺎﻥ ‪ Hamiltonian‬ﻷﻨـﻪ‬


‫ﺼﻭﺭﺓ ﻤﺅﺜﺭﺓ ﻟﻠﺩﺍﻟﺔ ﻫﺎﻤﻠﺘﻭﻨﻴﺎﻥ ﺍﻟﻤﻴﻜﺎﻨﻴﻜﺎ ﺍﻟﺘﻘﻠﻴﺩﻴﺔ‪.‬‬

‫⇐ ‪p2‬‬ ‫ﻭﺒﻤﺎ ﺃﻥ ‪ p‬ﻫﻭ ﻤﺅﺜﺭ ﻫﻴﺭﻤﻴﺘﻲ‬


‫ﺠﻬﺩ ﺤﻘﻴﻘﻲ‬ ‫=‪H‬‬
‫‪p2‬‬
‫)‪+ V (x‬‬
‫‪2m‬‬

‫‪- ٧٧ -‬‬
‫‪PDF created with pdfFactory Pro trial version www.pdffactory.com‬‬
‫ﺱ‪ /‬ﺍﻟﺩﺍﻟﺔ ﺍﻟﻤﻌﻁﺎﺓ‪:‬‬
‫‪N‬‬
‫= )‪ψ (x‬‬
‫‪x +a2‬‬
‫‪2‬‬

‫)‪ (a‬ﺍﺤﺴﺏ ‪ N‬ﺍﻟﻼﺯﻤﺔ ﺍﻟﻤﻌﺎﻴﺭﺓ ﺍﻟﺩﺍﻟﺔ ‪. ψ‬‬


‫‪ ،‬ﻤﺎ ﻗﻴﻡ ‪ n‬ﺍﻟﺘﻲ ﺘﺅﺩﻱ ﺇﻟﻰ ﺘﻜﺎﻤل ﻤﺘﺠﻤﻊ‪.‬‬ ‫‪vn‬‬ ‫ﻟﺤﺴﺎﺏ‬ ‫‪ψ‬‬ ‫)‪ (b‬ﺍﺴﺘﺨﺩﺍﻡ ﺍﻟﺩﺍﻟﺔ‬
‫ﺒﺎﺴﺘﺨﺩﺍﻡ ﺍﻟﺩﺍﻟﺔ ﺍﻟﻤﻭﺠﻴﺔ ﻓﻲ ﻓﺭﺍﻍ ﻜﻤﻴﺔ ﺍﻟﺤﺭﻜﺔ‪.‬‬ ‫‪p2‬‬ ‫)‪ (c‬ﺍﺤﺴﺏ‬
‫=‪∆ x‬‬ ‫‪x1 − x2‬‬ ‫)‪ (a‬ﺍﺤﺴﺏ‬

‫=‪Δx‬‬ ‫‪p1 − p 2‬‬

‫؟‬ ‫‪Δx Δp‬‬ ‫ﻟﺤﺴﺎﺏ‬


‫∞‬ ‫∞‬
‫‪dx‬‬ ‫‪dx‬‬ ‫!)‪D (2n − 3‬‬
‫‪N2‬‬ ‫∫‬
‫∞‪−‬‬ ‫‪(x + a 2 ) 2‬‬
‫‪2‬‬
‫‪=1‬‬ ‫∫‬‫‪0‬‬ ‫) ‪(a + x‬‬
‫‪2‬‬ ‫‪2 n‬‬
‫‪= 2n −1‬‬
‫‪2a‬‬ ‫!)‪(2n − 2‬‬
‫∞‪+‬‬ ‫∞‪+‬‬
‫‪dx‬‬ ‫‪‬‬ ‫‪x‬‬ ‫‪1‬‬ ‫‪x ‬‬
‫∫‬
‫∞‪−‬‬
‫‪2‬‬ ‫‪2‬‬
‫‪= 2 2‬‬ ‫‪2‬‬
‫‪+ 3 + tu −1 ( )‬‬
‫‪(x + a )  2a (x + a ) 2a‬‬ ‫∞‪a  −‬‬
‫‪D‬‬ ‫‪2a 3‬‬
‫=‬ ‫⇒‬ ‫‪N‬‬ ‫=‬
‫‪2a 3‬‬ ‫‪D‬‬
‫‪2a 3‬‬ ‫‪1‬‬
‫= ‪∴ψ‬‬
‫‪D x +a2‬‬
‫‪2‬‬

‫)‪(b‬‬
‫∞‪+‬‬
‫‪2a 2‬‬ ‫‪a2‬‬
‫=‬ ‫∫‬
‫‪n‬‬
‫‪x‬‬
‫‪D‬‬ ‫∞‪−‬‬ ‫‪(x 2 + a 1 ) 2‬‬

‫ﻓﻘﻁ ﻋﻨﺩﻤﺎ ‪n < 3‬‬ ‫‪xn‬‬ ‫ﺘﺘﻘﺎﺭﺏ‬

‫‪n = 0, 1, 2,‬‬ ‫⇐‬

‫‪xn‬‬ ‫‪=0‬‬ ‫ﺍﻟﻤﻜﺎﻤل ﻟﺯﻭﺍﻴﺎ‬

‫‪- ٧٨ -‬‬
‫‪PDF created with pdfFactory Pro trial version www.pdffactory.com‬‬
+∞
2a 3 x2
= ∫
n
x dx
D −∞ (x 2 + a 2 ) 2
+∞
x 2 ax  x 1 
∫ (x + a )
2 2 2
= 2 2
+ + tam −1 (x/a) 
 2 (x + a ) 2a  −∞
= D / 2a , n2 = 2a 2

N
φ (p) =
2Dh
∫ ax e −2p x/h
+∞
a a dx e −i px/h
=
D h ∫
−∞ x2 +a2
p = hk → p/h = k

Z = x tiy ‫ﻟﻠﺘﻜﺎﻤل ﻨﺴﺘﺨﺩﻡ ﻜﻭﻨﺘﻭﺭ ﻨﺼﻑ ﺩﺍﺌﺭﺓ ﻓﻲ ﺍﻟﻤﺴﺘﻭﻯ ﺍﻟﻤﺭﻜﺏ‬


= 2 D i R (−ia)
2Di − k ( −ik) D −ka
= e = e
2ia a
φ (p) = D/h e − pa/h p>0
= a/h e −pa/ h p>0

2a
p2 =
h ∫
0
p 2 e − pa/h ap

= ∫ φ * p 2 φ ap
2
p
ap h2
=z p2 = z2
h a2

2a h 3
= ∫ z 2 e − z az
2
p ( )
h a 0
3
h h
= 2( ) Γ (3) = 4 ( ) 2
a a
Γ (n + 1 ) = n !
Γ (2 + 1) 2!
p =0 Δp = p2

Δ p = 2h/a Δx = Δx

- ٧٩ -
PDF created with pdfFactory Pro trial version www.pdffactory.com
.‫ﺃﺜﺒﺕ ﺃﻥ ﺍﻟﻤﺅﺜﺭ‬ ψ (D ) = ψ ( − D ) ‫ ﻟﻭ ﺃﻥ ﺍﻟﺩﺍﻟﺔ ﺍﻟﻤﻭﺠﻴﺔ ﺘﺤﻘﻕ ﺍﻟﺸﺭﻁ‬/‫ﺱ‬
h d
∠=
i dθ

‫ﻤﺅﺜﺭ ﻫﻴﺭﻤﻴﺘﻲ‬

∠ − ∠
*
=0 ‫ﻹﺜﺒﺎﺕ ﻫﺫﺍ ﻴﺠﺏ ﺃﻥ‬
D
h d
∠ = ∫
−D
dθ ψ * ( θ )
i dθ
ψ

D
h d *
∠ =− ∫
*
dθ ψ ψ
−D
i dθ

h  
D d d

∠ − ∠ = ∫ +
*
dθ ( ψ *
( θ ) dθ
ψ ) (ψ dθ
ψ * )
i  −D 
D
h d
=
i ∫
−D


( ψ* ψ )

h
[
= ψ ( D ) ψ ( D ) − ψ * ( − D ) ψ (D )
i
*
]
ψ (D ) =ψ ( − D )
= 0

∠ = ∠
*
‫∴ ﺍﻟﻤﺅﺜﺭ‬
∞ k/2
1 1
3.c. ∫
−∞
dk N 2 = N 2 ∫
− k/2
dk = N 2 k =
2D
⇒ N=
2D k

.‫ ﻓﻲ ﺍﻟﺤﺎﻟﺘﻴﻥ ﻤﺘﺴﺎﻭﻴﺔ‬N ‫ﻭﻋﻠﻴﻪ ﻓﺈﻥ ﻗﻴﻡ‬


‫( ﻴﻤﻜﻥ ﺘﻘﺩﻴﺭﻩ ﺒﺎﻟﻤﺴﺎﻓﺔ ﺒﻴﻥ ﻨﻘﻁﺘـﻴﻥ ﻋﻨـﺩﻫﺎ‬p) ‫ﻋﺭﺽ ﺍﻟﺩﺍﻟﺔ ﻤﻥ ﺍﻟﺠﺯﺀ‬
:‫ ﻭﻫﺫﺍ ﻴﺤﺩﺙ ﻋﻨﺩ‬3.d ‫ﺘﺨﺘﻔﻲ‬ sin
kx
2

 x = + k
2D
kx
sin =0  2D
2  x = − k

4D k u
⇒ Δx= , Δk = − (− ) = k
.k ‫ﻭﻫﻲ ﻨﺘﻴﺠﺔ ﻻ ﺘﻌﺘﻤﺩ ﻋﻠﻰ‬ k 2 2
4D
⇒ Δ x. Δ k = . k = 4D
k

- ٨٠ -
PDF created with pdfFactory Pro trial version www.pdffactory.com
‫ﺍﻟﺩﻭﺍل ﺍﻟﺨﺎﺼﺔ ﻭﺍﻟﻘﻴﻡ ﺍﻟﺨﺎﺼﺔ‪Eigenfunctions and Eigenvalues :‬‬

‫ﻟﻭ ﺃﺨﺫﻨﺎ ﻤﻌﺎﺩﻟﺔ ﺸﺭﻭﺩﻨﺠﺭ ﺍﻟﺘﻲ ﺘﻌﺘﻤﺩ ﻋﻠﻰ ﺍﻟﺯﻤﻥ‪:‬‬


‫)‪∂ψ (x, t‬‬ ‫)‪h 2 ∂ 2 ψ(x, t‬‬
‫‪ih‬‬ ‫‪=−‬‬ ‫)‪+ v (x) ψ (x, t‬‬ ‫)‪(1‬‬
‫‪∂t‬‬ ‫‪2m ∂x 2‬‬

‫ﻭﻟﺤل ﻫﺫﻩ ﺍﻟﻤﻌﺎﺩﻟﺔ ﻨﻘﺼﻬﺎ ﻟﻤﻌﺎﺩﻟﺘﻴﻥ ﻨﻕ‬


‫)‪ψ (x, t) , T(t) u (x‬‬
‫)‪dT (t)  h 2 d 2 u (x‬‬ ‫‪‬‬
‫)‪⇒ ih u (x‬‬ ‫‪= −‬‬ ‫‪2‬‬
‫‪+ v (x) u (x) ‬‬
‫‪dt‬‬ ‫‪ 2m dx‬‬ ‫‪‬‬

‫ﺒﺎﻟﻘﺴﻤﺔ ﻋﻠﻰ )‪u (x) T (t‬‬


‫‪(sin 1‬‬
‫‪dT (t)dt‬‬ ‫)‪2m) (a 2 u (x) / dx 2 ) + V (x)u (x‬‬
‫‪⇒ ih‬‬ ‫=‬
‫)‪T (t‬‬ ‫)‪u (x‬‬

‫ﻭﻫﺫﺍ ﻜﻤﻴﺔ ﻴﻤﻜﻥ ﺘﺤﻘﻴﻘﻪ ﻓﻘﻁ ﺍﻓﺘﺭﺍﺽ ﺃﻥ ﻜﻼ ﺍﻟﻁﺭﻓﻴﻥ ﻴﺴﺎﻭﻱ ﺜﺎﺒﺕ ﻭﺍﻟﺫﻱ‬
‫ﻨﺴﻤﻴﻪ ‪E‬‬
‫)‪d T (t‬‬ ‫)‪dT (E‬‬ ‫‪i1‬‬ ‫‪i‬‬
‫‪⇒ ih‬‬
‫‪dt‬‬
‫⇒ )‪= E T (t‬‬ ‫∫‬ ‫)‪T (E‬‬
‫‪= ∫ − E dt = − Et‬‬
‫‪ih‬‬ ‫‪h‬‬
‫‪i‬‬
‫‪i‬‬ ‫‪− ET‬‬
‫‪lm T (t) = - Et ⇒ T (t) = Ce t‬‬
‫‪h‬‬

‫ﻭﺍﻟﻤﻌﺎﺩﻟﺔ ﺍﻟﺜﺎﻨﻴﺔ‪:‬‬
‫)‪h 2 d 2 u (x‬‬
‫‪−‬‬ ‫)‪+ v (x) u (x) = Eu (u‬‬
‫‪2m dx 2‬‬

‫ﻤﻌﺎﺩﻟﺔ ﺸﺭﻭﺩﻨﺠﺭ ﺍﻟﻐﻴﺭ ﻤﻌﺘﻤﺩﺓ ﻋﻠﻰ ﺍﻟﺯﻤﻥ‪ time indgader S-E :‬ﻭﻫـﺫﻩ‬
‫ﻤﻌﺎﺩﻟﺔ ﻫﻲ ﺩﺍﻟﺔ‬ ‫)‪ψ (x, t‬‬ ‫ﺍﻟﻤﻌﺎﺩﻟﺔ ﺘﻔﺭﻕ ﻋﻥ )‪ (1‬ﻭﺍﻟﺘﻲ ﺘﺼﻑ ﺍﻟﺘﻐﻴﺭ ﺍﻟﺯﻤﻨﻲ ﻟﻠﺩﺍﻟﺔ‬
‫ﺨﺎﺼﺔ‪ .Ligea valu equation .‬ﻭﻟﻔﻬﻡ ﻤﺎﺫﺍ ﻴﻌﻨﻲ ﻫﺫﺍ ﺍﻟﻜﻼﻡ ﺩﻋﻨﺎ ﻨﻌـﺩﺩ ﻤـﺭﺓ‬
‫ﺃﺨﺭﻯ ﻟﻤﻔﻬﻭﻡ ﺍﻟﻤﺅﺜﺭﺓ‪.‬‬
‫ﺒﺸﻜل ﻋﺎﻡ ﻤﺅﺜﺭ ﻤﺎ‪ :‬ﻴﺅﺜﺭ ﻋﻠﻰ ﺩﺍﻟﺔ ﻭﻴﺄﺨﺫﻫﺎ ﻟﺩﺍﻟﺔ ﺃﺨﺭﻯ‪ ،‬ﻤﺜﺎل ﻋﻠﻰ ﺫﻟﻙ‪:‬‬

‫‪- ٨١ -‬‬
‫‪PDF created with pdfFactory Pro trial version www.pdffactory.com‬‬
‫‪1.‬‬ ‫‪of (x) = f (x) + x 2‬‬
‫] )‪of (x) = [f (x‬‬
‫‪2‬‬
‫‪2.‬‬
‫‪3.‬‬ ‫)‪of (x) = f (3 x 2 + 1‬‬
‫‪3‬‬
‫‪ d f (x) ‬‬
‫‪4. of (x) = ‬‬ ‫‪‬‬
‫‪ dx ‬‬
‫)‪5. of (x) = df / dx − 2f (x‬‬
‫)‪6. of (x) = λf (x‬‬

‫ﻜل ﻫﺫﻩ ﺍﻷﻤﺜﻠﺔ ﺘﺅﺜﺭ ﻓﻲ ﺍﻟﺨﺎﺼﻴﺔ ﻫﻲ ﺒﺈﻋﻁﺎﺀ ﺩﺍﻟﺔ )‪ f (x‬ﺘﻭﺠﺩ ﻗﺎﻋﺩﺓ ﺍﻟﺘﻲ‬
‫ﺘﺤﺩﺩ )‪ of (x‬ﻟﻨﺎ‪.‬‬
‫ﻴﻭﺠﺩ ﻨﻭﻉ ﺨﺎﺹ ﻤﻥ ﺍﻟﻤﺅﺜﺭﺍﺕ ﻴﺴﻤﻰ ﺍﻟﻤﺅﺜﺭﺍﺕ ﺍﻟﺨﻁﻴﺔ‪.‬‬
‫ﻭﻫﺫﻩ ﺍﻟﻤﺅﺜﺭﺍﺕ ﻟﻪ ﺍﻟﺨﺎﺼﻴﺔ‪:‬‬

‫] )‪∠ [f 1 (x) + f 2 (x) ] = ∠ [f 1 (x) + ∠ f 2 (x‬‬

‫ﻭﺒﻭﺠﻭﺩ ‪ c‬ﻜﺭﻗﻡ ﻤﻭﺠﺏ ﺍﺨﺘﻴﺎﺭﻱ‪.‬‬


‫)‪∠cf(x) = c * ∠ f(x‬‬ ‫ﻤﻼﺤﻅﺔ‪ :‬ﻴﻭﺠﺩ ﻤﺅﺜﺭﺍﺕ ﻋﻜﺴﻴﺔ ﺍﻟﺨﻁﻴﺔ ﻭﻟﻬﺎ‬

‫ﻭﻋﻠﻴﻪ ﻓﺈﻨﻪ ﻓﻲ ﺍﻟﻘﺎﺌﻤﺔ ﺍﻟﺴﺎﺒﻘﺔ ﻓﺈﻥ ﺍﻟﻤﺅﺜﺭﻴﻥ ﺍﻷﺨﻴﺭﻴﻥ ﻫﻤـﺎ‬ ‫)‪∠ cf (x) = c * ∠ f (x‬‬

‫ﺍﻟﺨﻁﻴﺎﻥ‪.‬‬
‫ﺍﻟﻤﺅﺜﺭ ﺍﻟﺨﻁﻲ ﺴﻴﻨﻘل ﺩﺍﻟﺔ ﻤﺎ ﻟﺩﺍﻟﺔ ﺃﺨﺭﻯ‪.‬‬
‫)‪df (x‬‬
‫= )‪∠ f (x‬‬ ‫)‪− 2 f (x‬‬
‫‪ax‬‬

‫ﻭﻴﻤﻜﻨﻨﺎ ﺘﺸﺒﻪ ﺍﻟﺩﻭﺍل ﻜﻤﺘﺠﻬﺎﺕ ﻓﻲ ﺜﻼﺜﺔ ﺃﺒﻌﺎﺩ ﻭﺘﺄﺜﻴﺭ ﺍﻟﻤﺅﺜﺭﺍﺕ ﻴﻜﻭﻥ ﺒﻨﻘل‬
‫)ﺘﺤﻭﻴل( ﺍﻟﻤﺘﺠﻪ ﻟﻤﺘﺠﻪ ﺁﺨﺭ‪ ،‬ﻭﻴﺄﺨﺫ ﺍﻟﻤﺘﺠﻪ ﻜﻭﺤﺩﺓ )‪ (amitvedre‬ﻭﻤـﺅﺜﺭ ﻴﻨﻘـل‬
‫ﻨﻘﻁﺔ ﻋﻠﻰ ‪ unit sphere‬ﻟﻨﻘﻁﺔ ﺃﺨﺭﻯ ﻭﻤﺅﺜﺭ ﻤﺎ ﻴﻤﻜﻥ ﺍﻋﺘﺒـﺎﺭﻩ ﺒﻘﺎﺒـل ﺩﻭﺭﺍﻥ‬
‫ﺍﻟﻨﻘﻁﺔ ﺤﻭل ﻤﺤﻭﺭ ‪ z‬ﺒﺜﻼﺜﻴﻥ ‪. 30‬‬
‫‪°‬‬

‫‪A → A′‬‬ ‫ﺜﻼﺙ ﺤﺎﻻﺕ‪:‬‬

‫‪- ٨٢ -‬‬
‫‪PDF created with pdfFactory Pro trial version www.pdffactory.com‬‬
‫‪B → B′‬‬

‫‪C → C′‬‬

‫ﻻﺤﻅ ﺃﻥ ﺍﻟﻨﻘﻁﺔ ﻋﻠﻰ ﺍﻟﻘﻁﺏ ﺘﻨﻘل ﻋﻠﻰ ﻨﻔﺴﻬﺎ )ﻭﻜﺫﻟﻙ ﺍﻟﻨﻘﻁﺔ ﺍﻟﺘﻲ ﻋﻠـﻰ‬
‫ﺍﻟﻘﻁﺏ ﺍﻟﺠﻨﻭﺒﻲ( ﻭﻫﺫﻩ ﺍﻟﺤﺎﻟﺔ ﺍﻟﺨﺎﺼﺔ ﻟﻤﺜﺎل ﻤﺅﺜﺭ ﻤﺜل ﻤﻌﺎﺩﻟﺔ )‪ ( 4 – 6‬ﻭﺍﻟـﺫﻱ‬
‫ﻴﻤﻜﻥ ﻜﺘﺎﺒﺘﻪ ﻜﻤﺎ ﻴﻠﻲ‪:‬‬
‫‪H u E (x) = E‬‬ ‫)‪u E (x‬‬ ‫)‪(4-10‬‬

‫ﻤﻨﻁﻭﻕ ﺍﻟﻤﻌﺎﺩﻟﺔ‪ ،H :‬ﻤﺅﺜﺭ ﺍﻟﻬﺎﻤﻠﺘﻴﻭﻨﻴﺎﻥ ﻴﺅﺜﺭ ﻋﻠﻰ ﻨﻭﻑ ﻤﻌﻴﻥ ﻤﻥ ﺍﻟﺩﻭﺍل‬


‫ﺴﻴﻌﻁﻴﻨﺎ ﻤﻥ ﺠﺩﻴﺩ ﺍﻟﺩﺍﻟﺔ ﺍﻟﺘﻲ ﻴﺅﺜﺭ ﻋﻠﻴﻬﺎ ﻤﻀﺭﻭﺒﺔ ﻓﻲ ﺜﺎﺒﺕ‪.‬‬
‫ﻫﺫﺍ ﺍﻟﺜﺎﺒﺕ ﻴﺴﻤﻰ ‪ eigeulalw‬ﻭﻤﻥ ﺍﻟﻤﻌﺎﺩﻟﺔ ﻴﻌﺘﻤﺩ ﻋﻠﻰ ‪ E‬ﻭﺤل ﺍﻟﻤﻌﺎﺩﻟـﺔ‬
‫ﻴﻌﺘﻤﺩ ﻋﻠﻰ ‪.E‬‬
‫ﻴﺴﻤﻰ ‪ eigeufaction‬ﺍﻟﺫﻱ ﻴﻨﺎﻅﺭ ﺍﻟﻘﻴﻤﺔ ﺍﻟﺨﺎﺼﺔ ‪E‬‬ ‫)‪u E (x‬‬ ‫ﻭﺤل ﺍﻟﻤﻌﺎﺩﻟﺔ‬
‫ﻟﻠﻤﺅﺜﺭ ‪.H‬‬

‫)‪∠ f (x) = λ f (x‬‬ ‫ﻤﺜﺎل‪ :‬ﺤل ﻤﺴﺄﻟﺔ ﺍﻟﻘﻴﻤﺔ ﺍﻟﺨﺎﺼﺔ‬


‫ﺤﻴﺙ‪:‬‬
‫)‪h df (x‬‬
‫= )‪∠ f (x‬‬ ‫)‪− β x f (x‬‬
‫‪i ax‬‬

‫ﻭﻟﻬﺎ ﺸﺭﻁ ﺍﻷﻁﺭﺍﻑ )ﺍﻟﺸﺭﻭﻁ ﺍﻟﺤﺩﻴﺔ( ‪.bouday‬‬ ‫‪a ≤x≤a‬‬ ‫ﻓﻲ ﺍﻟﻤﻨﻁﻘﺔ‬

‫)‪f (a) = f ( − a‬‬

‫‪- ٨٣ -‬‬
‫‪PDF created with pdfFactory Pro trial version www.pdffactory.com‬‬
:‫ﺍﻟﺤل‬
∠ f (x) = λ f (x)
h a f (x)
− β v f (x) = λ f (x)
i ax

= [β + t (x) + λ f (x)]= [β x + λ ]+ (x)


df (x) i i
dx h h
df i x
= (β + λ) ax
f h
i x2
h f = (β + λx )+c
h 2
iβ x 2
+ iλλxh
2
f (x) = Ae

energy eigenvahes ‫ ﺘﺴﻤﻰ ﻗﻴﻡ ﺍﻟﻁﺎﻗﺔ ﺍﻟﺨﺎﺼﺔ‬H ‫ﺍﻟﻘﻴﻤﺔ ﺍﻟﺨﺎﺼﺔ ﻟﻠﻤﺅﺜﺭ‬


p 2 op
H= + V (x)
2m

the Eigenvalne Poeem for a ‫ﻤﺴﺄﻟﺔ ﺍﻟﻘﻴﻤﺔ ﺍﻟﺨﺎﺼﺔ ﻟﺠﺴﻡ ﻓﻲ ﺼﻨﺩﻭﻕ‬


.Particle in box


h 2 a 2 u (x)
+ V (x) u (x) = Eu (x) (4 – 6) ‫ﺒﺎﻋﺘﺒﺎﺭ‬
2m ax 2

:‫ﺤﻴﺙ‬
V (x) = ∞ x <0
= 0 °<x<a
=∞ a<x

:(4 – 6 ) ‫ﻤﻥ‬
u (x) = 0 x<0
=0 a<x

.( 4 – 6 ) ‫ ﺍﻟﻤﻌﺎﺩﻟﺔ‬V ( x ) = 0 ‫ﺩﺍﺨل ﺍﻟﺼﻨﺩﻭﻕ ﺤﻴﺙ‬


d 2 u (x) 2mE
+ 2 u (x) = 0
dx 2 h

.‫ ﺍﻟﻤﻌﺎﺩﻟﺔ ﺘﺄﺨﺫ ﺍﻟﺸﻜل‬،E < 0 ‫ﻟﻭﻥ ﺃﻥ‬

- ٨٤ -
PDF created with pdfFactory Pro trial version www.pdffactory.com
d 2 u (x) 2m E
2
− K 2 u (x) = 0 , k2 =
dx h2

e − lx , e kx ‫ﺍﻟﺤل ﺍﻟﻌﺎﻡ ﻫﻭ ﺇﺘﺤﺎﺩ ﺨﻁﻲ ﻤﻥ‬


k2 =
2mE
,E ‫⇐ ﺒﻘﻴﻡ ﺇﻴﺠﺎﺒﻴﺔ‬
h2

d 2 u (x)
+ k 2 u (x) = 0
dx 2

A sin Kx + B con lex ‫ﺍﻟﺤﺎﻟﺔ ﺍﻟﻌﺎﻡ‬

u ( 0 ) = 0 ‫ﻭﻟﻜﻥ ﻟﺯﻭﻡ‬

:‫ﻴﺤﺼﺭﻨﺎ ﻻﺨﺘﻴﺎﺭ ﺍﻟﺸﺭﻁ‬


u (x) = Asin k
u (a ) = 0

⇐ ka = nD n = 1, 2, 3.........

.‫ﻭﻋﻠﻴﻪ ﻓﺈﻥ ﻗﻴﻡ ﺍﻟﻁﺎﻗﺔ ﺍﻟﺨﺎﺼﺔ‬


h 2u 2 h 2 D 2 u 2
En = = n = 1, 2, 3.........
2m 2ma 2

.‫ﻭﺒﺈﻤﻜﺎﻨﻨﺎ ﺍﺨﺘﺒﺎﺭ ﺃﻥ ﺍﻟﺤﻠﻭل ﻤﻌﺎﻴﺭﺓ ﻟﻭ ﻜﺎﻨﺕ ﻗﻴﻤﺔ‬


A = 2la
2 hDx
u n (x ) = sin
a a

.‫ﻭﺍﻟﺤﻠﻭل ﻟﻬﺎ ﺍﻟﺨﺎﺼﻴﺔ‬


a a
2 nD χ m Dx
∫ dx u (x ) u m (x) = ∫ dx
*
n
sin sin
0 0
a a a
1 
a
(n − m) Dx (n + m) Dx 
= ∫
a0 
cos (
a
) − cos (
a
)

sin (n − m) D sin (n + m) D
= −
( n − m )D ( n + m )D
= 0 whem n± m
= 1 whem n =m

- ٨٥ -
PDF created with pdfFactory Pro trial version www.pdffactory.com
‫‪a‬‬

‫‪∫ dx u‬‬ ‫‪(x ) u m (x) = smn‬‬ ‫)‪..................(4 − 23‬‬


‫*‬
‫‪n‬‬
‫‪0‬‬

‫* ﻭﻫﺫﺍ ﻴﻌﻨﻲ ﺃﻥ ﺍﻟﺩﻭﺍل ﺍﻟﺨﺎﺼﺔ ﺍﻟﻤﻨﺎﻅﺭﺓ ﻟﻘﻴﻡ ﺨﺎﺼﺔ ﻤﺨﺘﻠﻔﺔ ﺘﻜـﻭﻥ ﻤﺘﻌﺎﻤـﺩﺓ‬
‫‪.ortagoral‬‬

‫* ﻟﻭ ﺃﻥ ﺍﻟﺩﻭﺍل ﺍﻟﺨﺎﺼﺔ ﻤﻌﺎﻴﺭﺓ‪ .‬ﺒﺩﻗﺔ‪ ،‬ﻜﻤﺎ ﻫﻭ ﻅـﺎﻫﺭ ﻤـﻥ )‪ ،(4-23‬ﻴـﺴﻤﻰ‬


‫‪ thonormality covelition‬ﺸﺭﻁ ﺍﻟﺘﻁﺒﻴﻊ ﻭﺍﻟﺘﻌﺎﻤﺩ‪.‬‬
‫ﺍﺴﺘﻨﺒﺎﻁ ﺒﻌﺽ ﺍﻟﻤﻌﻠﻭﻤﺎﺕ ﺍﻟﻔﻴﺯﻴﺎﺌﻴﺔ ﻤﻥ ﺍﻟﺤﻠﻭل ﺍﻟﺨﺎﺼﺔ ‪.eigensalutions‬‬

‫‪ -1‬ﺍﻟﺤﺎﻟﺔ ﺍﻟﺘﻲ ﻟﻬﺎ ﺃﻗل ﻁﺎﻗﺔ‪ ،‬ﺍﻟﺤﺎﻟﺔ ﺍﻟﺩﻨﻴﺎ ‪ ،grourdstate‬ﺘﻭﺼـﻑ ﺒــ )‪u1 (x‬‬
‫ﻭﺘﻘﺎﺒﻠﻬﺎ ﺍﻟﻘﻴﻤﺔ ﺍﻟﺨﺎﺼﺔ‪.‬‬
‫‪D 2h 2‬‬
‫= ‪E1‬‬
‫‪2ma 2‬‬

‫ﻻﺤﻅ ﺃﻨﻪ ﻤﻥ ﺍﻟﻤﻴﻜﺎﻨﻴﻜﺎ ﺍﻟﺘﻘﻠﻴﺩﻴﺔ ﺃﻥ ﺃﻗل ﻁﺎﻗﺔ ﻟﺠﺴﻡ ﺴﺎﻜﻥ ‪ atesr‬ﻓﻲ ﻓﺠﻭﺓ‪،‬‬
‫ﻓﺈﻥ ﻤﺠﻤﻭﻉ ﻁﺎﻗﺔ ﺍﻟﻭﻀﻊ ﻭﻁﺎﻗﺔ ﺍﻟﺤﺭﻜﺔ = ‪ 0‬ﻭﻟﻜﻥ ﻨﺠﺩ ﻓﻲ‬ ‫‪V(x)=0 , p = 0‬‬ ‫ﻟﻪ‬
‫ﻤﻴﻜﺎﻨﻴﻜﺎ ﺍﻟﻜﻡ ﺃﻥ ﺃﻗل ﻁﺎﻗﺔ ﻗﻴﻤﺘﻬﺎ ﻟﻴﺴﺕ ﺼﻔﺭ ﺃﻱ ﺘﻭﺠﺩ ﻗﻴﻤﺔ "ﻟﻠﻁﺎﻗﺔ ﺍﻟﺩﻨﻴﺎ" ﻫـﺫﺍ‬
‫ﻟﻤﺎﺫﺍ‪.‬‬ ‫‪Δn Δp ≥ D‬‬ ‫ﻤﺭﺘﺒﻁ ﺒﻤﺒﺩﺃ ﺍﻟﻼﺘﺤﺩﻴﺩ‬
‫ﻭﺫﻟﻙ ﻷﻥ ﻷﻱ ﺩﺍﻟﺔ ﺤﻘﻴﻘﻴـﺔ )‪ R(x‬ﻗﻴﻤـﺔ‬ ‫‪p =0‬‬ ‫‪ -2‬ﺒﻤﺎ ﺃﻥ ﺍﻟﺤﻠﻭل ﺤﻘﻴﻘﻴﺔ ﻓﺈﻥ‬
‫ﺇﻻ ﻓﻲ ﺤﺎﻟـﺔ‬ ‫‪p‬‬ ‫‪= p‬‬
‫‪x‬‬
‫∫ ﻭﻫﺫﺍ ﻻ ﻴﺘﻔﻕ ﻤﻊ ﺍﻟﻤﻁﻠﻭﺏ‬ ‫)‪dx R(x‬‬
‫‪h d‬‬
‫⇒ )‪R (x‬‬ ‫ﺘﺨﻴﻠﻴﺔ‬
‫‪l dx‬‬

‫ﻓﺈﻥ ‪p 2 ≠ 0‬‬ ‫ﻤﻥ ﻨﺎﺤﻴﺔ ﺃﺨﺭﻯ‬ ‫ﺃﻥ ‪p = 0‬‬

‫‪p 2 = 2mE‬‬ ‫ﻭﺒﻤﺎ ﺃﻨﻪ ﻓﻲ ﺩﺍﺨل ﺍﻟﺼﻨﺩﻭﻕ‬

‫‪p2‬‬ ‫‪= 2mE n‬‬

‫‪- ٨٦ -‬‬
‫‪PDF created with pdfFactory Pro trial version www.pdffactory.com‬‬
‫‪ -3‬ﻭﺒﺎﺯﺩﻴﺎﺩ ﻋﺩﺩ ﺍﻟﻌﻘﺩ ﻓﻲ ﺍﻟﺤل‪ ،‬ﻓﺈﻥ ﻫﺫﺍ ﻴﻘﺎﺒل ﻁﺎﻗﺔ ﺃﻋﻠﻰ ﻜﻤﺎ ﻫﻭ ﻤﻭﻀﺢ ﻓﻲ‬
‫ﺍﻟﺸﻜل ﺍﻟﺘﺎﻟﻲ ﻫﺫﺍ ﻴﻤﻜﻥ ﻓﻬﻤﻪ ﻭﺫﻟﻙ ﻷﻥ ﻁﺎﻗﺔ ﺍﻟﺤﺭﻜﺔ ﺘﺯﺩﺍﺩ ﺒـ ‪urith caruature‬‬
‫‪ of the solntions‬ﺍﻟﻘﻴﻤﺔ ﺍﻟﻤﺘﻭﻗﻌﺔ ﻟﻁﺎﻗﺔ ﺍﻟﺤﺭﻜﺔ‪.‬‬
‫‪h2‬‬ ‫)‪d 2 u (x‬‬
‫‪u‬‬ ‫=‬
‫‪2m‬‬ ‫∫‬ ‫)‪dx u * (x‬‬
‫‪d x2‬‬

‫ﻓﺭﻀﻴﺔ ﺍﻟﻔﻙ‪:‬‬
‫‪The Expansion Postulate and its physical Interpretation‬‬
‫ﺍﻟﺘﻲ ﺘﺤﻘﻕ ﺍﻟـﺸﺭﻁ ﺍﻟﺤـﺩﻱ‬ ‫)‪ψ(x‬‬ ‫ﻨﻅﺭﻴﺔ ﻓﻭﺭﻴﺭ ﺘﻨﺹ ﻋﻠﻰ ﺃﻥ ﺃﻱ ﺩﺍﻟﺔ‬
‫ﻴﻤﻜﻥ ﻜﺘﺎﺒﺘﻬﺎ‪.‬‬ ‫‪ψ (0 ) and ψ ( a ) = 0‬‬

‫‪nD x‬‬
‫‪ψ ( x ) = ∑ Cu sin‬‬
‫‪a‬‬

‫‪sin‬‬
‫‪nDx‬‬
‫ﺘﺘﻨﺎﺴـﺏ ﻤـﻊ‬ ‫∞‬ ‫∞‬
‫ﻭﺒﻤﺎ ﺃﻥ ﺍﻟﺩﻭﺍل ﺍﻟﺨﺎﺼﺔ )ﺍﻟﻤﻤﻴﺯﺓ( ﻟــ ‪H‬‬
‫‪a‬‬

‫ﻓﺒﺈﻤﻜﺎﻨﻨﺎ ﻜﺘﺎﺒﺔ ﺍﻟﺩﻭﺍل ﺍﻟﺨﺎﺼﺔ ﻟـ )‪ Un (x‬ﻜﺎﻟﺘﺎﻟﻲ‪:‬‬

‫)‪ψ (x) = ∑ A n U n (x‬‬

‫ﻭﻤﻥ ﺸﺭﻁ ﺍﻟﺘﻌﺎﻤﺩ ﻭﺍﻟﻤﻌﺎﻴﺭﺓ ﻓﺈﻨﻪ ﺒﺈﻤﻜﺎﻨﻨﺎ ﺇﻴﺠﺎﺩ ‪An‬؟‬

‫‪- ٨٧ -‬‬
‫‪PDF created with pdfFactory Pro trial version www.pdffactory.com‬‬
a a

∫ dx u (x) ψ (x) =
*
m ∫ dx u *m (x) ∑ A n U n (x)
0 0
a
= ∑ An ∫ dx u *m (x) = ∑ A n Smn
0
a
An = ∫
0
dx u *m (x) ψ (x)

‫ ﺩﺍﻟﺘﻪ ﺍﻟﻤﻭﺠﻴﺔ ﺘﻌﻁﻲ ﺒـ‬.‫ ﺍﻋﺘﺒﺭ ﺠﺴﻴﻡ ﻓﻲ ﺼﻨﺩﻭﻕ‬/‫ﻤﺜﺎل‬


ψ (x) = A ( x / a ) 0 < x < a/2
x
= A (1 − ) a <x < a
a 2

‫ﻟﻜﻲ ﺘﺤﻘﻕ‬ ‫ﺤﻴﺙ‬


a
12
∫ =1 A=
2
dx ψ (x)
0
a

‫ ؟‬En ‫ﺍﺤﺴﺏ ﺍﺤﺘﻤﺎل ﺃﻥ ﻗﻴﺎﺱ ﺍﻟﻁﺎﻗﺔ ﻴﻌﻁﻴﻨﺎ ﺍﻟﺩﺍﻟﺔ ﺍﻟﺨﺎﺼﺔ‬

.‫ ﻓﻲ ﺍﻟﻔﻙ‬An ‫ﻨﺤﻥ ﻨﺭﻴﺩ ﺤﺴﺎﺏ‬


a
2 nD x
An = ∫
0
dx ψ (x)
α
sin
a
a
2 nDx
An = ∫
0
dx ψ (x)
α
sin
a
12
= QA =
a
a/2 x 2 nDx 0  x nDx
An = ∫ 0
dx A
a a
sin
a
+ ∫ a/2
dx ( A  1 −  ) sin
 a a
24  a/2 x nDx 0 x nDx 2 nDx 
=  ∫0 dx ( ) sin + ∫ dx (1 − ) sin sin 
a  a a a/2 a a α a 
24 D/a
An =
a ∫ 0

24 D/a u
An =
a ∫ 0
an
D
sin nu (1 − (−1) n )

،‫ ﻭﺫﻟﻙ ﻟﻭﺠﻭﺩ ﺍﻟﺤﺩ ﺍﻟﺜﺎﻨﻲ ﺍﻟﺘﻜﺎﻤل ﻴﻤﻜﻥ ﺤﺴﺎﺒﻪ ﺒـﺴﻬﻭﻟﺔ‬0 ← An ‫ ← ﻗﻴﻤﺔ‬N


.‫ﻓﻘﻁ ﻟﻘﻴﻡ ﻓﺭﺩﻴﺔ‬

- ٨٨ -
PDF created with pdfFactory Pro trial version www.pdffactory.com
‫‪24‬‬ ‫‪1‬‬
‫= ‪An‬‬ ‫‪2 2 ( − 1) n +1‬‬
‫‪D‬‬ ‫‪πn‬‬
‫‪a6‬‬
‫‪An = 4 4‬‬ ‫‪n odd‬‬
‫‪D n‬‬
‫‪=0‬‬ ‫‪n euen‬‬

‫ﻤﺜﺎل‪ /‬ﻓﻲ ﺤﺎﻟﺔ ﻭﺠﻭﺩ ﺠﺴﻴﻡ ﺩﺍﺨل ﺼﻨﺩﻭﻕ )‪ 3‬ﺃﺒﻌﺎﺩ( ﻤﻥ ﺨـﻼل ﺍﻟﺤـل ﺍﻟـﺫﻱ‬
‫ﺘﻭﺼﻠﻨﺎ ﺇﻟﻴﻪ ﻓﻲ ﺤﺎﻟﺔ ﺒﻌﺩ ﻭﺍﺤﺩ‪.‬‬

‫‪nD‬‬ ‫‪nD‬‬
‫= ‪u a = nD ⇒ u‬‬ ‫‪⇒ p = hu = h‬‬
‫‪a‬‬ ‫‪a‬‬
‫‪Dhn x‬‬ ‫‪Dhn y‬‬ ‫‪Dhn z‬‬
‫= ‪⇒ px‬‬ ‫= ‪, py‬‬ ‫‪,‬‬ ‫= ‪pz‬‬
‫‪a‬‬ ‫‪b‬‬ ‫‪c‬‬
‫‪2‬‬ ‫‪2‬‬
‫‪1 2‬‬ ‫‪1‬‬ ‫‪D 2h 2 n x n y‬‬ ‫‪n 2z‬‬
‫=‪E‬‬ ‫= ‪p‬‬ ‫= ) ‪( p + py + pz‬‬
‫‪2‬‬ ‫‪2‬‬ ‫‪2‬‬
‫(‬ ‫‪+‬‬ ‫)‪+ 2‬‬
‫‪2m‬‬ ‫‪2m x‬‬ ‫‪2m a 2 b 2‬‬ ‫‪c‬‬

‫ﻫﺫﻩ ﺍﻟﻤﻌﺎﺩﻟﺔ ﺘﻌﻁﻲ ﻤﺴﺘﻭﻴﺎﺕ ﺍﻟﻁﺎﻗﺔ ﻟﺠﺴﻴﻡ ﻓﻲ ﺼﻨﺩﻭﻕ ﺠﻬﺩ ﻓﻲ ﺜﻼﺜﺔ ﺃﺒﻌﺎﺩ‪.‬‬
‫‪n x hx‬‬ ‫‪n x hy‬‬ ‫‪n x hz‬‬
‫‪ψ = C sin‬‬ ‫‪sin‬‬ ‫‪sin‬‬
‫‪a‬‬ ‫‪b‬‬ ‫‪c‬‬

‫ﻟﻭ ﻋﻭﻀﻨﺎ ﺒﻬﺫﻩ ﺍﻟﻤﻌﺎﺩﻟﺔ ﻓﻲ ﻤﻌﺎﺩﻟﺔ ﺴﺘﺭﻭﺩﻨﺠﺭ ﻓﻨﺘﺤـﺼل ﻋﻠـﻰ ﻗـﻴﻡ ‪E‬‬
‫ﺃﻋﻼﻩ‪.‬‬
‫ﻓﻲ ﺤﺎﻟﺔ ﻤﻜﺴﺏ ) ‪.(a = b = c‬‬
‫‪D 2h 2‬‬ ‫‪D 2h 2 2‬‬
‫=‪E‬‬ ‫= ) ‪( nx + ny + nz‬‬
‫‪2‬‬ ‫‪2‬‬ ‫‪2‬‬
‫‪k‬‬
‫‪2m‬‬ ‫‪2m 2‬‬

‫ﻭﺍﻟﺩﺍﻟﺔ ﺍﻟﻤﻭﺠﺒﺔ ﺍﻟﺘﻲ ﺘﻘﺎﺒﻠﻬﺎ‪.‬‬


‫‪n x hx‬‬ ‫‪n x hy‬‬ ‫‪n x hz‬‬
‫‪ψ = C sin‬‬ ‫‪sin‬‬ ‫‪sin‬‬
‫‪a‬‬ ‫‪b‬‬ ‫‪c‬‬

‫‪- ٨٩ -‬‬
‫‪PDF created with pdfFactory Pro trial version www.pdffactory.com‬‬
‫ﻻﺤﻅ ﺃﻥ ﺍﻟﻁﺎﻗﺔ ﺘﻌﺘﻤﺩ ﻋﻠﻰ ‪ .n2‬ﻫﺫﺍ ﻴﻌﻨﻲ ﺃﻥ ﻜل ﺍﻟﺤﺎﻻﺕ ﺍﻟﺘﻲ ﺘﻨﺎﻅﺭ ﻜل‬
‫‪nx , n y , n z‬‬ ‫ﻭﺍﻟﺘﻲ ﺘﻌﻁﻲ ﻨﻔﺱ ﺍﻟﻁﺎﻗﺔ‪ .‬ﻭﻟﻜﻥ ﻋﻨﺩﻤﺎ ﻨﻐﻴﺭ ﺍﻷﺭﻗـﺎﻡ‬ ‫‪D,ny ,nz‬‬ ‫ﺍﻷﺭﻗﺎﻡ‬
‫ﺒﺩﻭﻥ ﻤﺎ ﻨﻐﻴﺭ ﻗﻴﻤﺔ ‪ n‬ﻓﺈﻥ‪ Ψ‬ﺘﺘﻐﻴﺭ ﻭﻋﻠﻴﻪ ﻓﺈﻨﻪ ﻴﻭﺠﺩ ﻤﺴﺘﻭﻴﺎﺕ ﻤﻥ ﺍﻟﻁﺎﻗـﺔ ﻟﻬـﺎ‬
‫ﺍﻟﻌﺩﻴﺩ ﻤﻥ ﺍﻟﺩﻭﺍل ﺍﻟﻤﻭﺠﻴﺔ ﻭﻫﺫﺍ ﻤﺎ ﻴﺴﻤﻰ ﺒﺎﻟﺘﻜﺭﺍﺭ ‪.degeneracy‬‬

‫ﺍﻟﻁﺎﻗﺔ‬ ‫‪nx , n y , n z‬‬ ‫‪g‬‬


‫‪3 E1‬‬ ‫) ‪(1,1,1,‬‬ ‫‪1‬‬
‫‪6 E1‬‬ ‫) ‪(2 , 1, 1 ) (1, 2 ,1) (1,1, 2‬‬ ‫‪2‬‬
‫‪9 E1‬‬ ‫) ‪(2 , 2 ,1 ) (2 , 1, 2 ) ( 2 , 2 , 2‬‬ ‫‪3‬‬
‫‪11 E 1‬‬ ‫( ) ‪(3 ,1, 1‬‬

‫ﻓﺭﻀﻴﺔ ﺍﻟﻔﻙ‪:‬‬
‫‪Expansion Postulate and Its physical Interpretation‬‬
‫ﺍﻟﺘﻲ ﺘﺤﻘﻕ ﺍﻟﺸﺭﻭﻁ ﺍﻟﺤﺩﻴﺔ‬ ‫)‪ψ(x‬‬ ‫ﺇﻥ ﻨﻅﺭﻴﺔ ﻓﻭﺭﻴﺭ ﺘﻨﺹ ﻋﻠﻰ ﺃﻥ ﺃﻱ ﺩﺍﻟﺔ‬
‫ﻴﻤﻜﻥ ﻜﺘﺎﺒﺘﻬﺎ ﺒﺎﻟﺸﻜل ﺍﻟﺘﺎﻟﻲ‪.‬‬ ‫‪ψ (0 ) = ψ ( a ) = 0 ⇐ bowdary canditions‬‬

‫‪nD x‬‬
‫‪ψ ( x ) = ∑ Cu sin‬‬
‫‪a‬‬

‫ﻭﺒﻤﺎ ﺃﻥ ﺍﻟﺩﻭﺍل ﺍﻟﺨﺎﺼﺔ ﻟـ ‪ H‬ﻟﻠﺒﺌﺭ ﺍﻟﻼﻨﻬﺎﺌﻲ ‪ infinite well‬ﺘﺘﻨﺎﺴﺏ ﻤـﻊ‬


‫ﻓﺒﺈﻤﻜﺎﻨﻨﺎ ﻜﺘﺎﺒﺔ ﺍﻟﻤﻌﺎﺩﻟﺔ ﺍﻟﺴﺎﺒﻘﺔ ﺒﺩﻻﻟﺔ ﺍﻟﺩﻭﺍل ﺍﻟﺨﺎﺼﺔ ﻟـ )‪Un (x‬‬ ‫‪sin‬‬
‫‪nDx‬‬
‫‪a‬‬

‫)‪ψ (x) = ∑ A n U n (x‬‬

‫ﻭﺒﺎﺴﺘﺨﺩﺍﻡ ﻋﻼﻗﺔ ﺍﻟﻤﻌﺎﻴﺭﺓ‬


‫‪a‬‬

‫∫‬‫‪0‬‬
‫‪dx u *m (x) u n (x) = smn ↔ orthonormelnty‬‬

‫ﻴﻤﻜﻨﻨﺎ ﺇﻴﺠﺎﺩ ﻗﻴﻤﺔ ‪.An‬‬

‫‪- ٩٠ -‬‬
‫‪PDF created with pdfFactory Pro trial version www.pdffactory.com‬‬
‫‪a‬‬ ‫‪a‬‬

‫∫‬ ‫= )‪dx u (x) ψ (x‬‬


‫*‬
‫‪m‬‬ ‫∫‬ ‫)‪dx u *m (x‬‬ ‫∑‬ ‫)‪A n U n (x‬‬
‫‪0‬‬ ‫‪0‬‬
‫‪a‬‬
‫=‬ ‫∑‬ ‫‪An‬‬ ‫∫‬ ‫= )‪dx u *m u n (x‬‬ ‫∑‬ ‫‪A n Smn‬‬
‫‪1‬‬
‫‪0‬‬
‫‪442443‬‬
‫‪Smn‬‬
‫‪a‬‬
‫= ‪An‬‬ ‫∫‬
‫‪0‬‬
‫)‪dx u *m (x) ψ (x‬‬

‫‪a‬‬
‫= ‪⇒ An‬‬ ‫∫‬‫‪0‬‬
‫)‪dx u *n (x) ψ (x‬‬ ‫‪,‬‬ ‫‪sin = 1, if m = n‬‬

‫ﻭﻜﻤﺎ ﻓﻌﻠﻨﺎ ﻓﻲ ﺤﺎﻟﺔ ﺍﻨﺘﺸﺎﺭ ﺍﻟﺤﺯﻤﺔ ﺍﻟﻤﻭﺠﻴﺔ ﺍﻟﺤﺭﺓ‪ ،‬ﺒﺈﻤﻜﺎﻨﻨﺎ ﺤﺴﺎﺏ ﺍﻟﺘﻁﻭﺭ‬
‫ﻤﻊ ﺍﻟﺯﻤﻥ‪ .‬ﻭﺒﻤﺎ ﺃﻥ ﻜل ﻤﻥ ﺍﻟﺩﻭﺍل ﺍﻟﺨﺎﺼﺔ‬ ‫)‪ψ (x‬‬ ‫ﺍﻟﺯﻤﻨﻲ ﻟﻬﺫﻩ ﺍﻟﺩﺍﻟﺔ ﺍﻻﺨﺘﻴﺎﺭﻴﺔ‬
‫‪ ، ( e‬ﺒﺸﻜل ﻋﺎﻡ ﻴﻤﻜﻨﻨـﺎ‬ ‫‪− iEnt/h‬‬
‫)‬ ‫ﺘﺘﻁﻠﺏ ﺩﺍﻟﺔ ﺍﻋﺘﻤﺎﺩ ﻋﻠﻰ ﺍﻟﺯﻤﻥ ﺨﺎﺼﺔ ﺒﻬﺎ‬ ‫)‪U n (x‬‬

‫ﻜﺘﺎﺒﺔ‪.‬‬
‫= ) ‪ψ (x, t‬‬ ‫∑‬ ‫‪A n U n (x) e − iEnt/h‬‬

‫ﻭﻟﺘﺄﻭﻴل ﺍﻟﻤﻌﺎﻤﻼﺕ ‪ An‬ﻓﺒﺈﻤﻜﺎﻨﻨﺎ ﺤﺴﺎﺏ ﺍﻟﻘﻴﻤﺔ ﺍﻟﻤﺘﻭﻗﻌﺔ ﻟﻠﻁﺎﻗﺔ ﻓﻲ ﺃﻱ ﺤﺎﻟﺔ‬


‫‪ψ (x) = 0‬‬ ‫‪ ، H = p‬ﻭﺨﺎﺭﺝ ﺍﻟـﺼﻨﺩﻭﻕ‬ ‫‪2‬‬
‫‪/2m‬‬ ‫ﺍﺨﺘﻴﺎﺭﻴﺔ )‪ (x‬ﻭﺒﻤﺎ ﺃﻥ ﺩﺍﺨل ﺍﻟﺼﻨﺩﻭﻕ‬
‫)‪H U n (x) = E n U n (x‬‬ ‫ﻭﺒﻤﺎ ﺃﻥ‬
‫‪a‬‬ ‫‪a‬‬
‫‪H‬‬ ‫=‬ ‫∫‬ ‫= )‪dx ψ (x) Hψ (x‬‬
‫*‬
‫∫‬ ‫‪dx ψ * (x) H‬‬ ‫∑‬ ‫)‪A n U n (x‬‬
‫‪0‬‬ ‫‪0‬‬
‫‪a‬‬
‫=‬ ‫∑‬ ‫‪An‬‬ ‫∫‬ ‫)‪dx ψ * (x) E n U n (x‬‬ ‫)‪(a‬‬
‫‪0‬‬
‫‪a‬‬
‫‪= ∑ En‬‬ ‫∫‬ ‫)‪dx ψ * (x) A n U n (x‬‬
‫‪0‬‬

‫‪= ∑ En An‬‬
‫‪2‬‬

‫‪a‬‬
‫= ‪1‬‬ ‫∫‬ ‫)‪dx ψ * (x‬‬ ‫∑‬ ‫)‪A n U n (x‬‬ ‫)‪(B‬‬
‫‪0‬‬

‫∑‬ ‫∑‬
‫‪2‬‬
‫=‬ ‫= ‪An‬‬ ‫‪An‬‬

‫ﺍﻟﻤﻌﺎﺩﻟﺔ ﺍﻟﺴﺎﺒﻘﺔ )‪ (a‬ﻤﻊ ﺸﺭﻁ ﺍﻟﺘﻁﺒﻴﻊ ﻴﻘﺘﺭﺡ ﺃﻥ‬


‫‪2‬‬
‫‪An‬‬

‫‪- ٩١ -‬‬
‫‪PDF created with pdfFactory Pro trial version www.pdffactory.com‬‬
‫)‪A n = ∫ dx U *n (x) ψ (x‬‬

‫ﻴﻌﻁﻴﻨـﺎ ﺍﻟﻘﻴﻤـﺔ‬ ‫)‪ψ (x‬‬ ‫ﻴﻤﻜﻥ ﺘﺄﻭﻴﻠﻬﺎ ﺒﺎﺤﺘﻤﺎل ﺃﻥ ﻗﻴﺎﺱ ﺍﻟﻁﺎﻗﺔ ﻷﻱ ﺩﺍﻟـﺔ‬
‫ﺍﻟﺨﺎﺼﺔ ‪.En‬‬
‫ﻤﺜﺎل‪ :‬ﺨﺫ ﺠﺴﻡ ﻓﻲ ﺼﻨﺩﻭﻕ‪ .‬ﺩﺍﻟﺔ ﺍﻟﻤﻭﺠﺔ‪.‬‬
‫)‪ψ (x) = A (x/a‬‬ ‫‪0 < x < a/2‬‬
‫‪x‬‬
‫) ‪= A (1 −‬‬ ‫‪a/2 < x < a‬‬
‫‪a‬‬

‫‪ ∫ dx‬ﺍﺤﺴﺏ ﺍﺤﺘﻤـﺎل ﺃﻥ ﺍﻟﻘﻴـﺎﺱ‬ ‫ﻭﺫﻟﻙ ﻟﺘﺤﻘﻕ‬ ‫ﺤﻴﺙ‬


‫‪a‬‬
‫‪12‬‬
‫‪ψ (x) = 1‬‬ ‫=‪A‬‬
‫‪2‬‬

‫‪0‬‬
‫‪a‬‬

‫ﻟﻠﻁﺎﻗﺔ ﻴﻌﻁﻴﻨﺎ ﺍﻟﻘﻴﻤﺔ ﺍﻟﺨﺎﺼﺔ ‪ .En‬ﻨﺤﻥ ﻨﺭﻴﺩ ﺃﻥ ﻨﺤﺴﺏ ‪ An‬ﻓﻲ ﺍﻟﻔﻙ‪.‬‬


‫‪a‬‬
‫)‪ψ (x‬‬ ‫‪2‬‬ ‫‪nDx‬‬
‫= ‪An‬‬ ‫∫‬
‫‪0‬‬
‫‪dx‬‬
‫↑‬ ‫‪α‬‬
‫‪sin‬‬
‫‪a‬‬
‫‪12 x‬‬
‫‪a a‬‬
‫‪24  a/2‬‬ ‫‪x‬‬ ‫‪nDx‬‬ ‫‪0‬‬ ‫‪x‬‬ ‫‪nDx ‬‬
‫‪a ‬‬‫‪‬‬ ‫∫‬‫‪0‬‬
‫‪dx ( ) sin‬‬
‫‪a‬‬ ‫‪a‬‬
‫‪+‬‬ ‫∫‬ ‫‪a/2‬‬
‫‪dx (1 −‬‬
‫‪a‬‬
‫‪) sin‬‬
‫‪a ‬‬

‫ﻟﻠﺘﻜﺎﻤل ﺍﻷﻭل‪.‬‬ ‫=‪u‬‬


‫‪Dx‬‬ ‫‪a‬‬
‫‪, dx = du‬‬ ‫ﺒﺘﻐﻴﻴﺭ ﺍﻟﻤﺘﻐﻴﺭﺍﺕ‬
‫‪a‬‬ ‫‪D‬‬

‫ﻟﻠﺘﻜﺎﻤل ﺍﻟﺜﺎﻨﻲ‬ ‫=‪D-u‬‬


‫‪Dx‬‬
‫‪a‬‬
‫‪24‬‬ ‫‪a‬‬ ‫‪u‬‬
‫= ‪An‬‬
‫‪a‬‬ ‫∫ ‪∫ D an D sin (nu) +‬‬
‫‪D/2‬‬
‫‪24‬‬ ‫‪u‬‬
‫=‬
‫‪D‬‬ ‫∫‬
‫‪0‬‬
‫‪dn‬‬
‫‪D‬‬
‫) ‪sin nu (1 − (−1) n‬‬

‫‪ An‬ﻟﻘﻴﻡ ‪ n‬ﻤﺯﺩﻭﺠﺔ ← ‪ 0‬ﻷﻥ ﺍﻟﺤﺩ ﺍﻷﺨﻴﺭ ﻭﻟﻘﻴﻤﺔ ‪ n‬ﺍﻟﻔﺭﺩﻴﺔ‪.‬‬


‫‪24‬‬ ‫‪1‬‬
‫= ‪An‬‬ ‫‪2.‬‬ ‫‪( − 1 ) n +1‬‬
‫‪a‬‬ ‫‪πn 2‬‬
‫‪96‬‬
‫ﻭﻋﻠﻴﻪ‪.‬‬
‫‪2‬‬
‫⇒‬ ‫‪An‬‬ ‫=‬ ‫‪n odd‬‬
‫‪D 4n 4‬‬
‫‪=0‬‬ ‫‪neven‬‬

‫‪- ٩٢ -‬‬
‫‪PDF created with pdfFactory Pro trial version www.pdffactory.com‬‬
‫‪Momentum Eiglnhetion of the Fremantle‬‬
‫ﺍﻟﺘﻤﺎﺜل ‪:Parity‬‬

‫ﺍﻟﺩﻭﺍل ﺍﻟﺨﺎﺼﺔ ﻟﺠﺴﻡ ﺤﺭ )‪ (sin kx, cos kx‬ﻭﻜﺫﻟﻙ ﺍﻟﺩﻭﺍل ﺍﻟﺨﺎﺼﺔ ﻟﺠﺴﻡ‬
‫‪−a /2 →a /2‬‬ ‫ﻓﻲ ﺼﻨﺩﻭﻕ ﻤﻤﺘﺩ ﻤﻥ‬
‫‪2‬‬ ‫‪nDx‬‬
‫= )‪U n (x‬‬ ‫‪sin‬‬ ‫)‪(n = 2, 4, 6, ..............‬‬
‫‪a‬‬ ‫‪a‬‬
‫‪2‬‬ ‫‪nDx‬‬
‫= )‪U n (x‬‬ ‫‪cos‬‬ ‫)‪(n = 1, 3, 5, ..............‬‬
‫‪a‬‬ ‫‪a‬‬

‫ﻟﻭ ﺃﺨـﺫﻨﺎ‬ ‫‪x →−x‬‬ ‫ﻫﺫﻩ ﺍﻟﺩﻭﺍل ﺇﻤﺎ ﺃﻥ ﺘﻜﻭﻥ ﺯﻭﺠﻴﺔ ﺃﻭ ﻓﺭﺩﻴﺔ ﺘﺤﺕ ﺍﻟﺘﻐﻴﻴﺭ‬
‫ﺩﻭﺍل‬ ‫)‪ψ (x‬‬ ‫ﻭﻜﺎﻨﺕ ﺍﻟﻘﻴﻡ ﺍﻻﺒﺘﺩﺍﺌﻴـﺔ ﻟﻠﺤﺎﻟـﺔ‬ ‫‪x= 0‬‬ ‫ﺠﺴﻡ ﻓﻲ ﺼﻨﺩﻭﻕ ﻤﺘﻤﺎﺜل ﺤﻭل‬
‫ﺯﻭﺠﻴﺔ‪.‬‬

‫ﺤﻴﺙ ﺃﻥ ﺍﻟﺠﻤﻊ‬ ‫= )‪ψ (x‬‬ ‫∑‬ ‫‪An‬‬


‫‪2‬‬
‫‪cos‬‬
‫‪nDx‬‬
‫ﻴﺠﺏ ﺃﻥ ﺘﻜﻭﻥ‬ ‫)‪ψ (x‬‬ ‫ﻫﺫﺍ ﻴﻌﻨﻲ ﺃﻥ‬
‫‪a‬‬ ‫‪a‬‬

‫ﻋﻨﺩ ﻭﻗﺕ ﺁﺨﺭ ﺘﺼﺒﺢ ﺍﻟﺩﺍﻟﺔ‬ ‫)‪(n =1, 3, 5, ..............‬‬ ‫ﻴﻤﺘﺩ ﻋﻠﻰ‬
‫‪2‬‬ ‫‪nDx − iEnt/h‬‬
‫‪ψ (x , t) = ∑ An‬‬ ‫‪cos‬‬ ‫‪e‬‬
‫‪a‬‬ ‫‪a‬‬

‫ﻭﻫﺫﻩ ﺍﻟﺩﺍﻟﺔ ﻻ ﺯﺍﻟﺕ ﺯﻭﺠﻴﺔ ﻓﻲ ‪ x‬ﻋﻨﺩ ﺃﻭﻗﺎﺕ ﺃﺨﺭﻯ‪ .‬ﻭﻨﻔﺱ ﺍﻟﺸﻲﺀ ﻴﺤﺩﺙ‬
‫ﻓﻲ ﺤﺎﻟﺔ ﺍﻟﻘﻴﻡ ﺍﻷﻭﻟﻰ ﺍﻻﺒﺘﺩﺍﺌﻴﺔ ﺘﻜﻭﻥ ﻓﺭﺩﻴﺔ‪.‬‬

‫ﻭﻋﻠﻴﻪ‪ :‬ﻟﻠﺼﻨﺩﻭﻕ ﺍﻟﻤﺘﻤﺎﺜل ﻭﻤﺘﻤﺤﻭﺭ ﺤﻭل ‪ x = 0‬ﻓﺈﻥ ﺍﻟﻔﺭﺩﻴﺔ ﻭﺍﻟﺯﻭﺠﻴـﺔ‬


‫ﻫﻲ ﺨﻭﺍﺹ ﻻ ﺘﻌﺘﻤﺩ ﻋﻠﻰ ﺍﻟﺯﻤﻥ‪ .‬ﻭﻋﻠﻴﻪ ﺒﺄﻨﻬﺎ )ﺍﻟﺯﻭﺠﻴـﺔ ﻭﺍﻟﻔﺭﺩﻴـﺔ( ﺜﻭﺍﺒـﺕ‬
‫ﺍﻟﺴﺭﻋﺔ‪.‬‬

‫ﻓﻠﻭ ﺍﺴﺘﺤﺩﺜﻨﺎ ﻤﺅﺜﺭ ‪ Parihyopenator p‬ﻤﺅﺜﺭ ﺍﻟﺘﻤﺎﺜل ﻭﺍﻟﺫﻱ ﻗﺎﻋﺩﺘـﻪ ﺃﻥ‬


‫)‪ψ (x‬‬ ‫ﻭﻫﻜﺫﺍ ﻷﻱ ﺩﺍﻟﺔ ﻤﻭﺠﺒﺔ‬ ‫‪x →−x‬‬ ‫ﻴﻌﻜﺱ‬

‫)‪p ψ (x) = ψ ( - x‬‬

‫‪- ٩٣ -‬‬
‫‪PDF created with pdfFactory Pro trial version www.pdffactory.com‬‬
‫ﻭﻟﺩﺍﻟﺔ ﻤﻭﺠﺒﺔ ﺯﻭﺠﻴﺔ‪.‬‬

‫)‪p ψ ( + ) (x) = ψ ( + ) ( x‬‬

‫ﻭﻟﺩﺍﻟﺔ ﻓﺭﺩﻴﺔ‪.‬‬

‫)‪p ψ ( − ) (x) = − ψ ( − ) ( x‬‬

‫ﻭﻫﺎﺘﻴﻥ ﺍﻟﻤﻌﺎﺩﻟﺘﻴﻥ ﻫﻲ ﻤﻌﺎﺩﻻﺕ ﺫﺍﺕ ﻗﻴﻡ ﺨﺎﺼﺔ ﻭﺍﻟﺫﻱ ﻭﻀﺤﻨﺎﻩ ﻫـﺫﺍ ﺃﻥ‬
‫ﺍﻟﺩﻭﺍل ﺍﻟﺯﻭﺠﻴﺔ ﻫﻲ ﺩﻭﺍل ﺨﺎﺼﺔ ﻟـ ‪ p‬ﻟﻬﺎ ﻗﻴﻤﺔ ﺨﺎﺼﺔ ‪ + 1‬ﺒﻴﻨﻤﺎ ﺍﻟﺩﻭﺍل ﺍﻟﻔﺭﺩﻴﺔ‬
‫ﻫﻲ ﺩﻭﺍل ﺨﺎﺼﺔ ﻟـ ‪ p‬ﻟﻬﺎ ﻗﻴﻤﺔ ﺨﺎﺼﺔ ‪-1‬‬
‫ﻟﻴﺴﺕ ﻓﻘﻁ‬ ‫( ‪cos‬‬
‫‪nDx‬‬
‫( ‪) , sin‬‬
‫‪nDx‬‬
‫)‬ ‫ﻭﻓﻲ ﻤﺴﺄﻟﺔ ﺍﻟﺠﺴﻡ ﻓﻲ ﺼﻨﺩﻭﻕ ﻓﺈﻥ ﺍﻟﺩﻭﺍل‬
‫‪a‬‬ ‫‪a‬‬
‫ﻫـﻲ ﺍﻟﻘـﻴﻡ‬ ‫‪±1‬‬ ‫ﺩﻭﺍل ﺨﺎﺼﺔ ﻟـ ‪ H‬ﺒل ﻫﻲ ﺩﻭﺍل ﺨﺎﺼﺔ ﻟـ ‪ P‬ﺍﻟﻘﻴﻡ ﺍﻟﺨﺎﺼـﺔ‬
‫ﺍﻟﻤﻤﻜﻨﺔ ﻓﻘﻁ‪.‬‬
‫ﺒﺎﻟﺘﺄﺜﻴﺭ ﺒـ ‪ P‬ﻤﺭﺓ ﺃﺨﺭﻯ‪.‬‬ ‫)‪PU (x) = D u (x‬‬ ‫ﺍﻓﺘﺭﺍﺽ ﺃﻨﻨﺎ ﻋﻨﺩﻨﺎ‪.‬‬
‫)‪P 2 U (x) = P D u (x) = D 2 u ( x‬‬

‫ﻭﺫﻟﻙ ﻷﻥ ﻻﻨﻌﻜﺎﺴﻴﺔ ﺍﻻﺜﻨﻴﻥ )ﻤﺭﺘﻴﻥ( ﻴﺠﺏ ﺃﻻ ﻴﻐﻴﺭ ﺸﻴﺌﺎﹰ‪.‬‬ ‫)‪P 2 U (x) = U (x‬‬ ‫ﻟﻜﻥ‬
‫ﻭﺃﻱ ﺩﺍﻟﺔ ﺍﺨﺘﻴﺎﺭﻴﺔ ﻴﻤﻜﻨﻨﺎ ﻜﺘﺎﺒﺘﻬﺎ ﻜﻤﺠﻤﻭﻉ ﻟـﺩﻭﺍل ﺯﻭﺠﻴـﺔ‬ ‫‪⇒ λ 2 =1 ⇒ λ = ± 1‬‬

‫ﻭﻓﺭﺩﻴﺔ‪.‬‬
‫= )‪ψ (x‬‬
‫‪1‬‬
‫] )‪[ψ (x) + ψ (- x) ] + 1 [ψ (x) − ψ (- x‬‬
‫‪2‬‬ ‫‪2‬‬
‫ﻭﻫﺫﺍ ﻜﻤﺎ ﻋﻤﻠﻨﺎ ﺒﺎﻟﺩﻭﺍل ﺍﻟﺨﺎﺼﺔ ﻟـ ‪ ⇐ H‬ﺃﻱ ﺩﺍﻟﺔ ﻴﻤﻜﻥ ﺃﻥ ﺘﻔﻙ ﺒﺩﻻﻟـﺔ‬
‫ﺍﻟﺩﻭﺍل ﻟﻠﻤﺅﺜﺭ ﺍﻟﺠﺩﻴﺩ‪.‬‬
‫ﺇﻥ ﻅﻬﻭﺭ "ﺍﻟﺯﻭﺠﻴﺔ" ﻭ "ﺍﻟﻔﺭﺩﻴﺔ" ﺠﺎﺩ ﻷﻨﻨﺎ ﺭﻜﺯﻨﺎ ﺍﻟﺼﻨﺩﻭﻕ ﺤـﻭل ‪x = 0‬‬
‫ﻭﺇﺫﺍ ﻜﻨﺎ ﻭﻀﻌﻨﺎﻩ ﺒﻴﻥ ‪ a , 0‬ﻻ ﺸﻲﺀ ﺴﻴﺘﻐﻴﺭ ﻭﺴﺘﺒﻘﻰ ﺍﻟﺘﻤﺎﺜل )ﺍﻟﺘﻨـﺎﻅﺭ( ﺘﺤـﺕ‬
‫ﺍﻻﻨﻌﻜﺎﺱ ﻋﻨﺩ ‪ .x = a/2‬ﺇﻥ ﺍﻟﺩﺭﺱ ﺍﻟﻤﻁﻠﻭﺏ ﺘﻌﻠﻤﻪ ﻫﻨﺎ ﻫﻭ ﺃﻨﻨﺎ ﺒﻭﻀﻊ ﻤﺴﺄﻟﺔ ﻓﻲ‬
‫ﻤﻴﻜﺎﻨﻴﻜﺎ ﺍﻟﻜﻡ ﻴﺠﺏ ﺃﻥ ﻨﺭﺍﻋﻲ ﺒﺎﻫﺘﻤﺎﻡ ﺘﻤﺎﺜل ﻭﺘﻨـﺎﻅﺭ ﺍﻟﻬﺎﻤﻠﺘﻭﻨﻴـﺎﻥ‪ ،‬ﻭﺍﺨﺘﻴـﺎﺭ‬

‫‪- ٩٤ -‬‬
‫‪PDF created with pdfFactory Pro trial version www.pdffactory.com‬‬
‫ﺍﻻﺤﺩﺍﺜﻴﺎﺕ ﺒﻁﺭﻴﻘﺔ ﺘﻅﻬﺭ ﺍﻟﺘﻤﺎﺜل ﺒﻭﻀﻭﺡ ﻭﺍﻟﺤﻘﻴﻘﺔ ﺍﻟﻬﺎﻤﺔ ﻫﻭ ﺘﻤﺎﺜـل )ﺘﻨـﺎﻅﺭ(‬
‫ﺍﻟﻬﺎﻤﻠﺘﻭﻨﻴﺎﻥ‪ .‬ﻫﺫﺍ ﺭﺒﻤﺎ ﺃﻥ ﻴﺭﻯ ﺒﻭﻀﻭﺡ ﺒﺎﻟﺴﺅﺍل ﺘﺤﺕ ﺃﻱ ﺤﻴﺜﻴﺎﺕ ﺴﺘﺒﻘﻰ ﺍﻟﺩﺍﻟﺔ‬
‫ﺍﻟﺯﻭﺠﻴﺔ ﺯﻭﺠﻴﺔ ﻋﻨﺩ ﻜل ﺍﻷﺯﻤﻨﺔ‪.‬‬
‫‪One – Dimensional Potential‬‬ ‫ﺍﻟﺠﻬﺩ ﺫﻭ ﺍﻟﺒﻌﺩ ﺍﻟﻭﺍﺤﺩ‬

‫ﻓﻲ ﻫﺫﺍ ﺍﻟﻔﺼل ﺴﻭﻑ ﻨﺤل ﻤﺴﺎﺌل ﺒﺴﻴﻁﺔ ﻟﻠﺤﺭﻜﺔ ﻓﻲ ﺒﻌـﺩ ﻭﺍﺤـﺩ‪ .‬ﻫـﺫﻩ‬
‫ﺍﻟﻤﺴﺎﺌل ﺫﺍﺕ ﺃﻫﻤﻴﺔ ﻭﺫﻟﻙ ﻷﻨﻬـﺎ ﺘﻭﻀـﺢ ﺒﻌـﺽ ﺍﻟﻅـﻭﺍﻫﺭ ﺍﻟﻐﻴـﺭ ﺘﻘﻠﻴﺩﻴـﺔ‬
‫‪ nonclaesical efpeots‬ﻭﻜﺫﻟﻙ ﻷﻥ ﺍﻟﻌﺩﻴﺩ ﻤﻥ ﺍﻟﺤـﺎﻻﺕ ﺍﻟﻔﻴﺯﻴﺎﺌﻴـﺔ ﻫـﻲ ﻓـﻲ‬
‫ﻤﺤﺼﻠﺘﻬﺎ ﻓﻲ ﺒﻌﺩ ﻭﺍﺤﺩ ﺒﺎﻟﺭﻏﻡ ﻤﻥ ﺃﻨﻨﺎ ﻨﻘﻴﺱ ﻓﻲ ﺜﻼﺜﺔ ﺃﺒﻌﺎﺩ‪.‬‬
‫ﻋﺘﺒﺔ ﺍﻟﺠﻬﺩ )ﻤﻌﺎﻤﻠﻲ ﺍﻟﻨﻔﺎﺫ ﻭﺍﻻﻨﻌﻜﺎﺱ( ‪:The Potential step‬‬

‫ﺒﺎﻟﻨﻅﺭ ﺇﻟﻰ ﺸﻜل ﺍﻟﻤﻭﻀﺢ‪.‬‬

‫‪V (x) = 0‬‬ ‫‪x < 0‬‬


‫‪= V° 0‬‬ ‫‪x>0‬‬

‫ﻤﻌﺎﺩﻟﺔ ﺸﺭﻭﺩﻨﺠﺭ‪.‬‬
‫)‪h 2 d 2 u (x‬‬
‫‪−‬‬ ‫)‪+ v (x) u (x) = E u (x‬‬
‫‪2m dx 2‬‬

‫ﻴﻤﻜﻥ ﻜﺘﺎﺒﺘﻬﺎ‪:‬‬
‫)‪d 2 u (x‬‬
‫‪2‬‬
‫‪2m‬‬
‫‪+ 2‬‬ ‫‪[ E - v (x) ] u (x) = 0‬‬
‫‪dx‬‬ ‫‪h‬‬

‫‪- ٩٥ -‬‬
‫‪PDF created with pdfFactory Pro trial version www.pdffactory.com‬‬
‫ﺒﻜﺘﺎﺒﺔ‪:‬‬
‫‪2m‬‬
‫‪h‬‬ ‫‪2‬‬
‫[‬
‫‪E − V0 = q 2‬‬ ‫]‬ ‫‪,‬‬
‫‪2m E‬‬
‫‪h‬‬ ‫‪2‬‬
‫‪=K2‬‬

‫ﻓﺈﻥ ﺍﻟﺤل ﺍﻟﻌﺎﻡ‪.‬‬ ‫‪V (x) = 0 ← x< 0‬‬ ‫ﻋﻨﺩ‬

‫‪u (x) = e ikz + R e − ikz‬‬

‫ﻭﻫﺫﺍ ﻴﻘﺎﺒل ﻓﻴﺽ ﻴﺘﺤﺭﻙ ﻓﻲ ﺍﻻﺘﺠﺎﻩ ﺍﻟﻤﻭﺠﺏ ﻟـ ‪ x‬ﻭﻤﻘﺩﺍﺭ ﺍﻟﻔﻴﺽ ﻫﻭ‪:‬‬


‫‪h‬‬‫* ‪‬‬ ‫‪∂ψ‬‬ ‫‪∂ψ * ‬‬
‫=‪j‬‬ ‫‪‬‬ ‫‪ψ‬‬ ‫)‪(x‬‬ ‫‪−‬‬ ‫= ‪ψ‬‬
‫‪‬‬
‫‪2im‬‬ ‫‪∂x‬‬ ‫‪∂x ‬‬
‫‪‬‬ ‫‪644444‬‬ ‫‪complex coujngate‬‬
‫‪4‬‬‫‪474444444‬‬ ‫‪8 ‬‬
‫‪h  iux‬‬
‫=‬ ‫‪( e + R e ) ( iue‬‬
‫‪* iux‬‬ ‫‪2ikx‬‬
‫‪− Rik e ) − ( ike + ikR * e − iux ) (e iux + R * e − iux ) ‬‬
‫‪− iux‬‬ ‫‪iux‬‬

‫‪2im ‬‬ ‫‪‬‬


‫‪‬‬ ‫‪‬‬
‫‪h‬‬
‫‪2im‬‬
‫[{‬
‫) ‪ik (1 − RR * ) + ik ( R * e 2ikx − Re −ikx‬‬ ‫]‬
‫[‬
‫=} ) ‪− − ik (1 − RR * ) + ik ( R * e 2ikx − Re − ikx‬‬
‫‪h‬‬
‫‪2im‬‬
‫]‬
‫) ‪[ 2 ik ( 1 − R) = hk (1 − R 2‬‬
‫‪m‬‬

‫ﻜﻤﻭﺠﺔ ﺴﺎﻗﻁﺔ‪ .‬ﻭﻟﻭ ﻟـﻡ ﻴﻜـﻥ‬ ‫‪hk‬‬


‫ﺒﻔﻴﺽ ﻗﺩﺭﺓ‬ ‫‪e ikx‬‬ ‫ﺒﺈﻤﻜﺎﻨﻨﺎ ﺃﻥ ﻨﻨﻅﺭ ﻟـ‬
‫‪m‬‬
‫ﻜﺤل ﻟﻜل ﻗﻴﻡ ‪ x‬ﻭﻋﻠﻴﻪ ﻨﻌﺯﻱ ‪ R‬ﻟﻭﺠﻭﺩ ﺍﻟﺠﻬﺩ‬ ‫‪e ikx‬‬ ‫ﻫﻨﺎﻙ ﺠﻬﺩ ﺒﺈﻤﻜﺎﻨﻨﺎ ﺃﻥ ﻨﺨﺘﺎﺭ‬
‫ﻟﻘﻴﻡ‪x > 0‬‬ ‫‪hk‬‬
‫‪R‬‬
‫‪2‬‬
‫ﺒﻔﻴﺽ ﻤﻨﻌﻜﺱ ﻗﺩﺭﻩ‬ ‫‪R e - ikx‬‬ ‫ﻫﺫﺍ ﺍﻟﺠﻬﺩ ﻴﻨﺸﺊ ﺍﻟﻤﻭﺠﺔ ﺍﻟﻤﻨﻌﻜﺴﺔ‬
‫‪m‬‬

‫ﺒﺎﻤﻜﺎﻨﻨﺎ ﺃﻥ ﻨﻜﺘﺏ ﺍﻟﺤل‪.‬‬

‫‪U (x) = T e ijx‬‬

‫ﻭﻟﻜـﻥ ﻓـﻲ‬ ‫‪−x‬‬


‫‪←‬‬ ‫ﻓﻲ ﺍﺘﺠﺎﻩ‬ ‫∞‪+‬‬ ‫ﻭﻟﻜﻥ ﻫﺫﺍ ﺍﻟﺤﺩ ﻴﺼﻑ ﻤﻭﺠﺔ ﻗﺎﺩﻤﺔ ﻤﻥ‬
‫ﺤﺎﻟﺔ ﻤﻭﺠﺔ ﺴﺎﻗﻁﺔ ﻤﻥ ﺍﻟﻴﺴﺎﺭ ﻓﺈﻥ ﺍﻟﻤﻭﺠﺔ ﺍﻟﻭﺤﻴﺩﺓ ﻋﻠﻰ ﺍﻟﻴﻤﻴﻥ ﻴﻤﻜﻥ ﺃﻥ ﺘﻜـﻭﻥ‬
‫ﻤﻭﺠﺔ ﻨﺎﻓﺫﺓ ‪ transmitted were‬ﻭﺍﻟﻔﻴﺽ ﺍﻟﻤﻘﺎﺒل ﻟﻬﺎ‪.‬‬
‫‪j‬‬
‫‪h‬‬
‫‪2im‬‬
‫[‬
‫‪T * e - iux ( Tiq e iqx ) − (−iq T * e −iqx ) T e iqx‬‬ ‫]‬

‫‪- ٩٦ -‬‬
‫‪PDF created with pdfFactory Pro trial version www.pdffactory.com‬‬
‫ﻭﺒﻤﺎ ﺃﻨﻪ ﻻ ﻴﻭﺠﺩ ﺍﻋﺘﻤﺎﺩ ﻋﻠﻰ ﺍﻟﺯﻤﻥ ﻓﻲ ﻫﺫﻩ ﺍﻟﻤﺴﺄﻟﺔ ﻓﺈﻥ ﻗـﺎﻨﻭﻥ ﺍﻟﺤﻔـﻅ‬
‫ﻴﻌﻨﻲ ﺃﻥ )‪ j(x‬ﻻ ﺘﻌﺘﻤﺩ ﻋﻠﻰ ‪ .x‬ﻭﻋﻠﻴﻪ ﻓﺈﻥ ﺍﻟﻔﻴﺽ‬ ‫∂‬ ‫∂‬
‫‪P (x, t) + j (x, t) = 0‬‬ ‫)ﺍﻟﺒﻨﺎﺀ(‬
‫‪∂t‬‬ ‫‪∂t‬‬
‫ﻋﻠﻰ ﺍﻟﺠﺎﻨﺏ ﺍﻷﻴﺴﺭ ﻴﺠﺏ ﺃﻥ ﻴﺴﺎﻭﻱ ﺍﻟﻔﻴﺽ ﻋﻠﻰ ﺍﻟﺠﺎﻨﺏ ﺍﻷﻴﻤﻥ‪.‬‬
‫‪hk‬‬ ‫‪2‬‬ ‫‪hi‬‬ ‫‪2‬‬
‫⇒‬ ‫‪(1 − R‬‬ ‫= )‬ ‫‪T‬‬
‫‪m‬‬ ‫‪m‬‬

‫)‪U (x) = U (x‬‬ ‫ﻭﺇﻥ ﺍﺘﺼﺎﻟﻴﺔ ‪ contiuwity‬ﺍﻟﺩﺍﻟﺔ ﺍﻟﻤﻭﺠﻴﺔ ﻴﺅﺩﻱ‬

‫‪e iux + R e - iux = T e 2ikx ⇒ at x = 0 ⇒ 1 + R = T‬‬ ‫)‪(a‬‬

‫ﻭﻜﺫﻟﻙ ﻓﺈﻥ ﻤﻴل ﺍﻟﺩﺍﻟﺔ ﺍﻟﻤﻭﺠﺒﺔ ﻤﺘﺼل ﻋﻨﺩ ‪ x = 0‬ﺒﺎﻟﺭﻏﻡ ﻤﻥ ﺃﻥ ﺍﻟﺠﻬـﺩ‬


‫∈‪−‬‬ ‫ﻤﻥ‬ ‫(‬
‫‪d 2 u (x) 2m‬‬
‫]‪+ 2 [E − V (x) u (x) = 0‬‬ ‫ﻏﻴﺭ ﻤﺘﺼل ﻭﻫﺫﺍ ﻴﺘﻀﺢ ﺒﺘﻜﺎﻤل ﺍﻟﻤﻌﺎﺩﻟﺔ‬
‫‪dx 2‬‬ ‫‪h‬‬

‫ﺭﻗﻡ ﻤﻭﺠﺏ ﻓﺈﻥ‪.‬‬ ‫∈‬ ‫ﺤﻴﺙ‬ ‫∈‪+‬‬ ‫ﺇﻟﻰ‬


‫∈‬ ‫∈‬
‫‪du‬‬ ‫‪du‬‬ ‫‪d du‬‬ ‫‪2m‬‬
‫‪( )∈ − ( ) − ∈ = ∫ dx‬‬ ‫‪= ∫ dx 2 (V (x) − E ) u (x) = 0‬‬
‫‪dx‬‬ ‫‪dx‬‬ ‫∈‪−‬‬
‫∈‪dx dx −‬‬ ‫‪h‬‬

‫ﻋﻨﺩﺌـﺫ ﻓـﺈﻥ ﺍﻟﺘﻜﺎﻤـل‬ ‫)‪V°ο 8 (x − a‬‬ ‫ﻤﻼﺤﻅﺔ ﺍﻟﺠﻬﺩ ﻴﺤﺘﻭﻱ ﻋﻠﻰ ﺤﺩ ﻤﺜل‪:‬‬
‫ﻴﻌﻁﻴﻨﺎ‬ ‫∈‪a +‬‬ ‫ﺇﻟﻰ‬ ‫∈‪+ a −‬‬ ‫ﻟﻠﻤﻌﺎﺩﻟﺔ ﻤﻥ‬
‫∈‪a +‬‬
‫‪du‬‬ ‫‪du‬‬ ‫‪2m‬‬ ‫‪2m‬‬
‫‪( ) at ∈ − ( ) a − ∈ = 2‬‬
‫‪dx‬‬ ‫‪dx‬‬ ‫‪h‬‬ ‫∫‬
‫∈‪a −‬‬
‫= )‪V°ο 8 (x − a) u (x‬‬ ‫)‪V u (a‬‬
‫‪h 2 °ο‬‬

‫ﺍﺘﺼﺎل ﺍﻟﺘﻔﺎﻀل ﻟﻠﺠﻬﺩ ﻓﻲ ﺍﻟﻤﺴﺄﻟﺔ ﻴﻌﻁﻲ‪:‬‬


‫‪K −q‬‬ ‫‪2k‬‬
‫= ‪R‬‬ ‫‪,‬‬ ‫=‪T‬‬
‫‪K +q‬‬ ‫‪k+q‬‬

‫ﻭﻤﻥ ﻫﺫﺍ ﻴﻤﻜﻨﻨﺎ ﺤﺴﺎﺏ ﺍﻟﻔﻴﺽ ﺍﻟﻤﻨﻌﻜﺱ ﻭﺍﻟﻨﺎﻓﺫ ‪flected and transmitted‬‬
‫‪feuxes‬‬
‫‪hk‬‬ ‫‪h k k −q 2‬‬
‫= ‪Refleted j‬‬ ‫=‬
‫‪2‬‬
‫‪R‬‬ ‫(‬ ‫)‬
‫‪m‬‬ ‫‪m k+q‬‬
‫‪hq‬‬ ‫‪hq‬‬ ‫‪2k 2‬‬ ‫‪hk 4 hq‬‬
‫= ‪Transmitte d j‬‬ ‫= ‪T2‬‬ ‫(‬ ‫= )‬ ‫)‪(C‬‬
‫‪m‬‬ ‫‪m‬‬ ‫‪k +q‬‬ ‫‪m (k + q) 2‬‬

‫ﻻﺤﻅ ﺍﻵﺘﻲ‪:‬‬

‫‪- ٩٧ -‬‬
‫‪PDF created with pdfFactory Pro trial version www.pdffactory.com‬‬
‫‪ -١‬ﺒﺨﻼﻑ ﺍﻟﻤﻴﻜﺎﻨﻴﻜﺎ ﺍﻟﺘﻘﻠﻴﺩﻴﺔ ﻭﺍﻟﺘﻲ ﻁﺒﻘﺎﹰ ﻟﻬﺎ‪ :‬ﺠﺴﻡ ﻴﺴﻴﺭ ﻓﻭﻕ ﺠﻬﺩ ﻓﺈﻨﻪ ﺴﻭﻑ‬
‫ﻴﺘﺒﺎﻁﺄ ﻭﺫﻟﻙ ﻟﺤﻔﻅ ﺍﻟﻁﺎﻗﺔ‪ .‬ﻭﻟﻜﻨﻪ ﻻ ﻴﻤﻜﻥ ﺃﻥ ﻴﻨﻌﻜﺱ‪ .‬ﺃﻤﺎ ﻫﺫﺍ ﻓﻨﻼﺤﻅ ﺃﻥ ﺠـﺯﺀ‬
‫ﻤﻌﻴﻥ ﻤﻥ ﺍﻟﺠﺴﻴﻤﺎﺕ ﺍﻟﺴﺎﻗﻁﺔ ﺍﻨﻌﻜﺱ‪ .‬ﻭﻫﺫﺍ ﺒﺎﻟﻁﺒﻊ ﻨﺘﻴﺠـﺔ ﻟﻠﻁﺒﻴﻌـﺔ ﺍﻟﻤﻭﺠﻴـﺔ‬
‫ﻟﻠﺠﺴﻴﻤﺎﺕ ﻓﺈﻥ ﺍﻻﻨﻌﻜﺎﺱ ﺍﻟﺠﺯﻱﺀ ﻟﻠﻀﻭﺀ ﺍﻟﺴﺎﻗﻁ ﻋﻨﺩ ﺍﻟﺘﻤـﺎﺱ ﺒـﻴﻥ ﻭﺴـﻁﻴﻥ‬
‫ﻓﻅﺎﻫﺭﺓ ﻤﻌﺘﺎﺩﺓ‪.‬‬
‫‪ -٢‬ﻤﻥ ﺨﻼل ﺍﻟﺘﻌﺒﻴﺭﺍﺕ )ﻤﻌﺎﺩﻟﺔ ‪ (C‬ﻨﻼﺤﻅ ﺃﻥ ﻗﺎﻨﻭﻥ ﺍﻟﺤﻔﻅ )ﺍﻟﺒﻘﺎﺀ( ﻤﺘﺤﻘﻕ ﺃﻱ‬
‫ﺃﻥ ﺍﻟﻔﻴﺽ ﻋﻠﻰ ﺍﻟﺠﺎﻨﺏ ﺍﻷﻴﺴﺭ = ﺍﻟﻔﻴﺽ ﻋﻠﻰ ﺍﻟﺠﺎﻨﺏ ﺍﻷﻴﻤﻥ‪.‬‬
‫‪hk‬‬ ‫‪k −q 2‬‬ ‫‪hq 4k 2‬‬
‫( ‪(1 −‬‬ ‫=) )‬
‫‪m‬‬ ‫‪k+q‬‬ ‫‪m (k + q) 2‬‬
‫‪hk‬‬ ‫‪4qk‬‬ ‫‪h k 4qk‬‬
‫=‬
‫)‪m (k + q‬‬ ‫‪2‬‬
‫‪m (k + q) 2‬‬

‫ﻓﺈﻥ ﺍﻟﻨـﺴﺒﺔ ﺒـﻴﻥ ﺍﻟﻔـﻴﺽ‬ ‫= ‪k‬‬


‫‪2m‬‬
‫‪2‬‬
‫‪2m‬‬
‫‪E ≈ 2 ( E − Vο ) = q ⇐ E >> Vο‬‬ ‫‪ -٣‬ﻓﻲ ﺤﺎﻟﺔ‬
‫‪h‬‬ ‫‪h‬‬
‫‪ο← R‬‬
‫‪2‬‬
‫ﺍﻟﻤﻨﻌﻜﺱ ﻭﺍﻟﺴﺎﻗﻁ‬
‫‪JR‬‬ ‫‪( k − q) 2‬‬
‫=‬ ‫‪→0‬‬
‫‪JT‬‬ ‫‪4kq‬‬

‫ﻭﻫﺫﻩ ﺍﻟﻨﺘﻴﺠﺔ ﺘﺘﻔﻕ ﻤﻊ ﺍﻟﺤﺩﺱ ﻭﺍﻟﺫﻱ ﻴﺩل ﻋﻠﻰ ﺃﻨﻪ ﻋﻨﺩ ﺍﻟﻁﺎﻗﺎﺕ ﺍﻟﺤﺎﻟﻴـﺔ‬
‫ﻓﺈﻥ ﻭﺠﻭﺩ ﻋﻘﺒﺔ ﺍﻟﺠﻬﺩ ﻤﺎ ﻫﻭ ﺇﻻ ﺘﺸﻭﻴﻪ ﻗﻠﻴل ﻋﻠﻰ ﺍﻨﺘﺸﺎﺭ ﺍﻟﻤﻭﺠﻪ‪.‬‬

‫ﺘﺼﺒﺢ ﻜﻤﻴﺔ ﺘﺨﻴﻠﻴﺔ ﻓﻲ ﻫـﺫﻩ‬ ‫=‪q‬‬


‫‪2m‬‬
‫)‪(E − Vο‬‬ ‫ﻓﻲ ﻫﺫﻩ ﺍﻟﺤﺎﻟﺔ‬ ‫‪E < Vο‬‬ ‫‪ -٤‬ﻟﻭ ﺃﻥ‬
‫‪h2‬‬

‫ﺍﻟﺤﺎﻟﺔ ﻓﺈﻥ ﺍﻟﺤﻠﻭل ‪ x > 0‬ﻴﻜﻭﻥ ﻟﻬﺎ‪.‬‬


‫‪− q x‬‬
‫‪U (x) = T e i (iq) x = T e‬‬

‫ﻓﻲ ﻫﺫﻩ ﺍﻟﺤﺎﻟﺔ‪:‬‬


‫*‬
‫‪ k −i q   k −i q ‬‬ ‫‪k −i q k +i q‬‬
‫‪= R R = ‬‬ ‫‪‬‬ ‫= ‪‬‬ ‫‪=1‬‬
‫‪2‬‬ ‫*‬
‫‪R‬‬ ‫‪ k +i q ‬‬
‫‪‬‬ ‫‪k‬‬ ‫‪+‬‬ ‫‪i‬‬ ‫‪q‬‬ ‫‪‬‬ ‫‪‬‬ ‫‪k‬‬ ‫‪+‬‬ ‫‪i‬‬ ‫‪q‬‬ ‫‪k‬‬ ‫‪−‬‬ ‫‪i‬‬ ‫‪q‬‬

‫‪- ٩٨ -‬‬
‫‪PDF created with pdfFactory Pro trial version www.pdffactory.com‬‬
‫ﻟﻴﺴﺕ‬ ‫‪u = Te‬‬
‫‪−qx‬‬
‫ﻭﻓﻲ ﻫﺫﻩ ﺍﻟﺤﺎﻟﺔ ﺘﻭﺠﺩ ﻤﻭﺠﺔ ﻤﻨﺘﺸﺭﺓ ﻋﻥ ﻴﻤﻴﻥ ﺍﻟﺠﻬﺩ ﻷﻥ‬
‫ﻤﻭﺠﺔ ﺭﺍﺤﻠﺔ‪ .‬ﻭﻫﻜﺫﺍ‪ ،‬ﻜﻤﺎ ﻓﻲ ﺍﻟﻤﻴﻜﺎﻨﻴﻜﺎ ﺍﻟﺘﻘﻠﻴﺩﻴﺔ ﻴﻜﻭﻥ ﺍﻨﻌﻜﺎﺱ ﻜﻠﻲ‪ .‬ﻻﺤﻅ )ﻤﻥ‬
‫ﻨﺎﺤﻴﺔ ﺃﺨﺭﻯ( ﺃﻥ‪.‬‬
‫‪2k‬‬
‫=‪T‬‬ ‫‪≠0‬‬
‫‪k+i q‬‬

‫ﻭﻫﺫﺍ ﻴﻌﻨﻲ ﺃﻥ ﺠﺯﺀ ﻤﻥ ﺍﻟﻤﻭﺠﺔ ﻴﺨﺘﺭﻕ ‪ Penetrates‬ﺇﻟﻰ ﺩﺍﺨل ﺍﻟﻤﻨﻁﻘـﺔ‬


‫ﺍﻟﻤﻤﻨﻭﻋﺔ )ﺍﻟﻤﺤﺭﻤﺔ( ﻭﻅﺎﻫﺭﺓ ﺍﻻﺨﺘﺭﺍﻕ ﻫﺫﻩ ﻤﻥ ﻅﻭﺍﻫﺭ ﺍﻟﻤﻭﺠﺎﺕ ﻭﺴﻨﺭﻯ ﺒﻌـﺩ‬
‫ﻗﻠﻴل ﺃﻥ ﻫﺫﻩ ﺴﺘﺴﻤﺢ ﺒـ "‪ "tunneling‬ﺍﻻﺨﺘﺭﺍﻕ ﻤﻥ ﺨﻼل ﺍﻟﺠﻬﺩ‪ .‬ﻭﺍﻟـﺫﻱ ﻤـﻥ‬
‫ﻨﺎﺤﻴﺔ ﺍﻟﻔﻴﺯﻴﺎﺀ ﺍﻟﺘﻘﻠﻴﺩﻴﺔ ﺴﻴﻤﻨﻊ ﺒﺎﻟﻜﺎﻤل ﺍﻟﺠﺴﻴﻤﺎﺕ ﻭﻴﻤﻨﻌﻬﺎ ﻤﻥ ﺍﻻﻓﺘـﺭﺍﻕ‪ .‬ﻭﻓـﻲ‬
‫ﺍﻟﺤﻘﻴﻘﺔ ﻓﺈﻨﻪ ﻻ ﻴﻭﺠﺩ ﻓﻴﺽ ﻤﻥ ﺍﻟﺠﺴﻴﻤﺎﺕ ﻟﻠﻴﻤﻴﻥ ﻭﺫﻟﻙ ﻷﻥ )‪ j (x‬ﺘﺨﺘﻔﻲ ﻟﻠﺤﻠﻭل‬
‫ﺍﻟﺤﻘﻴﻘﻴﺔ ﺤﺘﻰ ﻟﻭ ﺍﻟﻤﻌﺎﻤل ﻟﻸﻤﺎﻡ ﺃﺨﺫ ﻜﻤﻴﺔ ﻤﺭﻜﺒﺔ ﻅﺎﻫﺭﺓ ﺍﻻﻨﻌﻜﺎﺱ ﺍﻟﻜﻠﻲ ﺭﻴﺎﻀﻴﺎﹰ‬
‫ﺘﻜﺎﻓﺊ ﻟﻠﺫﻱ ﻴﺤﺩﺙ ﻟﻠﻀﻭﺀ ﻋﻨﺩﻤﺎ ﻴﺴﻘﻁ ﻋﻠﻰ ﺴﻁﺢ )ﺘﻤﺎﺱ( ﺒـﻴﻥ ﻭﺴـﻁﻴﺔ ﻟﻬـﺎ‬
‫ﻤﻌﺎﻤل ﺍﻨﻜﺴﺎﺭ ﻤﺨﺘﻠﻑ )‪ ( n2 > n1‬ﺒﺯﺍﻭﻴﺔ ﺃﻜﺒﺭ ﻤﻥ ﺍﻟﺯﺍﻭﻴﺔ ﺍﻟﺤﺭﺠﺔ‪ .‬ﻓﺈﻥ ﺍﻟﻀﻭﺀ‬
‫ﻴﻌﺎﻨﻲ ﻤﻥ ﺍﻨﻌﻜﺎﺱ ﻜﻠﻲ ﻭﻟﻜﻥ ﺴﻴﻜﻭﻥ ﻫﻨﺎﻙ ﺍﻀﻤﺤﻼل ﻟﻠﻤﺠﺎل ﺍﻟﻜﻬﺭﻭﻤﻴﻐﻨﺎﻁﻴﺴﻲ‬
‫ﺍﻟﺫﻱ ﻴﺨﺘﺭﻕ ﺍﻟﻤﻨﻁﻘﺔ ﺍﻟﻤﺤﺭﻤﺔ‪.‬‬

‫ﺒﺌﺭ ﺍﻟﺠﻬﺩ ‪:The Potential well‬‬


‫ﻨﺄﺨﺫ ﺍﻟﺠﻬﺩ ﺍﻟﻤﻭﻀﻊ ﺒـ‬
‫‪V (x) = 0‬‬ ‫‪x <−a‬‬
‫‪= − Vο‬‬ ‫‪−a < x <a‬‬
‫‪=o‬‬ ‫‪a<x‬‬

‫‪2mE‬‬ ‫) ‪2m (E + Vο‬‬


‫= ‪k2‬‬ ‫‪,‬‬ ‫= ‪q2‬‬
‫‪h2‬‬ ‫‪h2‬‬

‫ﺍﻟﺤﻠﻭل‪:‬‬

‫‪- ٩٩ -‬‬
‫‪PDF created with pdfFactory Pro trial version www.pdffactory.com‬‬
‫‪x‬‬ ‫‪< −a‬‬ ‫‪⇒ u ( x ) = e ikx + R e −ikx‬‬
‫‪-a< x‬‬ ‫‪<a‬‬ ‫‪⇒ u ( x ) = A e iqx + B e -iqx‬‬
‫‪x‬‬ ‫‪< a‬‬ ‫‪⇒ u ( x ) = T e ikx‬‬

‫ﻤﻥ ﺍﻟﻴﺴﺎﺭ ﻭﻓـﻴﺽ ﻤـﻨﻌﻜﺱ ﻤـﻥ‬ ‫‪hk‬‬


‫ﻭﻫﺫﻩ ﺘﻁﺎﺒﻕ ﻟﻔﻴﺽ ﻗﺎﺩﻡ )ﺴﺎﻗﻁ(‬
‫‪m‬‬

‫‪. hk‬‬ ‫‪T‬‬


‫‪2‬‬
‫ﻭﻓﻴﺽ ﻨﺎﻓﺫ‬ ‫‪hk‬‬
‫‪R‬‬
‫‪2‬‬
‫ﺍﻟﻴﻤﻴﻥ‬
‫‪m‬‬ ‫‪m‬‬

‫ﺃﻤﺎ ﺩﺍﺨل ﺍﻟﺒﺌﺭ ﻓﺈﻨﻪ ﺘﻭﺠﺩ ﻤﻭﺠﺎﺕ ﺫﺍﻫﺒﺔ ﻓﻲ ﻜﻼ ﺍﻻﺘﺠـﺎﻫﻴﻥ ﻭﺫﻟـﻙ ﻷﻥ‬
‫ﻭﻁﺒﻘﺎﹰ ﻟﻘﺎﻨﻭﻥ ﺤﻔﻅ ﺍﻟﻔﻴﺽ‪.‬‬ ‫‪x=±‬‬ ‫ﺍﻻﻨﻌﻜﺎﺱ ﻴﺤﺩﺙ ﻤﻥ ﺍﻟﻁﺭﻓﻴﻥ ﻋﻨﺩ‬
‫‪hk‬‬ ‫‪hq‬‬ ‫‪hk‬‬
‫= ) ‪(1− R‬‬ ‫=) ‪− B‬‬
‫‪2‬‬ ‫‪2‬‬ ‫‪2‬‬ ‫‪2‬‬
‫‪( A‬‬ ‫‪T‬‬
‫‪m‬‬ ‫‪m‬‬ ‫‪m‬‬

‫ﻭﺒﺘﻁﺎﺒﻕ ﺍﻟﺩﻭﺍل ﺍﻟﻤﻭﺠﻴﺔ ﻭﺘﻔﺎﻀﻼﺘﻬﺎ ﻴﻌﻁﻴﻨﺎ ﺃﺭﺒﻊ ﻤﻌﺎﺩﻻﺕ‪.‬‬


‫‪e - ika + R e + ika = A e -iqa + B e iqa‬‬
‫) ‪ik ( e - ika − R e ika ) = iq ( A e -iqa − B e iqa‬‬
‫‪A e + iqa + B e - iqa T e ika‬‬
‫‪iq ( Ae iqa − B e - iqa = ik T e ika‬‬

‫ﻭﺒﺤل ﻫﺫﻩ ﺍﻟﻤﻌﺎﺩﻻﺕ‬


‫‪(q 2 − k 2 ) sin 2qa‬‬
‫‪R = ie −2ika‬‬
‫‪2k q cos 2qa − i (q 2 + k 2 ) sin 2qa‬‬
‫‪2kq‬‬
‫‪T = e −2ika‬‬
‫‪2k q cos 2qa − i (q 2 + k 2 ) sin 2qa‬‬

‫‪q 2 − k 2 << 2k q‬‬ ‫ﻓﺈﻨﻪ ﻻ ﻴﻭﺠﺩ ﻋﻤﻠﻴﺎﹰ ﺍﻨﻌﻜـﺎﺱ ﻭﺫﻟـﻙ ﻷﻥ‬ ‫‪E >> Vο‬‬ ‫ﻭﻓﻲ ﺤﺎﻟﺔ‬
‫←‪ο‬‬ ‫ﻓﺈﻥ ﺍﻟﻨﻔﺎﺫﻴﺔ‬ ‫‪E→ο‬‬ ‫ﻭﻜﺫﻟﻙ ﻋﻨﺩﻤﺎ‬

‫ﻓﻲ ﺤﺎﻟﺔ ‪ sin 2 ga = 0‬ﻓﻲ ﺤﺎﻟﺔ ﺍﻟﻁﺎﻗﺎﺕ ﻤﻭﺠﺒﺔ ﺍﻟﻤﻌﻁﺎﺓ ﺒـ‬


‫ﻟﻬﺫﻩ ﺍﻟﻘﻴﻡ ﻤﻥ ﺍﻟﻁﺎﻗﺔ ﻻ ﻴﻭﺠﺩ ﺍﻨﻌﻜﺎﺱ ‪ .R = 0‬ﻭﻫﺫﺍ ﻨﻤﻭﺫﺝ ﻟﻤﺎ ﻴﺤﺩﺙ ﻓﻲ‬
‫ﺍﺴﺘﻁﺎﺭﺓ ﺍﻟﻜﺘﺭﻭﻨﺎﺕ ﺫﺍﺕ ﻁﺎﻗﺔ ﻤﻨﺨﻔﻀﺔ )‪ ( 0.1 eV‬ﺒﻭﺍﺴـﻁﺔ ﻏـﺎﺯﺍﺕ ﻓﺎﻋﻠـﺔ‬
‫)ﻨﻴﻭﻥ‪ ،‬ﺃﺭﺠﻭﻥ( ﻭﺍﻟﺫﻱ ﻴﻭﺠﺩ ﺒﻬﺎ ﻨﻔﺎﺫﻴﺔ ﻋﺎﻟﻴﺔ ﻏﻴﺭ ﻁﺒﻴﻌﻴﺔ‪ .‬ﻫﺫﻩ ﺍﻟﻅـﺎﻫﺭﺓ ﺘـﻡ‬

‫‪- ١٠٠ -‬‬


‫‪PDF created with pdfFactory Pro trial version www.pdffactory.com‬‬
‫ﻤﺸﺎﻫﺩﺘﻬﺎ ﺃﻭﻻﹰ ﺒﻭﺍﺴﻁﺔ ‪ Townsend , Ramsauer‬ﻭﻭﺼﻔﺕ ﺒﺄﻨﻪ ﺭﻨﻴﻥ ﺍﻟﻨﻔﺎﺫﻴـﺔ‬
‫‪.transmission resonance‬‬
‫ﻭﻟﺘﻘﺩﻴﻡ ﻤﻨﺎﻗﺸﺔ ﺩﻗﻴﻘﺔ ﻓﺈﻨﻪ ﻤﻥ ﺍﻟﻀﺭﻭﺭﻱ ﺇﺩﺨﺎل )ﻭﻀﻊ ﻓـﻲ ﺍﻟﺤـﺴﺒﺎﻥ(‬
‫ﺍﻋﺘﺒﺎﺭﺍﺕ ﺜﻼﺜﺔ ﻭﺒﻠﻐﺔ ﺍﻟﻤﻭﺠﺎﺕ‪ ،‬ﻓﺈﻥ ﺍﻟﺘﺄﺜﻴﺭ ﺒﺴﺒﺏ ﺍﻟﺘﺩﺍﺨل ﺍﻟﻬﺩﺍﻡ ﺒﻴﻥ ﺍﻟﻤﻭﺠـﺎﺕ‬
‫ﺍﻟﻤﻨﻌﻜﺴﺔ ﻋﻨﺩ ‪ c = - a‬ﻭﺍﻟﻤﻭﺠﺎﺕ ﺍﻟﻤﻨﻌﻜﺴﺔ ﻤﺭﺓ ﻭﺍﺜﻨﻴﻥ ﻭﺜﻼﺜـﺔ ﻋﻨـﺩ ‪x = 0‬‬
‫ﻭﻫﺫﺍ ﻫﻭ ﺍﻟـﺫﻱ ﻴـﺼﻑ – ‪Fairy‬‬ ‫=‪λ‬‬
‫‪2D 2D 4a‬‬
‫=‬ ‫=‬ ‫ﺍﻟﺭﻨﻴﻥ‪⇐ 2 qa = nh .‬‬ ‫ﻭﺸﺭﻁ‬
‫‪q‬‬ ‫‪nD n‬‬

‫‪ .Perotintenforometeer‬ﻭﻫﺫﺍ ﻫﻭ ﺸﺭﻁ ﻋﻤل ﺠﻬﺎﺯ‪.‬‬


‫‪V (x) = ο‬‬ ‫ﻓﻲ ﺍﻟﻤﻌﺎﺩﻟﺔ‬ ‫‪Vο‬‬ ‫ﺴﺎﻟﺒﺔ ) ﺃﻱ ﺃﻥ‬ ‫‪Vο‬‬ ‫ﺍﻟﺤﻠﻭل ﻓﻲ ‪ E < 0‬ﺒﺸﺭﻁ ﺃﻥ‬
‫‪= Vο → > ο‬‬
‫=‬ ‫‪ο‬‬

‫ﺴﺘﻜﻭﻥ )ﻜﻤﺎ ﺴﻨﺭﻯ ﺍﻟﺤﻠﻭل ﻤﺠﺯﺃﺓ ‪ .(discrete‬ﺨﺫ‬


‫‪2mE‬‬
‫‪= −K2‬‬
‫‪h2‬‬

‫∞‬ ‫ﺍﻟﺤﻠﻭل ﺨﺎﺭﺝ ﺍﻟﺒﺌﺭ ﻭﺍﻟﺘﻲ ﺘﻜﻭﻥ ﻤﻘﻴﺩﺓ ﻋﻨﺩ‬


‫‪u ( x ) = C1 e kx‬‬ ‫‪x < −a‬‬
‫‪u ( x ) = C 2 e − kx‬‬ ‫‪a <x‬‬
‫‪u ( x ) = e + ikx + Re −ikx‬‬ ‫‪x<−a‬‬
‫‪u ( x ) = A e iqx + Be −iqx‬‬ ‫‪−a<x‬‬
‫‪u ( x ) = T e iux‬‬ ‫‪a<x‬‬

‫ﻭﺒﻤﺎ ﺃﻨﻨﺎ ﻨﺘﻌﺎﻤل ﻤﻊ ﺩﻭﺍل ﺤﻘﻴﻘﻴﺔ ﻓﺈﻨﻪ ﻓﻲ ﻫﺫﻩ ﺍﻟﺤﺎﻟﺔ ﻤﻥ ﺍﻟﻤﻨﺎﺴﺏ ﺃﻥ ﻨﻜﺘﺏ‬
‫ﺍﻟﺤﻠﻭل ﺩﺍﺨل ﺍﻟﺒﺌﺭ ﺒﺎﻟﺸﻜل‪.‬‬

‫‪u ( x ) = A cos qx + B sin qx‬‬ ‫‪− a<x<a‬‬

‫ﺒﺘﻁــﺎﺒﻕ ﺍﻟﺤﻠــﻭل‬ ‫‪Vο > E‬‬ ‫ﻭﺫﻟــﻙ ﻷﻥ‬ ‫= ‪q2‬‬


‫‪2m‬‬
‫‪(Vο − E ) > ο‬‬ ‫ﻻﺤــﻅ ﺃﻥ‬
‫‪h2‬‬
‫ﻴﻌﻁﻴﻨﺎ‬ ‫‪x = ±a‬‬ ‫ﻭﺘﻔﺎﻀﻼﺘﻬﺎ ﻋﻨﺩ ﺍﻟﺠﻭﺍﺏ‬

‫‪- ١٠١ -‬‬


‫‪PDF created with pdfFactory Pro trial version www.pdffactory.com‬‬
C1 e− ka = A cos qa − B sin q a
u C1 e− ka = [A ( sin qa) q ]+ B (cos q a) = q ( A sin qa + B cos q a)
C 2 e − ka = A cos qa + B sin q a
− k C 2 e − ka = q (A sin qa − B cos q a)

.K ‫ﻤﻥ ﻫﺫﻩ ﺍﻟﻤﻌﺎﺩﻻﺕ ﻴﻤﻜﻨﻨﺎ ﻜﺘﺎﺒﺔ‬


A sin qa − Bcos qa
k=q (1)
A sin qa + Bcos qa
A sin qa + Bcos qa
=q (2)
A sin qa − Bcos qa

‫ ﻭﻫﺫﺍ ﻴﻌﻨﻲ ﺃﻥ ﺍﻟﺤﻠـﻭل ﺇﻤـﺎ‬A B = 0 ‫ﻭﻫﺎﺘﻴﻥ ﺍﻟﻤﻌﺎﺩﻟﺘﻴﻥ ﻤﻌﺎﹰ ﺘﻌﻨﻲ ﺃﻥ‬


( A = 0 ) x = ‫ ( ﺃﻭ ﻓﺭﺩﻴﺔ‬B = 0 ) x = ‫ﺯﻭﺠﻴﺔ‬

.‫ﻭﺍﻟﺩﻭﺍل ﺘﻘﺭﻴﺒﺎﹰ ﺸﻜﻠﻬﺎ ﺍﻟﻤﻭﻀﺢ ﻓﻲ ﺍﻟﺸﻜل‬

‫ )ﺍﻟﺘﻲ ﻟﻴﺱ ﻟﻬﺎ ﻋﻘﺩ( ﺘﻜﻭﻥ ﺯﻭﺠﻴﺔ ﻭﺍﻟـﺸﺭﻭﻁ‬groudstate ‫ﻭﺍﻟﺤﺎﻟﺔ ﺍﻟﺩﻨﻴﺎ‬


.‫ﺍﻟﺘﻲ ﺘﺤﺩﺩ ﺍﻟﻁﺎﻗﺔ‬
(1) ‫ﻤﻥ ﺍﻟﻤﻌﺎﺩﻟﺔ‬

K = q tam qa (B = 0 ) ‫ﺤﻠﻭل ﺯﻭﺠﻴﺔ‬

- ١٠٢ -
PDF created with pdfFactory Pro trial version www.pdffactory.com
‫ﻤﻥ ﺍﻟﻤﻌﺎﺩﻟﺔ )‪(2‬‬

‫‪K = q tam qa‬‬ ‫ﺤﻠﻭل ﺯﻭﺠﻴﺔ ) ‪(A = 0‬‬

‫ﺩﻋﻨﺎ ﻨﺨﺘﺒﺭ ﻫﺫﻩ ﺍﻟﺤﻠﻭل ﻤﻨﻔﺼﻠﺔ‪.‬‬


‫)‪ (a‬ﺍﻟﺤﻠﻭل ﺍﻟﺯﻭﺠﻴﺔ‪:‬‬

‫)ﺘﺘﻨﺎﺴﺏ ﻤﻊ ﻋﻤﻕ ﺍﻟﺒﺌﺭ(‬ ‫ﺍﺴﺘﺨﺩﻡ ﺍﻟﺭﻤﺯ‬


‫‪2m Vο a 2‬‬
‫=‪λ‬‬
‫‪h2‬‬

‫) ﺘﺘﻨﺎﺴﺏ ﻤﻊ ﻋﺭﺽ ﺍﻟﺒﺌﺭ(‬ ‫‪y= q a‬‬

‫ﺇﻥ ﻨﻘﺎﻁ ﺍﻟﺘﻘﺎﻁﻊ ﺘﺤﺩﺩ ﺍﻟﻘﻴﻡ ﺍﻟﺨﺎﺼﺔ ‪ .‬ﻭﻫﺫﻩ ﺘﺘﻤﺜل ﻓﺌﺔ ﻤﺘﻘﻁﻌﺔ ‪discete ser‬‬

‫ﺘﻤﻀﻲ‪.‬‬ ‫‪λ − y2‬‬


‫ﻓﺈﻥ ﺍﻟﻤﻨﺤﻨﻴﺎﺕ ﻟﻠﻤﻌﺎﺩﻟﺔ‬ ‫‪λ‬‬ ‫ﻭﻜﻠﻤﺎ ﻜﺒﺭﺕ ﻗﻴﻤﺔ‬
‫‪y‬‬

‫ﺃﻱ‪ :‬ﻋﻨﺩﻤﺎ ﻴﺯﺩﺍﺩ ﻋﻤﻕ ﺍﻟﺠﻬﺩ ﺃﻭ ﻴﺴﺘﻌﺭﺽ ﻓﺈﻨﻪ ﻴﻭﺠﺩ ﺍﻟﻤﺯﻴﺩ ﻤﻥ ﺍﻟﺤﺎﻻﺕ‬
‫‪λ‬‬ ‫ﺍﻟﻤﺭﺒﻭﻁﺔ )ﺍﻟﻤﻘﻴﺩﺓ( ‪ lundstates‬ﻭﺍﻟﺸﻜل ﻴﺒﻴﻥ ﻜﺫﻟﻙ ﺃﻨﻪ ﻻ ﻴﻬﻡ ﻜﻡ ﺘﻜﻭﻥ ﻗﻴﻤﺔ‬
‫ﺼﻐﻴﺭﺓ ﻓﺈﻨﻪ ﺴﻴﻜﻭﻥ ﺩﺍﺌﻤﺎﹰ ﻋﻠﻰ ﺍﻷﻗل ﺤﺎﻟﺔ ﻤﺭﺒﻭﻁﺔ ﻭﺍﺤﺩﺓ‪ .‬ﻭﻫﺫﺍ ﺃﻤﺭ ﻴﻤﻴﺯ ﺍﻟﺠﻬﺩ‬
‫ﺍﻟﺠﺎﺫﺏ ‪ ttractiuepot‬ﻓﻲ ﺒﻌﺩ ﻭﺍﺤﺩ ﻭﻟﻴﺱ ﺼﺤﻴﺤﺎﹰ ﻓﻲ ﺜﻼﺜﺔ ﺃﺒﻌﺎﺩ ﻭﺍﻟﺫﻱ ﻴـﺴﻠﻙ‬
‫ﻓﺈﻥ ﺍﻟﻘـﻴﻡ ﺍﻟﺨﺎﺼـﺔ‬ ‫‪λ‬‬ ‫ﺴﻠﻭﻙ ﺍﻟﺤﻠﻭل ﺍﻟﻔﺭﺩﻴﺔ )ﺴﺘﻨﺎﻓﺱ ﻓﻴﻤﺎ ﺒﻌﺩ(‪ .‬ﻭﻜﻠﻤﺎ ﻜﺒﺭﺕ‬
‫ﻭﺒﻨﻘﺎﻁ ﺍﻟﺘﻘﺎﻁﻊ ﺍﻟﻤﻌﻁﺎﺓ ﺒﺎﻟﻤﻌﺎﺩﻟﺔ‪.‬‬ ‫‪λ‬‬ ‫ﺘﺅﻭل ﻭﺘﺼﺒﺢ ﻤﺘﺴﺎﻭﻴﺔ ﺍﻟﻤﺴﺎﻓﺔ ﻓﻲ‬
‫ﻭﻫﺫﺍ ﻫﻭ ﺸﺭﻁ ﺍﻟﻘﻴﻡ ﺍﻟﺨﺎﺼـﺔ ﻟﻠﺤﻠـﻭل‬ ‫‪qa = y = (n +‬‬
‫‪1‬‬
‫‪)D‬‬ ‫‪, n = 0, 1, 2, ..........‬‬
‫‪2‬‬

‫ﺍﻟﺯﻭﺠﻴﺔ ﻟﺼﻨﺩﻭﻕ ﻤﺎ ﻻ ﻨﻬﺎﻴﺔ ﻤﺘﻤﺭﻜﺯ ﺤﻭل ﺍﻟﻨﻘﻁﺔ ﺍﻷﺼل‪.‬‬

‫‪- ١٠٣ -‬‬


‫‪PDF created with pdfFactory Pro trial version www.pdffactory.com‬‬
Figure 5-11. Location of discrete eigenvalues for even solutions in
square well. The rising curves represent tan y; the falling curves are
λ − y 2 / y for different values of λ

.‫ ﻫﻨﺎ ﺴﻴﻜﻭﻥ ﺸﺭﻁ ﺍﻟﻘﻴﻤﺔ ﺍﻟﺨﺎﺼﺔ‬:‫( ﺍﻟﺤﻠﻭل ﺍﻟﻔﺭﺩﻴﺔ‬b)


λ − y2
= − cos y
y

‫ﻓﺈﻥ ﺍﻟﺭﺴﻤﺔ ﺍﻟﺘﺎﻟﻴﺔ ﺘﺒﺩﻭ ﻤﺜل ﺍﻟﺭﺴﻤﺔ ﺃﻋﻼﻩ ﻭﻟﻜـﻥ‬ D


− cory = ( + y ) ‫ﻭﺒﻤﺎ ﺃﻥ‬
2

‫ﻓﺈﻨﻬﺎ ﺘﺴﻠﻙ ﺒﻨﻔﺱ ﺍﻟﻁﺭﻴﻘـﺔ‬ λ ‫ﻭﻜﻠﻤﺎ ﻜﺒﺭﺕ‬ D


‫ ﻤﺘﺯﺤﺯﺤﺔ ﺒﻤﻘﺩﺍﺭ‬tan ‫ﻤﻨﺤﻨﻴﺎﺕ‬
2
y=nD , n = 1, 2, 3, ................. ‫ﺴﺘﺴﺘﺒﺩل ﺒـ‬ 1
y=(n + ) ‫ﻭﻟﻜﻥ ﻤﻌﺎﺩﻟﺔ‬
2

- ١٠٤ -
PDF created with pdfFactory Pro trial version www.pdffactory.com
Figure 5-12. Location of discrete eigenvalues for odd solutions in
square well. The rising curves represent – cot y; the falling curves are
λ − y 2 / y for different values of λ Note that thee is no eigenvalue for
λ < (π / 2) 2

λ−
D2
>ο .‫ﻭﻋﻠﻰ ﻋﻜﺱ ﺍﻟﺤﻠﻭل ﺍﻟﺯﻭﺠﻴﺔ ﻓﺈﻨﻪ ﺴﻴﻜﻭﻥ ﺘﻘﺎﻁﻊ ﻟـﻭﻥ ﺃﻥ‬
4

λ>
D2
= 2.40 ‫ ﻭﻻﺤﻅ ﺃﻥ ﺘﻘﺎﻁﻊ ﻴﺤﺩﺙ ﻋﻨﺩﻤﺎ‬،‫ﺍﻨﻅﺭ ﺇﻟﻰ ﺍﻟﺸﻜل ﺃﻋﻼﻩ‬
4

‫ ﻭﻋﻠﻴـﻪ‬x = 0 ‫ﻤﻥ ﺍﻟﺸﻜل ﻨﻼﺤﻅ ﺃﻥ ﺍﻟﺤﻠﻭل ﺍﻟﻔﺭﺩﻴﺔ ﺘﺨﺘﻔﻲ )ﺘﺘﻼﺸﻰ( ﻋﻨﺩ‬


‫ﻓﺈﻥ ﻤﺴﺄﻟﺔ ﺍﻟﺤﺎﻻﺕ ﺍﻟﻤﻘﻴﺩﺓ ﻟﻠﺤﻠﻭل ﺍﻟﻔﺭﺩﻴﺔ ﺴﺘﻜﻭﻥ ﻫﻲ ﻨﻔﺴﻬﺎ ﻋﻨﺩﻤﺎ ﻴﻜﻭﻥ ﻟﻬﺎ ﺒﺌﺭ‬
.‫ﺍﻟﺠﻬﺩ ﻫﻭ ﺍﻟﻤﻭﻀﺢ ﻓﻲ ﺍﻟﺸﻜل ﺍﻟﺘﺎﻟﻲ‬

- ١٠٥ -
PDF created with pdfFactory Pro trial version www.pdffactory.com
‫ﺴﻴﻜﻭﻥ ﺼﺤﻴﺢ‪.‬‬ ‫‪U (ο ) = 0‬‬ ‫ﻭﺫﻟﻙ ﻷﻥ ﺍﻟﺸﺭﻁ )ﻋﻨﺩ ﻫﺫﺍ ﺍﻟﺠﻬﺩ(‬

‫ﻭﻤﻥ ﺍﻟﺤﺴﺎﺒﺎﺕ ﺍﻟﺘﻔﺼﻴﻠﻴﺔ ﻓﺈﻨﻪ ﻴﻤﻜﻨﻨﺎ ﺘﺄﻜﻴﺩ ﺍﻟﻔﻬﻡ ﺍﻟﻌﺎﻡ ﻟﻠﺴﺒﺏ ﻓﻲ ﺍﻟﺤﺼﻭل‬
‫ﻋﻠﻰ ﻗﻴﻡ ﺨﺎﺼﺔ )ﻏﻴﺭ ﻤﺘﺼﻠﺔ(‪ .‬ﻫﺫﻩ ﺍﻟﻘﻴﻡ ﺘﻅﻬﺭ ﻭﺫﻟﻙ ﻷﻥ ﺍﻟﺩﺍﻟﺔ ﺍﻟﻤﻭﺠﺒﺔ ﻴﺠـﺏ‬
‫ﺃﻥ ﺘﺘﻼﺸﻰ ﻋﻨﺩ ∞ ‪.‬‬

‫ﺤﺎﺠﺯ ﺍﻟﺠﻬﺩ )ﺍﻻﺨﺘﺭﺍﻕ ﺍﻟﻨﻔﻘﻲ( ‪The Potential Barrier‬‬

‫ﺨﺫ ﺍﻟﺠﻬﺩ ﻜﺎﻟﺘﺎﻟﻲ‪:‬‬


‫‪V (x) = 0‬‬ ‫‪x <−a‬‬
‫‪= − Vο‬‬ ‫‪−a < x <a‬‬
‫‪=o‬‬ ‫‪a<x‬‬

‫ﺃﻱ ﺃﻨﻬﺎ ﺍﻟﻁﺎﻗـﺎﺕ ﺍﻟﺘـﻲ ﻻ‬ ‫‪E < Vο‬‬ ‫ﻭﻟﻌﻨﺎ ﻨﺤﺼﺭ ﺍﻟﻤﻨﺎﻗﺸﺔ ﻟﻠﻁﺎﻗﺎﺕ ﺒﺤﻴﺙ‬
‫ﺘﺴﻤﺢ ﺒﺎﺨﺘﺭﺍﻕ ﺍﻟﺤﺎﺠﺯ ﻁﺒﻘﺎﹰ ﻟﻠﻤﻴﻜﺎﻨﻴﻜﺎ ﺍﻟﺘﻘﻠﻴﺩﻴﺔ‪.‬‬

‫ﺩﺍﺨل ﺍﻟﺤﺎﺠﺯ )ﺍﻟﺤﻠﻭل( ﻟﻠﻤﻌﺎﺩﻟﺔ‪.‬‬


‫‪d 2 u (x) 2m‬‬
‫‪+ 2 ( E − Vο ) u (x) = 0‬‬
‫‪dx 2‬‬ ‫‪h 42‬‬
‫‪1‬‬ ‫‪4 43‬‬ ‫‪4‬‬

‫)‪d 2 u (x‬‬
‫‪2‬‬
‫‪− K 2 u (x) = 0‬‬
‫‪dx‬‬
‫‪2m‬‬
‫) ‪K 2 = 2 ( Vο - E‬‬
‫‪h‬‬

‫ﺍﻟﺤل ﺍﻟﻌﺎﻡ‪.‬‬

‫‪U ( x ) = A e − kx + Be kx‬‬ ‫‪x <a‬‬

‫‪- ١٠٦ -‬‬


‫‪PDF created with pdfFactory Pro trial version www.pdffactory.com‬‬
‫) ‪(E < Vο‬‬ ‫ﺃﻱ ﺃﻨﻪ ﺘﻭﺠﺩ ﻨﻔﺎﺫﻴﺔ ﺒﺎﻟﺭﻏﻡ ﻤﻥ ﺃﻥ ﺍﻟﻁﺎﻗﺔ ﺃﻗل ﻤﻥ ﻗﻤـﺔ ﺍﻟﺤـﺎﺠﺯ‬
‫ﻭﻫﺫﻩ ﻅﺎﻫﺭﺓ ﻤﻭﺠﻴﺔ ﻭﻓﻲ ﻤﻴﻜﺎﻨﻴﻜﺎ ﺍﻟﻜﻡ ﻨﻔﺱ ﺍﻟﻅﺎﻫﺭﺓ ﺘﻼﺤﻅ ﻤﻥ ﺍﻟﺠﺴﻴﻤﺎﺕ ﻫﺫﻩ‬
‫ﻫﻲ ﻅﺎﻫﺭﺓ ﺍﻻﺨﺘﺭﺍﻕ ﺍﻟﻨﻔﻘﻲ ﻟﺠﺴﻴﻡ ﺨﻼل ﺤﺎﺠﺯ ﻋﺎﺩﺓ ﻤﺎ ﺘﻼﺤﻅﻪ‪.‬‬
‫ﺇﻥ ﺩﺍﻟﺔ ﺍﻟﻤﻭﺠﺔ ﻻ ﺘﺘﻼﺸﻰ ﺩﺍﺨل ﺍﻟﺤﺎﺠﺯ ﻜﻤﺎ ﻴﻌﻨﻲ ﺍﺤﺘﻤﺎل ﻭﺠﻭﺩ ﺍﻟﺠـﺴﻡ‬
‫ﺒﻁﺎﻗﺔ ﺴﺎﻟﺒﺔ‪ .‬ﻭﻟﻜﻥ ﻜﻴﻑ ﻴﻤﻜﻥ ﺃﻥ ﻟﻬﺫﺍ ﻤﻌﻨﻰ؟‬
‫ﻟﻭ ﻨﻅﺭﻨﺎ ﻟﻤﺒﺩﺃ ﺍﻟﻼﺘﻌﻴﻴﻥ ﻟﺘﺴﻌﻔﻨﺎ ﻤﻥ ﺍﻟﺘﻨﺎﻗﺽ ﻤﻊ ﺍﻟﻤﻴﻜﺎﻨﻴﻜﺎ ﺍﻟﺘﻘﻠﻴﺩﻴﺔ‪.‬‬
‫ﺇﻥ ﺘﺠﺭﺒﺔ ﺩﺭﺍﺴﺔ ﺍﻟﺠﺴﻡ ﺩﺍﺨل ﺤﺎﺠﺯ ﺍﻟﺠﻬﺩ ﻴﺠﺏ ﺃﻥ ﺘﺤﻴﺯﻩ ﺒﺩﻗﺔ ﻗﺩﺭﻫﺎ‬

‫‪Δ x << 2a‬‬

‫ﻭﻫﺫﻩ ﺍﻟﺘﺠﺭﺒﺔ )ﺍﻟﻘﻴﺎﺴﺎﺕ( ﺴﺘﻨﻘل ﻟﻜﻤﻴﺔ ﺍﻟﺤﺭﻜـﺔ ﻟﻠﺠـﺴﻴﻡ ﺒﻌـﺩﻡ ﺘﺤﺩﻴـﺩ‬


‫ﻭﻟﻤـﺸﺎﻫﺩﺓ‬ ‫>> ‪Δ E‬‬
‫‪h2‬‬
‫ﻭﻫﺫﺍ ﻴﻘﺎﺒﻠﻪ ﺍﻨﺘﻘﺎل ﻟﻠﻁﺎﻗﺔ ﻗـﺩﺭﻩ‬ ‫>> ‪Δ p‬‬
‫‪h‬‬
‫ﺘﻜﺭﺭﻫﺎ‪.‬‬
‫‪8ma‬‬ ‫‪2a‬‬

‫‪E − Vο‬‬ ‫ﻁﺎﻗﺔ ﺤﺭﻜﺔ ﺴﺎﻟﺒﺔ‪ ،‬ﻓﺈﻥ ﻫﺫﺍ ﺍﻟﻼﺘﻌﻴﻴﻥ ﻴﺠﺏ ﺃﻥ ﻴﻜﻭﻥ ﺃﻗل ﺒﻜﺜﻴـﺭ ﻤـﻥ‬
‫‪h2 k2‬‬
‫>> ‪>> E‬‬
‫‪h2‬‬
‫ﺒﺤﻴﺙ ﺃﻥ‬
‫‪2m‬‬ ‫‪8ma 2‬‬

‫ﻭﺒﺸﻜل ﻋﺎﻡ ﻓﺈﻥ ﺍﻟﺤﻭﺍﺠﺯ ﺍﻟﺘﻲ ﺘﺤﺩﺙ ﻓﻲ ﺍﻟﻅﻭﺍﻫﺭ ﺍﻟﻔﻴﺯﻴﺎﺌﻴﺔ ﺍﻟﺤﻘﻴﻘﻴـﺔ ﻻ‬


‫ﺘﻜﻭﻥ ﻤﺭﺒﻌﺔ‪ ،‬ﻭﻟﻤﻨﺎﻗﺸﺔ ﺒﻌﺽ ﺍﻟﺘﻁﺒﻴﻘﺎﺕ ﻴﺠﺏ ﺃﻭﻻﹰ ﺃﻥ ﻨﺴﺘﻨﺘﺞ ﺘﻌﺒﻴﺭﺍﺕ ﺘﻘﺭﻴﺒﻴـﺔ‬
‫ﺨﻼل ﺤﻭﺍﺠﺯ ﺠﻬﺩ ﻏﻴﺭ ﻤﻨﺘﻅﻤﺔ ﻭﺍﻟﻁﺭﻴﻘﺔ ﻟﻌﻤل ﻫﺫﺍ ﻤـﻊ‬ ‫‪T‬‬
‫‪2‬‬
‫ﻟﻤﻌﺎﻤل ﺍﻟﻨﻔﺎﺫﻴﺔ‬
‫ﺍﻋﺘﺒﺎﺭ ﺃﻨﻪ ﻻ ﺘﻭﺠﺩ ﺤﻠﻭل ﺩﻗﻴﻘﺔ ﻟﻤﻌﻅﻡ ﺃﺸﻜﺎل ﺍﻟﺠﻬﺩ ﻫﻭ ﻋـﻥ ﻁﺭﻴﻘـﺔ ﺘﻘﺭﻴـﺏ‬
‫)‪ Wentzel- kreners – Birllouin (WKB‬ﺒﻤﻼﺤﻅﺔ ﺍﻟﻤﻌﺎﺩﻟﺔ‬
‫‪2‬‬
‫‪ 4ku‬‬ ‫‪ − 4ka‬‬
‫‪≈  2‬‬ ‫‪ e‬‬
‫‪2‬‬
‫‪T‬‬
‫‪K +u‬‬
‫‪2‬‬
‫‪‬‬

‫ﻭﺍﻟﺘﻲ ﺘﺘﻜﻭﻥ ﻤﻥ ﺤﺩﻴﻥ‪ ،‬ﻓﺈﻥ ﺍﻟﺤﺩ ﺍﻟﺜﺎﻨﻲ ﻫﻭ ﺍﻟﻤﻬﻴﻤﻥ ﻭﺍﻟﻐﺎﻟـﺏ‪ .‬ﻭﺒﻜﺘﺎﺒـﺔ‬


‫ﺍﻟﻤﻌﺎﺩﻟﺔ ﻤﻊ ﺍﻟﺸﻜل ‪.itsholdbe4‬‬

‫‪- ١٠٧ -‬‬


‫‪PDF created with pdfFactory Pro trial version www.pdffactory.com‬‬
‫‪2‬‬ ‫)‪2 (ka) (ua‬‬
‫‪lm T‬‬ ‫‪= 2u (2a) + 2lm‬‬
‫‪123‬‬ ‫‪(ka) 2 + (ua) 2‬‬

‫ﺘﺤﺕ ﻜل ﺍﻟﺤﻴﺜﻴﺎﺕ ﻓﺈﻥ ﺍﻟﺤﺩ ﺍﻷﻭل ﻫﻭ ﺍﻟﻐﺎﻟﺏ )ﺍﻟﻤﻬﻴﻤﻥ( ﻋﻠﻰ ﺍﻟﺤﺩ ﺍﻟﺜﺎﻨﻲ‬
‫ﻷﻱ ﻗﻴﻤﺔ ‪ Ka‬ﻭﺍﻟﻁﺭﻴﻘﺔ ﺍﻟﺘﻲ ﺴﻨﺘﺒﻌﻬﺎ ﻫﻲ ﻤﻌﺎﻟﺠﺔ ﺤﺎﺠﺯ ﻤﻨﺤﻨﻰ ﻜﺤﻭﺍﺠﺯ ﻤﺭﺒﻌﺔ‬
‫ﻤﺘﺠﺎﻭﺭﺓ ‪ uxtaposition‬ﻭﺒﻤﺎ ﺃﻥ ﻤﻌﺎﻤﻼﺕ ﺍﻟﻨﻔﺎﺫﻴﺔ ﻤﻀﺭﻭﺒﺔ ﻋﻨﺩﻤﺎ ﺘﻜﻭﻥ ﺼﻐﻴﺭﺓ‬
‫ﻭﻋﻨﺩﻤﺎ ﻴﻜﻭﻥ ﻤﻌﻅﻡ ﺍﻟﻔﻴﺽ ﻤﻨﻌﻜﺱ‪ ،‬ﻓﺈﻥ ﺍﻟﻨﻔﺎﺫﻴﺔ ﻀﺭﺏ ﻜـل ﺸـﺭﻴﺤﺔ ﺘﻜـﻭﻥ‬
‫ﻤﺴﺘﻘﻠﺔ ﻓﺈﻥ ﻫﺫﺍ ﻗﻠﻴل ﺍﻻﺤﺘﻤﺎل‪.‬‬
‫‪∑ lm T‬‬
‫‪2‬‬ ‫‪2‬‬
‫‪lm T‬‬ ‫~‬
‫> ‪~ − 2∑ Δx < k‬‬

‫‪~−2‬‬ ‫∫‬ ‫‪dx‬‬


‫‪2m‬‬
‫] ‪[V (x) − E‬‬
‫‪barier‬‬ ‫‪h2‬‬

‫ﻫﻭ ﻤﺘﻭﺴﻁ ﻗﻴﻤﺔ ‪ k‬ﻟﺫﻟﻙ ﺍﻟﺤﺎﺠﺯ‪.‬‬ ‫‪u‬‬ ‫ﻫﻭ ﺍﻟﻌﺭﺽ‬ ‫‪∆x‬‬ ‫ﺤﻴﺙ‬

‫ﻻﺤﻅ ﺃﻥ ﻫﺫﺍ ﺍﻟﺘﻌﺒﻴﺭ ﻓﻴﻪ ﺘﻘﺭﻴﺏ ﻜﺒﻴﺭ ﻟﺴﺒﺒﻴﻥ ﺨﺎﺼﺔ ﻋﻨﺩ ﻨﻘﻁﺔ ﺍﻻﻨﻌﻁﺎﻑ‬
‫ﻫﻲ ﺩﺍﻟﺔ ﻤﺘﻐﻴﺭﺓ ﺒﺒﻁﺊ‬ ‫)‪V (x‬‬ ‫ﺤﻴﺙ ﺍﻟﻁﺎﻗﺔ ﻭﺍﻟﺠﻬﺩ ﻴﺘﺴﺎﻭﻴﺎﻥ‪ .‬ﻭﻤﻥ ﺍﻟﻤﻬﻡ ﺃﻥ ﻴﻜﻭﻥ‬
‫ﻓﻲ ‪ x‬ﻭﺇﻻ ﻓﺈﻥ ﺘﻘﺭﻴﺏ ﺍﻟﺠﻬﺩ ﺍﻟﻤﻨﺤﻨﻰ ﻤﺭﺒﻌﺎﺕ ﻤﺘﺭﺍﺼﺔ ﻴﻜﻭﻥ ﻤﻤﻜﻨﻨﺎﹰ ﻟﻭ ﻜﺎﻨـﺕ‬
‫ﻫﻭ ﺘﻘﺭﻴﺏ ﻏﻴﺭ‬ ‫‪T‬‬
‫‪2‬‬ ‫‪ 4ku ‬‬
‫‪~ ‬‬ ‫‪‬‬ ‫ﻫﺫﻩ ﺍﻟﻤﺭﺒﻌﺎﺕ ﻀﻴﻘﺔ ﺠﺩﺍﹰ ﻭﻋﻨﺩﻫﺎ ﻜﺫﻟﻙ ﺍﻟﻤﻌﺎﺩﻟﺔ‬
‫‪‬‬ ‫‪‬‬

‫ﺠﻴﺩ‪ .‬ﻭﻟﻜﻥ ﻤﻥ ﺍﻟﻤﻨﺎﺴﺏ ﻜﺘﺎﺒﺔ‪.‬‬


‫ﺤﻴﺙ ﺃﻥ ﺍﻟﺘﻜﺎﻤل ﻴﻜﻭﻥ ﻋﻠﻰ ﺍﻟﻤﻨﻁﻘﺔ ﺍﻟﺘﻲ ﻓﻴﻬﺎ ﻴﻜـﻭﻥ‬ ‫‪T‬‬
‫‪2‬‬
‫∫ ‪~e‬‬
‫‪−2‬‬ ‫)‪dx (2m/h 2 ) 2 (V (x) − E‬‬

‫ﺍﻟﺠﺫﺭ ﺍﻟﺘﺭﺒﻴﻌﻲ ﺤﻘﻴﻘﻲ‪.‬‬


‫ﺍﻟﺘﺄﺜﻴﺭ ﺍﻟﻨﻔﻘﻲ ‪Tunneling Phenomena‬‬

‫ﺘﻌﺘﺒﺭ ﻫﺫﻩ ﺍﻟﻅﺎﻫﺭﺓ ﺸﺎﺌﻌﺔ ﻓﻲ ﺍﻟﻔﻴﺯﻴﺎﺀ ﺍﻟﺫﺭﻴﺔ ﻭﺍﻟﻨﻭﻭﻴﺔ ﻭﺴـﻭﻑ ﻨﻨـﺎﻗﺵ‬


‫ﺃﻤﺜﻠﺔ‪.‬‬
‫ﺍﻻﻨﺒﻌﺎﺙ )ﺍﻻﻟﻜﺘﺭﻭﻨﻲ( ﺍﻟﺒﺎﺭﺩ ‪.Cold Emission‬‬

‫‪- ١٠٨ -‬‬


‫‪PDF created with pdfFactory Pro trial version www.pdffactory.com‬‬
‫ﺨﺫ ﺍﻟﻜﺘﺭﻭﻨﺎﺕ ﻓﻲ ﻤﻌﺩﻥ‪ .‬ﺇﻥ ﻫﺫﻩ ﺍﻻﻟﻜﺘﺭﻭﻨﺎﺕ ﻤﻘﻴﺩﺓ ﺩﺍﺨل ﺍﻟﻤﻌﺩﻥ ﺒﻭﺍﺴﻁﺔ‬
‫ﺠﻬﺩ ﻭﺍﻟﺫﻱ ﻜﺘﻘﺭﻴﺏ ﺃﻭﻟﻲ‪ ،‬ﻴﻤﻜﻥ ﻭﺼﻔﻪ ﺒﺼﻨﺩﻭﻕ ﺫﻭ ﻋﻤﻕ ﻤﺤﺩﺩ ﻜﻤﺎ ﻫﻭ ﻤﻭﻀﺢ‬
‫ﺒﺎﻟﺸﻜل‪.‬‬

‫ﻻﺤﻅ ﺃﻥ ﻤﺴﺘﻭﻴﺎﺕ ﺍﻟﻁﺎﻗﺔ ﻟﻼﻟﻜﺘﺭﻭﻨﺎﺕ ﻗﺭﻴﺒﺔ ﺠﺩﺍﹰ ﻤﻥ ﺒﻌﻀﻬﺎ ﻭﺫﻟـﻙ ﻷﻥ‬


‫ﺍﻟﺼﻨﺩﻭﻕ ﻋﺭﻴﺽ ﺠﺩﺍﹰ‪.‬‬
‫ﻤﻥ ﺨﻭﺍﺽ ﺍﻹﻟﻜﺘﺭﻭﻨﺎﺕ )ﻭﺍﻟﺘﻲ ﺘﻭﺼﻑ ﺒﻤﺒﺩﺃ ﺒﺎﻟﻭﻟﻲ( ﺃﻨﻬﺎ ﺘﺘﺭﺘـﺏ ﻓـﻲ‬
‫ﻤﺴﺘﻭﻴﺎﺕ ﺍﻟﻁﺎﻗﺔ ﺒﺤﻴﺙ ﺃﻨﻪ ﻻ ﻴﻭﺠﺩ ﺃﻜﺜﺭ ﻤﻥ ﺍﻟﻜﺘﺭﻭﻨﻴﻥ ﻴﻤﻜﻥ ﺃﻥ ﻴﺸﻐﻠﻬﺎ ﻤﺴﺘﻭﻯ‬
‫ﻭﺍﺤﺩ ﻤﻥ ﻤﺴﺘﻭﻴﺎﺕ ﺍﻟﻁﺎﻗﺔ‪ .‬ﻭﻫﻜﺫﺍ ﻓﺄﻗل ﺤﺎﻟﺔ ﻟﻠﻁﺎﻗﺔ ﻓﻲ ﺍﻟﻤﻌﺩﻥ‪ ،‬ﻜل ﺍﻟﻤﺴﺘﻭﻴﺎﺕ‬
‫ﺇﻟﻰ ﺤﺩ ﻤﺴﺘﻭﻯ ﻁﺎﻗﺔ ﻴﺴﻤﻰ ﻤﺴﺘﻭﻯ ﻁﺎﻗﺔ ﻓﻴﺭﺴﻲ )ﻭﺍﻟﺘﻲ ﺘﻌﺘﻤـﺩ ﻋﻠـﻰ ﻜﺜﺎﻓـﺔ‬
‫ﺍﻹﻟﻜﺘﺭﻭﻨﺎﺕ( ﺘﻜﻭﻥ ﻤﻌﺒﺄﺓ‪ .‬ﻭﻋﻨﺩ ﺩﺭﺠﺔ ﺤﺭﺍﺭﺓ ﺃﻋﻠﻰ ﻤﻥ ﺍﻟﺼﻔﺭ ﺍﻟﻤﻁﻠـﻕ ﻓـﺈﻥ‬
‫ﺒﻌﺽ ﻫﺫﻩ ﺍﻻﻟﻜﺘﺭﻭﻨﺎﺕ ﺘﻜﻭﻥ ﺤﺭﺍﺭﻴﺔ ﻤﻬﻴﺠﺔ ﻟﻤﺴﺘﻭﻴﺎﺕ‪ .‬ﺍﻟﻔﺭﻕ ﺒـﻴﻥ ﻤـﺴﺘﻭﻯ‬
‫ﻁﺎﻗﺔ ﻫﻭ ﺍﻟﺫﻱ ﻴﻠﺯﻡ ﻻﺤﻀﺎﺭ ﺍﻟﻜﺘﺭﻭﻥ ﻟﻠﺨﺎﺭﺝ‪ ،‬ﺩﺍﻟﺔ ﺍﻟﺸﻐل‪ .‬ﺍﻹﻟﻜﺘﺭﻭﻨﺎﺕ ﻴﻤﻜـﻥ‬
‫ﺇﺯﺍﻟﺘﻬﺎ ﺒﺈﻋﻁﺎﺌﻬﺎ ﺒﺎﻟﻔﻭﺘﻭﻨﺎﺕ )ﻜﻤﺎ ﻓﻲ ﺍﻟﺘﺄﺜﻴﺭ ﺍﻟﻜﻬﺭﻭﻀﻭﺌﻲ( ﻭﺇﻤﺎ ﺒﺎﻟﺤﺭﺍﺭﺓ‪ .‬ﻜﺫﻟﻙ‬
‫ﻴﻤﻜﻨﻨﺎ ﺇﺯﺍﻟﺘﻬﺎ ﺒﺎﻟﺘﺄﺜﻴﺭ ﻋﻠﻴﻬﺎ ﺒﻤﺠﺎل ﻜﻬﺭﺒﺎﺌﻲ ﺨﺎﺭﺠﻲ‪ .‬ﺇﻥ ﺍﻻﻨﺒﻌﺎﺙ ﺍﻹﻟﻜﺘﺭﻭﻨـﻲ‬
‫ﺍﻟﺒﺎﺭﺩ ﻴﺤﺩﺙ ﻨﺘﻴﺠﺔ ﺘﺄﺜﻴﺭ ﻤﺠﺎل ﻜﻬﺭﺒﺎﺌﻲ ﺨﺎﺭﺠﻲ ﻭﺍﻟﺫﻱ ﺒﺩﻭﺭﻩ ﻴﻐﻴﺭ ﺍﻟﺠﻬﺩ ﺍﻟﺫﻱ‬
‫ﻓﻲ ﻫﺫﻩ ﺍﻟﺤﺎﻟﺔ ﻓﺈﻥ ﻫﺫﺍ ﻫﻭ ﺍﻟﺠﻬﺩ‬ ‫‪ω −eε x ←W‬‬ ‫ﻴﺭﻯ ﺒﺎﻟﻜﺘﺭﻭﻨﺎﺕ ﺃﻱ ﻴﺘﻐﻴﺭ ﺍﻟﺠﻬﺩ‬
‫ﺍﻟﺫﻱ ﺘﺭﺍﻩ ﺍﻹﻟﻜﺘﺭﻭﻨﺎﺕ ﻭﻫﺫﺍ ﻴﻜـﺎﻓﺊ ﺍﻟﻔـﺭﻕ }‪) {V (x) − E‬ﺍﻟـﺸﻜل ‪ (b‬ﻓﻠـﻭ ﺃﻥ‬
‫ﺍﻻﻟﻜﺘﺭﻭﻥ ﻋﻨﺩ ﺍﻟﻘﻤﺔ ﻟﻠﺒﺤﺭ ﻤﻥ ﺍﻟﻤﺴﺘﻭﻴﺎﺕ ﻓﻲ ﻫﺫﻩ ﺍﻟﺤﺎﻟﺔ ﻓﺈﻥ ﻤﻌﺎﻤل ﺍﻟﻨﻔﺎﺫﻴﺔ‪.‬‬

‫‪- ١٠٩ -‬‬


‫‪PDF created with pdfFactory Pro trial version www.pdffactory.com‬‬
‫‪a‬‬
‫‪2m‬‬
‫‪2‬‬
‫‪−2‬‬ ‫∫‬ ‫‪dx‬‬
‫‪h2‬‬
‫) ‪( V−eε x‬‬
‫‪0‬‬
‫‪T ~e‬‬
‫‪3‬‬
‫‪1‬‬ ‫)‪(A + Bx‬‬ ‫‪2‬‬

‫∫‬ ‫)‪dx (A + Bx‬‬ ‫‪2‬‬


‫=‬
‫‪3‬‬
‫‪B‬‬
‫‪2‬‬
‫‪1‬‬ ‫‪1‬‬ ‫‪3‬‬
‫‪ 2m‬‬ ‫‪‬‬ ‫‪ 2m ‬‬ ‫‪(ω − eεεx‬‬
‫‪W/eε‬‬

‫‪]οw/eε‬‬
‫‪2‬‬ ‫‪2‬‬ ‫‪2‬‬

‫∫‬ ‫‪dx  2 ( w − eεεx‬‬


‫‪h‬‬ ‫‪‬‬
‫‪= 2 ‬‬
‫‪h ‬‬ ‫‪3‬‬
‫‪ο‬‬ ‫‪− eε‬‬
‫‪2‬‬
‫‪3‬‬
‫‪2m 2w 2‬‬
‫=‬
‫‪h‬‬ ‫‪3eε‬‬
‫‪4 2mw w‬‬
‫‪−‬‬ ‫) (‬
‫‪−2‬‬ ‫‪3 h2‬‬
‫‪⇒ T =e =e‬‬
‫‪2‬‬ ‫‪eε‬‬

‫ﻭﻫﺫﺍ ﻫﻭ ﻗﺎﻨﻭﻥ ‪Fowler – Nordheim‬‬

‫ﻭﻤﻥ ﺘﻁﺒﻴﻘﺎﺕ ﺍﻻﻨﺒﻌﺎﺙ ﺍﻟﺒﺎﺭﺩ ﻫﻭ ‪Scanning Tunneling Micros cepe‬‬


‫‪ STM‬ﻭﻫﺫﻩ ﺍﻷﺩﺍﺓ ﺘﻌﺘﻤﺩ ﻋﻠﻰ ﺍﻟﺤﺴﺎﺴﻴﺔ ﺍﻟﻌﺎﻟﻴﺔ ﺠﺩﺍﹰ ﻟﻠﻤﺴﺎﻓﺔ ﻓﻲ ﺍﻻﻨﺒﻌﺎﺙ ﺍﻟﺒﺎﺭﺩ‪.‬‬
‫ﺇﻥ ﺘﻴﺎﺭ ﺍﻹﻟﻜﺘﺭﻭﻨﺎﺕ ﺍﻟﻤﺤﺙ ﺒﺠﻬﺩ ﻜﻬﺭﺒﺎﺌﻲ ﺨﺎﺭﺠﻲ ﻟﻠﻔﺭﻕ ﻓﻲ ﺍﻟﺠﻬـﺩ ﺴـﻁﺢ‬
‫ﺍﻟﻤﻌﺩﻥ ﻭﻨﻬﺎﻴﺔ ﺇﺒﺭﺓ ﺤﺎﺩﺓ ﻓﻭﻕ ﺴﻁﺢ ﻴﺘﻐﻴﺭ ﺃﺴﻴﺎﹰ ﺒﺎﻟﻤﺴﺎﻓﺔ ﺒﻴﻥ ﺍﻹﺒـﺭﺓ ﻭﺍﻟـﺴﻁﺢ‬
‫ﻭﻫﺫﺍ ﻴﺴﻤﺢ ﺒﺎﻟﺤﺼﻭل ﻋﻠﻰ ﻤﻌﻠﻭﻤﺎﺕ ﻤﻨﻔﺼﻠﺔ ﻋﻥ ﺘﻭﺒﻭﺠﺭﺍﻓﻴﺔ ﺍﻟﺴﻁﺢ‬

‫ﻭﻫﺫﺍ ﻗﺩ ﺘﻡ ﺍﺴﺘﺨﺩﺍﻡ ‪ STM‬ﻟﺩﺭﺍﺴﺔ ﺃﺴﻁﺢ ﺍﻟﻤﻌﺎﺩﻥ ﻭﺒﻌﺽ ﺍﻟﻤﻭﺼﻼﺕ ﻭﻗﺩ‬


‫ﺘﻡ ﺤﺩﻴﺜﺎﹰ ﻨﻘل ﺫﺭﺍﺕ ﻭﺠﺯﻴﺌﺎﺕ ﻋﻠﻰ ﺍﻷﺴﻁﺢ ﻭﺫﻟﻙ ﺒﺘﺨﺯﻴﻨﻬﺎ‪.‬‬

‫‪- ١١٠ -‬‬


‫‪PDF created with pdfFactory Pro trial version www.pdffactory.com‬‬
‫ﺍﻟﺘﺄﺜﻴﺭ ﺍﻟﻨﻔﻘﻲ ﻓﻲ ﺍﻟﻤﻭﺍﺩ ﻓﺎﺌﻘﺔ ﺍﻟﺘﻭﺼﻴل ‪.Tunneling in superconductors‬‬

‫‪The Harmonic Oscillator‬‬ ‫ﺍﻟﻤﺘﺫﺒﺫﺏ ﺍﻟﺘﻭﺍﻓﻘﻲ ﺍﻟﺒﺴﻴﻁ‬

‫ﻓﻲ ﻫﺫﺍ ﺍﻟﻤﺜﺎل ﺴﻨﺠﺩ ﺃﻥ ﺍﻟﻤﻌﺎﺩﻟﺔ ﺍﻟﺘﻔﺎﻀﻠﻴﺔ ﺍﻟﻤﻁﻠﻭﺏ ﺤﻠﻬﺎ ﺘﺨﺘﻠـﻑ ﻋـﻥ‬
‫ﺍﻷﻤﺜﻠﺔ ﺍﻟﺴﺎﺒﻘﺔ ﻭﺃﺤﺩ ﺃﺴﺒﺎﺏ ﻤﻨﺎﻗﺸﺔ ﻫﺫﻩ ﺍﻟﻤﺴﺄﻟﺔ ﻫﻭ ﺘﻌﻠﻡ ﺤل ﻤﺜل ﻫﺫﻩ ﺍﻟﻤﺴﺎﺌل‪.‬‬
‫ﺍﻟﻬﺎﻤﻠﺘﻭﻨﻴﺎﻥ ﺍﻟﺘﻘﻠﻴﺩﻱ ﻟﻪ ﺍﻟﺸﻜل‪.‬‬
‫‪p2 1 2‬‬
‫=‪H‬‬ ‫‪+ kx‬‬ ‫)‪(1‬‬
‫‪2m 2‬‬
‫‪= k. E = p.E‬‬

‫ﻭﻋﻠﻴﻪ ﻓﺈﻥ ﻤﻌﺎﺩﻟﺔ ﺍﻟﻘﻴﻤﺔ ﺍﻟﺨﺎﺼﺔ‪.‬‬


‫‪h 2 d 2 u (x) 1‬‬
‫‪2‬‬
‫‪+ k x h 2 (x) E.u‬‬ ‫)‪(2‬‬
‫‪2m dx‬‬ ‫‪2‬‬

‫= ‪k = mω 2 ⇐ ω‬‬
‫‪k‬‬
‫)‪ (3‬ﺒﺘﻌﺭﻴﻑ ﺘﺭﺩﺩ ﺍﻟﻤﺘﺫﺒﺫﺏ ﻭﻜﺘﺎﺒﺔ ﺍﻟﻁﺎﻗﺔ ﻋﻠﻰ ﺍﻟﺸﻜل‬
‫‪m‬‬
‫‪2E‬‬
‫=∈‬ ‫)‪(4‬‬
‫‪hω‬‬

‫= ‪y‬‬
‫‪mω‬‬
‫‪x‬‬ ‫)‪ (5‬ﻭﺘﻐﻴﻴﺭ ﺍﻟﻤﺘﻐﻴﺭﺍﺕ‬
‫‪h‬‬
‫‪d 2 u ( x) m2ω 2 2‬‬ ‫‪2mE‬‬
‫‪2‬‬
‫‪−‬‬ ‫‪2‬‬
‫‪x u + 2 u=0‬‬
‫‪dx‬‬ ‫‪h‬‬ ‫‪h‬‬
‫‪‬‬ ‫‪‬‬
‫‪‬‬ ‫‪‬‬
‫‪d u ( x) mω  mω 2 2 E ‬‬
‫‪2‬‬
‫‪−‬‬ ‫‪x‬‬ ‫‪u =0‬‬
‫‪dx 2‬‬ ‫‪h  12‬‬ ‫{ ‪h3‬‬ ‫‪hω ‬‬
‫‪ 2‬‬ ‫‪← ‬‬
‫‪ y‬‬ ‫‪‬‬
‫‪2‬‬ ‫‪2‬‬
‫‪1 d u‬‬ ‫‪d u‬‬
‫‪+ (∈ − y 2 ) u = 0 ⇒ 2 + (∈− y 2 ) u = 0‬‬ ‫)‪(6‬‬
‫‪mω dx‬‬ ‫‪2‬‬
‫‪dy‬‬
‫‪h‬‬

‫ﻜل ﺍﻟﻜﻤﻴﺎﺕ ﻫﻨﺎ ﺒﺩﻭﻥ ﻭﺤﺩﺍﺕ‪.‬‬


‫ﺤﻠﻭل ﻫﺫﻩ ﺍﻟﻤﻌﺎﺩﻟﺔ ﻫﻲ ﺍﻟﺩﻭﺍل ﺍﻟﺫﺍﺌﺒﺔ‪.‬‬
‫‪− y3‬‬
‫‪2‬‬
‫‪e‬‬
‫= ‪un‬‬ ‫)‪H n ( y‬‬
‫‪2n n D‬‬

‫‪- ١١١ -‬‬


‫‪PDF created with pdfFactory Pro trial version www.pdffactory.com‬‬
‫ﺘﻌﺭﻑ ﺒﻜﺜﻴﺭﺍﺕ ﻤﺤﺩﻭﺩﺓ‪.‬‬ ‫)‪H n ( y‬‬ ‫ﺤﻴﺙ‬
‫‪H 0 (y) = 1‬‬
‫‪H1 (y) = 2 y‬‬
‫‪H 2 (y) = 4 y 2 − 2‬‬
‫‪H 3 (y) = 8 y 2‬‬

‫ﺍﻟﻨﻅﺭ ﻟﻠﺤﻠﻭل ﻋﻨﺩ ﻤﺎ ﻻ ﻨﻬﺎﻴﺔ )ﺃﻭﻻﹰ( ﺜﻡ ﻋﻨﺩ ﺍﻟﺼﻔﺭ‪.‬‬


‫ﻓﺈﻥ ﺍﻟﺤﺩ ﺍﻟـﺫﻱ‬ ‫∞ → ‪y2‬‬ ‫ﻭﻷﻱ ﻗﻴﻤﺔ ﺨﺎﺼﺔ )ﺫﺍﺘﻴﺔ( ‪ ،E. eijenvalne‬ﻋﻨﺩﻤﺎ‬
‫ﻴﺘﻀﻤﻥ ‪ E‬ﻴﻌﺒﺭ ﻤﻬﻤﻭل‪ .‬ﻭﻟﻬﺫﺍ ﺍﻟﺴﺒﺏ ﻴﺘﻁﻠﺏ ﺃﻥ )‪ asymptotically u (y‬ﺘﺤﻘـﻕ‬
‫ﺍﻟﻤﻌﺎﺩﻟﺔ‪.‬‬
‫)‪d 2 u ο (y‬‬
‫‪− y 2 u ο (y) = 0‬‬ ‫)‪(7‬‬
‫‪d y2‬‬

‫ﻴﻤﻜﻨﻨﺎ ﻤﻥ ﻜﺘﺎﺒﺔ ﺍﻟﻤﻌﺎﺩﻟﺔ ﺒﺎﻟﺸﻜل‪.‬‬ ‫ﺒﺎﻟﻀﺭﺏ ﻓﻲ‬


‫‪du ο‬‬
‫‪2‬‬
‫‪dy‬‬

‫‪d duο 2‬‬ ‫‪d‬‬


‫‪) − y2‬‬ ‫‪(u 0 ) = 0‬‬
‫‪2‬‬
‫(‬ ‫)‪(8‬‬
‫‪dy dy‬‬ ‫‪dy‬‬

‫ﺃﻭ ﻤﺎ ﻴﻜﺎﻓﺅﻫﺎ‪.‬‬
‫‪d  d uο 2‬‬ ‫‪2‬‬
‫‪) − y 2u 0  = − 2 y2u 0‬‬
‫‪2‬‬
‫‪‬‬ ‫(‬ ‫)‪(9‬‬
‫‪dy  dy‬‬ ‫‪‬‬

‫ﻭﻟﻭ ﺃﻫﻤﻠﻨﺎ ﺍﻟﺤﺩ ﻋﻠﻰ ﺍﻟﺠﺎﻨﺏ ﺍﻷﻴﻤﻥ ﻤﻥ ﺍﻟﻤﻌﺎﺩﻟﺔ ﻓﺈﻨﻪ ﺴﻴﺴﺭ ﺍﻟﺤل ﻭﺒﺘﻜﺎﻤل‬
‫ﺍﻟﻤﻌﺎﺩﻟﺔ‪.‬‬
‫‪d  d uο 2‬‬ ‫‪2‬‬ ‫‪du‬‬
‫‪) − y2u0  = 0⇒ ( ο )2 − y2u0 = C‬‬
‫‪2‬‬
‫(‪‬‬
‫‪dy  dy‬‬ ‫‪‬‬ ‫‪dy‬‬
‫‪d uο‬‬ ‫‪1‬‬
‫) ‪= ( C + y 2 uο 2‬‬ ‫‪2‬‬
‫‪dy‬‬

‫ﻴﺠﺏ ﺃﻥ ﺘﺘﻼﺸﻰ ﻋﻥ ∞ ‪ ،‬ﻓﺈﻥ ﻫﺫﺍ ﻴﺤـﺘﻡ ﺃﻥ‬ ‫ﺒﻤﺎ ﺃﻥ ﻜﻼﹰ ﻤﻥ‬


‫‪d uο‬‬
‫)‪, uο ( y‬‬
‫‪dy‬‬

‫‪ C = 0‬ﻭﻫﻜﺫﺍ‪.‬‬

‫‪- ١١٢ -‬‬


‫‪PDF created with pdfFactory Pro trial version www.pdffactory.com‬‬
‫‪d uο‬‬ ‫‪d uο‬‬
‫‪dy‬‬
‫‪= ± yuο‬‬ ‫∫‬ ‫‪uο‬‬
‫‪= ± ∫ y dy‬‬ ‫)‪(10‬‬

‫ﺍﻟﺤﻠﻭل ﺍﻟﻤﺘﺒﺩﻟﺔ ﻋﻨﺩ ∞ ‪ ،‬ﻫﻲ‪.‬‬


‫‪−y2‬‬
‫‪U ο (y) = e‬‬ ‫‪2‬‬

‫ﻜﻤﻴﺔ ﻤﻬﻤﻠﺔ ﻤﻘﺎﺭﻨﺔ ﺒـ‬ ‫‪2y U ο2‬‬ ‫‪= 2ye − y‬‬


‫‪2‬‬
‫ﺒﺈﻤﻜﺎﻨﻨﺎ ﺍﻵﻥ ﺍﺨﺘﺒﺎﺭ ﺃﻥ‬
‫‪d‬‬ ‫‪d 2 − y2‬‬ ‫‪2‬‬
‫) ‪(y 2 U 2ο‬‬ ‫=‬ ‫‪(y e ) ~ − 4 y 3 e − y‬‬
‫‪dy‬‬ ‫‪dy‬‬

‫ﻭﺫﻟﻙ ﺒﺎﻋﺘﺒﺎﺭ ﻗﻴﻤﺔ ‪ y‬ﻜﺒﻴﺭﺓ‪.‬‬


‫ﺒﺤﻴﺙ ﺃﻥ‬ ‫)‪h (y‬‬ ‫ﺍﻵﻥ ﺴﻨﺴﺘﺤﺩﺙ ﺩﺍﻟﺔ ﺠﺩﻴﺩﺓ‬
‫‪2‬‬
‫‪U ( y ) = h (y) e − y‬‬
‫‪2‬‬
‫)‪.......... ........(1 2‬‬

‫ﻭﻋﻠﻴﻪ ﻓﺈﻥ ﺍﻟﻤﻌﺎﺩﻟﺔ ﺍﻟﺘﻔﺎﻀﻠﻴﺔ ﺘﺼﺒﺢ‪.‬‬


‫)‪d 2 h (y‬‬ ‫)‪dh (y‬‬
‫‪2‬‬
‫‪− 2y‬‬ ‫‪+ (∈ − 1) h (y) = 0‬‬ ‫)‪.............(13‬‬
‫‪dy‬‬ ‫‪dy‬‬

‫ﻭﺸﻜل ﺍﻟﻤﻌﺎﺩﻟﺔ ﻻ ﻴﺒﺩﻭ ﺒﺴﻴﻁﺎﹰ‪ ،‬ﻭﻟﻜﻨﻨﺎ ﻨﺎﻗﺸﻨﺎ ﺍﻟﺴﻠﻭﻙ ﻋﻨﺩ ﻤـﺎ ﻻ ﻨﻬﺎﻴـﺔ‪،‬‬
‫ﻭﺍﻵﻥ ﻨﺭﻴﺩ ﺃﻥ ﻨﻨﻅﺭ ﻟﻠﺴﻠﻭﻙ ﻋﻨﺩ ‪ .y = 0‬ﺩﻋﻨﺎ ﻨﺤﺎﻭل ﻜﺘﺎﺒﺔ ﺍﻟﺤل ﻋﻠـﻰ ﺸـﻜل‬
‫ﻤﻔﻜﻭﻙ ﺴﻠﺴﻠﺔ ﺍﻟﻘﻭﺓ‪.Power series expansion .‬‬
‫∞‬
‫‪h (y ) = ∑ a m y m‬‬ ‫)‪..................(14‬‬
‫‪m =0‬‬

‫∞‬ ‫∞‬ ‫‪m−1‬‬ ‫∞‬

‫∑‬
‫‪m =0‬‬
‫‪m (m - 1) a m y‬‬ ‫‪m-2‬‬
‫‪− 2y∑ m a m y‬‬
‫‪m =0‬‬
‫)‪+ (∈ −1‬‬ ‫∑‬
‫‪m =0‬‬
‫‪a m ym = 0‬‬
‫‪14‬‬ ‫‪4244‬‬ ‫‪3‬‬
‫∞‬ ‫‪m‬‬
‫‪∑ m am y‬‬
‫‪m =0‬‬

‫ﻓﻘﻁ‪.‬‬ ‫‪ym‬‬ ‫ﺒﺄﺨﺫ ﻤﻌﺎﻤﻼﺕ‬

‫‪(m + 2) (m + 1) a m+ 2 − 2 (m + 1) a m + (∈ −1) a m = 0‬‬

‫‪(m + 2) (m + 1) a m+ 2 = ( 2m -∈ +1) a m‬‬ ‫)‪.......... ..... (15‬‬

‫‪- ١١٣ -‬‬


‫‪PDF created with pdfFactory Pro trial version www.pdffactory.com‬‬
‫‪.a‬‬ ‫‪1‬‬
‫‪, a0‬‬ ‫ﺇﺫﺍ ﻋﺭﻓﻨﺎ‬ ‫‪am‬‬ ‫ﻭﻫﺫﻩ ﻫﻲ ﺍﻟﻤﻌﺎﺩﻟﺔ ﺍﻻﺴﺘﻁﺭﺍﺩ ﻟﻠﺤﺼﻭل ﻋﻠﻰ‬

‫) ‪(m > N‬‬ ‫ﻭﻟﻘﻴﻡ ﺍﺨﺘﻴﺎﺭﻴﺔ ∈‪ ،‬ﻨﺠﺩ ﺃﻥ ﻋﻨﺩﻤﺎ ﺘﻜﻭﻥ ‪ m‬ﻜﺒﻴﺭﺓ‬


‫‪2‬‬
‫~ ‪am + 2‬‬ ‫‪am‬‬
‫‪m‬‬

‫ﻭﻫﺫﺍ ﻴﻌﻨﻲ ﺃﻥ ﺍﻟﺤﻠﻭل ﺘﻘﺭﻴﺒﺎﹰ )ﺒﺎﺨﺘﻴﺎﺭ ﺍﻟﺤﻠﻭل ﺍﻟﺯﻭﺠﻴﺔ(‪ .‬ﻭﻫﺫﺍ‬

‫)‪h ( y ) = (atolyromialiny‬‬

‫‪‬‬ ‫‪2‬‬ ‫‪22‬‬ ‫‪23‬‬ ‫‪‬‬


‫‪+ a N  y N + y N+2 +‬‬ ‫‪yN+4 +‬‬ ‫‪y N + 6 + .......... ‬‬
‫‪‬‬ ‫‪N‬‬ ‫) ‪N (N + 2‬‬ ‫) ‪N (N + 2 ) (N + 4‬‬ ‫‪‬‬

‫ﻭﺒﺎﻟﺘﺒﺴﻴﻁ‬
‫‪‬‬ ‫‪N‬‬
‫‪−1‬‬ ‫‪N‬‬
‫‪N‬‬
‫‪+1‬‬
‫‪‬‬
‫‪N‬‬ ‫‪‬‬ ‫‪(y‬‬ ‫‪2 2‬‬
‫)‬ ‫‪(y‬‬ ‫‪2‬‬
‫)‬ ‫‪2‬‬
‫‪(y‬‬ ‫‪2 2‬‬
‫)‬ ‫‪‬‬
‫‪a N y 2 ( −1 )! ‬‬ ‫‪+‬‬ ‫‪+‬‬ ‫‪+ .......... ‬‬
‫‪2‬‬ ‫! )‪ ( N − 1‬‬ ‫‪N‬‬
‫!) (‬
‫‪N‬‬
‫!)‪( + 1‬‬ ‫‪‬‬
‫‪ 2‬‬ ‫‪2‬‬ ‫‪2‬‬ ‫‪‬‬

‫)ﻟﻠﻤﻼﺌﻤﺔ( ﻓﺈﻥ ﺍﻟﻤﺘﺴﻠﺴﻠﺔ ﺘﺄﺨﺫ ﺍﻟﺸﻜل‪.‬‬ ‫‪N = 2k‬‬ ‫ﺒﺎﺨﺘﻴﺎﺭ‬


‫‪ (y 2 ) k − 1 (y 2 ) k‬‬ ‫‪(y 2 ) k + 1‬‬ ‫‪‬‬
‫‪y 2 (k − 1 )! ‬‬ ‫‪+‬‬ ‫‪+‬‬ ‫‪+ .......... ‬‬
‫! )‪ (k − 1‬‬ ‫!‪k‬‬ ‫! )‪(k + 1‬‬ ‫‪‬‬

‫‪- ١١٤ -‬‬


‫‪PDF created with pdfFactory Pro trial version www.pdffactory.com‬‬
‫‪ 2‬‬ ‫‪(y 2 ) 2‬‬ ‫‪(y 2 ) k −1 ‬‬
‫‪= y 2 (k − 1) !  e y − {1 + y 2 +‬‬ ‫‪+ .................. +‬‬ ‫‪‬‬
‫‪‬‬ ‫!‪2‬‬ ‫‪(k − 2)! ‬‬

‫‪y2 ey‬‬ ‫ﻭﻫﺫﺍ ﺸﻜل ﻤﺘﻌﺩﺩ ﺍﻟﺤﺩﻭﺩ ‪ +‬ﺜﺎﺒﺕ ‪x‬‬

‫‪4 (y ) = h (y ) e‬‬
‫‪−y2‬‬
‫‪2‬‬
‫ﻭﺒﺎﻟﺘﻌﻭﻴﺽ ﻫﺫﻩ ﺍﻟﻤﻌﺎﺩﻟﺔ ﻓﻲ ﺍﻟﻤﻌﺎﺩﻟﺔ‬
‫ﻭﺍﻟﺤل ﺍﻟﻤﻘﺒﻭل ﻴﻤﻜﻥ ﺃﻥ ﻴﻭﺠﺩ‬ ‫∞‬ ‫ﻓﺈﻨﻨﺎ ﺴﻨﺘﺤﺼل ﻋﻠﻰ ﺤل ﻻ ﻴﺘﻼﺸﻰ ﻋﻨﺩ‬
‫ﻟﻭ ﺃﻥ ﻤﻌﺎﺩﻟﺔ ﺍﻻﺴﺘﻁﺭﺍﺩ ﺘﻨﺘﻬﻲ‪ ،‬ﺃﻱ ﺃﻥ ﻟﻭ ﺃﻥ‪.‬‬
‫‪( N + 1 ) ( N + 2 ) a n + 2 = 2N − ∈+ 1 a n = 0‬‬
‫‪1424‬‬ ‫‪3‬‬
‫‪=0‬‬

‫‪⇒ ∈ = 2N + 1‬‬ ‫)‪(17‬‬


‫‪2E‬‬
‫‪= 2N + 1‬‬
‫‪hw‬‬

‫ﻭﺍﻟﻨﺘﺎﺌﺞ ﻫﻲ‪:‬‬

‫‪ -1‬ﺃﻨﻪ ﻴﻭﺠﺩ ﻗﻴﻡ ﺫﺍﺘﻴﺔ ﻤﻨﻔﺼﻠﺔ )ﻏﻴﺭ ﻤﺘﺼﻠﺔ( ﻤﺘﺴﺎﻭﻴﺔ ﺍﻟﻤﺴﺎﻓﺎﺕ ﻤﻥ ﺍﻟﻤﻌﺎﺩﻟـﺔ‬
‫)‪ (4‬ﻭﺍﻟﻤﻌﺎﺩﻟﺔ )‪ (17‬ﻴﻤﻜﻨﻨﺎ ﺍﻟﺤﺼﻭل ﻋﻠﻰ ﺍﻟﻤﻌﺎﺩﻟﺔ ﺍﻟﺘﺎﻟﻴﺔ‪.‬‬
‫‪1‬‬ ‫‪1‬‬
‫=‪E‬‬ ‫‪hw (2m + 1) = (n + ) h w‬‬ ‫)‪(18‬‬
‫‪2‬‬ ‫‪2‬‬

‫ﻭﻫﺫﻩ ﻗﺭﻴﺒﺔ ﻤﻥ ﻤﻌﺎﺩﻟﺔ ‪ Planck‬ﻟﻺﺸﻌﺎﻉ‪.‬‬


‫‪1‬‬
‫‪E = (n + ) hw‬‬
‫‪2‬‬

‫ﺍﻟﻬﻴﺭﻤﻴﺘﻲ)‪Hn (y‬‬ ‫‪ -2‬ﻤﺎ ﻋﺩﺍ ﺜﻭﺍﺒﺕ ﺍﻟﻤﻌﺎﻴﺭﺓ‪ ،‬ﻓﺈﻥ ﻤﺘﻌﺩﺩ ﺍﻟﺤﺩﻭﺩ ﻫﻭ ﻤﺘﻌﺩﺩ ﺍﻟﺤﺩﻭﺩ‬
‫ﻭﺍﻟﺫﻱ ﻴﻤﻜﻥ ﺇﻴﺠﺎﺩ ﺨﻭﺍﺼﻪ ﺒﺴﻬﻭﻟﺔ ﻓﻲ ﺃﻱ ﻜﺘﺎﺏ ﻓﻲ ﺍﻟﻔﻴﺯﻴﺎﺀ ﺍﻟﺭﻴﺎﻀﻴﺔ‪.‬‬
‫ﻭﻟﻭ ﻗﺼﺭﻨﺎ ﺃﻨﻔﺴﻨﺎ ﻟﻠﺨﻭﺍﺹ ﺍﻟﺘﺎﻟﻴﺔ ﻤﻨﻬﺎ‪:‬‬

‫ﺘﺤﻘﻕ ﺍﻟﻤﻌﺎﺩﻟﺔ ﺍﻟﺘﻔﺎﻀﻠﻴﺔ‪.‬‬ ‫)‪Hn (y‬‬

‫‪∈ = 2 n +1‬‬ ‫ﺘﺤﺕ ﺸﺭﻁ‬


‫) ‪d 2 H n (y‬‬ ‫)‪dH n (y‬‬
‫‪2‬‬
‫‪− 2y‬‬ ‫‪+ 2nH n (y) = 0‬‬
‫‪dy‬‬ ‫‪dy‬‬

‫‪- ١١٥ -‬‬


‫‪PDF created with pdfFactory Pro trial version www.pdffactory.com‬‬
ecussionedations ‫ﻭﻫﺫﻩ ﺘﺤﻘﻕ ﺍﻟﻌﻼﻗﺎﺕ ﺍﻻﺴﺘﻁﺭﺍﺩﻴﺔ ﺍﻟﺘﺎﻟﻴﺔ‬
H n +1 − 2y H n + 2n H n −1 = 0
dH n
H n +1 + − 2y H n + 2n H n = 0
dy

∑ H n (y)
Zn
= e 2zy − z
2
‫ﻭﻜﺫﻟﻙ‬
n n!

H n (y) = (−1) n e y
2 d n − y2
e ‫ﻭ‬
dy n

‫ﻭﺍﻟﺘﻁﺒﻴﻊ ﻟﻜﺜﻴﺭﺍﺕ ﺤﺩﻭﺩ ﻫﻴﺭﻤﻴﺕ ﻴﻜﻭﻥ ﺒﺤﻴﺙ‬



2
dy e − y H n (y) 2 = d n n! D
−∞

.‫ﻫﺫﻩ ﻗﺎﺌﻤﺔ ﺒﺒﻌﺽ ﻋﺩﻴﺩﺍﺕ ﺤﺩﻭﺩ ﻫﻴﺭﻤﻴﺕ‬


H 0 (y) = 1
H1 (y) = 2 y
H 2 (y) = 4 y 2 − 2
H 3 (y) = 8 y 2 − 12 y
H 4 (y) = 16 y 4 − 48 y 2 + 12
H 5 (y) = 32 y 5 − 160 y 3 + 129 y

(‫ﻭﻤﻌﺎﺩﻻﺕ ﺍﻟﻘﻴﻡ ﺍﻟﺫﺍﺘﻴﺔ )ﺍﻟﺨﺎﺼﺔ‬


d2u n mk 2 2mEn
2
= 2
x un − un
dx h h2
‫و‬
d2u* e mk 2mEn *
2
= 2 x 2 u* e − u e
dx h h2

‫ ﺍﻟﻤﻌﺎﺩﻟﺔ ﺍﻟﺜﺎﻨﻴﺔ‬،‫ﺒﺎﻟﻤﻌﺎﺩﻟﺘﻴﻥ ﺍﻟﺴﺎﺒﻘﺘﻴﻥ ﻋﻠﻰ ﺍﻟﺘﻭﺍﻟﻲ‬ un ‫ﻭ‬ u*e ‫ﻭﻋﻨﺩ ﻀﺭﺏ‬


.‫ﺘﻁﺭﺡ ﻤﻥ ﺍﻷﻭﻟﻰ‬
d  * dun du x  2m
*

ux − u x  = 2 (E x − E n ) u *x u n
dx  dx dx  h

- ١١٦ -
PDF created with pdfFactory Pro trial version www.pdffactory.com
‫ﻓﺈﻥ ﺍﻟﺠﺎﻨـﺏ ﺍﻷﻴـﺴﺭ‬ ‫∞‪+‬‬ ‫←‬ ‫∞‪−‬‬ ‫ﻭﺒﺘﻜﺎﻤل ﻫﺫﻩ ﺍﻟﻤﻌﺎﺩﻟﺔ ﻋﻠﻰ ‪ x‬ﻤﻥ‬
‫ﻭﻫﻜﺫﺍ‪.‬‬ ‫∞ ‪x= ±‬‬ ‫ﻴﺘﻼﺸﻰ‪ ،‬ﺒﻤﺎ ﺃﻥ ﺍﻟﻤﻌﺎﺩﻻﺕ ﺍﻟﺨﺎﺼﺔ ﻭﺘﻔﺎﻀﻼﺘﻬﺎ ﺘﺘﻼﺸﻰ ﻋﻨﺩ‬
‫∞‬
‫‪(E‬‬ ‫‪x‬‬
‫) ‪− En‬‬ ‫∫‬ ‫‪dx u *e (x) u n (x) = 0‬‬
‫∞‪−‬‬

‫ﻫﻲ ﻤﺘﻌﺎﻤﺩﺓ‪.‬‬ ‫‪Ee ± En‬‬ ‫ﻭﺍﻟﺫﻱ ﻴﻌﻨﻲ ﺃﻥ ﺍﻟﺩﻭﺍل ﺍﻟﺨﺎﺼﺔ ﺍﻟﺘﻲ ﻟﻬﺎ‬
‫ﺘﻭﻀﺢ ﺃﻥ ﺃﻗل ﺤﺎﻟﺔ ﻟﻬﺎ ﻁﺎﻗﺔ ﻭﻫﺫﻩ ﻫﻲ ﻁﺎﻗﺔ ﻨﻘﻁـﺔ‬ ‫‪1‬‬
‫) ‪E = h w (n +‬‬ ‫‪ -3‬ﺍﻟﻤﻌﺎﺩﻟﺔ‬
‫‪2‬‬

‫ﺍﻟﺼﻔﺭ ‪.Zero – point evergy‬‬


‫ﺇﻥ ﻭﺠﻭﺩﻫﺎ ﻫﻭ ﺘﺄﺜﻴﺭ ﻜﻤﻲ ﺒﺤﺕ‪ ،‬ﻭﻴﻤﻜﻥ ﺘﺄﻭﻴﻠﻪ ﺒﺩﻻﻟﺔ ﻤﺒﺩﺃ ﺍﻟﻼﺘﻌﻴﻴﻥ‪ .‬ﺇﻨﻬﺎ‬
‫ﻁﺎﻗﺔ ﻨﻘﻁﺔ ﺍﻟﺼﻔﺭ ﺍﻟﻤﺴﺌﻭﻟﺔ ﻋﻥ ﺍﻟﺤﻘﻴﻘﺔ ﺃﻥ ﺍﻟﻬﻴﻠﻴﻭﻡ ﻻ ﻴﺘﺠﻤـﺩ ﻋﻨـﺩ ﺩﺭﺠـﺎﺕ‬
‫ﺍﻟﺤﺭﺍﺭﺓ ﺍﻟﻘﻠﻴﻠﺔ‪ ،‬ﻭﻟﻜﻨﻪ ﻴﺒﻘﻰ ﻓﻲ ﺍﻟﺤﺎﻟﺔ ﺍﻟﺴﺎﺌﻠﺔ ﺤﺘﻰ ﺩﺭﺠﺎﺕ ﺤـﺭﺍﺭﺓ ﻤﻘـﺩﺍﺭﻫﺎ‬
‫‪ 10-3k‬ﻭﺫﻟﻙ ﻋﻨﺩ ﺍﻟﻀﻐﻁ ﺍﻟﻌﺎﺩﻱ‪ .‬ﺍﻟﺘﺭﺩﺩ ﻟﻬﺎ ﻴﻜﻭﻥ ﻜﺒﻴﺭﺍﹰ ﻟﻠﺫﺭﺍﺕ ﺍﻟﺨﻔﻴﻔﺔ‪ .‬ﻭﺍﻟﺫﻱ‬
‫ﻫﻭ ﺍﻟﺴﺒﺏ ﻓﻲ ﺃﻥ ﺍﻟﻨﻴﺘﺭﻭﺠﻴﻥ ﻻ ﻴﻅﻬﺭ ﻫﺫﺍ ﺍﻟﺘﺄﺜﻴﺭ ﻤﺜل ﺍﻟﻬﻴﻠﻴﻭﻡ‪ .‬ﺇﻨﻪ ﻜﺫﻟﻙ ﻴﻌﺘﻤﺩ‬
‫ﻋﻠﻰ ﺍﻟﺨﻭﺍﺹ ﺍﻟﺘﻔﻀﻴﻠﻴﺔ ﻟﻠﻘﻭﻯ ﺍﻟﻔﺎﻋﻠﺔ ﺒﻴﻥ ﺍﻟﺠﺯﻴﺌﺎﺕ ﻭﺍﻟﺘﻲ ﺘﺸﺭﺡ ﺍﻟﺴﺒﺏ ﻓﻲ ﺃﻥ‬
‫ﺍﻟﻬﻴﺩﺭﻭﺠﻴﻥ ﻴﺘﺠﻤﺩ‪.‬‬

‫‪- ١١٧ -‬‬


‫‪PDF created with pdfFactory Pro trial version www.pdffactory.com‬‬
‫ﺍﻟﺒﻨﺎﺀ ﺍﻟﻌﺎﻡ ﻟﻤﻴﻜﺎﻨﻴﻜﺎ ﺍﻟﻜﻡ )ﺍﻟﻤﻴﻜﺎﻨﻴﻜﺎ ﺍﻟﻤﻭﺠﻴﺔ(‪:‬‬

‫ﺴﻨﺘﻌﺭﺽ ﻓﻲ ﻫﺫﺍ ﺍﻟﻔﺼل ﻟﺩﺭﺍﺴﺔ ﺍﻟﻤﺒﺎﺩﺉ ﺍﻷﺴﺎﺴﻴﺔ ﺍﻟﺘﻲ ﺫﻜﺭﺕ ﺁﻨﻔﺎﹰ ﻓـﻲ‬
‫ﺴﻴﺎﻕ ﺤل ﺒﻌﺽ ﺍﻟﻤﺴﺎﺌل ﺍﻟﻔﻴﺯﻴﺎﺌﻴﺔ ﻤﻥ ﻫﺫﻩ ﻨﻅﺭﻴﺔ ﺍﻟﻤﻔﻜﻭﻙ ﻭﻤﻌﻨﺎﻫﺎ ﺍﻟﻔﻴﺯﻴـﺎﺌﻲ‬
‫ﻓﻲ ﺍﻟﻤﺅﺜﺭﺍﺕ ﻤﺘﺠﻬﺎﺕ ﺍﻟﺤﺎﻟﺔ‪ ،‬ﺍﻟﺘﻜﺭﺍﺭ ﻭﺍﻟﻨﻬﺎﻴﺔ ﺍﻟﺘﻘﻠﻴﺩﻴﺔ‪ .‬ﻭﺴﻨﻘﺩﻡ ﺘﺭﻤﻴﺯ ﺩﻴـﺭﺍﻙ‬
‫ﺒﺎﺨﺘﺼﺎﺭ‪.‬‬
‫ﺍﻟﺩﻭﺍل ﺍﻟﺨﺎﺼﺔ ﻭﺍﻟﻘﻴﻡ ﺍﻟﺨﺎﺼﺔ‪:‬‬

‫"ﻤﺅﺜﺭ ﺍﻟﻬﺎﻤﻠﺘﻭﻨﻴﺎﻥ"‬

‫ﺇﻥ ﺤﺎﻟﺔ ﺃﻱ ﻨﻅﺎﻡ ﻓﻴﺯﻴﺎﺌﻲ ﻴﻭﺼﻑ ﺒﺎﻟﺩﺍﻟﺔ ﺍﻟﻤﻭﺠﻴﺔ ﻭﺍﻟﺘﻲ ﺘﺤﺘﻭﻱ ﻋﻠﻰ ﻜل‬
‫ﺍﻟﻤﻌﻠﻭﻤﺎﺕ ﻋﻥ ﻫﺫﺍ ﺍﻟﻨﻅﺎﻡ‪ .‬ﺇﻥ ﺍﻟﺩﺍﻟﺔ ﺍﻟﻤﻭﺠﻴﺔ ﺘﻌﺘﻤﺩ ﻋﻠﻰ ﺍﻟﺯﻤﻥ‪ ،‬ﻭﺘﻁﻭﺭﻫﺎ ﻤـﻊ‬
‫ﺍﻟﺯﻤﻥ ﻴﻌﻁﻲ ﺒـ‪:‬‬
‫∂‬
‫‪ih‬‬ ‫)‪ψ (x, t) = H ψ (x, t‬‬ ‫)‪(1‬‬
‫‪xt‬‬

‫ﺇﻥ ﺍﻟﺩﺍﻟﺔ ﺍﻟﻤﻭﺠﻴﺔ ﺁﺜﺭﻨﺎ ﻋﻠﻴﺎ ﺒﻤﺅﺜﺭ ‪ ،H‬ﺍﻟﻬﺎﻤﻠﺘﻭﻨﻴﺎﻥ ﻭﺍﻟـﺫﻱ ﻴﻠﻌـﺏ ﺩﻭﺭﺍﹰ‬
‫ﻜﺒﻴﺭﺍﹰ ﻓﻲ ﻤﻴﻜﺎﻨﻴﻜﺎ ﺍﻟﻜﻡ‪ .‬ﺍﻟﻤﺅﺜﺭ ‪ H‬ﻟﻨﻅﺎﻡ ﻤﻥ ﺠﺴﻡ ﻭﺍﺤﺩ ﻓﻲ ﺠﻬﺩ ‪ V‬ﻟﻪ ﺍﻟﺸﻜل‪.‬‬
‫‪p sp2‬‬
‫=‪H‬‬ ‫)‪+ V (x‬‬
‫‪2m‬‬

‫ﻭﺇﺫﺍ ﻜﺎﻨﺕ )‪ V (x‬ﻟﻴﺱ ﻟﻬﺎ ﺍﻋﺘﻤﺎﺩ ﻭﺍﻀﺢ ﻋﻠﻰ ﺍﻟﺯﻤﻥ ﻓﺈﻥ ﺍﻟﻤﻌﺎﺩﻟـﺔ )‪(١‬‬
‫ﻴﻤﻜﻥ ﺤﻠﻬﺎ ﺒـ‪:‬‬

‫‪ψ (x, t) = u E (x) e − Et/h‬‬

‫ﺤﻴﺙ‪:‬‬

‫)‪H u E (x) = E u E (x‬‬

‫‪- ١١٨ -‬‬


‫‪PDF created with pdfFactory Pro trial version www.pdffactory.com‬‬
‫ﻫﻲ ﺍﻟﺩﻭﺍل ﺍﻟﺨﺎﺼﺔ ﻟﻠﻬﺎﻤﻠﺘﻭﻨﻴﺎﻥ ﻭ ‪ E‬ﻫـﻲ ﺍﻟﻘـﻴﻡ‬ ‫)‪u E (x‬‬ ‫ﺇﻥ ﺤﻠﻭل ﻫﺫﻩ ﺍﻟﻤﻌﺎﺩﻟﺔ‬
‫ﺍﻟﺨﺎﺼﺔ ﻭﻓﻲ ﻫﺫﺍ ﺍﻟﻔﺼل ﺴﻨﺫﻜﺭ ﻋﻠﻰ ﺨﺎﺼﻴﺘﻥ ﻤﻬﻤﺘﻴﻥ ﻟﻠﺩﻭﺍل ﺍﻟﺨﺎﺼﺔ ﻟـ ‪.H‬‬
‫‪ -1‬ﺍﻟﺩﻭﺍل ﺍﻟﺨﺎﺼﺔ ﺍﻟﻤﻁﺎﺒﻘﺔ ﻟﻘﻴﻡ ﺨﺎﺼﺔ ﻤﺨﺘﻠﻔﺔ )ﺃﻱ ﻗﻴﻡ ﻤﺨﺘﻠﻔﺔ ﻟـ ‪ (E‬ﺘﻜـﻭﻥ‬
‫ﻫﺫﻩ ﺍﻟﺩﻭﺍل ﻤﺘﻌﺎﻤﺩﺓ ﺃﻱ‪:‬‬

‫∫‬ ‫‪dx u *E (x) u‬‬


‫‪E′‬‬
‫‪(x) = 0‬‬ ‫‪E ≠ E′‬‬

‫ﺘﻜـﻭﻥ‬ ‫)‪ψ (x‬‬ ‫‪ -2‬ﺇﻥ ﺍﻟﺩﻭﺍل ﺍﻟﺨﺎﺼﺔ ﺘﻜﻭﻥ ﻓﺌﺔ ﺍﻟﻬﺎﻤﻠﺔ – ﺃﻱ ﺃﻥ – ﺩﺍﻟﺔ ﺍﺨﺘﻴﺎﺭﻴﺔ‬
‫ﻤﺭﺒﻌﺔ ﺍﻟﺘﻜﺎﻤل ﻟﻜﻲ‪.‬‬

‫∫‬ ‫)‪dx ψ * (x) ψ(x‬‬ ‫<‬ ‫∞‬ ‫)‪(6‬‬

‫ﻴﻤﻜﻥ ﺃﻥ ﺘﻤﺩ )ﺃﻭ ﺘﻔﻙ( ‪ expanded‬ﺒﺩﻻﻟﺔ ﺍﻟﻘﻴﻡ ﺍﻟﺨﺎﺼﺔ ﻟﻠﻬﺎﻤﺘﻠﻭﻨﻴﺎﻥ‪.‬‬


‫= )‪ψ (x‬‬ ‫∑‬ ‫‪E‬‬
‫)‪C E u E (x‬‬ ‫)‪(7‬‬

‫ﺇﻥ ﻁﻴﻑ ‪ H‬ﻴﻤﻜﻥ ﺃﻥ ﻴﻜﻭﻥ ﻤﻨﻔﺼل )ﻤﺠﺯﺃ( ﻜﻤﺎ ﻓـﻲ ﺤﺎﻟـﺔ ﺍﻟﻤﺘﺫﺒـﺫﺏ‬
‫ﻋﻨﺩﻤﺎ ‪ ، ∞ ← x‬ﻓﺈﻥ ﺍﻟﻘﻴﻡ ﺍﻟﺨﺎﺼﺔ ﻴﻤﻜـﻥ ﺃﻥ‬ ‫)‪0 ← V (x‬‬ ‫ﺍﻟﺘﻭﺍﻓﻘﻲ‪ .‬ﻭﻟﻭ ﺃﻥ ﺍﻟﺠﻬﺩ‬
‫ﺘﻜﻭﻥ ﻤﻨﻔﺼﻠﺔ ﻭﻜﺫﻟﻙ ﻤﺘﺼﻠﺔ‪.‬‬

‫ﻭﺇﻥ ﺍﻟﻤﻌﺎﺩﻟﺔ )‪ (7‬ﻴﻤﻜﻥ ﺘﻌﻭﻴﻀﻬﺎ ﺒـ‪:‬‬


‫= )‪ψ (x‬‬ ‫∑‬‫‪n‬‬
‫‪C n u n (x) +‬‬ ‫∫‬ ‫)‪dp C (p) u p (x‬‬ ‫)‪(8‬‬

‫ﺇﻥ ﺍﻟﺩﻭﺍل ﺍﻟﺨﺎﺼﺔ ﻴﻤﻜﻥ ﺃﻥ ﺘﻀﺭﺏ ﺒﺜﻭﺍﺒﺕ ﺒﺤﻴـﺙ ﺃﻥ ﺘـﺼﺒﺢ ﻤﻌـﺎﻴﺭﺓ‬


‫ﻭﺸﺭﻭﻁ ﺍﻟﻤﻌﺎﻴﺭﺓ ﺍﻟﻤﺘﻌﺎﻤﺩﺓ‪.‬‬

‫∫‬ ‫‪dx u *m (x) u n (x) = δ mn‬‬

‫∫‬ ‫)‪dx u *p (x) u p (x) = δ (1 − 2‬‬

‫∫‬ ‫‪dx u *m (x) u p (x) = 0‬‬

‫‪- ١١٩ -‬‬


‫‪PDF created with pdfFactory Pro trial version www.pdffactory.com‬‬
‫ﻴﻤﻜﻥ ﺘﺤﺩﻴﺩﻫﺎ ﺒﻤﻼﺤﻅﺔ ﺃﻥ ﻜل ﺩﺍﻟﺔ ﺨﺎﺼﺔ ﻟﻬـﺎ‬ ‫)‪ψ (x, t‬‬ ‫ﺇﻥ ﺍﻟﺩﺍﻟﺔ ﺍﻟﻤﻭﺠﻴﺔ‬
‫ﺍﻋﺘﻤﺎﺩ ﺴﺘﺤﺼل ﻋﻠﻰ ﺍﻟﺯﻤﻥ ﻴﻌﻁﻲ ﺒـ‪:‬‬
‫‪u E (x, t ) = u E (x) e −iEt/h‬‬
‫⇒‬
‫‪ψ (x, t) = ∑ C E u E (x) e −iEt/h‬‬
‫‪E‬‬

‫‪ψ (x, t) = ∑ C E u E (x) e −iEt/h‬‬ ‫∫‬


‫‪2 t/2m h‬‬
‫‪dp C(p) u (x) e −ip‬‬
‫‪E‬‬

‫ﻤﺸﺎﻫﺩﺍﺕ ﺃﺨﺭﻯ‪O These Obseruobeas :‬‬

‫ﺇﻥ ﺍﻟﻁﺎﻗﺔ ﻫﻲ ﻭﺍﺤﺩﺓ ﻤﻥ ﺍﻟﻤﺸﺎﻫﺩﺍﺕ ﻟﻨﻅﺎﻡ ﻤﺸﺎﻫﺩﺍﺕ ﺃﺨﺭﻯ ﻜﻤﻴﺔ ﺍﻟﺤﺭﻜﺔ‬


‫ﺴﻨﻨﺎﻗﺵ ﻜﻤﻴﺔ ﺍﻟﺤﺭﻜﺔ ﺍﻟﺯﺍﻭﻴﺔ )ﺍﻟﻔﺼل ﺍﻟﺘﺎﻟﻲ(‪.‬‬
‫ﻤﻌﺎﺩﻟﺔ‪ :‬ﻤﺅﺜﺭ ﻜﻤﻴﺔ ﺍﻟﺤﺭﻜﺔ‪.‬‬
‫‪h d‬‬
‫= )‪p op u p (x‬‬ ‫)‪u (x) = p u p (x‬‬
‫‪i dx p‬‬

‫ﻭﺍﻟـﺩﻭﺍل‬ ‫) ∞ < ‪(− ∞ < p‬‬ ‫ﺍﻟﺩﻭﺍل ﺍﻟﺨﺎﺼﺔ ﺘﻜـﻭﻥ ﺍﺘـﺼﺎل ‪aconitum‬‬
‫ﺍﻟﺨﺎﺼﺔ ﺘﺄﺨﺫ ﺍﻟﺸﻜل‪.‬‬
‫‪1‬‬
‫= )‪u p (x‬‬ ‫‪e ipx/ h‬‬
‫‪2D h‬‬

‫ﻭﺍﻟﺩﻭﺍل ﺍﻟﺨﺎﺼﺔ ﺘﻜﻭﻥ ﻓﺌﺔ ﻋﻴﺎﺭﻴﺔ ﻤﺘﻌﺎﻤﺩﺓ‪.‬‬


‫∞‪+‬‬

‫∫‬
‫∞‪−‬‬
‫)‪dx u *p (x) u p (x‬‬
‫‪1‬‬ ‫‪2‬‬
‫) ‪= δ (p1 − p 2‬‬

‫ﺇﻥ ﻤﺅﺜﺭ ﻜﻤﻴﺔ ﺍﻟﺤﺭﻜﺔ ﻤﺜل ﺍﻟﻬﺎﻤﻠﻴﺘﻭﻨﻴﺎﻥ ﻟﻪ ﻗﻴﻡ ﺨﺎﺼﺔ ﺤﻘﻴﻘﻴﺔ‪.‬‬


‫ﺇﻥ ﺍﻟﻤﺅﺜﺭﺍﺕ‪ ،‬ﺍﻟﺘﻲ ﻟﻬﺎ ﻗﻴﻡ ﺨﺎﺼﺔ ﺤﻘﻴﻘﻴﺔ‪ ،‬ﺘﺴﻤﻰ ﻤﺅﺜﺭﺍﺕ ﻫﻴﺭﻤﻴﺘﺎﻥ‪ .‬ﻭﻷﻥ‬
‫ﻜل ﺍﻟﻤﺅﺜﺭﺍﺕ ﺍﻟﻔﻴﺯﻴﺎﺌﻴﺔ ﺘﺸﺘﺭﻁ ﻓﻲ ﻫﺫﻩ ﺍﻟﺼﻔﺔ‪ ،‬ﻓﻠﻬﺫﺍ ﺍﻟﺴﺒﺏ ﺘﻭﺼﻑ ﺒﻤـﺅﺜﺭﺍﺕ‬
‫ﻫﻴﺭﻤﻴﺘﻴﺔ‪.‬‬

‫‪- ١٢٠ -‬‬


‫‪PDF created with pdfFactory Pro trial version www.pdffactory.com‬‬
‫ﻨﻅﺭﻴﺔ ﺍﻟﻤﻔﻜﻭﻙ ﻭﺘﻤﺎﺜﻠﻬﺎ ﻤﻊ ﺍﻟﻤﺘﺠﻬﺎﺕ‪:‬‬

‫ﻟﻤﺅﺜﺭ ﺍﺨﺘﻴﺎﺭﻱ ﻭﺍﻟﺫﻱ ﻴﺭﻤﺯ ﻟﻪ ﺒـ ‪ ،A‬ﺴﻴﻜﻭﻥ ﻫﻨﺎﻙ ﺩﻭﺍل ﺨﺎﺼﺔ ﺘﻁﺎﺒﻕ‬


‫ﻟﻘﻴﻡ ﺨﺎﺼﺔ ﺤﻘﻴﻘﻴﺔ ‪.a‬‬

‫)‪A u a (x) = a u a (x‬‬

‫ﻭﺍﻟﺩﻭﺍل ﺍﻟﺨﺎﺼﺔ ﺘﺸﻜل ﻓﺌﺔ ﻤﺘﻌﺎﻤﺩﺓ ﻭﻴﻤﻜﻥ ﺃﻥ ﺘﺼﺒﺢ ﻤﻌﺎﻴﺭﺓ ﺒﺤﻴﺙ‪:‬‬

‫∫‬ ‫‪dx u *a1 (x) u a2 (x) = δa 1 , a 2‬‬

‫ﻷﻥ )‪u a (x‬‬ ‫ﺘﻜﻭﻥ ﻓﺌﺔ ﻜﺎﻤﻠﺔ ﻭﺍﻟﺘﻲ ﻴﻤﻜﻥ ﺃﻥ ﺘﺤﺩﺩ ﺒﻪ‬ ‫)‪u a (x‬‬ ‫ﻭﺍﻟﺩﻭﺍل ﺍﻟﺨﺎﺼﺔ‬
‫= )‪ψ(x‬‬ ‫∑‬ ‫‪a‬‬
‫)‪C a u a (x‬‬

‫ﻤﻥ ﺸﺭﻁ ﺍﻟﻤﻌﺎﻴﺭﺓ ﻭﺍﻟﺘﻌﺎﻤﺩ‪:‬‬

‫)‪C a = ∫ dx u *a (x) ψ (x‬‬

‫ﺘﻔﺴﻴﺭ ﻤﻌﺎﻤﻼﺕ ﺍﻟﻔﻙ‪:‬‬

‫ﺃﻥ ﺘﻔﺴﻴﺭ ‪:Ca‬‬


‫ﻟﻭ ﻫﻨﺎﻙ ﻤﺅﺜﺭ ﺘﻡ ﻗﻴﺎﺴﻪ ﻟﺘﺠﻤﻊ ﻤﻥ ﺍﻷﻨﻅﻤﺔ ﻜل ﻤﻨﻬﺎ ﻴﻭﺼـﻑ ﺒﺩﺍﻟـﺔ )‪(x‬‬
‫ﻭﺍﻟﺘﻲ ﺘﻜﻭﻥ ﻤﻌﺎﻴﺭﺓ ﻟﻠﻭﺤﺩﺓ ﺒﺤﻴﺙ‪.‬‬

‫‪∫ dx ψ‬‬ ‫‪(x) ψ (x) = 1‬‬


‫*‬

‫ﻋﻨﺩﺌﺫ‪:‬‬

‫‪ -1‬ﻨﺘﻴﺠﺔ ﺃﻱ ﻗﻴﺎﺱ ﻤﻌﻁﻰ ﻴﻤﻜﻥ ﺃﻥ ﻴﻜﻭﻥ ﻓﻘﻁ ﻭﺍﺤﺩﺓ ﻤﻥ ﺍﻟﻘﻴﻤﺔ ﺍﻟﺨﺎﺼﺔ ‪.a‬‬
‫‪Ca‬‬
‫‪2‬‬
‫‪ -2‬ﺍﺤﺘﻤﺎل ﺃﻥ ﺍﻟﻘﻴﻤﺔ ﺍﻟﺨﺎﺼﺔ ‪ a‬ﺃﻥ ﺘﻭﺠﺩ ﻫﻭ‬
‫‪ -3‬ﻭﻜﻤﻘﺘﻀﻰ ﻟﻬﺫﺍ ﺍﻟﺘﺄﻭﻴل ﻓﺈﻥ ﺍﺤﺘﻤﺎل ﺃﻥ ﻗﻴﻤﺔ ﺍﻟﻤﺅﺜﺭ ‪ A‬ﻟﻨﻅﺎﻡ ﻴﻜﻭﻥ ﻟﻬﺎ ﻭﺍﺤﺩ‬
‫ﻤﻥ ﺍﻟﻘﻴﻤﺔ ﺍﻟﺨﺎﺼﺔ ﻫﻭ ‪.unity‬‬

‫‪- ١٢١ -‬‬


‫‪PDF created with pdfFactory Pro trial version www.pdffactory.com‬‬

2
Ca =1
a

:‫ﻭﻫﺫﺍ ﻴﻨﺘﺞ ﻤﻥ‬


 
1= ∫ dx ψ * (x) ψ (x) = ∫ dx  ∑ C *a u *a (x)  ψ (x)
 a 
= ∑ Ca C a
*

:‫ﻭﻫﺫﻩ ﺍﻟﻤﻌﺎﺩﻟﺔ ﺘﻌﻁﻰ ﺍﻟﺘﺎﻟﻲ‬


∑a
C a C *a = ∑ ∫
a
dx u *a (x) ψ (x) ∫ dy u a (y) ψ * (y)

= ∫ ∫ dx dy ψ * (y) ψ (x) ∑ u a (y) u *a (x) = 1


a

⇒ ∑a
u a (y) u (x) = δ (x − y)
*
a

‫ﺇﻥ ﻫﺫﻩ ﺍﻟﺨﺎﺼﻴﺔ ﻟﻠﺩﻭﺍل ﺍﻟﺨﺎﺼﺔ ﺘﻭﺼﻑ ﺒـ ﺍﻟﻌﻼﻗﺔ ﺍﻟﻜﺎﻤﻠﺔ ﻭﻫﻲ ﻤﻁﺎﺒﻘﺔ‬
.‫ﻟﻤﻨﻁﻭﻕ ﻨﻅﺭﻴﺔ ﺍﻟﻤﻔﻜﻭﻙ‬

- ١٢٢ -
PDF created with pdfFactory Pro trial version www.pdffactory.com
‫‪The Vector Space Analogy‬‬ ‫ﺍﻟﺘﻨﺎﻅﺭ ﻤﻊ ﻓﻀﺎﺀ ﺍﻟﻤﺘﺠﻪ‬

‫ﺍﻟﻤﺅﺜﺭﺍﺕ ﻭﺍﻟﻜﻤﻴﺎﺕ ﺍﻟﻤﺸﺎﻫﺩﺓ ‪:operators and observables‬‬

‫ﻓﻲ ﻓﺭﺍﻍ )ﻓﻀﺎﺀ( ﺍﻟﻤﺘﺠﻬﺎﺕ‪ ،‬ﺍﻟﻤﺅﺜﺭ ﻴﺤﻭل ‪ transforms‬ﻤﺘﺠﻪ ﺇﻟﻰ ﻤﺘﺠـﻪ‬


‫ﺁﺨﺭ ﺇﻥ ﺍﻟﻤﺅﺜﺭﺍﺕ ﺍﻟﺨﻁﻴﺔ ﻟﻬﺎ ﺍﻟﺨﺎﺼﻴﺔ‪.‬‬

‫)‪A (α1 ψ1 (x) + α 2 ψ 2 (x) ) = α 1 A ψ1 (x) + α 2 A ψ 2 (x‬‬

‫ﻭﻟﻭ ﻜﺎﻨﺕ ﻫﺫﻩ ﺍﻟﻤﺅﺜﺭﺍﺕ ﺘﻤﺜل ﻜﻤﻴﺎﺕ ﻤﺸﺎﻫﺩﺓ‪ ،‬ﻓﺈﻨﻬـﺎ ﻴﺠـﺏ ﺃﻥ ﺘﻜـﻭﻥ‬
‫ﻫﻴﺭﻤﻴﺘﻴﺔ‪ .‬ﻭﻟﻠﻤﺅﺜﺭ ﺍﻟﻬﻴﺭﻤﻴﺘﻲ‪.‬‬

‫‪A = A‬‬ ‫‪A − A‬‬ ‫‪=0‬‬


‫*‬ ‫*‬

‫‪ψ‬‬ ‫ﺃﻱ ﺃﻨﻪ ﻟﻠﺤﺎﻟﺔ‬

‫∫‬ ‫)‪dx ψ * (x) Aψ (x) = ∫ dx (A ψ (x) ) ψ (x‬‬


‫*‬
‫)‪(a‬‬

‫ﻤﻥ ﻫﺫﺍ ﻴﻨﺘﺞ ﺃﻥ ﻟﻠﻤﺅﺜﺭﺍﺕ ﺍﻟﻬﻴﺭﻤﻴﺘﻴﺔ‬

‫∫‬ ‫‪dx φ * Aψ = ∫ dx (A φ ) ψ‬‬


‫*‬
‫)‪(b‬‬

‫‪ψ‬‬ ‫‪,‬‬ ‫‪φ‬‬ ‫ﻭﻷﻱ ﺯﻭﺝ ﻤﻥ ﺍﻟﺩﻭﺍل‬


‫ﻭﻹﺜﺒﺎﺕ ﻫﺫﺍ‪ ،‬ﺨﺫ‪.‬‬

‫)‪ψ (x) = φ (x) + λ ψ (x‬‬

‫ﻫﻭ ﺜﺎﺒﺕ ﻤﺭﻜﺏ‪ ،‬ﻋﻨﺩﺌﺫ ﻴﻨﺘﺞ ﻤﻥ ﺍﻟﻤﻌﺎﺩﻟﺔ )‪ (a‬ﺃﻥ‪:‬‬ ‫‪λ‬‬ ‫ﺤﻴﺙ‬


‫∫‬ ‫)‪dx ψ * (x) A ψ = ∫ dx (φ * + λ * ψ * ) A (φ + λ ψ‬‬

‫=‬ ‫∫‬ ‫‪dx φ * A φ + λ‬‬ ‫∫‬


‫‪2‬‬
‫‪dx ψ * Aψ‬‬

‫‪+ λ ∫ dx φ * Adx φ * Aψ + λ * ∫ ψ * Aφ‬‬

‫‪- ١٢٣ -‬‬


‫‪PDF created with pdfFactory Pro trial version www.pdffactory.com‬‬
‫ﺍﻟﻤﺭﺍﻓﻕ ﻟﻬﺫﺍ ﻫﻭ‬
‫∫‬ ‫] ) ‪dx (AψA* ψ = ∫ dx [A (φ + λψ) ] [ (φ + λ ψ‬‬
‫*‬

‫‪= ∫ dx (AφA* φ + λ‬‬ ‫∫‬ ‫‪dx (AψA* ψ + λ * ∫ dx (AψA* φ + λ ∫ dx (AφA* ψ‬‬


‫‪2‬‬

‫ﻭﺍﻟﻤﺅﺜﺭ ﻫﻴﺭﻤﻴﺕ‪ ،‬ﺍﻟﺠﻭﺍﻨﺏ ﺍﻟﻴﺴﺭﻯ ﻤﻥ ﺍﻟﻤﻌـﺎﺩﻟﺘﻴﻥ )‪ (d) , (c‬ﻭﺍﻟﺤـﺩﻴﻥ‬


‫ﻤﻌﺎﻤﻼﺕ ﻤﺴﺘﻘﻠﺔ )ﻫﻲ ﻤﺭﻜﺒﺔ( ﺒﺈﻤﻜﺎﻨﻨﺎ ﺘﺴﺎﻭﻱ‬ ‫‪λ* , λ‬‬ ‫ﺍﻷﻭﻟﻴﻥ ﻤﺘﺴﺎﻭﻴﻴﻥ‪ .‬ﻭﺒﻤﺎ ﺃﻥ‬
‫ﻤﻌﺎﻤﻼﺘﻬﻡ ﺒﺎﻨﻔﺼﺎل‪ ،‬ﻭﻴﻨﺘﺞ ﻟﻨﺎ ﺍﻟﻤﻌﺎﺩﻟﺔ‬

‫ﻭﻟﻭ ﺃﻥ ﻤﺅﺜﺭ ‪ A‬ﻏﻴﺭ ﻫﻴﺭﻤﻴﺘﻲ‪ ،‬ﻓﺒﺈﻤﻜﺎﻨﻨـﺎ ﺘﻌﺭﻴـﻑ ﺍﻟﻤـﺅﺜﺭ ﺍﻟﻤﺭﺍﻓـﻕ‬


‫ﺍﻟﻬﻴﺭﻤﻴﺘﻲ ‪hermitian conjugatioput A+‬‬

‫∫‬ ‫‪dx (AφA* ψ = ∫ dxφ * A + ψ‬‬ ‫⊗⊗‬

‫ﻭﺫﻟﻙ ﻷﻱ ﺯﻭﺝ ﻤﻥ ﺍﻟﺩﻭﺍل ﺍﻟﻤﻭﺠﻴﺔ‪.‬‬


‫‪A = A+‬‬ ‫ﻻﺤﻅ ﺃﻥ ﻟﻠﻤﺅﺜﺭ ﺍﻟﻬﻴﺭﻤﻴﺘﻲ‬ ‫⊗‬

‫ﺍﻟﻤﺅﺜﺭﺍﺕ ﺍﻟﺘﺒﺎﺩﻟﻴﺔ ‪[ A , B ] = 0‬‬ ‫⊗‬

‫‪( AB ) + = B + A +‬‬ ‫ﻟﻠﻤﺅﺜﺭﺍﺕ ﺍﻟﻬﻴﺭﻤﻴﺘﻴﺔ‬ ‫⊗‬

‫‪- ١٢٤ -‬‬


‫‪PDF created with pdfFactory Pro trial version www.pdffactory.com‬‬
‫ﺍﻟﻨﻬﺎﻴﺔ ﺍﻟﻜﻼﺴﻴﻜﻴﺔ )ﺍﻟﺘﻘﻠﻴﺩﻴﺔ( ﻟﻠﻨﻅﺭﻴﺔ ﺍﻟﻜﻤﻴﺔ ﻭﺍﻻﻋﺘﻤﺎﺩ ﺍﻟﺯﻤﻨﻲ‬
‫‪Time Dependence The classical limit of Quantum theory‬‬
‫ﻤﻥ ﺍﻷﺴﺌﻠﺔ ﺍﻟﻤﻬﻤﺔ ﺍﻟﺘﻲ ﻨﺤﻥ ﺒﺼﺩﺩﻫﺎ ﺍﻵﻥ ﻫﻲ ﺍﻟﻨﻬﺎﻴﺔ ﺍﻟﺘﻘﻠﻴﺩﻴﺔ ﻟﻠﻨﻅﺭﻴـﺔ‬
‫ﺍﻟﻜﻤﻴﺔ‪ .‬ﻭﻟﻤﻨﺎﻗﺸﺔ ﻫﺫﻩ ﺍﻟﻤﺴﺄﻟﺔ‪ ،‬ﻓﺈﻨﻨﺎ ﻨﺒﺩﺃ ﺒﺩﺭﺍﺴﺔ ﺍﻟﺘﻁﻭﺭ ﺍﻟﺯﻤﻨﻲ ﻟﻠﻘﻴﻡ ﺍﻟﻤﺘﻭﻗﻌـﺔ‬
‫ﻟﻠﻤﺅﺜﺭﺍﺕ ﺍﻟﻤﺘﻐﻴﺭﺓ ﻤﻊ ﺍﻟﺯﻤﻥ‪ .‬ﻤﻨﻬﺎ ﻴﻤﻜﻥ ﺃﻥ ﺘﺘﻐﻴـﺭ ﻤـﻊ ﺍﻟـﺯﻤﻥ ﻭﺫﻟـﻙ ﻷﻥ‬
‫ﺍﻟﻤﺅﺜﺭﺍﺕ ﻟﻬﺎ ﺍﻋﺘﻤﺎﺩ ﻭﺍﻀﺢ ﻋﻠﻰ ﺍﻟﺯﻤﻥ‪ ،‬ﻋﻠﻰ ﺴﺒﻴل ﺍﻟﻤﺜﺎل‪ ،‬ﺍﻟﻤـﺅﺜﺭ ‪، α + pt / m‬‬
‫ﻭﺇﻨﻪ ﻴﺘﻐﻴﺭ ﻤﻊ ﺍﻟﺯﻤﻥ‪ ،‬ﻭﺫﻟﻙ ﻷﻥ ﺍﻟﻘﻴﻤﺔ ﺍﻟﻤﺘﻭﻗﻌﺔ ﺘﺅﺨﺫ ﺒﺎﻟﻨﺴﺒﺔ ﻟﻠﺩﺍﻟـﺔ ﺍﻟﻤﻭﺠﻴـﺔ‬
‫ﺍﻟﺘﻲ ﺒﻨﻔﺴﻬﺎ ﺘﺘﻐﻴﺭ ﻤﻊ ﺍﻟﺯﻤﻥ‪ .‬ﻟﻭ ﻜﺘﺒﻨﺎ‪.‬‬

‫‪A t = ∫ ψ * (x, t) A ψ (x , t) dx‬‬

‫ﻋﻨﺩﺌﺫ‪:‬‬
‫‪d‬‬ ‫‪∂A‬‬
‫)‪A t = ∫ ψ * (x, t‬‬ ‫‪ψ (x , t) dx‬‬
‫‪dt‬‬ ‫‪∂t‬‬
‫)‪∂ψ * (x, t‬‬ ‫)‪∂ψ(x, t‬‬
‫∫‪+‬‬ ‫‪A ψ (x , t ) dx + ∫ ψ * (x , t ) A‬‬ ‫‪di‬‬
‫‪∂t‬‬ ‫‪∂t‬‬
‫‪∂A‬‬
‫*‬
‫‪1‬‬ ‫‪‬‬ ‫‪1‬‬ ‫‪‬‬
‫=‬ ‫‪+ ∫  H ψ (x, t)  A ψ (x, t) dx + ∫ ψ * (x, t) A  H ψ (x, t) ‬‬
‫‪∂t‬‬ ‫‪t‬‬ ‫‪ ih‬‬ ‫‪‬‬ ‫‪ ih‬‬ ‫‪‬‬
‫‪∂A‬‬ ‫‪i‬‬ ‫‪i‬‬
‫=‬ ‫‪+ ∫ ψ * (x, t) H A ψ (x, t) dx + − ∫ ψ * (x, t) AHψ (x, t)dx‬‬
‫‪∂t t h‬‬ ‫‪h‬‬
‫‪∂A‬‬
‫=‬ ‫‪+‬‬
‫‪i‬‬
‫] ‪[H , A‬‬
‫‪∂t‬‬ ‫‪t‬‬ ‫‪h‬‬ ‫‪t‬‬

‫ﻨﻼﺤﻅ ﺃﻨﻪ ﺇﺫﺍ ﻜﺎﻥ ﺍﻟﻤﺅﺜﺭ ‪ A‬ﻏﻴﺭ ﻤﻌﺘﻤﺩ ﻋﻠﻰ ﺍﻟﺯﻤﻥ ﺒﻭﻀﻭﺡ ﺤﻴﻨﺌﺫ = ‪0‬‬
‫‪dA‬‬ ‫‪dA‬‬
‫⇒‬ ‫= ‪=0‬‬
‫‪dt‬‬ ‫‪dt‬‬

‫⇒‬
‫‪d‬‬
‫= ‪A t‬‬
‫‪i‬‬
‫‪[H , A‬‬ ‫]‬
‫‪dt‬‬ ‫‪h‬‬ ‫‪t‬‬

‫ﻭﺇﺫﺍ ﻜﺎﻥ ﺍﻟﻤﺅﺜﺭ ﺘﺒﺎﺩﻟﻲ ﻤﻊ ‪ ،H‬ﻓﺈﻥ ﺍﻟﻘﻴﻤﺔ ﺍﻟﻤﺘﻭﻗﻌﺔ ﺘﻜﻭﻥ ﺩﺍﺌﻤﺎﹰ ﺜﺎﺒﺕ‪.‬‬

‫‪- ١٢٥ -‬‬


‫‪PDF created with pdfFactory Pro trial version www.pdffactory.com‬‬
‫ﻭﻋﻠﻴﻪ ﻓﺒﺈﻤﻜﺎﻨﻨﺎ ﺃﻥ ﻨﻘﻭل ﺃﻥ ﺍﻟﻘﻴﻤﺔ ﺍﻟﻘﻴﺎﺴﻴﺔ ﺍﻟﻤﺸﺎﻫﺩﺓ ﻫﻲ ﺜﺎﺒﺕ ﻟﻠﺤﺭﻜـﺔ‪.‬‬
‫ﻭﻟﻭ ﺃﻥ ﺍﻟﻬﺎﻤﻠﺘﻭﻨﻴﺎﻥ ﻫﻭ ﻭﺍﺤﺩ ﻤﻥ ﺍﻟﻔﺌﺎﺕ ﺍﻟﻜﺎﻤﻠﺔ ﻟﻠﻤﺸﺎﻫﺩﺍﺕ ﺍﻟﻤﺘﺒﺎﺩﻟﺔ‪ ،‬ﻋﻨﺩﺌﺫ ﻜل‬
‫ﺍﻟﻤﺅﺜﺭﺍﺕ ﺘﻜﻭﻥ ﺜﺎﺒﺕ ﻓﻲ ﺍﻟﺤﺭﻜﺔ‪.‬‬
‫ﺨﺫ‪:‬‬
‫)‪( 1‬‬ ‫‪A=x‬‬
‫)‪(2‬‬ ‫‪A=p‬‬

‫ﺃﻭﻻﹰ‪:‬‬
‫‪d‬‬
‫= ‪x‬‬
‫‪i‬‬
‫] ‪[H , x‬‬
‫‪dt‬‬ ‫‪h‬‬
‫‪i  p2‬‬ ‫‪‬‬
‫=‬ ‫‪ 2m + v (x) , χ ‬‬
‫‪h ‬‬ ‫‪‬‬

‫‪ ،‬ﺃﻱ ﻤﺜﻼﹰ )‪.v (x‬‬ ‫‪χ‬‬ ‫ﺘﺘﺒﺎﺩل ﻤﻊ ﺃﻱ ﺩﺍﻟﺔ ﻓﻲ‬ ‫‪χ‬‬

‫‪[ V (x) , χ ] = 0‬‬


‫⇒‬ ‫‪[p‬‬ ‫‪2‬‬
‫‪, χ‬‬ ‫?=]‬
‫‪[p‬‬ ‫‪2‬‬
‫‪,‬‬ ‫‪χ ] = p [ p , χ ] + [ p , χ ]p‬‬
‫‪h‬‬
‫‪=2 p‬‬
‫‪i‬‬
‫‪d‬‬ ‫‪i/ 2/‬‬ ‫‪p‬‬ ‫‪p‬‬
‫⇒‬ ‫‪χ = . t/‬‬ ‫=‬ ‫)‪(a‬‬
‫‪dt‬‬ ‫‪h/ i/‬‬ ‫‪2/ m‬‬ ‫‪m‬‬

‫ﺜﺎﻨﻴﺎﹰ‪:‬‬
‫] ‪p = [H , p‬‬
‫‪d‬‬ ‫‪i‬‬
‫‪dt‬‬ ‫‪h‬‬
‫‪i  p2‬‬ ‫‪‬‬
‫=‬ ‫‪‬‬ ‫‪+ v (x) , p ‬‬
‫‪h  2m‬‬ ‫‪‬‬

‫‪ P‬ﺘﺘﺒﺎﺩل ﻤﻊ ﺃﻱ ﺩﺍﻟﺔ ﻓﻲ ‪p‬‬

‫⇒‬ ‫=‬
‫‪i‬‬
‫] ‪[ V (x) , p‬‬ ‫‪=−‬‬
‫‪i‬‬
‫] )‪[ p , V (x‬‬
‫‪h‬‬ ‫‪h‬‬

‫‪- ١٢٦ -‬‬


‫‪PDF created with pdfFactory Pro trial version www.pdffactory.com‬‬
[ p , V (x) ] ? ‫ﻟﺤﺴﺎﺏ‬
h d
pV (x) ψ (x) − V (x) P ψ (x) = [ V (x) ψ (x) ]− h V (x) d ψ (x)
i dx i dx
h dψ ( x) h dV(x) h dψ (x)
= V (x) + ψ (x) − V (x)
i dx i dx i dx
h dV(x)
= . ψ (x )
i dx
h dV(x)
⇒ [ p , V (x)]=
i dx

:‫ﻭﻋﻠﻴﻪ‬
d i h/ dV (x) dV (x)
p =− . =− (b)
dt h/ i dx t dx t

( b ) , ( a ) ‫ﺒﺎﺘﺤﺎﺩ‬
d p d
χ = ⇒m χ = p
dt m dt
d d d d2 dV (x)
p t
= .m χ = m 2 χ t =−
dt dt dt dt dx
d2 dV (x)
⇒m 2
χ t =−
dt dx t

‫ﻫﺫﻩ ﺍﻟﻤﻌﺎﺩﻟﺔ ﺘﺒﺩﻭ ﻤﺸﺎﺒﻬﺔ ﻜﺜﻴﺭﺍﹰ ﻟﻤﻌﺎﺩﻟﺔ ﺍﻟﺤﺭﻜﺔ ﻟﺠﺴﻡ ﺘﻘﻠﻴﺩﻱ ﻴﺘﺤﺭﻙ ﻓـﻲ‬
V (x) ‫ﺠﻬﺩ‬
d2χ dV (x)
m 2
=−
dt dx

χ= χ ‫ﻭﺍﻟﺸﻲﺀ ﺍﻟﻭﺤﻴﺩ ﺍﻟﺫﻱ ﻴﻤﻨﻌﻨﺎ ﺃﻥ ﻨﻌﻤل ﺍﻟﺘﻁﺎﺒﻕ‬


:‫ﻫﻭ‬
V( χ )
dV d
=/
dx dχ

- ١٢٧ -
PDF created with pdfFactory Pro trial version www.pdffactory.com
‫ﺘﻤﺕ ﺍﻟﺤﻴﺜﻴﺎﺕ ﻭﺍﻟﺘﻲ ﻋﻨﺩﻫﺎ ﺍﻟﻤﺘﻁﺎﺒﻘﺔ ﺍﻟﺴﺎﺒﻘﺔ ﺘـﺼﺒﺢ ﺘﻘﺭﻴﺒـﺎﹰ ﻤﺘـﺴﺎﻭﻴﺔ‬
‫ﺍﻟﺤﺭﻜﺔ ﻫﻲ ﻓﻲ ﺍﻷﺴﺎﺱ ﺘﻘﻠﻴﺩﻴﺔ‪ ،‬ﻜﻤﺎ ﺘﻡ ﻤﻼﺤﻅﺘﻪ ﺃﻭﻻﹰ ﺒﻭﺍﺴـﻁﺔ ‪ hrenfen‬ﻫـﺫﺍ‬
‫ﻴﻠﺯﻡ ﺃﻥ ﺩﺍﻟﺔ ﺍﻟﺠﻬﺩ ﺘﺘﻐﻴﺭ ﺒﺒﻁﻲﺀ ﻤﻊ ﺍﻟﻤﺘﻐﻴﺭ‪.‬‬
‫)‪dV (x‬‬
‫‪F (x) = −‬‬
‫‪dχ‬‬

‫) ‪)F′ ( χ )+ (χ − χ‬‬
‫‪2‬‬

‫‪⇒ F (x) = F ( χ )+ (x − χ‬‬ ‫‪F′′ ( χ‬‬ ‫)‬


‫‪2‬‬

‫) ‪ (Δ χ‬ﺼﻐﻴﺭﺍﹰ ﻓﺈﻥ ﺍﻟﺤﺩﻭﺩ ﺍﻟﻌﻠﻴﺎ ﻤـﻥ ﺍﻟﻔـﻙ‬ ‫‪2‬‬


‫(‬
‫‪= χ− χ‬‬
‫‪2‬‬
‫)‬ ‫ﻭﻟﻭ ﺃﻥ ﺍﻟﻼﺘﻌﻴﻴﻥ‬
‫ﻴﻤﻜﻥ ﺇﻫﻤﺎﻟﻬﺎ‪.‬‬
‫‪F (χ ) ≅ F ( χ )+ (χ − χ )F′ ( x‬‬ ‫)‬
‫‪≅ F( χ‬‬ ‫)‬
‫ﻭﻟﻤﺠﺎﻻﺕ ﻋﻴﻨﻴﺔ ‪ macroscopic helve‬ﻓـﺈﻥ ﺍﻟﻤﻌﺎﺩﻟـﺔ ﺍﻷﺨﻴـﺭﺓ ﺘﻌﺘﺒـﺭ‬
‫ﺼﺤﻴﺤﺔ ﻭﻫﺫﺍ ﻴﺠﻌﻠﻨﺎ ﺃﻥ ﻨﺤﺴﺏ ﻭﻨﻀﻴﻑ ﻤﺩﺍﺭ ﺍﻹﻟﻜﺘﺭﻭﻥ ﺃﻭ ﺍﻟﺒﺭﻭﺘﻭﻥ ﻓﻲ ﺘﻌﺠل‬
‫ﺒﺎﺴﺘﺨﺩﺍﻡ ﺍﻟﻤﻌﺎﺩﻻﺕ ﺍﻟﺘﻘﻠﻴﺩﻴﺔ‪.‬‬
‫ﺱ‪ /‬ﺇﺫﺍ ﻜﺎﻥ ‪ B , A‬ﻤﺅﺜﺭﻴﻥ ﻫﻴﺭﻤﻴﺘﻴﻴﻥ‪ ،‬ﺃﺜﺒﺕ ﺃﻥ‪:‬‬

‫‪ -1‬ﺍﻟﻤﺅﺜﺭ ‪ AB‬ﻴﻜﻭﻥ ﻓﻘﻁ ﻫﻴﺭﻤﻴﺘﻲ ﻟﻭ ﺃﻥ ‪ B , A‬ﻴﺘﺒﺎﺩﻟﻴﺎﻥ‪AB = BA ،‬‬

‫‪ -2‬ﺍﻟﻤﺅﺜﺭ ‪ (A + B)n‬ﻫﻭ ﻜﺫﻟﻙ ﻫﻴﺭﻤﻴﺘﻲ‪.‬‬


‫ﺍﻟﺤل‪/‬‬

‫‪ -1‬ﻟﻤﺅﺜﺭﻴﻥ ﻫﻴﺭﻤﻴﺘﻴﻴﻥ‪:‬‬

‫‪(AB) + = B + A + = BA‬‬

‫ﻭﻫﺫﺍ ﻴﺴﺎﻭﻱ ‪ AB‬ﻓﻘﻁ ﻟﻭ ﻜﺎﻥ ‪AB = BA‬‬

‫‪- ١٢٨ -‬‬


‫‪PDF created with pdfFactory Pro trial version www.pdffactory.com‬‬
‫‪-2‬‬
‫)‪[ (A + B) ] = (A + B‬‬
‫‪n +‬‬
‫‪(A + B)+ ................(A + B)+‬‬
‫‪+‬‬

‫)‪= (A + B)(A + B)..................... (A + B‬‬


‫)‪= (A + B‬‬
‫‪n‬‬

‫‪A A+‬‬ ‫ﻴﻜﻭﻨﻭ ﻫﻴﺭﻤﻴﻴﺘﺎﻥ ﻷﻱ ﻤﺅﺜﺭ ﻤﺜل‬ ‫) ‪i (A − A+‬‬ ‫‪, A + A+‬‬ ‫ﺱ‪ /‬ﺃﺜﺒﺕ ﺃﻥ‬
‫ﺍﻟﺤل‪/‬‬

‫‪( A + A+ )+‬‬ ‫‪= A + + (A + ) + = A + + A‬‬

‫]) ‪[i ( A - A‬‬


‫‪+‬‬ ‫‪+‬‬
‫(‬ ‫)‬
‫) ‪= (− i ) A + − (A + ) + = (−i) (A + − A‬‬
‫) ‪=i (A − A+‬‬

‫‪( A A + ) + = (A + ) + A + = A A +‬‬

‫ﺱ‪ /‬ﺃﺜﺒﺕ ﺃﻨﻪ ﺇﺫﺍ ﻜﺎﻨﺕ ‪ H‬ﻫﻭ ﻤﺅﺜﺭ ﻫﻴﺭﻤﻴﺘﻲ‪ ،‬ﻓﺈﻥ ﺍﻟﻤﺅﺜﺭ ﺍﻟﻬﻴﺭﻤﻴﺘﻲ ﺍﻟﻤﺭﺍﻓﻕ ﻟـ‬
‫ﺍﻟﻤﺅﺜﺭ ‪e - iH‬‬ ‫∑ ( ﻫﻭ‬ ‫)ﻭﺍﻟﺫﻱ ﻴﻌﺭﻑ ﺒـ‬
‫∞‬
‫!‪in Hn / n‬‬ ‫‪e iH‬‬
‫‪n =0‬‬

‫ﺍﻟﺤل‪/‬‬
‫‪+‬‬
‫‪ ∞ in n ‬‬ ‫‪(−i) n‬‬
‫∑ = ‪(e ) =  ∑ H ‬‬
‫‪iH‬‬
‫‪‬‬‫‪+‬‬
‫‪‬‬ ‫‪Hn‬‬ ‫) (‬ ‫‪+‬‬

‫!‪ 0 n‬‬ ‫‪‬‬ ‫!‪n‬‬


‫‪(−i) n n − iH‬‬
‫∑=‬ ‫‪H =e‬‬
‫!‪n‬‬

‫‪H 4 =1‬‬ ‫ﺱ‪ /‬ﺨﺫ ﺍﻟﻤﺅﺜﺭ ﺍﻟﻬﻴﺭﻤﻴﺘﻲ ‪ H‬ﻭﺍﻟﺫﻱ ﻟﻪ ﺍﻟﺨﺎﺼﻴﺔ‬

‫ﺃﻭﺠﺩ ﺍﻟﻘﻴﻡ ﺍﻟﺨﺎﺼﺔ )ﺍﻟﺫﺍﺘﻴﺔ( ﻟﻠﻤﺅﺜﺭ ‪H‬؟‬


‫ﺃﻭﺠﺩ ﺍﻟﻘﻴﻡ ﺍﻟﺨﺎﺼﺔ )ﺍﻟﺫﺍﺘﻴﺔ( ﻟﻠﻤﺅﺜﺭ ‪ H‬ﻟﻭ ﺃﻥ ‪ H‬ﻏﻴﺭ ﻤﻘﻴـﺩ ﺒـﺄﻥ ﻴﻜـﻭﻥ‬
‫ﻫﻴﺭﻤﻴﺘﻲ؟‬

‫‪- ١٢٩ -‬‬


‫‪PDF created with pdfFactory Pro trial version www.pdffactory.com‬‬
‫‪H 4 =1‬‬ ‫ﺍﻟﺤل‪ /‬ﻜﻭﻥ‬

‫‪ψ‬‬ ‫ﻟﻜل ﻗﻴﻡ‬ ‫‪(H 4 − 1) ψ = 0‬‬ ‫ﻤﻌﻨﺎﻩ‬


‫‪( H + 1) ( H − 1) ( H + i) ( H − i) ψ = 0‬‬ ‫ﻭﻫﺫﺍ ﻴﻤﻜﻥ ﻜﺘﺎﺒﺘﻪ‪:‬‬
‫‪(H −i)ψ=0‬‬ ‫ﻭﻋﻠﻴﻪ ﻓﺈﻥ‬

‫‪Hψ − iψ = 0 ⇒ Hψ = i ψ‬‬ ‫ﻭﻓﻲ ﻫﺫﻩ ﺍﻟﺤﺎﻟﺔ‬


‫ﺃﻱ ﺃﻥ ﺍﻟﻘﻴﻡ ﺍﻟﺨﺎﺼﺔ = ‪i‬‬

‫‪( H − i ) =/ 0‬‬ ‫ﺃﻭ‬


‫‪(H −i)ψ = φ‬‬ ‫ﻓﻲ ﻫﺫﻩ ﺍﻟﺤﺎﻟﺔ ﻨﻀﻊ‬
‫ﻓﻲ ﻫﺫﻩ ﺍﻟﺤﺎﻟﺔ‬ ‫‪( H + 1 ) ( H − 1 ) ( H + i) φ = 0‬‬ ‫⇐‬

‫‪(H +i)φ=0‬‬ ‫⇒‬ ‫ﺃﻤﺎ‬

‫ﺍﻟﻘﻴﻤﺔ ﺍﻟﺨﺎﺼﺔ ﻫﻲ ‪– i‬‬ ‫⇐‬

‫‪(H +i)φ=x‬‬ ‫ﺃﻭ ﺃﻥ‬

‫‪⇒ ( H + 1) ( H − 1 ) x = 0‬‬

‫‪(H −1 ) x = 0‬‬ ‫ﺃﻤﺎ‬


‫‪( −1) ................+ 1‬‬ ‫⇐ ﺍﻟﻘﻴﻤﺔ ﺍﻟﺨﺎﺼﺔ‬
‫‪± 1, ± i‬‬ ‫ﻭﺒﻬﺫﻩ ﺍﻟﻁﺭﻴﻘﺔ ﻭﻀﺤﻨﺎ ﺃﻥ ﺍﻟﻘﻴﻡ ﺍﻟﺨﺎﺼﺔ ﺘﻜﻭﻥ‬

‫‪H = H+‬‬ ‫ﺇﺫﺍ ﻜﺎﻥ‬


‫ﻓﻘﻁ‪.‬‬ ‫‪±1‬‬ ‫ﻓﺈﻥ ﺍﻟﻘﻴﻡ ﺍﻟﺨﺎﺼﺔ ﻴﺠﺏ ﺃﻥ ﺘﻜﻭﻥ ﺤﻘﻴﻘﻴﺔ ﻭﻋﻠﻴﻪ ﻓﺎﻟﻘﻴﻡ ﺍﻟﺼﺤﻴﺤﺔ ﻫﻲ‬

‫‪- ١٣٠ -‬‬


‫‪PDF created with pdfFactory Pro trial version www.pdffactory.com‬‬
‫ﻫﻭ ‪ unitary‬ﻟﻘﺩ ﺃﺜﺒﺕ ﻤﻥ‬ ‫‪e iA‬‬ ‫ﺱ‪ /‬ﻭﻀﺢ ﺃﻨﻪ ﺇﺫﺍ ﻜﺎﻥ ‪ A‬ﻫﻭ ﻤﺅﺜﺭ ﻫﻴﺭﻤﻴﺘﻲ‪ ،‬ﻓﺈﻥ‬
‫) ‪(e‬‬
‫‪iH +‬‬
‫‪= e −iH‬‬ ‫ﻗﺒل ﺃﻨﻪ ﻟﻜل ﻤﺅﺜﺭ ﻫﻴﺭﻤﻴﺘﻲ‬
‫ﻭﻋﻠﻴﻪ‪:‬‬

‫‪(e ) (e )= e‬‬
‫‪iA +‬‬ ‫‪iA‬‬ ‫‪− iA‬‬
‫‪, e iA = 1‬‬

‫‪λ‬‬ ‫ﻭﻴﻤﻜﻥ ﺘﻭﻀﻴﺢ ﻫﺫﺍ ﻜﺫﻟﻙ ﻤﻥ ﺨﻼل ﻤﻌﺎﻤﻼﺕ ﻟﻬﺎ ﻗﻭﻯ ﻤﺨﺘﻠﻔﺔ ﻓﻲ‬
‫‪(iλi 2 2 (iλi 3 3‬‬ ‫‪(iλi 2 2 (iλi 3 3‬‬
‫‪(1 + iλλ +‬‬ ‫‪H +‬‬ ‫‪H + ..........) (1 − iλλ +‬‬ ‫‪H −‬‬ ‫)‪H + ..........‬‬
‫!‪2‬‬ ‫!‪3‬‬ ‫!‪2‬‬ ‫!‪3‬‬

‫ﻤﺤـﺼﻭل ﻋﻠـﻰ‬ ‫‪χ‬‬ ‫ﻭﺍﻟﻤﻭﻗﻊ‬ ‫‪p‬‬ ‫ﺱ‪ /‬ﺍﺴﺘﺨﺩﻡ ﻋﻼﻗﺎﺕ ﺍﻟﺘﺒﺩﻴل ﺒﻴﻥ ﻜﻤﻴﺔ ﺍﻟﺤﺭﻜﺔ‬
‫ﻋﻠـﻰ ﺍﻟـﺯﻤﻥ‪ ،‬ﻤﻌﻁـﻰ‬ ‫‪p‬‬ ‫ﻭ‬ ‫‪χ‬‬ ‫ﺍﻟﻤﻌﺎﺩﻻﺕ ﺍﻟﺘﻲ ﺘﺼﻑ ﺍﻋﺘﻤﺎﺩ ﻜـل ﻤـﻥ‬
‫ﺍﻟﻬﺎﻤﻠﺘﻭﻨﻴﺎﻥ‪.‬‬
‫‪2‬‬
‫‪(a ) H = p‬‬ ‫‪1‬‬
‫)∈ ‪+ m ( ω12 χ 2 + ω 2χ +‬‬
‫‪2m 2‬‬
‫‪p2 1‬‬
‫‪(b ) H = + m ω 2 χ 2 − A2‬‬
‫‪2m 2‬‬ ‫‪χ‬‬

‫ﺍﻟﺤل‪/‬‬
‫‪i  p2‬‬ ‫‪ i p‬‬
‫‪χ = [ H , χ ]= ‬‬ ‫{[‬
‫] ‪p, χ‬‬
‫‪d‬‬ ‫‪i‬‬
‫‪,‬‬ ‫= ‪χ‬‬
‫‪dt‬‬ ‫‪h‬‬ ‫‪h  2m‬‬ ‫‪ h m h/i‬‬
‫‪p‬‬
‫=‬
‫‪m‬‬
‫‪dp i‬‬
‫‪dt h‬‬
‫‪i‬‬
‫‪h‬‬
‫[‬
‫‪= [H, p ]= − p, m ω12 χ 2 /2 + ω 2 χ‬‬ ‫]‬
‫‪i h‬‬ ‫‪χ‬‬ ‫‪‬‬
‫‪= − ,  2 mω12 + ω 2  = − mω12 χ - ω 2‬‬
‫‪h i‬‬ ‫‪2‬‬ ‫‪‬‬
‫‪d x 1  dp ‬‬
‫‪2‬‬
‫‪ω‬‬
‫‪=   = − ω12 χ − 2‬‬
‫‪dt‬‬ ‫‪m  dt ‬‬ ‫‪m‬‬

‫ﻭﺍﻟﺤل ﻟﻠﻤﻌﺎﺩﻟﺔ ﻫﻭ‬


‫‪ω2‬‬ ‫)‪p(0‬‬
‫‪χ =−‬‬ ‫‪2‬‬
‫‪+ x (0 ) cos ω1 t +‬‬ ‫‪sin ω1 t‬‬
‫‪mω‬‬ ‫‪1‬‬
‫‪mω1‬‬

‫‪- ١٣١ -‬‬


‫‪PDF created with pdfFactory Pro trial version www.pdffactory.com‬‬
dχ p
= (b)
dt m
dp i x2 A  2A
= −  p , m ω2 − 2  = − mω 2 χ − 3
dt h 2 x  x

.‫ ﻭﻁﺎﻗﺘﻪ ﺘﻭﺼﻑ ﺒﻤﺅﺜﺭ ﻟﻬﺎﻤﻠﺘﻭﻨﻴﺎﻥ‬،‫ﺇﻟﻜﺘﺭﻭﻥ ﻴﺘﺫﺒﺫﺏ ﻓﻲ ﻤﺠﺎل ﻜﻬﺭﺒﺎﺌﻲ‬


p2
H= − ( e E ο cos wt ) χ
2m
 dH   dp   dx 
 , ,  ‫ﺍﺤﺴﺏ‬
 dt   dt   dt 

/‫ﺍﻟﺤل‬
i  p2  i p
= [ H, χ ]=  [p , x ]= p
dx i
, x =
dt h h  2m  h m m
i  p2 
= [ H, p ]= −  p , − (eE ο cos ω t )χ 
dp i
dt h h  2m 
=−
i
[p , − (eE ο cos ω t )χ ]= − i , h eE ο cos ω t = − eE ο cos ω t
h h i
dH ∂H
= = eE ο sin ω t
dt ∂t

- ١٣٢ -
PDF created with pdfFactory Pro trial version www.pdffactory.com
‫ﻤﻌﺎﺩﻟﺔ ﺸﺭﻭﺩﻨﺠﺭ ﻓﻲ ﺜﻼﺜﺔ ﺃﺒﻌﺎﺩ‪:‬‬
‫‪The Schrödinger Equation in three Dimensions‬‬
‫ﺍﻟﺠﻬﺩ ﺍﻟﻤﺭﻜﺯﻱ‪The Central Potential :‬‬

‫)‪V (x, y, t‬‬ ‫ﻓﻲ ﻫﺫﺍ ﺍﻟﻔﺼل ﺴﻭﻑ ﻨﻨﺎﻗﺵ ﺤﺎﻟﺔ ﻤﻬﻤﺔ ﻭﺍﻟﺘﻲ ﻴﻜﻭﻥ ﻓﻴﻬﺎ ﺍﻟﺠﻬﺩ‬
‫ﻓﻘﻁ‪ .‬ﻭﻟﻨﻅﺎﻡ ﻴﺘﻜﻭﻥ ﻤﻥ ﺠﺴﻴﻤﻴﻥ‪ ،‬ﺤﻴﺙ ﺍﻟﺠﻬﺩ ﻴﻌﺘﻤﺩ‬ ‫‪r = x2 + y 2 + Z 2‬‬ ‫ﻴﻌﺘﻤﺩ ﻋﻠﻰ‬
‫ﻋﻠﻰ ﺍﻟﻤﺴﺎﻓﺔ ﺍﻟﻔﺎﺼﻠﺔ ﺒﻴﻨﻬﻤﺎ ﻓﻘﻁ‪ ،‬ﻓﺈﻥ ﺍﻟﻬﺎﻤﻠﺘﻭﻨﻴﺎﻥ ﻟﻪ ﺍﻟﺸﻜل‪.‬‬
‫‪p12‬‬ ‫‪p 22‬‬
‫=‪H‬‬ ‫‪+‬‬ ‫) ‪+ V ( r1 − r2‬‬ ‫)‪(1‬‬
‫‪2m1‬‬ ‫‪2m 2‬‬

‫ﻭﺒﺎﻟﺘﺤﻠﻴل ﺍﻟﻌﺎﺩﻱ ﻟﻤﺭﻜﺯ ﺍﻟﻜﺘﻠﺔ ﻭﺍﻹﺤﺩﺍﺜﻲ ﺍﻟﻨﺴﺒﻲ‪.‬‬


‫‪m1 r1 + m 2 r 2‬‬
‫=‪R‬‬ ‫)‪( 2‬‬
‫‪m1 + m 2‬‬
‫‪r = r1 − r 2‬‬

‫ﻜﻤﻴﺔ ﺍﻟﺤﺭﻜﺔ ﺍﻟﻜﻠﻴﺔ ﻭﺍﻟﻨﺴﺒﻴﺔ‬


‫‪p = p1 + p 2‬‬
‫‪m 2 p1 − m1 p 2‬‬
‫=‪p‬‬ ‫)‪( 3‬‬
‫‪m1 + m 2‬‬

‫ﺍﻟﻬﺎﻤﻠﺘﻭﻨﻴﻥ ﻴﺄﺨﺫ ﺍﻟﺸﻜل‪.‬‬


‫‪2‬‬ ‫‪2‬‬
‫‪p‬‬ ‫‪p‬‬
‫=‪H‬‬ ‫‪+‬‬ ‫) ‪+ V( r‬‬ ‫)‪( 4‬‬
‫‪2M‬‬ ‫‪2M‬‬

‫ﻫﻲ ﺍﻟﻜﺘﻠﺔ ﺍﻟﻜﻠﻴﺔ ﻟﻠﻨﻅﺎﻡ‬ ‫‪M‬‬ ‫ﻫﻨﺎ‬

‫‪M = m1 + m 2‬‬ ‫)‪(5‬‬

‫‪μ‬‬ ‫ﺍﻟﻜﺘﻠﺔ ﺍﻟﻤﺨﺘﺯﻟﺔ )ﺍﻟﻤﺨﻔﻔﺔ(‬


‫‪m1 m 2‬‬
‫=‪μ‬‬ ‫)‪(6‬‬
‫‪M‬‬

‫ﺒﺈﻤﻜﺎﻨﻨﺎ ﺃﻥ ﻨﺘﺄﻜﺩ ﻤﻥ ﺃﻥ‪:‬‬

‫‪- ١٣٣ -‬‬


‫‪PDF created with pdfFactory Pro trial version www.pdffactory.com‬‬
‫‪[p‬‬ ‫‪i‬‬
‫‪, Rj‬‬ ‫‪]hi sij‬‬
‫‪[p‬‬ ‫‪i‬‬
‫‪, rj‬‬ ‫‪]hi sij‬‬ ‫) ‪(7‬‬

‫ﻭﺒﻤﺎ ﺃﻥ ﺍﻟﺠﻬﺩ ﻻ ﻴﻌﺘﻤﺩ ﻋﻠﻰ ﺇﺤﺩﺍﺜﻲ ﻤﺭﻜﺯ ﺍﻟﻜﺘﻠﺔ ‪ ، R‬ﻓﺈﻥ ﻤـﺅﺜﺭ ﻜﻤﻴـﺔ‬
‫ﺍﻟﺤﺭﻜﺔ ﺍﻟﻜﻠﻴﺔ ‪ P‬ﺘﺒﺎﺩﻟﻲ ﻤﻊ ‪ H‬ﻭﺒﺈﻤﻜﺎﻨﻨﺎ ﺍﻟﺤﺼﻭل ﻋﻠﻰ ﺍﻟﺩﻭﺍل ﺍﻟﺨﺎﺼﺔ ﻟﻜل ﻤﻥ‪P‬‬
‫ﻭ ‪ H‬ﺍﻟﺩﻭﺍل ﺍﻟﺨﺎﺼﺔ ﻟـ‪ P‬ﻟﻬﺎ ﺍﻟﺸﻜل‪.‬‬
‫‪1‬‬
‫= ) ‪U (P , R‬‬ ‫‪3‬‬
‫‪e i P, R / h‬‬ ‫)‪(8‬‬
‫)‪( 2 D h‬‬ ‫‪2‬‬

‫ﻭﺍﻟﻤﻌﺎﺩﻟﺔ ﺍﻟﺨﺎﺼﺔ ﻟـ ‪ H‬ﻴﻤﻜﻥ ﻜﺘﺎﺒﺘﻬﺎ ﻋﻠﻰ ﺍﻟﺼﻭﺭﺓ‪.‬‬

‫)‪ψ(R ,r) =U(P,R )uE (r‬‬ ‫)‪(9‬‬

‫ﺤﻴﺙ‪:‬‬
‫‪ p2‬‬ ‫‪‬‬
‫‪‬‬ ‫) ‪+ V (r ) u E (r ) = E u E (r‬‬ ‫)‪(10‬‬
‫‪ 2μ‬‬ ‫‪‬‬
‫‪P2‬‬
‫‪E = E tot −‬‬ ‫‪,‬‬ ‫‪M =w‬‬
‫‪2M‬‬

‫ﻫﻲ ﺍﻟﻁﺎﻗﺔ ﺍﻟﺩﺍﺨﻠﻴﺔ‪ ،‬ﺒﻌﺩ ﻁﺭﺡ ﻁﺎﻗﺔ ﺍﻟﺤﺭﻜﺔ ﻟﻠﺠﺴﻴﻤﻴﻥ‪.‬‬


‫ﻴﻤﻜﻨﻨﺎ ﻜﺘﺎﺒﺔ ﻫﺫﻩ ﺍﻟﻤﻌﺎﺩﻟﺔ ﻋﻠﻰ ﺍﻟﺼﻭﺭﺓ‬
‫‪ h2 2‬‬ ‫‪‬‬
‫‪ −‬‬ ‫) ‪Δ r + V (r ) u E (r ) = E u E (r‬‬ ‫)‪(1‬‬
‫‪ 2μ‬‬ ‫‪‬‬

‫‪φ‬‬ ‫ﺃﻭ‬ ‫‪θ‬‬ ‫ﺃﻱ ﻻ ﺘﻌﺘﻤﺩ ﻋﻠﻰ‬ ‫‪r‬‬ ‫ﻻﺤﻅ ﺃﻥ ﻫﺫﻩ ﺍﻟﻤﻌﺎﺩﻟﺔ ﻻ ﺘﻌﺘﻤﺩ ﻋﻠﻰ ﺍﺘﺠﺎﻩ‬

‫‪Consequences of Rotational Invariance‬‬ ‫ﻨﺘﺎﺌﺞ ﻋﺩﻡ ﺍﻟﺘﻐﻴﻴﺭ ﺒﺎﻟﺩﻭﺭﺍﻥ‬


‫ﻓﻲ ﻫﺫﺍ ﺍﻟﺠﺯﺀ ﺴﻨﻭﻀﺢ ﺃﻥ )‪ (11‬ﻴﻤﻜﻥ ﺃﻥ ﺘﻔﺼل ﺒﺤﻴـﺙ ﺃﻥ ﺍﻹﺤـﺩﺍﺜﻲ‬
‫ﺍﻟﺨﻁﻲ ‪ r‬ﻴﻅﻬﺭ ﻓﻲ ﻤﻌﺎﺩﻟﺔ ﻁﺎﻗﺔ ﺍﻟﻘﻴﻤﺔ ﺍﻟﺨﺎﺼﺔ‪.‬‬

‫‪- ١٣٤ -‬‬


‫‪PDF created with pdfFactory Pro trial version www.pdffactory.com‬‬
‫ﻓﻲ ﺍﻟﻤﻴﻜﺎﻨﻴﻜﺎ ﺍﻟﺘﻘﻠﻴﺩﻴﺔ‪ ،‬ﺍﻟﻔﺼل ﻴﻌﺘﻤﺩ ﻋﻠﻰ ﺍﺴﺘﺨﺩﺍﻡ ﻜﻤﻴﺔ ﺍﻟﺤﺭﻜﺔ ﺍﻟﺯﺍﻭﻴـﺔ‬
‫ﺒﺄﺨﺫ‪:‬‬
‫‪L =r×p‬‬ ‫) ‪(12‬‬
‫⇒‬
‫‪2‬‬
‫) ( ) (‬
‫‪L = r×p . r×p = r p − r −p‬‬
‫‪2‬‬ ‫‪2‬‬
‫(‬ ‫)‬‫‪2‬‬

‫‪= (r . p ) + L‬‬
‫‪1‬‬ ‫‪1‬‬
‫‪2‬‬ ‫‪2‬‬
‫‪p2‬‬ ‫‪2‬‬ ‫‪2‬‬
‫‪r‬‬ ‫‪r‬‬
‫‪= p 2r +‬‬
‫‪1 2‬‬
‫‪L‬‬ ‫)‪(13‬‬
‫‪r2‬‬

‫ﻭﻟﺠﻬﺩ ﻤﺭﻜﺯﻱ ) ‪ V‬ﺘﻌﺘﻤﺩ ﻋﻠﻰ ‪ r‬ﻓﻘﻁ( ﺍﻟﻘﻭﺓ ﺘﻜﻭﻥ ﻨﺼﻑ ﻗﻁﺭﻴﺔ ‪radial‬‬
‫ﻭﻻ ﺘﺒﺫل ﻋﺯﻡ ﻋﻠﻰ ﺍﻟﻨﻅﺎﻡ‪ .‬ﻭﻫﻜﺫﺍ ﻓﺈﻥ ‪ L‬ﺜﺎﺒـﺕ ﻟﻠﺤﺭﻜـﺔ‪ ،‬ﻭ ‪ L2‬ﻓﻘـﻁ ﻭﻟﻬـﺫﺍ‬
‫ﻓﺎﻟﻤﻌﺎﺩﻟﺔ‪.‬‬
‫‪1 2‬‬
‫=‪E‬‬ ‫)‪P + V (r‬‬
‫‪2μ‬‬

‫ﺘﺘﻀﻤﻥ ﺍﻹﺤﺩﺍﺜﻴﺎﺕ ﺍﻟﺨﻁﻴﺔ ﻓﻘﻁ‪ .‬ﻭﻨﻔﺱ ﺍﻷﻤﺭ ﺼﺤﻴﺢ ﻓﻲ ﻤﻴﻜﺎﻨﻴﻜﺎ ﺍﻟﻜﻡ‪.‬‬

‫ﻭﻓﻲ ﻫﺫﺍ ﺍﻟﻔﺼل‪:‬‬


‫‪ -1‬ﺴﻨﺤﺩﺩ ﻤﺅﺜﺭﺍﺕ ﻜﻤﻴﺔ ﺍﻟﺤﺭﻜﺔ ﻤﻥ ﺍﺤﺘﻴﺎﺝ ﺃﻥ ﺍﻟﻬﺎﻤﻠﺘﻭﻨﻴﻥ ﻏﻴﺭ ﻤﺘﻐﻴﺭ ﺘﺤـﺕ‬
‫ﺍﻟﺩﻭﺭﺍﻥ‪.‬‬

‫‪ -2‬ﺍﺸﺘﻘﺎﻕ ﺍﻟﻤﻌﺎﺩﻟﺔ ﺍﻟﺨﻁﻴﺔ‪.‬‬


‫ﻻ ﺘﻌﺘﻤﺩ ﻋﻠﻰ ﺍﻟﺩﻭﺭﺍﻥ(‬ ‫‪∆2‬‬ ‫ﻋﺩﻡ ﺍﻟﺘﻐﻴﺭ ﺘﺤﺕ ﺍﻟﺩﻭﺭﺍﻥ ﺤﻭل ﻤﺤﻭل ‪) Z‬ﻹﺜﺒﺎﺕ ﺃﻥ‬
‫‪Invariance under Rotations about the z-axis‬‬
‫ﺤﻭل ﻤﺤﻭﺭ ‪Z‬‬ ‫‪θ‬‬ ‫ﺨﺫ ﺤﺎﻟﺔ ﺨﺎﺼﺔ ﻟﺩﻭﺭﺍﻥ ﺨﻼل ﺯﺍﻭﻴﺔ‬
‫‪χ ′ = χ cos θ − y sin θ‬‬
‫‪y′ = χ sin θ − y cos θ‬‬ ‫)‪(14‬‬

‫ﺒﺈﻤﻜﺎﻨﻨﺎ ﺃﻥ ﻨﺭﻯ ﺃﻥ‬

‫‪- ١٣٥ -‬‬


‫‪PDF created with pdfFactory Pro trial version www.pdffactory.com‬‬
1 1
r ′ = ( x′ 2 + y′ 2 + Z ′ 2 ) 2
= ( χ 2 + y2 + Z 2 ) 2
=r
2 2 2 2
 ∂   ∂   ∂ ∂   ∂ ∂ 
  +   =  cos θ − sin θ  +  sin θ + cos θ 
 ∂ x′   ∂ y′   ∂x ∂ y  ∂x ∂ y 
2
∂  ∂ 
2

=   +  
 ∂x   ∂y 

‫ ﻻ ﻴﺘﺄﺜﺭ ﺒﺎﻟﺩﻭﺭﺍﻥ ﻭﺒﻤﺎ ﺃﻥ ﺍﻟﻬﺎﻤﻠﺘﻭﻨﻴﻥ ﻟﻪ ﺨﺎﺼـﺔ ﻋـﺩﻡ‬H ‫ﻭﻫﺫﺍ ﻴﻌﻨﻲ ﺃﻥ‬


‫ ﻭﻟﺘﺤﺩﻴﺩ ﺍﻟﻤﺅﺜﺭﺍﺕ ﺍﻟﺘﻲ ﺘﺘﺒﺎﺩل ﻤﻊ‬،‫ ﻓﻨﺤﻥ ﻨﺘﻭﻗﻊ ﻗﺎﻨﻭﻥ ﺍﻟﺤﻔﻅ‬Invariace ‫ﺍﻟﺘﻐﻴﻴﺭ‬
.‫ﻓﻘﻁ‬ θ ‫ ﻭﺍﺤﺘﻔﻅ ﺒﺎﻟﺤﺩﻭﺩ ﻟـ‬Z ‫ ﺨﺫ ﺩﻭﺭﺍﻥ ﻤﺘﻨﺎﻫﻲ ﻓﻲ ﺍﻟﺼﻐﺭ ﺤﻭل ﻤﺤﻭﺭ‬،H
χ′ = χ − θ y
y′ = y + x θ (15)

:‫ﻨﺤﻥ ﻨﻠﺯﻡ ﺃﻥ‬

H U E (x − θ y , y + θ x , Z ) = E U E (x − θ y , y + θ x , Z) (16)

f ( x + Δ ) = f (x) +
af
(Δ x) ‫ﻭﻁﺭﺤﻨﺎ ﻤﻨﻪ‬ θ ‫ﻭﻟﻭ ﻓﻜﻜﻨﺎ ﻫﺫﺍ ﻟﻠﺤﺩ ﺍﻷﻭل ﻓﻲ‬
ax

H U E (x , y , Z ) = E U E (x , y , Z) (17)

:‫ﻨﺘﺤﺼل ﻋﻠﻰ‬
 ∂ ∂   ∂ ∂ 
H  x − y  U E (x, t, z ) = E  x − y  U E (x, y, z ) (18)
 ∂y ∂x   ∂y ∂x 

.‫ﺍﻟﺠﺎﻨﺏ ﺍﻷﻴﻤﻥ ﻓﻲ ﻫﺫﻩ ﺍﻟﻤﻌﺎﺩﻟﺔ ﻴﻤﻜﻥ ﻜﺘﺎﺒﺘﻪ ﻋﻠﻰ ﺍﻟﺼﻭﺭﺓ‬


 ∂ ∂   ∂ ∂ 
 x − y  E U E ( x, y, z) =  x − y  H U E ( x, y, z ) (19)
 ∂y ∂x   ∂y ∂x 

.H ‫ﺘﺒﺎﺩﻟﻲ ﻤﻊ‬  ∂ ∂ 
 x − y  ‫ﺃﻱ ﺃﻥ ﺍﻟﻤﺅﺜﺭ‬
 ∂y ∂x 

:‫ﻓﺈﻥ‬ L =r , P ‫ﻟﻭ ﻋﺭﻓﻨﺎ‬


h ∂ ∂ 
L z =  x − y  = x Py − y Px (20)
i  ∂y ∂x 

.‫( ﻤﻌﺎﹰ‬19) ‫( ﻭ‬18) ‫ﺍﻟﻤﻌﺎﺩﻟﺘﻴﻥ‬

- ١٣٦ -
PDF created with pdfFactory Pro trial version www.pdffactory.com
‫‪(H L z − L z H) U E (x, y, z) = 0‬‬

‫ﺘﻜﻭﻥ ﻓﺌﺔ ﻭﻫﺫﺍ ﻴﻌﻨﻲ ﺃﻥ ﻋﻼﻗﺔ ﺍﻟﻤﺅﺜﺭ‪.‬‬ ‫) ‪U E (r‬‬ ‫ﺒﻤﺎ ﺃﻥ‬

‫‪[H , L ] = 0‬‬
‫‪z‬‬
‫)‪(21‬‬

‫ﻟﻠﻤﺅﺜﺭ‪.‬‬ ‫‪z−‬‬ ‫ﻫﻭ ﺍﻟﻤﺭﻜﺒﺔ‬ ‫‪Lz‬‬ ‫ﺘﻜﻭﻥ ﺼﺤﻴﺤﺔ‪.‬‬

‫‪L =r‬‬ ‫‪× P‬‬ ‫)‪(22‬‬

‫ﻭﻫﻭ ﻜﻤﻴﺔ ﺍﻟﺤﺭﻜﺔ ﺍﻟﺯﺍﻭﻴﺔ‪ .‬ﻭﻟﻭ ﺃﺨﺫﻨﺎ ﺍﻟﺩﻭﺭﺍﻥ ﺤﻭل ‪ y, x‬ﺒﺈﻤﻜﺎﻨﻨﺎ ﺃﻥ ﻨﺠﺩ‬
‫ﺍﻟﻌﻼﻗﺎﺕ‪.‬‬
‫‪[H , L ] = 0‬‬
‫‪[H , L ] = 0‬‬
‫‪x‬‬

‫‪y‬‬
‫)‪(23‬‬

‫ﻭﻫﻜﺫﺍ‪ ،‬ﻓﺈﻥ ﻤﺅﺜﺭﺍﺕ ﻜﻤﻴﺔ ﺍﻟﺤﺭﻜﺔ ﺍﻟﺯﺍﻭﻴﺔ ﺍﻟﺜﻼﺜﺔ ﻤﺘﺒﺎﺩﻟﺔ ﻤـﻊ ‪ H‬ﺃﻱ ﺃﻥ‬
‫ﻜﻤﻴﺔ ﺍﻟﺤﺭﻜﺔ ﺍﻟﺯﺍﻭﻴﺔ ﺜﺎﺒﺕ ﻓﻲ ﺍﻟﺤﺭﻜﺔ‪ .‬ﻭﻫﺫﺍ ﻴﻭﺍﺯﻱ ﺍﻟﻨﺘﻴﺠﺔ ﺍﻟﺘﻘﻠﻴﺩﻴﺔ ﻤﻥ ﺤﻴـﺙ‬
‫ﺃﻥ ﺍﻟﻘﻭﻯ ﺍﻟﻤﺭﻜﺯﻴﺔ ﻴﻨﺘﺞ ﻋﻨﻬﺎ ﺤﻔﻅ ﻜﻤﻴﺔ ﺍﻟﺤﺭﻜﺔ ﺍﻟﺯﺍﻭﻴﺔ‪.‬‬
‫ﻋﻼﻗﺎﺕ ﺍﻟﺘﺒﺩﻴل ﻟﻜﻤﻴﺔ ﺍﻟﺤﺭﻜﺔ ﺍﻟﺯﺍﻭﻴﺔ‪.‬‬
‫‪‬‬
‫‪[L‬‬ ‫[ ]‬
‫‪, L y = yp z − zp y , zp x − xp z‬‬ ‫]‬ ‫‪‬‬
‫‪‬‬
‫‪= [yp‬‬ ‫[ ]‬ ‫]‬
‫‪x‬‬

‫‪, zp y − zp y , zp x‬‬ ‫‪− [yp z , xp z ] ‬‬


‫‪+ [ zp‬‬ ‫]‬
‫‪z‬‬
‫‪‬‬
‫‪y‬‬
‫‪, xp z‬‬ ‫‪‬‬ ‫)‪(24‬‬
‫‪‬‬
‫‪= y [ Pz , z ] Px + x [z , p z ]p y‬‬ ‫‪‬‬
‫‪‬‬
‫=‬
‫‪h‬‬
‫‪(yp x − xp x ) = ih L z‬‬ ‫‪‬‬
‫‪i‬‬ ‫‪‬‬

‫ﻭﺒﺎﻟﻤﺜل‪:‬‬

‫‪[L‬‬ ‫‪y‬‬
‫]‬
‫‪, L z = ih L x‬‬ ‫)‪(25‬‬

‫‪[L‬‬ ‫‪z‬‬
‫‪, L x ] = ih L y‬‬ ‫)‪(26‬‬

‫ﻋﻼﻗﺎﺕ ﺍﻟﺘﺒﺩﻴل )‪ (24‬ﻭ )‪ (26‬ﻴﻤﻜﻥ ﺘﻠﺨﻴﺼﻬﺎ ﺒﺎﻟﻘﺎﻨﻭﻥ‬

‫‪- ١٣٧ -‬‬


‫‪PDF created with pdfFactory Pro trial version www.pdffactory.com‬‬
‫‪L× L = i h L‬‬

‫ﻭﺍﻟﻨﺘﻴﺠﺔ ﺍﻟﻤﺘﻭﻗﻌﺔ ﻋﻠﻰ ﻋﻼﻗﺎﺕ ﺍﻟﺘﺒﺩﻴل ﻫﺫﻩ ﻫﻲ ﺃﻨﻪ ﺘﻭﺠﺩ ﻓﻘـﻁ ﻤﺭﻜﺒـﺔ‬
‫ﻴﻤﻜﻥ ﺍﺨﺘﻴﺎﺭﻫﺎ ﻟﺘﺒﺎﺩل ﻤﻊ ‪ H‬ﻟﺘﻜﻭﻥ ﻓﺌﺔ ﻤﻥ ﺍﻟﻜﻤﻴﺎﺕ ﺍﻟﻤﺸﺎﻫﺩﺓ‪.‬‬ ‫‪L‬‬ ‫ﻭﺍﺤﺩﺓ ﻟـ‬
‫ﺸﺭﺡ ﻤﻔﺼل ﻟﻠﺨﻁﻭﺓ‪:‬‬
‫‪[ L , L ] = [yp‬‬ ‫‪− zp y , zp x − xp z‬‬ ‫]‬
‫[‬ ‫]‬ ‫[‬ ‫]‬
‫‪x‬‬ ‫‪y‬‬ ‫‪z‬‬

‫‪= [yp z , zp x ]− zp y , zp x − [ yp z , xp z ] + zp y , xp z‬‬

‫ﻭﻟﺘﻭﻀﻴﺢ ﻫﺫﺍ‪ ،‬ﻨﻔﺘﺭﺽ ﺃﻨﻪ ﻴﻭﺠﺩ ﻟﺩﻴﻨﺎ ﺍﻟﺩﺍﻟﺔ ﺍﻟﺨﺎﺼﺔ ﻟﻠﻤﺅﺜﺭ ‪ Lx‬ﻭﺍﻟﺫﻱ ﻟﻪ‬
‫ﻗﻴﻤﺔ ﺨﺎﺼﺔ ﻭﺍﻟﺘﻲ ﻫﻲ ﺘﻜﻭﻥ ﺩﺍﻟﺔ ﺨﺎﺼﺔ ﻟﻠﻤﺅﺜﺭ ‪ Ly‬ﺒﻘﻴﻤﺔ ﺨﺎﺼﺔ ‪.L2‬‬
‫ﻭﻫﻜﺫﺍ ﺒﺎﺴﺘﻁﺎﻋﺘﻨﺎ ﺍﺨﺘﻴﺎﺭ ﻓﺌﺔ ﻜﺎﻤﻠﺔ ﻤـﻥ ﺍﻟﻜﻤﻴـﺎﺕ ﺍﻟﻤـﺸﺎﻫﺩﺓ ﺍﻟﻤﺘﺒﺎﺩﻟـﺔ‬
‫‪L2 , H L z‬‬ ‫ﻟﻠﻤﺅﺜﺭﺍﺕ‬
‫ﻓﺼل ﺍﻟﻤﺘﻐﻴﺭﺍﺕ ﻓﻲ ﻤﻌﺎﺩﻟﺔ ﺸﺭﻭﺩﻨﺠﺭ‬
‫‪Separation of Variables is the Schrödinger Equation‬‬
‫‪ ، L‬ﻫﻨﺎ‬‫‪2‬‬
‫‪, Lz‬‬ ‫ﻓﻲ ﺍﻟﻔﺼل ﺍﻟﻘﺎﺩﻡ )‪ (11‬ﺴﻨﺤﺩﺩ ﺍﻟﻘﻴﻡ ﻭﺍﻟﺩﻭﺍل ﺍﻟﺨﺎﺼﺔ ﻟﻠﻤﺅﺜﺭﺍﺕ‬
‫ﺴﻨﻭﻀﺢ ﺃﻥ ﺍﺴﺘﺨﺩﺍﻤﻬﺎ ﻴﺒﺴﻁ ﻜﺜﻴﺭﺍﹰ ﺤل ﻤﻌﺎﺩﻟﺔ ﺸﺭﻭﺩﻨﺠﺭ‪ .‬ﻭﻫﺫﺍ ﻴﻨﺘﺞ ﻤﻥ ﺍﻟﻌﻼﻗﺔ‬
‫ﺍﻟﻤﺸﺘﻘﺔ ﺍﻟﺘﺎﻟﻴﺔ‪:‬‬
‫‪2‬‬
‫‪L = r×p‬‬ ‫] ) ‪( ) = [(r × p ) ] + [ (r × p ) ] + [ (r × p‬‬
‫‪2‬‬
‫‪x‬‬
‫‪2‬‬
‫‪y‬‬
‫‪2‬‬
‫‪z‬‬
‫‪2‬‬

‫∂ ‪‬‬ ‫∂‬ ‫∂ ‪‬‬ ‫‪∂ ‬‬


‫‪= − h 2  y − z‬‬ ‫‪  y − z ‬‬
‫‪ ∂z‬‬ ‫‪∂y‬‬ ‫‪  ∂z‬‬ ‫‪∂y ‬‬
‫∂ ‪‬‬ ‫∂‬ ‫∂ ‪‬‬ ‫‪∂ ‬‬
‫‪−h2  z − x‬‬ ‫‪ z − x ‬‬
‫‪ ∂x‬‬ ‫‪∂z‬‬ ‫‪  ∂x‬‬ ‫‪∂z ‬‬
‫∂ ‪‬‬ ‫∂ ‪∂ ‬‬ ‫‪∂ ‬‬
‫‪− h 2  x − y   x − y ‬‬
‫‪ ∂y‬‬ ‫‪∂x   ∂y‬‬ ‫‪∂z ‬‬
‫‪‬‬ ‫‪ ∂2‬‬ ‫‪∂2 ‬‬ ‫‪ ∂2‬‬ ‫‪∂2‬‬ ‫‪ 2  ∂2‬‬ ‫‪∂2‬‬ ‫‪‬‬
‫‪= − h 2  x 2  2 + 2  + y 2  2 + 2‬‬ ‫‪ + z  2 + 2‬‬ ‫‪‬‬
‫‪‬‬ ‫‪ ∂y ∂z ‬‬ ‫‪ ∂z ∂y‬‬ ‫‪‬‬ ‫‪ ∂x ∂y‬‬ ‫‪‬‬
‫‪ ∂2 ‬‬ ‫‪∂2‬‬ ‫‪∂2‬‬ ‫∂‬ ‫∂‬ ‫‪∂ ‬‬
‫‪− 2 xy ‬‬ ‫‪ − 2 yz‬‬ ‫‪− 2 zx‬‬ ‫‪− 2x − 2 y‬‬ ‫‪− 2z‬‬
‫‪ ∂x∂y ‬‬ ‫‪∂y∂z‬‬ ‫‪∂z∂x‬‬ ‫‪∂x‬‬ ‫‪∂y‬‬ ‫‪∂z ‬‬

‫‪- ١٣٨ -‬‬


‫‪PDF created with pdfFactory Pro trial version www.pdffactory.com‬‬
(r − p) = − h
2
2  ∂
 x
∂ ∂  ∂
+ y + z   x
∂ ∂ 
+ y + z 
 ∂x ∂y ∂z   ∂x ∂y ∂z 
∂2 ∂2 ∂2 ∂2 ∂2
= − h 2 (x 2 2 + y 2 2 + z 2 2 + 2xy + 2yz
∂x ∂y ∂z ∂x∂y ∂y∂z
∂2 ∂ ∂ ∂
+ 2zx +x +y +z ) (28)
∂z∂x ∂x ∂y ∂z

∂  ∂  ∂  ∂  ∂  ∂  ∂  ∂ 
=h2 { y y  − y  z  − z  y  + z  z 
∂ z  ∂ z  ∂z ∂y ∂y  ∂z  ∂y  ∂y
∂  ∂  ∂  ∂  ∂  ∂  ∂  ∂ 
+z  z  − z  x  − x  z  + x x 
∂x  ∂x ∂x  ∂z  ∂z  ∂x ∂ z  ∂ z 
∂  ∂  ∂  ∂  ∂  ∂  ∂  ∂ 
+x  x  −x  y  − y  x  + y y 
∂y  ∂y  ∂y  ∂z  ∂x  ∂y  ∂ x  ∂ z 
 ∂2   ∂2 ∂   ∂2 ∂ 
= y  y 2  − y z +  − z y + 
 ∂z   ∂ z ∂ y ∂ y  ∂ y∂ z ∂ z
 ∂2   ∂2   ∂2 ∂   ∂2 ∂ 
+ z z 2
+ z  z 2
− z  x +  − x z + 
 ∂y   ∂x   ∂ x ∂ z ∂ z  ∂z∂ x ∂x
 ∂2   ∂2   ∂2 ∂ 
+ x x 2  + x  x 2 
− x y + 
 ∂z   ∂y   ∂ y∂ z ∂ z
 ∂2 ∂   ∂2 
− y x +  + y y 
 ∂ x ∂ y ∂ y  ∂x∂z 
 ∂2 ∂2  2  ∂
2
∂2  2  ∂2 ∂2 
= y2  2 +  + x  2 +  + z  2 + 
∂ z ∂x∂z   ∂z ∂ y2  ∂ y ∂ x2 
 ∂2   ∂2   ∂2 
− 2yz   − 2z x   − 2xy  
∂ z ∂ y  ∂ z ∂ y  ∂ x ∂ y 
∂ ∂ ∂
− 2y − 2z − 2x
∂y ∂z ∂x

.‫( ﻴﻌﻁﻴﻨﺎ‬r . p ) ‫ﻭ‬


2 2
‫ﻭﻤﺠﻤﻭﻉ‬

( ) ∂∂x ∂2 ∂2  2 ∂ ∂ ∂ 
2
→ →
2
→2
+  r . p  = − h2 x2 + y2 + z2 + +  + h  x +y +z  (29)
  
2
∂ y2 ∂ z2   ∂x ∂y ∂ z 

- ١٣٩ -
PDF created with pdfFactory Pro trial version www.pdffactory.com
‫‪2‬‬
‫‪→2‬‬ ‫→ → ‪→ →‬‬ ‫→ →‬
‫‪+  r . p  = r . p + ih r . p‬‬ ‫)‪(30‬‬
‫‪‬‬ ‫‪‬‬

‫ﻻﺤﻅ ﺃﻨﻨﺎ ﻨﺘﻌﺎﻤل ﻤﻊ ﻤﺅﺜﺭﺍﺕ ﻭﻋﻠﻴﻪ ﻓﺈﻥ ﺍﻟﻤﺤﺎﻓﻅﺔ ﻋﻠﻰ ﺘﺭﺘﻴﺏ ﺍﻟﺤـﺩﻭﺩ‬
‫ﻤﻬﻡ ﺠﺩﺍﹰ ﻤﻥ )‪.(30‬‬
‫‪1 ‬‬ ‫‪→ →‬‬
‫‪2‬‬

‫‪+  r . p  − ih r . p ‬‬
‫→ →‬
‫‪p = 2 ‬‬
‫‪2‬‬ ‫‪2‬‬
‫)‪(31‬‬
‫‪r ‬‬ ‫‪‬‬ ‫‪‬‬ ‫‪‬‬

‫ﻻ ﻴﺘﺒﺎﺩﻻ‪.‬‬ ‫ﻭﻫﺫﻩ ﺍﻟﻨﺘﻴﺠﺔ ﺘﺨﺘﻠﻑ ﻤﻥ ﺍﻟﻨﺘﻴﺠﺔ ﺍﻟﺘﻘﻠﻴﺩﻴﺔ ﻭﺫﻟﻙ ﻷﻥ‬


‫→‬ ‫→‬
‫‪p . r‬‬

‫‪2‬‬
‫‪1‬‬ ‫‪1‬‬ ‫‪ ∂ ‬‬ ‫∂ ‪1‬‬
‫‪p = 2‬‬ ‫‪−h 2‬‬ ‫‪ r  − h 2‬‬ ‫ﻭﺒﺎﻋﺘﺒﺎﺭ‬
‫‪2‬‬ ‫‪2‬‬ ‫‪2‬‬

‫‪r‬‬ ‫‪r‬‬ ‫‪ ∂r‬‬ ‫‪r ∂r‬‬

‫‪ ( p = − i h‬ﻓﺈﻥ ﻤﻌﺎﺩﻟﺔ ﺸﺭﻭﺩﻨﺠﺭ ﺘﺄﺨﺫ ﺍﻟﺸﻜل‬ ‫∂‬


‫)ﻻﺤﻅ ﻫﻨﺎ ﺃﻨﻨﺎ ﻋﻭﻀﻨﺎ ﻋﻥ‬
‫→‬

‫‪∂r‬‬

‫‪h2  1‬‬ ‫∂ ‪ ∂  ∂  1‬‬ ‫‪1‬‬ ‫‪‬‬


‫‪−‬‬ ‫‪‬‬ ‫‪ r   r  +‬‬ ‫‪− 2 2‬‬ ‫‪2‬‬
‫‪ u E (r) + V (r) u E (r) = Eu E (r) #‬‬ ‫)‪(33‬‬
‫‪2μ  r 2‬‬ ‫‪ ∂r  ∂r r ∂r h r‬‬ ‫‪‬‬

‫ﻭﻟﻭ ﺃﻨﻨﺎ ﺃﺨﺫﻨﺎ ﺍﻟﺤﺴﺎﺒﺎﺕ ﺒﺎﻻﺤﺩﺍﺜﻴﺎﺕ ﺍﻟﻜﺭﻭﻴﺔ )ﺍﻟﻤﻭﻀﺤﺔ ﻓﻲ ﺍﻟﺸﻜل( ﻭﻫﺫﺍ‬


‫ﻫﻭ ﺍﻟﻁﺒﻴﻌﻲ ﻓﻲ ﻫﺫﻩ ﺍﻟﺤﺎﻟﺔ‪ ،‬ﺤﻴﻨﺌﺫ ﻓﺈﻥ ﺍﻟﻤﺅﺜﺭ ﺍﻟﻭﺤﻴﺩ ﺍﻟﺫﻱ ﻴﺤﺘﻭﻱ ﻋﻠﻰ ﺍﻟﺯﻭﺍﻴﺎ‬
‫‪ .‬ﻭﻟﻬﺫﺍ ﻓﻠﻭ ﺃﺨﺫﻨﺎ ﺍﻟﺩﻭﺍل ﻟﻬﺎ ﺍﻟﺸﻜل‪.‬‬ ‫‪2‬‬
‫ﻫﻭ‬ ‫‪y,θ‬‬ ‫ﺍﻟﻜﺭﻭﻴﺔ‬

‫)‪u E (r) = Yλ (θ , φ ) R Eλ (r‬‬ ‫)‪(34‬‬

‫ﺤﻴﺙ‬
‫‪2‬‬
‫) ‪Yλ (θ , φ ) = λ Yλ (θ , φ‬‬ ‫)‪(35‬‬

‫‪ ،‬ﻋﻨﺩﺌﺫ ﻓﺈﻥ ﺍﻟﻤﻌﺎﺩﻟﺔ ﺴﺘﻨﻔﺼل ﺇﻟـﻰ‬ ‫‪2‬‬


‫ﻫﻲ ﻤﻌﺎﺩﻟﺔ ﺍﻟﻘﻴﻤﺔ ﺍﻟﺨﺎﺼﺔ ﻟﻠﻤﺅﺜﺭ‬
‫)‪ (35‬ﻭﺍﻟﻤﻌﺎﺩﻟﺔ ﺍﻟﻘﻁﺭﻴﺔ )ﺍﻟﺨﻁﻴﺔ( ﻻﺤﻘﺎﹰ )‪.(39‬‬

‫‪- ١٤٠ -‬‬


‫‪PDF created with pdfFactory Pro trial version www.pdffactory.com‬‬
‫ﺍﻟﺸﻜل ﺃﻋﻼﻩ ﻴﺒﻴﻥ ﺍﻹﺤﺩﺍﺜﻴﺎﺕ ﺍﻟﻜﺭﻭﻴﺔ‪ ،‬ﻭﺍﻟﻌﻼﻗﺔ ﺒﻴﻥ ﺍﻹﺤﺩﺍﺜﻴﺎﺕ ﺍﻟﻜﺎﺭﺘﻴﺯﻴﺔ‬
‫)‪ (x,y,z‬ﻭﺍﻟﻜﺭﻭﻴﺔ )‪.(r, o, y‬‬
‫ﺇﻥ ﺍﻟﻁﺭﻴﻘﺔ ﺍﻟﻤﺘﺒﻌﺔ ﻫﻨﺎ ﻻ ﺘﺨﺘﻠﻑ ﻋﻥ ﺍﻟﻁﺭﻴﻘﺔ ﺍﻟﻌﺎﺩﻴﺔ ﺍﻟﻤﺘﻌﺎﺭﻑ ﻋﻠﻴﻬﺎ ﻓﻲ‬
‫ﻓﺼل ﺍﻟﻤﺘﻐﻴﺭﺍﺕ‪ .‬ﻭﻟﻜﻥ ﻫﻨﺎ ﻨﺅﻜﺩ ﻋﻠﻰ ﺩﻭﺭ ﺍﻟﺘﻤﺎﺜل ‪ symmetry‬ﻓﻲ ﺘﺤﺩﻴﺩ ﺍﻟﻔﺌـﺔ‬
‫ﺍﻟﻭﺍﺼﻠﺔ ﻟﻠﻤﺅﺜﺭﺍﺕ ﺍﻟﻤﺘﺒﺎﺩﻟﺔ )ﺍﻟﺘﺒﺎﺩﻟﻴﺔ( ‪.commuting epertes‬‬
‫ﻨﺭﻯ ﺃﻥ‪ λ‬ﻟﻬﺎ ﺃﺒﻌﺎﺩ ‪ ، h‬ﻭﻨﻜﺘﺒﻬﺎ ﺒﺎﻟﺸﻜل‪.‬‬
‫‪2‬‬

‫‪λ = l (l + 1) h 2‬‬ ‫)‪(36‬‬

‫ﻭﻨﺴﺘﺨﺩﻡ ﻫﺫﻩ ﺍﻟﺤﻘﻴﻘـﺔ ﻓـﻲ‬ ‫‪l = 0, 1, 2, 3, .............‬‬ ‫ﻓﻲ ﺍﻟﻔﺼل ﺍﻟﻘﺎﺩﻡ ﺴﻨﺜﺒﺕ ﺃﻥ‬
‫ﺍﻷﺠﺯﺍﺀ ﺍﻟﺘﺎﻟﻴﺔ‪ .‬ﺴﻨﻜﺘﺏ ﺍﻟﺩﻭﺍل ﺍﻟﺨﺎﺼﺔ ﻟﻜﻤﻴﺔ ﺍﻟﺘﺤﺭﻙ ﺍﻟﺯﺍﻭﻱ ﻋﻠـﻰ ﺍﻟـﺼﻭﺭﺓ‬
‫‪.‬‬ ‫‪z‬‬
‫ﻭ‬ ‫‪2‬‬
‫ﻫﻲ ﻜﺫﻟﻙ ﺩﺍﻟﺔ ﺨﺎﺼﺔ ﻟﻠﻤﺅﺜﺭ‬ ‫) ‪Ylm (θ , φ‬‬ ‫ﺘﺩل ﻋﻠﻰ‬ ‫‪l‬‬ ‫) ‪ ، Y (θ , φ‬ﺤﻴﺙ‬
‫‪lm‬‬

‫) ‪Ylm (θ , φ ) = mh Ylm (θ , φ‬‬ ‫)‪(37‬‬


‫‪z‬‬

‫‪−l ≤ m ≤ l‬‬ ‫ﻭﺴﻨﺠﺩ ﻜﺫﻟﻙ ﺃﻥ ‪ m‬ﻫﻭ ﺭﻗﻡ ﺼﺤﻴﺢ‬

‫‪- ١٤١ -‬‬


‫‪PDF created with pdfFactory Pro trial version www.pdffactory.com‬‬
‫ﺍﻟﺘﻲ‬ ‫‪Ylm‬‬ ‫ﻫﻲ ﺩﻭﺍل ﺨﺎﺼﺔ ﻟﻤﺅﺜﺭﺍﺕ ﻫﻴﺭﻤﻴﺘﻴﺔ‪ ،‬ﻓﺈﻨﻨﺎ ﻨﺘﻭﻗﻊ ﺃﻥ‬ ‫‪Ylm‬‬ ‫ﻭﺒﻤﺎ ﺃﻥ‬
‫ﺘﻨﺎﻅﺭ ﻗﻴﻡ ﺨﺎﺼﺔ ﻤﺨﺘﻠﻔﺔ ﺘﻜﻭﻥ ﻤﺘﻌﺎﻤﺩﺓ‪ .‬ﻭﺴﻨﺜﺒﺕ ﻓﻲ ﺍﻟﻔﺼل ﺍﻟﻘـﺎﺩﻡ ﺃﻥ ﻋﻨـﺩﻤﺎ‬
‫ﻴﻜﻭﻥ ﻫﻨﺎﻙ ﻤﻌﺎﻴﺭﺓ ﺤﻘﻴﻘﻴﺔ‪ ،‬ﻭﺍﻟﺘﻜﺎﻤل ﻋﻠﻰ ﻤﺩﻯ ﺍﻟﻤﺘﻐﻴﺭﺍﺕ ‪. φ ,θ‬‬
‫∫‬ ‫‪d Ω Yl m (θ , φ ) Yl‬‬ ‫) ‪(θ , φ‬‬
‫*‬
‫‪1 1‬‬ ‫‪2m2‬‬
‫‪D‬‬ ‫‪2D‬‬
‫‪= ∫ sin θ d θ ∫ dφ Yl m (θ , φ ) Yl‬‬ ‫‪(θ , φ ) = Sl1l2 S m1m2‬‬
‫*‬
‫)‪(38‬‬
‫‪1 1‬‬ ‫‪2m 2‬‬
‫‪ο‬‬ ‫‪ο‬‬

‫ﺍﻟﻤﻌﺎﺩﻟﺔ ﺍﻟﻨﺼﻑ ﻗﹸﻁﺭﻴﺔ ‪The Radial Equation‬‬

‫ﻋﻨﺩﻤﺎ ﻨﻌﻭﺽ )‪ (35‬ﻭ )‪ (36‬ﻓﻲ )‪ (33‬ﻭ )‪ ،(34‬ﻓﺈﻨﻨﺎ ﻨﺘﺤﺼل ﻋﻠﻰ ﺍﻟﻨﺼﻑ‬


‫ﻗﻁﺭﻴﺔ‪.‬‬
‫‪h 2 1 d  d  l ( l + 1) ‬‬
‫‪−‬‬ ‫‪r  −‬‬ ‫) ‪R (r) + V (r) R Elm = R Elm (r‬‬ ‫) ‪(39‬‬
‫‪2μ  r dr  dr ‬‬ ‫‪r 2  Elm‬‬

‫ﻻﺤﻅ ﺃﻨﻪ ﻻ ﻴﻭﺠﺩ ﺍﻋﺘﻤﺎﺩ ﻋﻠﻰ ‪ m‬ﻓﻲ ﻫﺫﻩ ﺍﻟﻤﻌﺎﺩﻟﺔ‪ .‬ﻭﻋﻠﻴﻪ‪ ،‬ﻓﺈﻨـﻪ ﻟﻘﻴﻤـﺔ‬
‫ﻤﻥ ﺍﻟﺘﻜﺭﺍﺭ ‪ ،deglnemacy‬ﻭﺫﻟﻙ ﻷﻥ ﻜل ﻗﻴﻡ ‪m‬‬ ‫)‪( 2l + 1‬‬ ‫ﺴﻴﻜﻭﻥ ﻫﻨﺎﻙ‬ ‫‪l‬‬ ‫ﻤﻌﻁﺎﻩ‬
‫ﺍﻟﻤﻤﻜﻨﺔ ﺴﻴﻜﻭﻥ ﻟﻬﺎ ﻨﻔﺱ ﺍﻟﻁﺎﻗﺔ‪ .‬ﻭﻟﻬﺫﺍ ﺤﺫﻓﻨﺎ ‪ m‬ﻤﻥ ﺍﻟﻤﻌﺎﺩﻻﺕ ﺍﻟﻨﺼﻑ ﻗﻁﺭﻴـﺔ‬
‫ﻭﻫﺫﻩ ﺍﻟﻤﻌﺎﺩﻟﺔ ﻴﻤﻜﻨﻨﺎ ﻜﺘﺎﺒﺘﻬﺎ ﻋﻠﻰ ﺍﻟﺼﻴﻐﺔ‪.‬‬
‫‪ d2 2 d ‬‬ ‫‪2μ ‬‬ ‫‪l (l + 1) h 2 ‬‬ ‫‪2 μE‬‬
‫‪ 2 +‬‬ ‫‪ R nl (r ) − 2  V (r ) +‬‬ ‫‪2‬‬ ‫‪ R nl (r ) + 2 R nl = 0‬‬ ‫)‪(40‬‬
‫‪ dr‬‬ ‫‪r dr ‬‬ ‫‪h ‬‬ ‫‪2μ r ‬‬ ‫‪h‬‬

‫‪ . R‬ﻭﺴـﻨﻘﻭﻡ‬ ‫‪ne‬‬
‫)‪(r‬‬ ‫ﺤﻴﺙ ﺍﺴﺘﺒﺩﻟﻨﺎ ﺍﻟﺭﻤﺯ ‪ E‬ﺒﺎﻟﺭﻤﺯ ‪ n‬ﻓﻲ ﺍﻟﺩﺍﻟـﺔ ﺍﻟﺨﺎﺼـﺔ‬
‫ﺒﺎﺨﺘﺒﺎﺭ ﻫﺫﻩ ﺍﻟﻤﻌﺎﺩﻟﺔ ﻟﻠﻌﺩﻴﺩ ﻤﻥ ﺃﺸﻜﺎل ﺍﻟﺠﻬﺩ ﻤﻘﻴﺩﺓ ﺒﺎﻟﺸﺭﻁ ﺃﻨﻬـﺎ ﺘـﺼﻴﺭ ﺇﻟـﻰ‬
‫ﻤﺎ ﻋﺩﺍ ﻟﻠﺤﺎﻟﺔ ﺍﻟﻤﻬﻤـﺔ ﺍﻟﺨﺎﺼـﺔ ﻟﺠﻬـﺩ ﻜﻭﻟـﻭﻡ‬ ‫‪1‬‬
‫ﺃﺴﺭﻉ ﻤﻥ‬ ‫∞‬ ‫ﺍﻟﺼﻔﺭ ﻋﻨﺩ‬
‫‪r‬‬
‫‪ Coulomb potential‬ﻭﺍﻟﺫﻱ ﺴﻴﻨﺎﻗﺵ ﻓﻲ ﺍﻟﻔﺼل )‪ .(12‬ﻭﺴـﻨﻔﺘﺭﺽ ﻜـﺫﻟﻙ ﺃﻥ‬
‫ﺍﻟﺠﻬﺩ ﻟﻴﺱ ﻤﻔﺭﺩ ‪.Singular‬‬

‫ﻋﻨﺩ ﻨﻘﻁﺔ ﺍﻷﺼل‪،‬‬ ‫‪1‬‬


‫ﻜﻔﺭﺩﻴﺔ‬
‫‪r‬‬

‫‪- ١٤٢ -‬‬


‫‪PDF created with pdfFactory Pro trial version www.pdffactory.com‬‬
‫‪lim r 2 V (r) = 0‬‬ ‫)‪(41‬‬
‫‪r →ο‬‬

‫ﻭﺇﻨﻪ ﻤﻥ ﺍﻟﻤﻨﺎﺴﺏ ﺃﺤﻴﺎﻨﺎﹰ ﺃﻥ ﻨﺴﺘﺤﺩﺙ ﺍﻟﺩﺍﻟﺔ‪.‬‬

‫)‪u nl (r) = r R nl (r‬‬ ‫)‪(42‬‬

‫ﻭﺒﻤﺎ ﺃﻥ‪:‬‬
‫‪ d 2 2 d  u re (r ) d  u 1 du  2  u 1 du ‬‬
‫‪ 2 +‬‬ ‫‪‬‬ ‫‪= − 2 +‬‬ ‫‪+‬‬ ‫‪− +‬‬
‫‪ dr‬‬ ‫‪r dr  r‬‬ ‫‪dr  r‬‬ ‫‪r dr  r  r r dr ‬‬
‫‪2‬‬ ‫‪1 du 1 d 2 u 1 du 2u 2 du‬‬
‫‪= 3 u− 2‬‬ ‫‪+‬‬ ‫‪−‬‬ ‫‪− +‬‬
‫‪r‬‬ ‫‪r dr r dr 2 r 2 dr r 3 r 2 dr‬‬
‫‪1 d2u‬‬
‫=‬ ‫)‪(43‬‬
‫‪r dr 2‬‬
‫∴ ﻤﻌﺎﺩﻟﺔ ﺸﺭﻭﺩﻨﺠﺭ ﻴﻤﻜﻨﻨﺎ ﻜﺘﺎﺒﺘﻬﺎ ﺍﻵﻥ )ﻤﻊ ﻀﺭﺏ ﻁﺭﻓﻴﻬﺎ ﻓﻲ ‪(r‬‬
‫‪d 2 u re (r) 2μ ‬‬ ‫‪l (l + 1 ) h 2‬‬ ‫‪‬‬
‫‪+‬‬ ‫‪‬‬ ‫‪E‬‬ ‫‪−‬‬ ‫‪V‬‬ ‫(‬‫‪r‬‬ ‫)‬ ‫‪−‬‬ ‫‪ u nl (r ) = 0‬‬ ‫) ‪(44‬‬
‫‪dr 2‬‬ ‫‪h2 ‬‬ ‫‪2μ r 2‬‬ ‫‪‬‬

‫ﻭﻫﺫﻩ ﺘﺒﺩﻭ ﻤﺸﺎﺒﻬﺔ ﻟﻠﻤﻌﺎﺩﻟﺔ ﻓﻲ ﺒﻌﺩ ﻭﺍﺤﺩ ﻤﺎ ﻋﺩﺍ‬


‫ﺘﻐﻴﺭ ﺒﺈﻀﺎﻓﺔ ﺍﻟﺤﺎﺠﺯ ﺍﻟﻤﺭﻜﺯ ﺍﻟﻤ‪‬ﻨﹶﻔﱢﺭ ‪repulsive centrifugal‬‬ ‫) ‪V (r‬‬ ‫)‪ (a‬ﺃﻥ ﺍﻟﺠﻬﺩ‬
‫‪l (l + 1)h 2‬‬
‫‪V (r ) → V (r ) +‬‬ ‫‪.barrier‬‬
‫‪2μ r 2‬‬

‫) ‪u = r R (r‬‬ ‫ﺍﻟﺸﻜل‪ :‬ﺍﻟﺠﻬﺩ ﺍﻟﻔﻌﺎل ﺍﻟﻤﺅﺜﺭ ﻓﻲ ﺍﻟﻤﻌﺎﺩﻟﺔ ﺍﻟﻨﺼﻑ ﻗﻁﺭﻴـﺔ ﻟــ‬


‫ﻋﻨﺩﻤﺎ ﻴﻜﻭﻥ ﺍﻟﺠﻬﺩ ﺍﻟﺤﻘﻴﻘﻲ ﻤﺭﺒﻊ‪.‬‬

‫‪- ١٤٣ -‬‬


‫‪PDF created with pdfFactory Pro trial version www.pdffactory.com‬‬
‫ﻭﻤﺤﺩﻭﺩﻴﺔ ﺍﻟﺩﺍﻟﺔ ﺍﻟﻤﻭﺠﻴﺔ ﻋﻨﺩ ﻨﻘﻁﺔ ﺍﻷﺼل ﻴﻠﺯﻡ‬ ‫) ‪u nl (r‬‬ ‫)‪ (b‬ﺍﻟﺘﻌﺭﻴﻑ ﻟﻠﺩﺍﻟﺔ‬
‫‪u nl (0 ) = 0‬‬ ‫ﺃﻥ‪:‬‬
‫ﻓﻲ ﺍﻟﺠﺎﻨﺏ ﺍﻷﻴﺴﺭ‪.‬‬ ‫∞‪V=r‬‬ ‫ﻤﻤﺎ ﻴﺠﻌﻠﻬﺎ ﻤﺜل ﻤﺴﺄﻟﺔ ﺍﻟﺒﻌﺩ ﺍﻟﻭﺍﺤﺩ ﺍﻟﺫﻱ ﻟﻪ‬
‫ﻭﺒﺈﺴﻘﺎﻁ ‪u nl‬‬ ‫ﺃﻭﻻﹰ‪ :‬ﻨﻌﺘﺒﺭ ﺍﻟﻤﻌﺎﺩﻟﺔ ﺍﻟﻨﺼﻑ ﻗﻁﺭﻴﺔ ﻗﺭﻴﺒﺎﹰ ﻤﻥ ﺍﻷﺼل ‪،Origin‬‬
‫ﻓﺈﻥ ﺍﻟﺤﺩﻭﺩ ﺍﻟﻤﻬﻴﻤﻨﺔ ﻓﻲ ﺍﻟﻤﻌﺎﺩﻟﺔ‪.‬‬ ‫‪r→ο‬‬ ‫ﻋﻨﺩﻤﺎ‬
‫)‪d 2 u l (l + 1‬‬
‫‪− 2 u~ο‬‬ ‫) ‪(47‬‬
‫‪dr 2‬‬ ‫‪r‬‬
‫ﻭﺒﺎﻟﺘﻌﻭﻴﺽ ‪Ansatz‬‬

‫‪u (r ) ~ r s‬‬

‫ﺴﻨﺠﺩ ﺃﻥ ﺍﻟﻤﻌﺎﺩﻟﺔ ﺘﻜﻭﻥ ﺼﺤﻴﺤﺔ ﺒﺸﺭﻁ ﺃﻥ‪:‬‬


‫‪s ( s −1) − l ( l + 1 ) = 0‬‬

‫‪s =−l‬‬ ‫ﺃﻭ‬ ‫‪s = l +1‬‬ ‫ﺃﻱ ﺃﻥ‬


‫ﻴـﺴﻤﻰ‬ ‫‪r l +1‬‬ ‫ﻭﺍﻟﺤل ﺍﻟﺫﻱ ﻴﺤﻘﻕ ﺍﻟﺸﺭﻁ ‪ u (ο) = 0‬ﺃﻱ ﺍﻟﺤل ﺍﻟﺫﻱ ﻴﺴﻠﻙ ﻤﺜـل‬
‫ﻫﻭ ﺍﻟﺫﻱ ﺍﻟﻐﻴﺭ ﻤﻨـﺘﻅﻡ‬ ‫‪r -l‬‬ ‫ﺍﻟﺤل ﺍﻟﻤﻨﺘﻅﻡ ‪ regular sohtim‬ﺍﻟﺤل ﺍﻟﺫﻱ ﻴﺴﻠﻙ ﻤﺜل‬
‫‪ irregnlar‬ﻭﺒﺩﻻﻟﺔ ﺍﻟﻤﻌﺎﺩﻟﺔ ﺍﻟﻨﺼﻑ ﻗﻁﺭﻴﺔ ) ‪ R (r‬ﻟﺩﻴﻨﺎ ﺍﻟﺤل ﺍﻟﻤﻨﺘﻅﻡ ﻴﺴﻠﻙ ﻤﺜل‬
‫ﻭﺍﻟﺤل ﺍﻟﻐﻴﺭ ﻤﻨﺘﻅﻡ ﻴﺴﻠﻙ ﻤﺜل‬
‫ﻋﻨﺩﻤﺎ ﺘﻜﻭﻥ ‪ r‬ﻜﺒﻴﺭﺓ ﺒﺈﻤﻜﺎﻨﻨﺎ ﺇﻫﻤﺎل ﺤﺩﻭﺩ ﺍﻟﺠﻬﺩ ﻭﺍﻟﻤﻌﺎﺩﻟﺔ ﺘﺼﺒﺢ‪.‬‬
‫‪d 2 u 2μ E‬‬
‫‪+ 2 u =0‬‬ ‫) ‪(50‬‬
‫‪dr 2‬‬ ‫‪h‬‬
‫ﻭﺸﺭﻁ ﻤﺭﺒﻊ ﺍﻟﺘﻜﺎﻤل‬
‫∞‬
‫) ‪1 = ∫ d r ψ (r‬‬ ‫∫‬ ‫) ‪r 2 dr ∫ d Ω R nl (r ) Ylm ( θ , ϕ‬‬ ‫)‪(51‬‬
‫‪2‬‬ ‫‪2‬‬ ‫‪2‬‬

‫‪ο‬‬
‫∞‬

‫∫‬ ‫) ‪r 2 dr R nl (r‬‬
‫‪2‬‬
‫=‬
‫‪ο‬‬

‫ﺃﻱ‪:‬‬
‫∞‬

‫∫‬ ‫) ‪dr u nl (r‬‬ ‫) ‪(52‬‬


‫‪2‬‬

‫‪ο‬‬

‫‪- ١٤٤ -‬‬


‫‪PDF created with pdfFactory Pro trial version www.pdffactory.com‬‬
‫∞‬ ‫ﻟﻜﻲ ﺘﺘﻼﺸﻰ ﺍﻟﺩﺍﻟﺔ ﻋﻨﺩ‬
‫‪E < 0‬‬ ‫* ﻓﻲ ﺤﺎﻟﺔ‬
‫‪2μ E‬‬
‫‪= −α2‬‬ ‫)‪(53‬‬
‫‪h2‬‬
‫‪d 2u‬‬
‫‪2‬‬
‫‪+α 2 u = 0‬‬ ‫ﺍﻟﺤل ﺍﻟﻤﻘﺎﺭﺏ‬
‫‪dr‬‬

‫‪u (r ) ~ e − α r‬‬ ‫)‪(54‬‬


‫‪E > 0‬‬ ‫* ﻓﻲ ﺤﺎﻟﺔ‬
‫ﻟﺩﻴﻨﺎ ﺍﻟﺤﻠﻭل ﻭﺍﻟﺘﻲ ﺘﻜﻭﻥ ﻤﻌﺎﻴﺭﺓ ﻓﻲ ﺩﺍﺨل ﺼﻨﺩﻭﻕ‪.‬‬
‫‪2μ E‬‬
‫‪2‬‬
‫‪=k2‬‬ ‫)‪(55‬‬
‫‪h‬‬
‫‪d 2u‬‬
‫‪2‬‬
‫‪−u2 =0‬‬
‫‪dr‬‬
‫ﻜﺒﻴﺭﺓ ﺒﺤﻴﺙ ﺃﻥ ) ‪ V(r‬ﻤﻬﻤﻠﺔ ﻴﻜﻭﻥ ﻫﻭ ﺍﺘﺤﺎﺩ ﺨﻁـﻲ‬ ‫‪γ‬‬ ‫ﺍﻟﺤل ﻋﻨﺩﻤﺎ ﺘﻜﻭﻥ‬
‫‪.e‬‬ ‫‪− ikr‬‬
‫ﻭ‬ ‫‪e ikr‬‬ ‫ﻤﻥ‬

‫‪- ١٤٥ -‬‬


‫‪PDF created with pdfFactory Pro trial version www.pdffactory.com‬‬
‫‪The Hydrogen Atom‬‬ ‫ﺫﺭﺓ ﺍﻟﻬﻴﺩﺭﻭﺠﻴﻥ‬

‫ﺫﺭﺓ ﺍﻟﻬﻴﺩﺭﻭﺠﻴﻥ ﺃﺒﺴﻁ ﺫﺭﺓ ﻭﺫﻟﻙ ﻷﻨﻬﺎ ﺘﺤﺘﻭﻱ ﻋﻠﻰ ﺇﻟﻜﺘﺭﻭﻥ ﻭﺍﺤﺩ‪ .‬ﻭﻟﻬﺫﺍ‬
‫ﺍﻟﺴﺒﺏ ﺘﺼﺒﺢ ﻤﻌﺎﺩﻟﺔ ﺸﺭﻭﺩﻨﺠﺭ ﻤﻌﺎﺩﻟﺔ ﺠﺴﻡ ﻭﺍﺤﺩ ﻭﺫﻟﻙ ﺒﻌﺩ ﻓﺼل ﺤﺭﻜﺔ ﻤﺭﻜﺯ‬
‫ﺍﻟﻜﺘﻠﺔ ﻭﺴﻭﻑ ﻨﺘﻌﺎﻤل ﻤﻊ ﺫﺭﺍﺕ ﻤﺸﺎﺒﻬﺔ ﻟﻠﻬﻴﺩﺭﻭﺠﻴﻥ‪ ،‬ﺃﻱ ﺃﻥ ﺍﻟﺫﺭﺍﺕ ﺍﻟﺘﻲ ﺘﺤﺘﻭﻱ‬
‫ﺘﺤﺘﻭﻱ ﻋﻠﻰ ﺃﻜﺜﺭ ﻤـﻥ ﺒﺭﻭﺘـﻭﻥ‬ ‫ﻋﻠﻰ ﺇﻟﻜﺘﺭﻭﻥ ﻭﺍﺤﺩ ﻓﻘﻁ‪ ،‬ﻭﻟﻜﻥ ﺴﻨﺴﻤﺢ‬
‫ﻭﺍﺤﺩ‪ .‬ﻭﺍﻟﺠﻬﺩ ﻓﻲ ﻫﺫﻩ ﺍﻟﺤﺎﻟﺔ ﻴﻜﻭﻥ‬
‫‪Ze2‬‬
‫‪V(r ) = −‬‬ ‫)‪(1‬‬
‫‪r‬‬

‫ﻭﻤﻌﺎﺩﻟﺔ ﺸﺭﻭﺩﻨﺠﺭ ﺍﻟﻨﺼﻑ ﻗﻁﺭﻴﺔ ﺘﻜﻭﻥ‬


‫‪ d2 2 d ‬‬ ‫‪2μ ‬‬ ‫‪Ze 2 l (l + 1) h 2 ‬‬
‫‪ 2 +‬‬ ‫‪ R + 2  E +‬‬ ‫‪−‬‬ ‫‪ R =0‬‬ ‫)‪(2‬‬
‫‪ dr‬‬ ‫‪r dr ‬‬ ‫‪h ‬‬ ‫‪r‬‬ ‫‪2μ r 2 ‬‬

‫ﺇﻨﻪ ﻤﻥ ﺍﻟﻤﻨﺎﺴﺏ‬ ‫‪E < 0‬‬ ‫ﻭﺴﻨﺭﻜﺯ ﺍﻟﺩﺭﺍﺴﺔ ﻋﻠﻰ ﺍﻟﺤﺎﻻﺕ ﺍﻟﻤﻘﻴﺩﺓ‪ ،‬ﺃﻱ ﺤﻠﻭل‬
‫ﺃﻥ ﻨﻌﻤل ﺘﻐﻴﻴﺭ ﻓﻲ ﺍﻟﻤﺘﻐﻴﺭﺍﺕ‪.‬‬
‫‪1‬‬
‫‪8µ E‬‬ ‫‪‬‬ ‫‪2‬‬
‫‪δ = ‬‬ ‫‪2‬‬
‫‪‬‬
‫‪‬‬ ‫‪r‬‬ ‫)‪(3‬‬
‫‪ h‬‬ ‫‪‬‬

‫ﻓﻲ ﻫﺫﻩ ﺍﻟﺤﺎﻟﺔ ﺘﺼﺒﺢ ﺍﻟﻤﻌﺎﺩﻟﺔ )‪ (2‬ﻋﻠﻰ ﺍﻟﺼﻭﺭﺓ ﺍﻟﺘﺎﻟﻴﺔ‪:‬‬


‫)‪d 2 R 2 dR l ( l + 1‬‬ ‫‪λ 1‬‬
‫‪+‬‬ ‫‪−‬‬ ‫‪R+ −  R = 0‬‬ ‫)‪(4‬‬
‫‪dδ‬‬ ‫‪2‬‬
‫‪δ dδ‬‬ ‫‪δ‬‬ ‫‪2‬‬
‫‪δ 4‬‬

‫ﻭﻗﺩ ﺍﺴﺘﺤﺩﺜﻨﺎ ﺍﻟﻤﻌﺎﻤل )ﺒﺩﻭﻥ ﻭﺤﺩﺓ(‬


‫‪1‬‬ ‫‪1‬‬
‫‪Z e2‬‬ ‫‪ μ‬‬ ‫‪‬‬ ‫‪2‬‬
‫‪ μ C2‬‬ ‫‪‬‬ ‫‪2‬‬

‫=‪λ‬‬ ‫‪‬‬
‫‪2 E‬‬
‫‪‬‬
‫‪‬‬ ‫‪= Zα ‬‬ ‫‪‬‬
‫‪‬‬
‫)‪(5‬‬
‫‪h‬‬ ‫‪‬‬ ‫‪‬‬ ‫‪2 E‬‬ ‫‪‬‬

‫= ‪ ، α‬ﻭﺍﻟﻁﺎﻗﺔ ﻤ‪‬ﻌﺒﺭ ﻋﻨﻬﺎ ﺒﻭﺤﺩﺍﺕ‬ ‫‪1‬‬


‫ﺍﻟﻤﻌﺎﺩﻟﺔ )‪ (4‬ﻤﻥ ﺍﻟﺴﻬل ﺤﻠﻬﺎ‪ .‬ﻭﺫﻟﻙ‬
‫‪137‬‬

‫ﺍﻟﻜﺘﻠﺔ ﻋﻨﺩ ﺍﻟﺴﻜﻭﻥ ‪. rest – mass‬‬

‫‪- ١٤٦ -‬‬


‫‪PDF created with pdfFactory Pro trial version www.pdffactory.com‬‬
‫‪The Energy Spectrum‬‬ ‫ﻁﻴﻑ ﺍﻟﻁﺎﻗﺔ‬

‫ﺴﻨﺤﺎﻭل ﺤل ﻤﻌﺎﺩﻟﺔ )‪ (4‬ﺒﻁﺭﻴﻘﺔ ﻤﻌﺭﻭﻓﺔ ﻟﺩﻴﻨﺎ ﺍﻵﻥ‪ .‬ﺃﻭﻻﹰ ﺴـﻨﺠﺩ ﺴـﻠﻭﻙ‬


‫ﻜﺒﻴﺭﺓ ﻓﺎﻟﺤﺩﻭﺩ ﺍﻟﻤﺘﺒﻘﻴﺔ ﻓﻲ ﺍﻟﻤﻌﺎﺩﻟﺔ‪.‬‬ ‫‪δ‬‬ ‫ﺍﻟﺩﺍﻟﺔ ﻋﻨﺩﻤﺎ ﺘﻜﻭﻥ ‪ δ‬ﻜﺒﻴﺭﺓ‪ .‬ﻭﻋﻨﺩﻤﺎ ﺘﻜﻭﻥ‬
‫‪d2R 1‬‬
‫‪− R=0‬‬ ‫) ‪(6‬‬
‫‪dδ 2‬‬ ‫‪4‬‬

‫ﻭﺍﻟﺤل ﺍﻟﻤﻘﺒﻭل ﻋﻨﺩ ﻤﺎ ﻻ ﻨﻬﺎﻴﺔ‬

‫∞ → ‪∞ ← R ⇐δ‬‬ ‫ﻋﻨﺩ‬ ‫‪⇐ R ~e‬‬


‫‪+δ‬‬
‫‪2‬‬

‫‪−δ‬‬
‫‪2‬‬
‫‪R~e‬‬

‫ﻭﻜﻤﺎ ﻋﺎﻟﺠﻨﺎ ﺍﻟﻤﺘﺫﺒﺫﺏ ﺍﻟﺘﻭﺍﻓﻘﻲ‬


‫‪δ‬‬
‫‪−‬‬
‫‪R= e‬‬ ‫‪2‬‬
‫) ‪G (δ‬‬ ‫) ‪(7‬‬

‫ﻭﺒﺎﻟﺘﻌﻭﻴﺽ ﻓﻲ ﺍﻟﻤﻌﺎﺩﻟﺔ )‪(4‬‬

‫ﺴﻨﺘﺤﺼل ﻋﻠﻰ ﻤﻌﺎﺩﻟﺔ ﻟـ )‪:G (s‬‬


‫‪d 2 G  2  dG  λ − 1 l (l + )‬‬
‫‪− 1 − ‬‬ ‫‪+‬‬ ‫‪−‬‬ ‫‪G =0‬‬ ‫)‪(8‬‬
‫‪dδ 2  δ  dδ  δ‬‬ ‫‪δ 2 ‬‬

‫ﻭﺍﻵﻥ ﻨﻜﺘﺏ ﻤﻔﻜﻭﻙ ﺍﻟﻘﻭﺓ ﻟـ ) ‪ G (δ‬ﻭﺍﻟﺘﻲ ﺘﺄﺨﺫ ﺍﻟﺸﻜل‪:‬‬


‫∞‬
‫‪G (δ ) = δ l‬‬ ‫‪∑a‬‬ ‫‪n‬‬
‫‪δn‬‬ ‫)‪(9‬‬
‫‪n =ο‬‬

‫ﻭﺒﺘﻌﻭﻴﺽ ﺍﻟﻤﻌﺎﺩﻟﺔ )‪ (a‬ﻓﻲ ﺍﻟﻤﻌﺎﺩﻟﺔ ﺍﻟﺘﻔﺎﻀـﻠﻴﺔ‪ ،‬ﺴـﻨﺠﺩ ﺍﻟﻌﻼﻗـﺔ ﺒـﻴﻥ‬


‫ﻭﻤﻌﺎﺩﻟﺔ ﺍﻻﺴﺘﻁﺭﺍﺩ ﻴﻤﻜﻥ ﺍﻟﺤﺼﻭل ﻋﻠﻴﻬﺎ ﻤﻥ ﺍﻟﻤﻌﺎﺩﻟـﺔ‬ ‫‪an‬‬ ‫ﺍﻟﻤﻌﺎﻤﻼﺕ ﺍﻟﻤﺨﺘﻠﻔﺔ‬
‫ﺍﻟﺘﻔﺎﻀﻠﻴﺔ ﺍﻟﺨﺎﻀﻌﺔ ﻟﻠﻌﻼﻗﺔ‬
‫∞‬
‫‪H (δ ) = ∑ a n δ n‬‬ ‫)‪(10‬‬
‫‪n =ο‬‬

‫‪- ١٤٧ -‬‬


‫‪PDF created with pdfFactory Pro trial version www.pdffactory.com‬‬
‫ﻭﺍﻟﺘﻲ ﺘﻜﻭﻥ‬
‫‪d 2 H  2 l + 2  d H λ −1− l‬‬
‫‪+‬‬ ‫‪− 1‬‬ ‫‪+‬‬ ‫‪H=0‬‬ ‫)‪(11‬‬
‫‪dδ 2  δ‬‬ ‫‪ dδ‬‬ ‫‪δ‬‬

‫ﻭﻫﺫﻩ ﺍﻟﻤﻌﺎﺩﻟـﺔ ﻴﻤﻜﻨﻨـﺎ ﺍﻟﺤـﺼﻭل ﻋﻠﻴﻬـﺎ ﺒـﺴﻬﻭﻟﺔ ﻭﺫﻟـﻙ ﺒﺘﻌـﻭﻴﺽ‬


‫ﻓﻲ ﻤﻌﺎﺩﻟﺔ )‪(8‬‬ ‫) ‪G (δ ) = δ l H ( p‬‬

‫ﻓﻲ ﻫﺫﻩ ﺍﻟﺤﺎﻟﺔ‪:‬‬


‫∞‬
‫‪‬‬ ‫‪ 2l + 2 ‬‬ ‫‪‬‬
‫‪∑ n ( n − 1 ) a‬‬ ‫‪n‬‬
‫‪δ n − 2 + n a n δ n −1 ‬‬
‫‪ δ‬‬
‫‪− 1 + (λ − 1 − l ) a n δ n −1  = 0‬‬
‫‪‬‬
‫) ‪(12‬‬
‫‪n=0‬‬ ‫‪‬‬ ‫‪‬‬

‫‪∑ {(n + 1)[n a‬‬ ‫]‬ ‫}‬


‫∞‬

‫‪n +1‬‬
‫‪+ (2 l + 2 ) a n + 1 + ( λ − 1 − l − n ) a n δ n −1 = 0‬‬
‫‪n=0‬‬

‫ﻭﺒﻤﺎ ﺃﻥ ﻫﺫﻩ ﺍﻟﻤﻌﺎﺩﻟﺔ ﺘﺘﻼﺸﻰ ﺤﺩ ﺒﺤﺩ‪ ،‬ﻭﺴﻨﺘﺤﺼل ﻋﻠﻰ ﻤﻌﺎﺩﻟﺔ ﺍﻻﺴﺘﻁﺭﺍﺩ‪.‬‬


‫‪a n +1‬‬ ‫‪n + l +1− λ‬‬
‫=‬ ‫)‪(13‬‬
‫‪an‬‬ ‫) ‪( n + 1 )(n + 2l + 2‬‬

‫ﻭﻟﻘﻴﻡ ﻜﺒﻴﺭﺓ ﻟﻘﻴﻤﺔ ‪ n‬ﻓﺈﻥ ﺍﻟﻨﺴﺒﺔ ﺘﺼﺒﺢ‬


‫‪a n +1‬‬
‫≈‬
‫‪1‬‬
‫) ‪(14‬‬
‫‪an‬‬ ‫‪n‬‬

‫ﻭﻜﻤﺎ ﻓﻌﻠﻨﺎ ﻓﻲ ﺤﺎﻟﺔ ﺍﻟﻤﺘﺫﺒﺫﺏ ﺍﻟﺘﻭﺍﻓﻘﻲ‪ ،‬ﻓﺈﻨﻨﺎ ﻟﻥ ﻨﺘﺤﺼل ﻋﻠﻰ ﺴﻠﻭﻙ ﻤﻘﺒﻭل‬
‫‪n = nr‬‬ ‫ﻭ‬ ‫‪l‬‬ ‫ﺇﻻ ﺇﺫﺍ ﺍﻨﺘﻬﺕ ﺍﻟﺴﻠﺴﻠﺔ )‪ (a‬ﺃﻱ ﺃﻨﻪ ﻟﻘﻴﻤﺔ ﻤﻌﻁﺎﺓ ﻟـ‬ ‫∞‬ ‫ﻟﻠﺩﺍﻟﺔ ) ‪ R ( p‬ﻋﻨﺩ‬
‫ﻴﺠﺏ ﺃﻥ ﻨﺘﺤﺼل ﻋﻠﻰ‪.‬‬

‫ﺩﻋﻨﺎ ﺍﻵﻥ ﻨﺴﺘﺤﺩﺙ ﺍﻟﺭﻗﻡ ﺍﻟﻜﻤﻲ ﺍﻷﺴﺎﺴﻲ ‪ N‬ﺍﻟﻤﻌﺭﻑ ﺒـ‬

‫‪n = nr + l + 1‬‬ ‫)‪(16‬‬

‫‪⇐ nr ≥ 0‬‬ ‫ﻭﻴﻨﺘﺞ ﻤﻥ ﺍﻟﺤﻘﻴﻘﺔ ﺃﻥ‬


‫‪1. n ≥ l +1‬‬
‫‪2. n is au int eger‬‬

‫‪- ١٤٨ -‬‬


‫‪PDF created with pdfFactory Pro trial version www.pdffactory.com‬‬
‫‪1‬‬
‫‪E = − µ c2‬‬
‫) ‪(Zα‬‬ ‫‪2‬‬
‫)‪(14‬‬
‫‪2‬‬ ‫‪n2‬‬
‫‪E = hw‬‬

‫ﺍﻟﺘﻲ ﺘﻅﻬﺭ‬ ‫‪µ‬‬ ‫ﻭﻫﺫﻩ ﺍﻟﻨﺘﻴﺠﺔ ﺸﺒﻴﻬﺔ ﺒﻨﻤﻭﺫﺝ ﺒﻭﺭ‪ .‬ﻻﺤﻅ ﺃﻥ ﺍﻟﻜﺘﻠﺔ ﺍﻟﻤﺨﺘﺯﻟﺔ‬
‫ﻓﻲ ﺍﻟﻤﻌﺎﺩﻟﺔ‪ .‬ﺇﻥ ﻭﺠﻭﺩ ﺍﻟﻜﺘﻠﺔ ﺍﻟﻤﺨﺘﺯﻟﺔ‪.‬‬
‫‪mµ‬‬
‫=‪µ‬‬ ‫)‪(18‬‬
‫‪m+ M‬‬

‫ﺤﻴﺙ ‪ m‬ﻜﺘﻠﺔ ﺍﻹﻟﻜﺘﺭﻭﻥ‪ ،‬ﻭ ‪ M‬ﻜﺘﻠﺔ ﺍﻟﻨﻭﺍﺓ‪ ،‬ﻴﻌﻨﻲ ﺃﻥ ﺍﻟﺘﺭﺩﺩﺍﺕ‪.‬‬


‫‪Ei − Ej mC 2 /2h‬‬
‫= ‪ωiy‬‬ ‫=‬ ‫‪(Zα )2  12 − 12 ‬‬ ‫)‪(19‬‬
‫‪h‬‬
‫‪1+‬‬
‫‪m‬‬ ‫‪n ∫ n ∫‬‬
‫‪M‬‬

‫ﻭﺍﻟﺫﻱ ﻴﺨﺘﻠﻑ ﻗﻠﻴﻼﹰ ﻻﺨﺘﻼﻑ ﺫﺭﺍﺕ ﺍﻟﻬﻴﺩﺭﻭﺠﻴﻥ ﺍﻟﺸﺒﻴﻬﺔ ﻋﻠـﻰ ﺍﻷﺨـﺹ‬


‫ﺍﻟﻔﺭﻕ ﺒﻴﻥ ﻁﻴﻑ ﺫﺭﺓ ﺍﻟﻬﻴﺩﺭﻭﺠﻴﻥ ﻭﺫﺭﺓ ﺍﻟﺩﻴﺘﻭﻴﺭﻭﻡ‪.‬‬

‫‪- ١٤٩ -‬‬


‫‪PDF created with pdfFactory Pro trial version www.pdffactory.com‬‬
‫‪The Degeneracy of the spectrum‬‬ ‫ﻅﺎﻫﺭﺓ ﺘﺤﻠل ﺍﻟﻁﻴﻑ‬

‫ﺍﻵﻥ ﺴﻨﻨﺎﻗﺵ ﺘﺤﻠل ﻁﻴﻑ ﺍﻟﻁﺎﻗﺔ‬


‫ﻭ‬ ‫‪λ = nr + l +1‬‬ ‫ﻟﻘﻴﻤﺔ ‪ ، λ =1‬ﺍﻟﺤﺎﻟﺔ ﺍﻟﺩﻨﻴﺎ‪ ،‬ﻴﺠﺏ ﺃﻥ ﻴﻜـﻭﻥ )ﻤـﻥ ﺍﻟﻤﻌﺎﺩﻟـﺔ‬
‫‪ . n‬ﻫﺫﺍ ﻴﻌﻨﻲ ﺃﻨﻪ ﺘﻭﺠﺩ ﺤﺎﻟﺔ ﻭﺤﻴﺩﺓ ﻓﺭﻴﺩﺓ ﻟﻠﺤﺎﻟﺔ ﺍﻟﺩﻨﻴﺎ ‪ ground stale‬ﺇﺫﺍ‬ ‫‪r‬‬
‫‪= o, l = 0‬‬

‫ﻴﻜﻭﻥ ﻋﻨﺩﻨﺎ ﺍﺤﺘﻤﺎﻟﻴﻴﻥ‬ ‫‪⇐ λ=2‬‬ ‫ﺃﺨﺫﻨﺎ‬


‫‪(i ) n r =1, l = 0‬‬
‫‪(ii ) n r = 0 , l =1‬‬

‫‪ (i ) n‬ﻓﺈﻥ ﺍﻟﻤﻌﺎﺩﻟﺔ )‪ ،(13‬ﺘﻜﺘﺏ ﺒﺎﻟﺸﻜل‬ ‫‪r‬‬


‫‪=1, l = 0‬‬ ‫ﺒﺎﻟﻨﺴﺒﺔ ﻟﻠﺤﺎﻟﺔ ﺍﻷﻭﻟﻰ‬
‫‪a n +1‬‬ ‫‪n + l +1 − λ‬‬ ‫‪n + l + 1− nr − l −1‬‬
‫=‬ ‫=‬
‫‪an‬‬ ‫)‪(n + 1)(n + 2l + 2 ) (n + 1)(n + 2 l + 2‬‬
‫‪n − nr‬‬
‫=‬ ‫) ‪(20‬‬
‫) ‪(n + 1)(n + 2 l + 2‬‬

‫ﻭﻫﺫﺍ ﻴﺒﻴﻥ ﺃﻥ‪:‬‬


‫‪a1‬‬ ‫‪−1‬‬ ‫‪−1‬‬ ‫‪a‬‬
‫=‬ ‫=‬ ‫‪⇒ a1 = − ο‬‬
‫‪aο‬‬ ‫‪(ο + ο + 2) 1 × 2‬‬ ‫‪2‬‬
‫∞‬
‫‪⇒ H (δ ) = ∑ a n δ n = a ο δ ο + a 1 δ1‬‬
‫‪n=ο‬‬

‫‪ a‬‬ ‫‪‬‬


‫‪= a ο +  − ο‬‬ ‫‪ δ‬‬
‫‪ 2‬‬ ‫‪‬‬
‫‪ δ‬‬
‫) ‪H (δ‬‬ ‫‪= a ο 1 − ‬‬ ‫)‪(21‬‬
‫‪ 2‬‬

‫ﺒﻴﻨﻤﺎ ﺍﻟﺘﻭﺯﻴﻊ ﺍﻟﺯﺍﻭﻱ ﻤﺘﻤﺎﺜل ﻜﺭﻭﻴﺎﹰ‬

‫ﻟﻨﺄﺨﺫ ﺍﻟﺤﺎﻟﺔ ﺍﻟﺜﺎﻨﻴﺔ‪(i i ) n r = 0, l = 1 .‬‬

‫ﻫﻨﺎ ﺍﻟﺩﺍﻟﺔ ﺍﻟﻨﺼﻑ ﻗﻁﺭﻴﺔ ﺜﺎﺒﺘﺔ‬

‫‪H (δ ) = a ο‬‬

‫ﻟﻜﻥ ﺍﻟﺠﺯﺀ ﺍﻟﺯﺍﻭﻱ ﻤﻥ ﺍﻟﺩﺍﻟﺔ ﺍﻟﻤﻭﺠﻴﺔ ﻴﺘﻀﻤﻥ ) ‪. Y (ο , ϕ‬‬


‫‪1m‬‬

‫‪- ١٥٠ -‬‬


‫‪PDF created with pdfFactory Pro trial version www.pdffactory.com‬‬
‫ﺍﻟﺘﺤﻠل ﻴﻜﻭﻥ )‪ (2 l +1‬ﻭﻋﻠﻴﻪ ﻴﻭﺠﺩ ﺜﻼﺙ ﻤﻥ ﻤﺜل ﻫﺫﻩ ﺍﻟﺤﺎﻻﺕ‪.‬‬

‫ﻴﻭﺠﺩ ﻟـﺩﻴﻨﺎ‬ ‫‪λ =3‬‬ ‫ﻓﻲ ﺤﺎﻟﺔ‬ ‫‪3 +1= 4 = 2 2‬‬ ‫ﻴﻜﻭﻥ‬ ‫‪λ =n=2‬‬ ‫ﺍﻟﺘﺤﻠل ﺍﻟﻜﻠﻲ ﻟﻠﺤﺎﻟﺔ‬
‫‪n r = 2, l = 0‬‬ ‫ﺍﻻﺤﺘﻤﺎﻻﺕ ﺍﻟﺘﺎﻟﻴﺔ‪:‬‬
‫ﻭﻋﻠﻴـﻪ ﻓـﺈﻥ ﺍﻟﺩﺍﻟـﺔ‬ ‫‪a1‬‬
‫‪= −1‬‬ ‫ﻭ‬ ‫‪a2‬‬
‫‪=−‬‬
‫‪1‬‬
‫ﻫﻨﺎ ﻴﻭﺠﺩ ﺤﺎﻟﺔ ﻭﺍﺤﺩﺓ ﻭﺍﻟﺘﻲ ﻟﻬﺎ‬
‫‪aο‬‬ ‫‪a1‬‬ ‫‪6‬‬

‫ﺍﻟﻨﺼﻑ ﻗﻁﺭﻴﺔ ﺘﺘﻀﻤﻥ‪.‬‬


‫‪‬‬ ‫‪1 ‬‬
‫‪H (δ ) = a ο 1 − δ + δ 2 ‬‬ ‫)‪(22‬‬
‫‪‬‬ ‫‪6 ‬‬

‫‪، H (δ ) = a‬‬ ‫‪ο‬‬


‫‪ δ ‬‬
‫‪1 − ‬‬ ‫ﻟﻬﺎ ﺜﻼﺙ ﺤﺎﻻﺕ‪ ،‬ﺒـ‬ ‫‪nr = 1 , l = 1‬‬
‫‪‬‬ ‫‪4 ‬‬

‫‪H (δ ) = a ο‬‬ ‫ﺤﺎﻻﺕ ﺒـ‬ ‫)‪5 = (2l +1‬‬ ‫ﻟﻬﺎ‬ ‫‪nr = 0 , l = 2‬‬ ‫ﺒﻴﻨﻤﺎ‬
‫ﺘﺤﻠل ﻟﻠﺤﺎﻻﺕ ﺍﻟﺘﻲ ﻟﻬﺎ ﻗﻴﻤﺔ ﺨﺎﺼـﺔ )ﺫﺍﺘﻴـﺔ(‬ ‫‪1+ 3 + 5 = 9 = 32‬‬ ‫ﻭﻫﻜﺫﺍ ﻴﻭﺠﺩ‬
‫‪1 + 3 + 5 + ... + [2 (n − 1) + 1]= n 2‬‬ ‫ﻴﻜﻭﻥ‬ ‫‪λ=n‬‬ ‫ﻭﺒﺸﻜل ﻋﺎﻡ‪ ،‬ﺍﻟﺘﺤﻠل ﻓﻲ ﺤﺎﻟﺔ‬ ‫‪λ =n=3‬‬

‫ﻭﻤﺴﺒﻘﺎﹰ ﻨﺤﻥ ﺘﻭﻗﻌﻨﺎ ﺘﺤﻠل )‪ (2l +1‬ﻟﻠﺠﻬـﺩ ﺍﻟﻨـﺼﻑ ﻗﻁـﺭﻱ‪ ،‬ﻭﺫﻟـﻙ ﻷﻥ‬
‫‪.‬‬ ‫‪2‬‬
‫‪ ،‬ﻭﻟﻜﻥ ﻴﻌﺘﻤﺩ ﻋﻠﻰ‬ ‫‪z‬‬
‫ﺍﻟﻬﺎﻤﻠﺘﻭﻨﻴﺎﻥ ﺍﻟﻨﺼﻑ ﻗﻁﺭﻱ ﻻ ﻴﻌﺘﻤﺩ ﻋﻠﻰ‬

‫ﻫﻨﺎ ﻴﻭﺠﺩ ﺘﺤﻠل ﺇﻀﺎﻓﻲ‪ .‬ﻭﺍﻟﺘﺤﻠل ﺍﻟﺨﺎﺹ ﺼﻔﺔ ﻤﻤﻴﺯﺓ ﻟﻠﺠﻬﺩ ‪. 1‬‬
‫‪r‬‬

‫‪- ١٥١ -‬‬


‫‪PDF created with pdfFactory Pro trial version www.pdffactory.com‬‬
‫ﺍﻟﺩﻭﺍل ﺍﻟﺫﺍﺘﻴﺔ ﺍﻟﻨﺼﻑ ﻗﻁﺭﻴﺔ ‪The Radial Eigen functions‬‬

‫ﻓﻲ ﻋﻼﻗـﺔ ﺍﻻﺴـﺘﻁﺭﺍﺩ )‪(13‬‬ ‫‪λ=n‬‬ ‫ﺒﺎﻟﺭﺠﻭﻉ ﻟﻠﺩﺍﻟﺔ ﺍﻟﺘﻔﺎﻀﻠﻴﺔ‪ .‬ﻟﻭ ﻋﻭﻀﻨﺎ‬
‫ﺒﺤﻴﺙ‪:‬‬
‫‪k + l +1− n‬‬
‫= ‪a k +1‬‬ ‫‪a‬‬ ‫) ‪(26‬‬
‫‪(k + 1)(k + 2l + 2 ) k‬‬

‫ﺴﻨﺠﺩ ﺃﻥ‪:‬‬
‫)‪n − (k + l + 1‬‬ ‫) ‪n − (k + l‬‬
‫)‪a k + 1 = (− 1‬‬
‫‪k +1‬‬
‫‪.‬‬
‫)‪(k + 1)(k + 2l + 2) k (k + 2l +1‬‬
‫)‪n − ( l + 1‬‬
‫‪.........‬‬ ‫‪a‬‬ ‫) ‪(27‬‬
‫‪1. ( 2l + 2) ο‬‬

‫ﺒﻤﺴﺎﻋﺩﺓ ﻫﺫﻩ ﺍﻟﻌﻼﻗﺔ ﺒﺈﻤﻜﺎﻨﻨﺎ ﺍﻟﺤﺼﻭل ﻋﻠﻰ ﻤﻔﻜﻭﻙ ﺴﻠﺴﻠﺔ ﺍﻟﻘﻭﺱ ﻟﻠﺩﺍﻟـﺔ‬
‫) ‪ . H (δ‬ﻭﻤﺎ ﻴﻜﺎﻓﺊ ﻫﺫﺍ‪.‬ﻨﺠﺩ ﺃﻥ ﺍﻟﻤﻌﺎﺩﻟﺔ ﻟـ ) ‪ H (δ‬ﺘﻜﻭﻥ ﻟﻤﻌﻘﺩﺓ ﺍﻟﺤﺩﻭﺩ‬
‫= ) ‪H (δ‬‬ ‫) ‪(δ‬‬ ‫)‪(28‬‬
‫)‪( 2 l + 1‬‬
‫‪n − l −1‬‬

‫ﻭﺒﻌﺩﻤﺎ ﻨﹸﺤ‪‬ﻭل ﺒﺎﻟﻌﻜﺱ ﻟﻠﻭﺼﻭل ﻟﻺﺤـﺩﺍﺜﻴﺎﺕ ﺍﻟﻨـﺼﻑ ﻗﻁﺭﻴـﺔ ‪ r‬ﻭﺒﻌـﺩ‬


‫ﺍﻟﻤﻌﺎﻴﺭﺓ ﻨﺠﺩ ﺃﻥ ﺍﻟﺩﻭﺍل ﺍﻟﻨﺼﻑ ﻗﻁﺭﻴﺔ ﺍﻷﻭﻟﻰ ﻴﻤﻜﻥ ﻜﺘﺎﺒﺘﻬﺎ ﻋﻠﻰ ﺍﻟﺼﻭﺭﺓ‬
‫‪‬‬ ‫‪h ‬‬ ‫‪‬‬
‫‪Rnl (r ):‬‬ ‫= ‪ a ο‬‬ ‫‪‬‬ ‫‪‬‬
‫‪‬‬ ‫‪µ c α ‬‬ ‫‪‬‬
‫‪3‬‬ ‫‪‬‬
‫‪ z‬‬ ‫‪‬‬ ‫‪2‬‬ ‫‪−‬‬
‫‪zr‬‬
‫‪‬‬
‫‪R10 (r ) = 2 ‬‬ ‫‪‬‬ ‫‪‬‬
‫‪aο‬‬
‫‪‬‬ ‫‪e‬‬
‫‪ aο‬‬ ‫‪‬‬ ‫‪‬‬
‫‪3‬‬ ‫‪‬‬
‫‪ z‬‬ ‫‪‬‬ ‫‪‬‬ ‫‪ − 2 aο‬‬ ‫‪‬‬
‫‪2‬‬ ‫‪zr‬‬

‫‪R20 (r ) = 2 ‬‬ ‫‪‬‬ ‫‪1 − z r‬‬ ‫‪e‬‬ ‫‪12 − 30‬‬


‫‪‬‬ ‫‪ 2a‬‬ ‫‪‬‬
‫‪ 2 aο‬‬ ‫‪‬‬ ‫‪‬‬ ‫‪ο‬‬ ‫‪‬‬ ‫‪‬‬
‫‪3‬‬ ‫‪‬‬
‫‪ z‬‬ ‫‪‬‬ ‫‪‬‬ ‫‪2 z r 2 ( z r )  − 3 aο‬‬
‫‪zr‬‬
‫‪2‬‬ ‫‪2‬‬
‫‪‬‬
‫= ) ‪R21 (r‬‬
‫‪1‬‬ ‫‪‬‬ ‫‪‬‬ ‫‪ 1−‬‬ ‫‪+‬‬ ‫‪e‬‬ ‫‪‬‬
‫‪3  2 a ο‬‬ ‫‪‬‬
‫‪‬‬ ‫‪‬‬ ‫‪3 aο‬‬ ‫‪27 a ο2 ‬‬ ‫‪‬‬
‫‪3‬‬ ‫‪‬‬
‫‪ z‬‬ ‫‪‬‬ ‫‪‬‬ ‫‪2 z r 2 ( z r )  − 3 aο‬‬
‫‪zr‬‬
‫‪2‬‬ ‫‪2‬‬
‫‪‬‬
‫‪R30 (r ) = 2 ‬‬ ‫‪‬‬
‫‪‬‬ ‫‪ 1−‬‬ ‫‪+‬‬ ‫‪e‬‬ ‫‪‬‬
‫‪ 3 aο‬‬ ‫‪‬‬ ‫‪‬‬ ‫‪3 aο‬‬ ‫‪27 a ο2 ‬‬ ‫‪‬‬

‫‪- ١٥٢ -‬‬


‫‪PDF created with pdfFactory Pro trial version www.pdffactory.com‬‬
‫ﻟﻭﺠﻭﺩ ﺍﻹﻟﻜﺘﺭﻭﻥ ﻋﻨﺩ ﻤﺴﺎﻓﺔ ‪r‬‬ ‫) ‪P (r‬‬ ‫ﺭﺴﻡ ﻜﺜﺎﻓﺔ ﺍﻻﺤﺘﻤﺎل ﺍﻟﻨﺼﻑ ﻗﻁﺭﻱ‬
‫ﻤﻥ ﻨﻘﻁﺔ ﺍﻷﺼل ﻴﻤﻜﻥ ﺇﻴﺠﺎﺩﻩ ﻤﻥ ﻫﺫﻩ ﺍﻟﺩﻭﺍل ﺍﻟﻤﻭﺠﻴﺔ ﻜﻤﺎ ﻫﻭ ﻤﻭﻀﺢ ﺒﺎﻟـﺸﻜل‬
‫)‪ (12-2‬ﻴﺠﺏ ﺃﻥ ﻨﺘﺫﻜﺭ ﺃﻥ ﺍﻟﺩﺍﻟﺔ ﺍﻟﻤﻭﺠﻴﺔ ﻟﻬﺎ ﺠﺯﺀ ﺯﺍﻭﻱ‪ ،‬ﺍﻟﺫﻱ ﻤﺭﺒﻌﻪ ﺍﻟﻤﻁﻠﻕ‬
‫ﻤﻭﻀﺤﺔ ﺒﺎﻟﺭﺴﻡ‬ ‫) ‪Pem (cosθ‬‬
‫‪2‬‬
‫ﺍﻟﺭﺴﻭﻤﺎﺕ ﻟﺩﻭﺍل ﻟﻴﺠﻨﺩﺭ ﺍﻟﻤﺸﺎﺭﻜﺔ‬ ‫) ‪Pem (cosθ‬‬
‫‪2‬‬
‫ﻴﻜﻭﻥ‬
‫)‪.(12-3‬‬
‫ﻭﺒﻤﻌﺭﻓﺔ ﺍﻟﺩﻭﺍل ﺍﻟﻤﻭﺠﻴﺔ ﺒﺈﻤﻜﺎﻨﻨﺎ ﺤﺴﺎﺏ ﺍﻟﻘﻴﻤﺔ ﺍﻟﻤﺘﻭﻗﻌﺔ‪.‬‬
‫∞‬
‫]) ‪= ∫ dr r 2 + k [ Rnl (r‬‬
‫‪k‬‬ ‫‪2‬‬
‫‪r‬‬
‫‪ο‬‬

‫ﻭﻫﺫﻩ ﺒﻌﺽ ﺍﻟﻘﻴﻡ ﺍﻟﻤﺘﻭﻗﻌﺔ ﺍﻟﻤﻔﻴﺩﺓ‪.‬‬


‫‪‬‬
‫= ‪r‬‬
‫‪aο‬‬
‫‪2z‬‬
‫‪[3n‬‬ ‫‪2‬‬
‫]‬
‫)‪− l (l + 1‬‬
‫‪‬‬
‫‪‬‬
‫‪‬‬
‫[‬ ‫]‬
‫‪2 2‬‬
‫‪a n‬‬
‫‪r2‬‬ ‫‪= ο 2 5 n 2 + 1 − 3 l (l + 1) ‬‬
‫‪2z‬‬ ‫‪‬‬
‫‪‬‬
‫‪1‬‬ ‫‪z‬‬ ‫)‪ (12 − 36‬‬
‫=‬ ‫‪‬‬
‫‪r‬‬ ‫‪aο n 2‬‬ ‫‪‬‬
‫‪1‬‬ ‫‪z‬‬ ‫‪2‬‬ ‫‪‬‬
‫=‬ ‫‪‬‬
‫‪r2‬‬ ‫‪2 3‬‬ ‫‪1‬‬ ‫‪‬‬
‫‪aο n  l + ‬‬
‫‪‬‬ ‫‪2‬‬ ‫‪‬‬

‫‪- ١٥٣ -‬‬


‫‪PDF created with pdfFactory Pro trial version www.pdffactory.com‬‬

You might also like